Combined Deck: 24.1, 23.2, 23.1, 22.2, 22.1, 21.1, 21.2, 20.1, 20.2 Flashcards

MCQs with remembered answeres from the last 5yrs of released stems for the ANZCA part 2 exam

1
Q

2hyp2.1A high mixed venous oxygen saturation (SvO2) is most likely to be associated with

a. COPD
b. PE / Tamponade
c. Acute MI
d. Severe liver failure
e. Sepsis

A

d. Severe liver failure
but could also be
e. Sepsis

LIFTL:

INTERPRETATION

High SvO2
- increased O2 delivery (increased FiO2, hyperoxia, hyperbaric oxygen)
- decreased O2 demand (hypothermia, anaesthesia, neuromuscular blockade)
- high flow states: sepsis, hyperthyroidism, severe liver disease

Low SvO2
- decreased O2 delivery:
1. decreased Hb (anaemia, haemorrhage, dilution)
2. decreased SaO2 (hypoxaemia)
3. decreased Q (any form of shock, arrhythmia)

  • increased O2 demand (hyperthermia, shivering, pain, seizures)
  • Causes of High SvO2 despite evidence of End-organ Hypoxia:
    1. microvascular shunting (e.g. sepsis)
    2. histotoxic hypoxia (e.g. cyanide poisoning)
    3. abnormalities in distribution of blood flow

Anesthesia Monitoring Of Mixed Venous Saturation:
https://www.ncbi.nlm.nih.gov/books/NBK539835/

In sepsis, ScvO2 less than 70% or SvO2 lower than 65% correlate with poor prognosis.[2] In application, certain studies have shown that maintaining a goal ScvO2 greater than 70% leads to reduced mortality.[11] Therefore, ScvO2 is used to guide treatment algorithms in the Surviving Sepsis Campaign (SSC).

Studies have shown that normal to higher levels of mixed venous oxygen saturation in patients with clinically worsening sepsis do not rule out tissue hypoxia due to the inability to utilize O2.[11][7] Therefore, several studies support the conclusion that abnormally low or high ScvO2 correlates with higher mortality in patients with septic shock.

How well did you know this?
1
Not at all
2
3
4
5
Perfectly
2
Q

20.1 A patient has foam sclerotherapy to treat a number of varicose veins. Following the procedure she stands, immediately loses consciousness and develops a unilateral limb weakness. The most likely mechanism is

a. Anaphylaxis
b. Intracranial bleed
c. Paradoxical gas embolus
d. Thromboembolic stroke

A

c. Paradoxical gas embolus

Although liquid-injection sclerotherapy is the criterion standard, foam sclerotherapy is becoming a popular alternative because of its efficacy and success rate.1 A potential complication of foam sclerotherapy is the formation of gas microemboli in the brain, which can lead to neurologic deficits.

https://www.degruyter.com/document/doi/10.7556/jaoa.2016.063/html?lang=en

How well did you know this?
1
Not at all
2
3
4
5
Perfectly
3
Q

20.2 If group A Rh-ve fresh frozen plasma is not available for use in an A Rh-ve patient, of the following your next best choice should be

a. A+
b. B-
c. AB+
d. O+
e. O-

A

a. A+

Group A Plasma component preference
1st choice: A
2nd Choice: AB
3rd Choice: B

[a] If the patient is a female of childbearing potential, O RhD negative red cells should be used until the patient’s blood group is established.
[b] Group A platelets with the A2 subgroup don’t express significant amounts of A antigen and are therefore preferable to other group A platelets when transfusing group O and B recipients.
[c] Apheresis platelets that have a low titre anti-A/B or pooled platelets pose a lower risk of haemolysis when transfusing ABO incompatible components.
[d] Plasma components that have low titre anti-A/B pose a lower risk of haemolysis when transfusing ABO incompatible components.
[e] Group A plasma may be used as per local institutional policies. 

If no A, use AB Rh + cryo (Ie; no anti A or anti B)

Cryo incompatible can be given, but large volumes are high risk for DIC

https://litfl.com/cryoprecipitate/

How well did you know this?
1
Not at all
2
3
4
5
Perfectly
4
Q

20.1 In the treatment of diabetic ketoacidosis, the most important initial therapeutic intervention is to

a) Electrolyte correction
b) Insulin
c) IV hydration
d) Bicarbonate

A

IV hydration

Fluid first (hartmanns or saline w k+) then insulin
BJA Developments in the management of diabetic ketoacidosis 2015

Diabetic ketoacidosis (DKA) is a medical emergency and bedside capillary ketone testing allows timely diagnosis and identification of successful treatment.

> 0.9% saline with premixed potassium chloride should be the main resuscitation fluid on the general wards and in theatre; this is because it complies with National Patient Safety Agency recommendations on the administration of potassium chloride.

> Weight-based fixed rate i.v. insulin infusion (FRIII) is now recommended rather than a variable rate i.v. insulin infusion (VRIII).

> The blood glucose must be kept above 14 mmol litre−1 with the FRIII.

> Precipitating factor(s) needs to be identified and treated. Surgery and also critical care may be indicated to manage the patient presenting with DKA.

How well did you know this?
1
Not at all
2
3
4
5
Perfectly
5
Q

22.2 The nerve labelled by the arrow marked P in the diagram is the

  1. Ulnar Nerve
  2. Axillary Nerve
  3. Median Nerve
  4. Medial Cutaneous nerve of the forearm
  5. Long Thoracic Nerve
  6. Dorsal Scapular Nerve
  7. Radial Nerve
  8. Suprascapular nerve
  9. Musculocutaneous Nerve
A
  1. Long Thoracic Nerve
How well did you know this?
1
Not at all
2
3
4
5
Perfectly
6
Q

22.2 The most likely diagnosis for the following electrocardiograph is
(comment that this was like a 2015A repeat - ECG below is from that paper + 2022 recalled options)

a. AF with BBB
b. sinus tachy with BBB
c. ventricular tachycardia
d. torsades

A

b. sinus tachy with BBB

The most correct answer would be Trifasicular block:
RBBB with LAD (RBBB with left anterior hemiblock) and 1st degree heart block

Barash 8E 2017:
The term bifascicular block often refers to block in the right bundle and one of the two major fascicles of the left bundle. RBBB with left anterior hemiblock is present when the ECG shows an RBBB with a left axis deviation (usually greater than −60 degrees) in the absence of an inferior myocardial infarction. Complete RBBB with right axis deviation (greater than 90 degrees) is indicative of RBBB and left posterior hemiblock in the absence of a lateral myocardial infarction or evidence of right-sided heart failure. The term trifascicular block is used to describe first-degree AV block in the presence of bifascicular block.

Is it necessary to insert a temporary pacemaker before general anesthesia for an asymptomatic patient with bifascicular or trifascicular block?
The risk for progression to complete heart block in asymptomatic patients with bifascicular block is low. Further, no clinical characteristics have been identified that accurately predict the risk of development of complete heart block. Therefore, routine PPM implantation in patients with asymptomatic bifascicular block is not recommended. Observations made in the perioperative period have suggested that development of complete heart block during general anesthesia is also rare; therefore, it is generally not recommended that patients undergo temporary pacemaker insertion before general anesthesia. However, it is advisable to have an external pacemaker available in the operating room.

How well did you know this?
1
Not at all
2
3
4
5
Perfectly
7
Q

23.1 The initial treatment of a trigeminocardiac reflex during skull base surgery should be

a. Tell surgeons to stop stimulus
b. Atropine
c. LA to site

A

a) Tell the surgeons to stop stimulus

https://www.ncbi.nlm.nih.gov/pmc/articles/PMC1821135/

https://jamanetwork.com/journals/jamaotolaryngology/fullarticle/1864754

Careful dissection for prevention and early intervention with stimulus removal and anticholinergic use as needed are paramount to ensure good outcomes

N.B
Trigeminocardiac reflex refers to the sudden development of bradycardia or even asystole with arterial hypotension from manipulation of any sensory branches of the trigeminal nerve. Although it has only rarely been associated with morbidity and tends to be self-limited with removal of the stimulus, it is an important phenomenon for head and neck surgeons to recognize and respond to

How well did you know this?
1
Not at all
2
3
4
5
Perfectly
8
Q

23.1 You are planning to extubate a patient following airway surgery. The patient has FAILED the cuff-leak test when

a. <110ml leak with cuff deflated
b. >110ml leak with cuff deflated
c. Audible leak with cuff deflated
d. No audible leak with cuff deflated
e. No audible leak with cuff pressure <30cm H2O

A

a. <110ml leak with cuff deflated

approach is to use 110 mL or 10% of tidal volume as the cut-off

https://litfl.com/cuff-leak-test/

How well did you know this?
1
Not at all
2
3
4
5
Perfectly
9
Q

21.1 The lung ultrasound finding most consistent with atelectasis is three or more

A. B lines
B. A lines
C. Comet tails
D. Z lines
E. Lung Pulse

A

comet tails or B-lines

useful resource: https://academic.oup.com/bjaed/article/16/2/39/2897763

Comet Tail artefact:
- a short path reverberation artefact that weakens with each reverberation, resulting in a vertical echogenic artefact that rapidly fades as it continues in to the ultrasound image.
https://litfl.com/comet-tail-artefact/

Short path reverberation artefact
- The ultrasound appearance of this artefact is a thin vertical bright or echogenic line that passes from the point of origin, to the deepest part of the ultrasound image.

  • When appearing deep to the pleural line these are known as B-lines.
  • Elsewhere in the body the identical artefact is known as ring down artefact.
  • Where these artefacts fade quickly they are called comet tail artefacts

https://litfl.com/short-path-reverberation-artefact/

Radiopedia “B-line distribution corresponds with sub-pleural thickened interlobular septa” - more consistent with homogenous atelectasis

How well did you know this?
1
Not at all
2
3
4
5
Perfectly
10
Q

21.1 Infection control management of patients with carbapenemase-producing Enterobacteriaceae (CPE)
infection should include all of the following EXCEPT
a) isolation
b) contact precautions
c) droplet precautions
d) screening at risk patients with rectal swab and urine mcs

A

c) droplet precautions

https://www.safetyandquality.gov.au/sites/default/files/migrated/Recommendations-for-the-control-of-Carbapenemase-producing-Enterobacteriaceae.pdf

How well did you know this?
1
Not at all
2
3
4
5
Perfectly
11
Q

23.1 A patient will open her eyes in response to voice, speak with inappropriate words and
withdraw to a painful stimulus. Her Glasgow Coma Scale score is

a. 6
b. 7
c. 8
d. 9
e. 10

A

e. 10

How well did you know this?
1
Not at all
2
3
4
5
Perfectly
12
Q

20.1
a. Arndt blocker
b. Cohen blocker
c. Microlaryngeal tube
d. Husaker tuber
e. Parker Flex-tip tube

A

B Cohen Blocker

How well did you know this?
1
Not at all
2
3
4
5
Perfectly
13
Q

21.1 A transhiatal oesophagectomy is performed via a

a) laparotomy + right thoracotomy
b) laparotomy + left neck incision
c) laparotomy + left neck incision + Right thoractomy
d) Laparotomy + left thoractomy

A

midline laparotomy and left cervical incision

https://academic.oup.com/bjaed/article/17/2/68/2907833

Transhiatal oesophagectomy classically involves laparotomy and dissection of the lower oesophagus through an enlarged diaphragmatic hiatus, followed by removal of the oesophagus and re-anastomosis via a left cervical incision, thereby avoiding thoracotomy altogether (Fig. 2e).
- useful in patients with malignancies of the lower third of the oesophagus where thoracotomy is undesirable, such as those who have previously undergone thoracic surgery. - Dissection around the mediastinum is frequently associated with arrhythmias and ventricular compression causing hypotension (although this frequently occurs in transhiatal surgery, it is not uncommonly encountered during the thoracic phase of other approaches).

How well did you know this?
1
Not at all
2
3
4
5
Perfectly
14
Q

22.2 The Glasgow Coma Score of a patient whose best responses are: opening eyes to pain, making incomprehensible sounds, and withdrawing from pain is
a) 6
b) 8
c) 9
d) 10
e) 12

A

B) 8

E=2
V=2
M=4
Total= 8

How well did you know this?
1
Not at all
2
3
4
5
Perfectly
15
Q

Following a severe spinal cord injury, return of reflexes is usually seen after

a. <1 day
b. 1-3 days
c. 7 days
d. 1-4 weeks
e. >1 month

A

Answer: b, 1-3 days

BJA 2013 Initial Management of Acute Spinal Cord Injury

Spinal shock is the loss of reflexes below the level of SCI resulting in the clinical signs of flaccid areflexia and is usually combined with hypotension of neurogenic shock.

There is a gradual return of reflex activity when the reflex arcs below redevelop, often resulting in spasticity, and autonomic hyperreflexia.

This is a complex process and a recent four-phase classification to spinal shock has been postulated:

areflexia (Days 0 – 1),
initial reflex return (Days 1 – 3),
early hyperreflexia (Days 4 – 28), and
late hyperreflexia (1 – 12 months)

How well did you know this?
1
Not at all
2
3
4
5
Perfectly
16
Q

23.1 A man has symptomatic carbon monoxide poisoning. His pulse oximetry (SpO2) and arterial blood gas (PaO2) would be expected to show

a. Normal SpO2, Normal PaO2
b. Normal SpO2, reduced PaO2
c. Reduced SpO2, normal PaO2
d. Reduced SpO2, reduced PaO2

A

a. Normal - Normal

A normal or high oximetry reading should be disregarded because saturation monitors cannot differentiate between carboxyhaemoglobin and oxyhaemoglobin, owing to their similar absorbances.
The blood gas PO2 will also be normal in these cases (despite the presence of tissue hypoxia).

file:///Users/newuser/Downloads/BTS%20Guideline%20for%20oxygen%20use%20in%20adults%20in%20healthcare%20and%20emergency%20settings.pdf

How well did you know this?
1
Not at all
2
3
4
5
Perfectly
17
Q

21.1 The most common cause of postoperative visual loss after spinal surgery is

a. Central retinal artery occlusion
b. Central retinal vein occlusion
c. Ischemic optic neuropathy
d. Haemorrhage
e. corneal abrasion

A

c. Ischemic optic neuropathy

Cardiac: Anterior
Spinal: Posterior
ION

How well did you know this?
1
Not at all
2
3
4
5
Perfectly
18
Q

23.1 Therapeutic privilege is defined as

A. Withholding information to obtain consent
B. Getting presents and money for treating someone.
C. Not telling pt info because of their religious or cultural beliefs.
D. Withholding information to the patient if you think it will cause harm

A

D. Withholding information to the patient if you think it will cause harm

https://www.sciencedirect.com/topics/medicine-and-dentistry/therapeutic-privilege

“Therapeutic privilege,” also known as “therapeutic nondisclosure,” is defined as the withholding of relevant health information from the patient if nondisclosure is believed to be in the best interests of the patient (President’s Commission, 1982; Berger, 2005). The two most common justifications for such nondisclosure are that the disclosure would create incapacitating emotional distress and that disclosure would violate a patient’s personal, cultural, or other social requirements (Crawley et al., 2001; Berger, 2005).

How well did you know this?
1
Not at all
2
3
4
5
Perfectly
19
Q

20.2 A Jehovah’s Witness patient attends for a revision total hip replacement and is medically optimized. You consider she is high risk for the procedure but after extensive discussion agree to proceed, including agreeing that you will not give blood under any circumstances. Your decision can be justified on the basis of

a) Paternalism
b) Non maleficence
c) Autonomy
d) Beneficence

A

a) Autonomy
- Obligation to respect the decision-making capacities of persons.

Non-maleficence: Obligation to avoid causing harm
- If refused to proceed.

Paternalism: A set of attitudes and practices in which the health provider determines that a patient’s wishes or choices should not be honored.
- If transfused patient against their wishes

Beneficence: Obligation to provide benefits and to balance benefits against risks; obligation of physician to act for the benefit of the patient
- Controversial interpretation in this case. Both proceeding and refusing to do case may be acting for the benefit of the patient, depending on how you look at the scenario.

BJA: ‘MORAL balance’ decision-making in critical care
https://www.bjaed.org/article/S2058-5349(18)30145-8/fulltext

How well did you know this?
1
Not at all
2
3
4
5
Perfectly
20
Q

22.1 A four-year-old boy is in refractory ventricular fibrillation. The recommended dose of amiodarone is

A

80mg

Age + 4 x 2-> 4 + 4 x 2 =16kg
5 x 16mg =80mg

How well did you know this?
1
Not at all
2
3
4
5
Perfectly
21
Q

21.1 An 84-year-old woman with dementia presents for surgery for a breast lump. She lives in a care facility and is accompanied by the nurse manager from the facility and her son. Neither have a written legal authority to act on her behalf. Regarding consent for her surgery

a) Anaesthetic consent is implied in surgical consent
b) Son can’t consent
c) Legal guardian can’t consent
d) Not required if 2 Doctors are in agreement about the need for surgery
e) nil consent required if would be in patients interest/not against wishes

A

a) anaesthetic consent implied in surgical consent

or

e) nil consent required if would be in patients interest/ not against wishes

https://journals.sagepub.com/doi/pdf/10.1177/0310057X1003800504

The anaesthetist should give the parent or relative the same kind of information as a patient and recommend the appropriate procedure. The consent of a parent or relative is lawful authority to proceed. If the parent or relative does not wish to take this role, it may be necessary for a court or a guardianship body to appoint someone else to make the decision. This may take some time, and if the procedure is medically necessary and cannot be deferred, then reasonable treatment may be administered without consent (this is a principle of the common law and, in some jurisdictions, is also in legislation: for example, in New South Wales, the Guardianship Act 1987 (NSW) s 37; and in Victoria

How well did you know this?
1
Not at all
2
3
4
5
Perfectly
22
Q

22.2 Regarding cardiopulmonary exercise testing before major surgery, oxygen pulse is the

a. Arterial oxygen content at peak HR
b. Arterial oxygen saturation at mean HR?
c. Arterial oxygen saturation at peak HR
d. PaO2 at peak HR
e. Oxygen consumption/min divided by HR

A

e. Oxygen consumption/min divided by HR

VO2/HR: oxygen consumption divided by HR, known as the ‘oxygen pulse’ (ml beat–1)

https://www.bjaed.org/article/S2058-5349(19)30021-6/fulltext

The objective of CPET is to determine functional capacity in an individual.
Deficiencies in CPET-derived variables—specifically:
1. ventilatory anaerobic threshold (AT)
2. peak O2 consumption (VO2peak)
3. ventilatory efficiency for carbon dioxide (VE/VCO2)
—are associated with poor postoperative outcomes (mortality, morbidity, admission to intensive care, and length of hospital stay) after intra-abdominal surgery.

  1. Does the oxygen pulse increase with exercise?
    The oxygen pulse is the VO2 divided by HR, and represents the product of the stroke volume and the arterial-venous oxygen difference. It can be seen in panel 2 and can be viewed as a surrogate for stroke volume, and as such should increase at the start of exercise before slowly reaching a plateau at its highest predicted value.
How well did you know this?
1
Not at all
2
3
4
5
Perfectly
23
Q

23.1 A feature that is atypical of multiple sclerosis is

A. Unilateral visual loss
B. Aphasia
C. Diplopia
D. Lower limb motor
E. Some sensory thing

A

B. Aphasia

UTD

How well did you know this?
1
Not at all
2
3
4
5
Perfectly
24
Q

22.2 An absolute contraindication to transoesophageal echocardiography is
A. Dysphagia
B. GORD
C. Oesophageal stricture
D. oesophageal webbing
E. oesophageal varices

A

C. Oesophageal stricture

https://www.asecho.org/wp-content/uploads/2014/05/2013_Performing-Comprehensive-TEE.pdf

How well did you know this?
1
Not at all
2
3
4
5
Perfectly
25
Q

22.2 A four-year-old boy is in refractory ventricular fibrillation. The recommended dose of amiodarone is

a) 40mg
b) 80mg
c) 120mg
d) 160mg

A

80mg

Age + 4 x 2-> 4 + 4 x 2 =16kg
5 x 16mg =80mg

16kg x 5mg/kg = 80mg

How well did you know this?
1
Not at all
2
3
4
5
Perfectly
26
Q

23.1 According to National Audit Project (NAP) 5, the incidence of awareness during general anaesthesia for lower segment caesarean section should be quoted as

a) 1:700
b) 1:3,000
c) 1:8,000
d) 1:19,000
e) 1:36,000

A

a) 1:670 (or 1:700)

How well did you know this?
1
Not at all
2
3
4
5
Perfectly
27
Q

23.1 According to the Fourth Consensus Guidelines for the Management of Post-operative Nausea and Vomiting (PONV) published in 2020, multimodal PONV prophylaxis should be implemented in adult patients

a. For everyone
b. 1 or more RF
c. 2 or more RF
d. 3 or more RF
e. 4 or more RF

A

b) 1 or more RF

How well did you know this?
1
Not at all
2
3
4
5
Perfectly
28
Q

20.1 In a Blalock–Taussig shunt, blood passes to the pulmonary artery via the

a. Aorta
b. Subclavian artery
c. IVC
d. SVC
e. Left atrium

A

B

How well did you know this?
1
Not at all
2
3
4
5
Perfectly
29
Q

22.2 The prevention of microbial contamination of living tissues or sterile materials is known as

a. disinfection
b. antisepsis
c. decontamination
d. asepsis
e. sterilisation

A

d. asepsis

Asepsis: the prevention of microbial contamination of living tissues or sterile materials.
Disinfection: the inactivation of non-sporing organisms using either thermal or chemical means.
Sterilisation: complete destruction of all micro-organisms, including spores.

https://www.anzca.edu.au/getattachment/e4e601e6-d344-42ce-9849-7ae9bfa19f15/PG28(A)-Guideline-on-infection-control-in-anaesthesia

How well did you know this?
1
Not at all
2
3
4
5
Perfectly
30
Q

22.1 The risk of a perioperative respiratory adverse event in a child is least likely to be increased by

a. Asthma
b. Infection 3 weeks ago
c. History of eczema
d. Passive smoking

A

History of eczema

APRICOT study

How well did you know this?
1
Not at all
2
3
4
5
Perfectly
31
Q

21.1 Predictors of successful awake extubation after volatile anaesthesia in infants include

a. 2mL/kg tidal volume,
b. grimacing
c. coughing
d. RR > 20

A

b. grimacing

conjugate gaze
facial grimace
eye opening
purposeful movement
tidal volume greater than 5 ml/kg

Source: SPANZA 2019 article

How well did you know this?
1
Not at all
2
3
4
5
Perfectly
32
Q

22.2 A patient with an acute subarachnoid haemorrhage arrives in the emergency department. Her Glasgow Coma Scale score is 10 and she has no motor deficit. A CT brain shows diffuse subarachnoid haemorrhage with no localised areas of blood > 1 mm thick, and no intracerebral nor intraventricular blood. Her World Federation of Neurosurgical Societies (WFNS) grade of subarachnoid haemorrhage is

a) 1
b) 2
c) 3
d) 4
e) 5

A

4

GCS 7-12

How well did you know this?
1
Not at all
2
3
4
5
Perfectly
33
Q

20.2 The normal response of serum growth hormone level to an oral glucose load is

A. Initially increases then normalises
B. Initially decreases then normalises
C. Initially increases and stays elevated
D. Initially decreases then stays decreased
E. No response

A

B. Initially decreases then normalises

Oral glucose tolerance test — The most specific dynamic test for establishing the diagnosis of acromegaly is an OGTT. When performing the test, we measure serum GH before and two hours after glucose administration; the criterion for the diagnosis of acromegaly is a GH concentration greater than 1 ng/mL. In normal subjects, serum GH concentrations fall to 1 ng/mL or less within two hours after ingestion of 75 g glucose. In contrast, the post-glucose values are greater than 2 ng/mL in over 85 percent of patients with acromegaly.

Following oral glucose administration in humans, a transient suppression of plasma GH levels for 2–3 h is observed followed by a delayed rise occurring at 3–5 h post glucose ingestion. This initial suppression seems to be related to a glucose-mediated increase in hypothalamic somatostatin release. Evidence supporting this hypothesis emerges from the findings that in healthy individuals, GH secretion in response to GHRH or GH secretagogue is diminished after an oral glucose load. Furthermore, the inhibitory effect of glucose is reversed with the acetylcholinesterase inhibitor pyridostigmine, a substance thought to suppress somatostatin release from the hypothalamus. These findings support the hypothesis that oral glucose load is associated with a somatostatin release into the hypophyseal portal blood suppressing GH levels. The delayed GH rise would result from a decrease in somatostatinergic tone and hence an increase in GHRH. Subsequently, the available pituitary stores of GH are released leading to a rebound rise in GH.

How well did you know this?
1
Not at all
2
3
4
5
Perfectly
34
Q

22.1 A risk factor for postoperative nausea and vomiting in adults is age less than

a. 20
b. 30
c. 40
d. 50
e. 60

A

50

4th consensus guidelines for management of PONV

How well did you know this?
1
Not at all
2
3
4
5
Perfectly
35
Q

An awake patient in the post-anaesthesia care unit complains of breathlessness. The FiO2 through the patient’s rebreather mask is 40%. An arterial blood gas taken at the time shows (ABG shown). The alveolar-arterial gradient (in mmHg) is approximately

Blood gas shows:
PaO2 135
PaCO2 48
SpO2 100%

The A-a gradient is:
A. 5
B. 30
C. 60
D. 90
E. 110

A

D 90

A-a = PAO2 - PaO2

Alveolar air equation gives PAO2

PAO2 = PiO2 - PaCO2 / R
PAO2 = 0.4 x (760 - 47) - 48 / 0.8

so, as PaO2 given as 135
A-a = 228 - 135 = 93

How well did you know this?
1
Not at all
2
3
4
5
Perfectly
36
Q

23.1 Application of a pacemaker magnet to a dual-chamber implanted pacemaker would be expected to convert the operating mode to

a. AOO
b. VOO
c. DOO
d. AAI

A

c) DOO

The pacing mode will be DOO when the programmed pacing mode is a dual chamber mode or an MVP mode (AAIR<=>DDDR, AAI<=>DDD), VOO when the programmed pacing mode is a single chamber ventricular mode, and AOO when the programmed pacing mode is a single chamber atrial mode.

How well did you know this?
1
Not at all
2
3
4
5
Perfectly
37
Q

21.2 You have been asked to provide general anaesthesia for a complex thoracic endovascular aortic aneurysm repair. After the placement of a lumbar drain the recommended safe time before the administration of intravenous heparin is

a) 1 hour
b) 4 hours
c) 6 hours
d) 12 hours

A

1 hour

ASRA: 1 hour

Although the occurrence of a bloody or difficult neuraxial needle placement may increase risk, there are no data to support mandatory cancellation of a case. Direct communication with the surgeon and a specific risk-benefit decision about proceeding in each case are warranted.

Currently, insufficient data and experience are available to determine if the risk of neuraxial haematoma is increased when combining neuraxial techniques with the full anticoagulation of cardiac surgery. We suggest postoperative monitoring of neurologic function and selection of neuraxial solutions that minimise sensory and motor block to facilitate detection of new/progressive neurodeficits.

NYSORA:
Administration of intravenous heparin intraoperatively should be delayed for at least 1 hour after epidural placement; a delay before administration of subcutaneous heparin is not required. In cases of full heparinization for CPB, additional precautions include delaying surgery for 24 hours in the event of a traumatic tap, tightly controlling the heparin effect and reversal, and removing catheters when normal coagulation is restored.

How well did you know this?
1
Not at all
2
3
4
5
Perfectly
38
Q

22.1 The dose of hydrocortisone that has equivalent glucocorticoid effect to 8 mg dexamethasone is

a) 12mg
b) 25mg
c) 50mg
d) 100mg
e) 200mg

A

200mg Hydrocortisone or 25mg Prednisolone

Conversion
Prednisone 1mg =
Hydrocortisone 4mg =
Dexamethasone 0.15mg =
Triamcinolone 0.8mg =
Methylprednisolone 0.8mg =
Betamethasone 0.15mg =

(https://litfl.com/corticosteroids-overview/)

How well did you know this?
1
Not at all
2
3
4
5
Perfectly
39
Q

23.1 Causes of exhaled carbon dioxide detection following oesophageal intubation include
all of the following EXCEPT

a. Massive bronchopleural fistula.
b. Carbonated drink.
c. Vigorous bag valve masking previously.
d. Previous gastric insufflation with CO2 for endoscopy.
e. Tracheoesophageal fistula.

A

A Massive bronchopleural fistula.

Nick Chrimes 2022 - Journal of Anaesthesia
‘Preventing unrecognised oesophageal intubation: a consensus guideline from the Project for Universal Management of Airways and international airway societies’

Causes of exhaled carbon dioxide detection despite oesophageal intubation

No alveolar ventilation occurring
-Prior ingestion of carbonated beverages or antacids
-Gastric insufflation of CO2 for upper gastrointestinal endoscopy
-Prolonged ventilation with facemask or poorly positioned supraglottic airway before attempting tracheal intubation
-Bystander rescue breaths

Some alveolar ventilation potentially occurring
-Tracheo-oesophageal fistula with tube tip proximal to fistula
-Proximal oesophageal intubation with uncuffed tube in a paediatric patient

How well did you know this?
1
Not at all
2
3
4
5
Perfectly
40
Q

19.1, 20.1 Soon after a peribulbar block, the patient’s eye rapidly becomes proptosed and tense, and the visual acuity is markedly decreased. A lateral canthotomy is indicated to:

a) Allow globe to continue to swell
b) Drain blood from behind eyeball
c) Allow the eye to proptose
d) Reduce pressure on the optic nerve

A

c) Allow the eye to proptose

Orbital Compartment Syndrome

The orbital compartment is a fixed space with limited capacity for expansion. If something like blood fills part of that space the pressure increases and may result in ischaemia of the optic nerve or the retina. A lateral canthotomy is a way of releasing this pressure.

You have up to approximately 2 hours before irreversible visual loss occurs. It may occur in less than 2 hours however, so speed is of the essence.

use local anesthetic but warn the patient that they may feel pain

Perform the canthotomy:
place the scissors across the lateral canthus and incise the canthus full thickness

Perform cantholysis:
Grasp the lateral lower eyelid with toothed forcepsPull the lower eyelid anteriorlyPoint the scissors toward the patient’s nose, place the blades either side of the lateral canthal tendon, and cut.

By cutting the canthal tendon,the counter pressure of the eyelid on the is relieved and the eye is allowed to proptose and pressure is relieved.

How well did you know this?
1
Not at all
2
3
4
5
Perfectly
41
Q

23.1 A 50-year-old man presents with a subarachnoid haemorrhage. He undergoes
cerebral angiography and the frontal view is shown below. His cerebral aneurysm is
in the

(exact image on exam)

a. Anterior choroidal
b. Anterior communicating artery
c. MCA
d. PCA

A

b) anterior communicating artery

https://case.edu/med/neurology/NR/SubarachnoidHemorrhageAComm3.htm

https://www.thieme-connect.com/products/ejournals/pdf/10.1055/s-0039-1681979.pdf

https://case.edu/med/neurology/NR/NRHome.htm (scroll down to subarachnoid imaging area)

How well did you know this?
1
Not at all
2
3
4
5
Perfectly
42
Q

21.1 The part of the lung that is typically divided into superior, medial, anterior, lateral and posterior bronchial segments is the

A. Right Upper lobe
B. Right Lower lobe
C. Left Upper lobe
D. Right Middle lobe
E. Lingula

A

RLL
1.Superior (apical bronchus 6)
-> most common site for foreign body or secretions to collect if patient laying flat in bed

How well did you know this?
1
Not at all
2
3
4
5
Perfectly
43
Q

20.2 The most common type of perioperative stroke is

a) Hypoxic
b) Thrombotic
c) Embolic
d) Hypotensive
e) Haemorrhagic

A

c) Embolic

Blue Book 2017
Perioperative Stroke

Epidemiology
A perioperative stroke is defined as one that occurs either intra-operatively or in the post-operative period within 30 days70. Perioperative strokes are associated with an increased length of stay and a six-fold increased mortality. Any combination of surgery and anaesthesia is associated with an increased risk of stroke irrespective of the type of surgery. This may relate to coagulation changes

The most common type of perioperative stroke is ischaemic stroke of embolic origin (heart or aorta). Hypotension is rarely the cause of perioperative stroke. Haemorrhagic stroke is uncommon which probably reflects the fact that severe hypertension during anaesthesia is a rare event, and anticoagulants have typically been withheld.

The risk of perioperative stroke varies depending on the type of the surgery and patients’ risk factors.

Procedural risk
Urgent surgery is associated with an increased risk of stroke when compared to elective surger.

Cardiac, vascular and brain surgeries are defined as “high-risk” as these have an increased risk of stroke when compared to other types of surgery. Valvular and aortic repair surgeries have a stroke risk as high as 8 to 10 per cent.

Perioperative strokes in non-high-risk surgery are relatively rare and are estimated to have an incidence of about 1/1000 cases80.

Patients’ risk factors
>Age
>history of previous stroke or transient ischaemic attack
>renal failure
>atrial fibrillation
>history of cardiovascular diseases
are identified risk factors for perioperative stroke.

Atrial fibrillation is associated with a two-fold increase in the risk of death and stroke after carotid endarterectomy.

How well did you know this?
1
Not at all
2
3
4
5
Perfectly
44
Q

21.1 In the morbidly obese the induction dose of propofol should be calculated based on

a. Lean body weight
b. Total body weight
c. Ideal body weight
d. Ideal body weight + 70%

A

Lean Body Weight

For infusion: Adjusted body weight
NDMB: Lean Body weight
Sux: Total body weight

Source: SOBA UK

How well did you know this?
1
Not at all
2
3
4
5
Perfectly
45
Q

A 50-year-old woman has had a headache for the last month which is relieved by lying flat. She has had no medical procedure to her spine such as epidural, spinal or lumbar puncture. Her brain magnetic resonance imaging (MRI) scan shows diffuse meningeal enhancement and brain sagging. Her neurologist suspects spontaneous intracranial hypotension and asks you to do an epidural blood patch. No spinal imaging has been performed to confirm a cerebrospinal fluid (CSF) leak. You should

A do LP to measure pressure if low do lumbar patch
B do blood patch at lumbar level with no further investigation
C do spine imaging if CSF leak present do blood patch at level
D do spine imaging if CSF leak present do lumbar blood patch
E refuse to do blood patch

A

B do blood patch at lumbar level with no further investigation

How well did you know this?
1
Not at all
2
3
4
5
Perfectly
46
Q

21.2 The image below on the left shows a normal central venous pressure (CVP) trace. The CVP
trace in the image below on the right is most consistent with

a) AF
b) MR
c) AR
d) TR
e) Pericardial constriction

A

TR

How well did you know this?
1
Not at all
2
3
4
5
Perfectly
47
Q

22.2 Despite an interscalene block being performed preoperatively for arthroscopic rotator cuff repair, a patient wakes up with posterior shoulder pain. The most appropriate procedure to consider would be a nerve block of the

a. Supraclavicular nerve
b. Suprascapular nerve
c. Medial pectoral
d. Vagus nerve

A

b. Suprascapular nerve

Suprascapula nerve (C5,6)
- innervates supra and infraspinatus
- comes off superior trunk of the brachial plexus, and is usually anaesthetised by an interscalene block
- sensory innervation to 70% posterior-superior shoulders and portion of the anterior axilla and the ACJ

Supraclavicular nerve (C3,4)
- provides sensory to the ‘cape’ of the shoulder
- component of the cervical plexus block
- lies outside the brachial plexus
- commonly missed during supraclavicular brachial plexus blocks

Subscapular nerve:
- subscapularis
- medial rotation shoulder

Dorsal scapular nerve:
- branch of the brachial plexus
- supplies rhomboid major muscle, rhomboid minor muscle, and levator scapulae muscle
- causes the scapula to be moved medially towards the vertebral column
- Dorsal scapular nerve syndrome can cause a winged scapula, with pain and limited motion

Thoracodorsal nerve:
- thoracodorsal nerve also branches from the posterior division of the brachial plexus
- this nerve innervates the latissimus dorsi muscle.

https://resources.wfsahq.org/atotw/the-shoulder-block/

How well did you know this?
1
Not at all
2
3
4
5
Perfectly
48
Q

21.1 A baby is brought to the emergency department three days after a term home birth. It has not been feeding well and has had few wet nappies. The child is grey in appearance and femoral pulses are difficult to palpate. You note an enlarged liver and marked tachycardia. Pulse oximetry reveals
saturations of 75% despite oxygen being administered. You suspect a duct-dependent circulation. The best initial management is

a) Intubation and controlled ventilation
b) 20ml/kg crystalloid bolus
c) Alprostadil (PGE1)
d) Stop administration of oxygen

A

c) Alprostadil (PGE1)

From Paediatric BASIC on CHD:
- Resuscitation of an infant or newborn in shock should follow a standard approach regardless of the aetiology.
- Any patient with a duct dependent lesion either for pulmonary blood flow, or systemic output, will require PGE1. The problem is that whether or not a duct dependent lesion is present is unclear in most cases. If CHD has been diagnosed antenatally, PGE1 should be started.
- The cyanosed neonate presenting with severe cyanosis (O2 <75) and/or in extremis should be started on PGE1; the assumption being that the duct has closed and needs to be reopened.

How well did you know this?
1
Not at all
2
3
4
5
Perfectly
49
Q

19.2 An 80-year-old woman is admitted to hospital with respiratory failure. Her arterial blood gas on oxygen 4 litres per minute via a Hudson mask is as follows: (ABG shown) Which of the following most accurately describes this blood gas result?

pH 7.2, pO2 91, pCO2 84, BE 16, HCO3- 43, Na 145

a) Metabolic alkalosis, acute resp acidosis + normal AG
b) Metabolic alkalosis resp acidaemia + abnormal AG
c) Mixed acidaemia
d) Respiratory Acidosis with incomplete compensation
e) Compensated Respiratory acidosis

A

d) Respiratory Acidosis with incomplete compensation

Uncertain of this answer, not enough info to calculate anion gap

pH 7.2 = acidaemia
pCO2 84 = respiratory acidosis
HCO3 43 = metabolic alkalosis as compensation
BE 16 = metabolic alkalosis

Boston rules:
Chronic fully compensated Respiratory acidosis
Expected compensation is 3-4 mmol/L rise for every 10mmHg rise in PCO2.
Expected metabolic compensation therefore is
HCO3 = 24 + 4 x ((84-40)/10)
= 24 + 4x (44/10)
= 24 + 4 x (4.4)
= 24 + 17.6
= 41.6

Metabolic acidosis
PaCO2 should be 1.5 x HCO3 + 8
= 72.5

Rules (from K.Brandis Acid-base rules anaesthesia mcq):
- 1 for 10 (acute resp acidosis), 4 for 10 (chronic resp acidosis)
- 2 for 10 (acute resp alkalosis), 5 for 10 (chronic resp alkalosis)
- 1.5xHCO + 8 = expected pCO2 in a metabolic acidosis
- 0.7xHCO3 + 22 = expected pCO2 in a metabolic alkalosis

https://www.anaesthesiamcq.com/AcidBaseBook/ab9_3.php

How well did you know this?
1
Not at all
2
3
4
5
Perfectly
50
Q

21.2 The main advantage of using noradrenaline (norepinephrine) over phenylephrine for the prevention of hypotension as a result of spinal anaesthesia for elective caesarean section is

a) Better APGAR
b) Better foetal acid-base balance
c) Less nausea & vomiting
d) Less maternal bradycardia

A

less bradycardia

How well did you know this?
1
Not at all
2
3
4
5
Perfectly
51
Q

20.1 ECG calibration, 10mm on Y axis is equal to:

a. 0.2 sec
b. 0.4sec
c. 1sec
d. 0.1mV
e. 1mV

A

b) 1mV

How well did you know this?
1
Not at all
2
3
4
5
Perfectly
52
Q

21.2 Stellate ganglion block is NOT contraindicated in patients with

a) Contralateral phrenic nerve palsy
b) Glaucoma
c) Recent MI
d) Arrhythmia

A

d) Arrhythmia
- caution if conduction disease however

Contraindications are current coagulopathy (or anticoagulated), recent myocardial infarction, pathologic bradycardia, and glaucoma.

Source Radiopaedia

Contralateral stellate ganglion/phrenic nerve block/neuropathy

How well did you know this?
1
Not at all
2
3
4
5
Perfectly
53
Q

22.1 A 68-year-old woman presents with a loud systolic murmur in the anaesthesia room before total
hip joint arthroplasty. A transthoracic echocardiogram is performed (image provided) and shows

a. AS
b. LVOT
c. MR

A

MR

How well did you know this?
1
Not at all
2
3
4
5
Perfectly
54
Q

22.1 The most reliable clinical indicator of opioid-induced ventilatory impairment (OIVI) is decreased

a. Sedation
b. Respiratory rate

A

SS /GCS

Repeat APMSE

How well did you know this?
1
Not at all
2
3
4
5
Perfectly
55
Q

21.1, 20.1 The coagulopathy that can result from intrahepatic cholestasis of pregnancy is due to

a. 2/7/9/10
b. All clotting factors made by the liver
c. Thrombocytopenia
d. Platelet dysfunction
e. Fibrinolysis

A

a. 2/7/9/10

Hypovitaminosis of Vitamin K

(Bile required for absorption)

Source: BMC Article
https://bmcpregnancychildbirth.biomedcentral.com/articles/10.1186/s12884-022-04875-w

How well did you know this?
1
Not at all
2
3
4
5
Perfectly
56
Q

21.2 A patient presents for endovascular clot retrieval after experiencing a right hemisensory loss and right homonymous hemianopia. The vessel most likely occluded is the left

a) ACA
b) MCA
c) PCA
d) AICA
e) PICA

A

Left PCA

How well did you know this?
1
Not at all
2
3
4
5
Perfectly
57
Q

22.1 A straight laryngoscope blade is likely to be more useful than the Macintosh blade when
performing direct laryngoscopy in patients with all of the following EXCEPT

A. Prominent maxillary Incisors
B. Undergoing manual inline stabilisation
C. Large Tongue
D. Floppy Epiglottis
E. Retrognathia

A

Not actually remembered answers

B undergoing manual inline stabilisation (posterior column problem, all others are anterior column issues)

https://www.anzca.edu.au/getattachment/9ef4cd97-2f02-47fe-a63a-9f74fa7c68ac/PS56BP-Guideline-on-equipment-to-manage-difficult-airways-Background-Paper

Straight laryngoscope blades:
These may be considered for patients with anterior column problems including:
- prominent maxillary incisors
- retrognathia
- large tongue and
- large floppy epiglottis

The Miller straight blade with its low profile produces a higher pressure on the submandibular tissues with the same force (pressure = force/area), and can be used to lift the epiglottis directly 75 to facilitate intubation.
There is evidence to support better success rates with straight blades as a rescue device when the Macintosh blade has failed. However, comparative studies of straight blades and videolaryngoscopy are lacking. As the paraglossal technique
for straight laryngoscope blades is different from Macintosh blades, training and ongoing volume of practice is recommended for optimal use. It should be
recognised that while straight blades provide better laryngoscopic views, the incidence of difficult intubation due to the narrower field of vision is increased .

McCoy (CLM) laryngoscope blades

When “McCoy” laryngoscope blades are in their flexed position, they apply pressure at the base of the tongue lifting the epiglottis anteriorly and are therefore, suitable for posterior column problems (e.g. manual inline stabilisation of head and neck) where the mandible and submandibular tissues are normal.

However, the effect of this levering action of McCoy blades has not been shown to consistently improve laryngeal view. When compared to Glidescope TM videolaryngoscopes, McCoy laryngoscope blades resulted in longer tracheal intubation times in bariatric patients.

Recommendations concerning Straight and McCoy laryngoscope blades:
Page 15 PG56(A)BP Difficult airway equipment BP 2021
Advanced equipment, such as videolaryngoscopy and the common availability of flexible bronchoscopes and intubation guides/bougies, may prove to be better alternatives to difficult airway management. While evidence is currently lacking, it is recommended that Straight and McCoy laryngoscope blades are not required in DATs unless operators have been trained in their use and have ongoing volume of practice (Weak recommendation for, level of evidence
moderate quality)

How well did you know this?
1
Not at all
2
3
4
5
Perfectly
58
Q

21.1, 23.1 In patients without other co-morbidities, bariatric weight loss surgery is indicated when the body mass index (kg/m2) is greater than

A

a. 35

Major updates (2022) to 1991 National Institutes of Health guidelines for bariatric surgery

Metabolic and bariatric surgery (MBS) is recommended for individuals with a body mass index (BMI) 35 kg/m2 , regardless of presence, absence, or severity of co-morbidities.

MBS should be considered for individuals with metabolic disease and BMI of 30-34.9 kg/m2

BMI thresholds should be adjusted in the Asian population such that a BMI 25 kg/m2 suggests clinical obesity, and individuals with BMI 27.5 kg/m2 should be offered MBS.

Long-term results of MBS consistently demonstrate safety and efficacy.

Appropriately selected children and adolescents should be considered for MBS.

https://www.soard.org/article/S1550-7289(22)00641-4/fulltext#:~:text=The%201991%20NIH%20Consensus%20Statement,surgery%20that%20is%20applied%20universally

Contraindications:
- Inflammatory disease of GI tract (ulcers, oesophagitis, Crohn’s)
- Upper GI bleeding
- Portal Htn
- Liver Cirrhosis
- Chronic Pancreatitis
- Laparascopic surgery may be technically difficult in patients weighing >180kg and may be considered a relative contraindication

How well did you know this?
1
Not at all
2
3
4
5
Perfectly
59
Q

21.2 Of the following, the lifestyle modification that is least effective in reducing essential
hypertension is

a) Stopping caffeine
b) Low salt diet
c) High potassium diet
d) Exercise
e) Alcohol cessation

A

a) Stopping caffeine

Eat a well-balanced diet that’s low in salt
Limit alcohol
Enjoy regular physical activity
Manage stress
Maintain a healthy weight
Quit smoking

Foods that are rich in potassium are important in managing high blood pressure (HBP or hypertension) because potassium lessens the effects of sodium. The more potassium you eat, the more sodium you lose through urine. Potassium also helps to ease tension in your blood vessel walls, which helps further lower blood pressure.

Source AHA

How well did you know this?
1
Not at all
2
3
4
5
Perfectly
60
Q

23.1 Diagnostic criteria for adult systemic inflammatory response syndrome include all of
the following EXCEPT

a. Leukopenia
b. Hypothermia
c. Tachycardia
d. Tachypnoea
e. Hypotension

A

e. Hypotension

https://www.safetyandquality.gov.au/sites/default/files/2022-06/sepsis_clinical_care_standard_2022.pdf

How well did you know this?
1
Not at all
2
3
4
5
Perfectly
61
Q

21.2, 22.2, 23.2 A 25-year-old woman is administered two doses of aprepitant for postoperative nausea and vomiting after a sleeve gastrectomy. She normally takes the oral contraceptive pill. You should advise her to use alternative contraception for the next

a) 3 days
b) 7 days
c) 14 days
d) 28 days

A

28 days

Aprepitant PI:
“Alternative or “back-up” measures of contraception should be used during treatment with this medicine and for one month following the last dose of this medicine.”

Pharmacokinetics:
- aprepitant is a CYP3A4 inhibitor
- caution is also advised with warfarin and phenytoin use

How well did you know this?
1
Not at all
2
3
4
5
Perfectly
62
Q

21.1 The modified Aldrete scoring system uses all of the following EXCEPT

a) BP
b) Pain score
c) Resp rate
d) sedation level

A

pain score

How well did you know this?
1
Not at all
2
3
4
5
Perfectly
63
Q

21.1 The management of a patient who has experienced a cardiac arrest within 10 days of cardiac surgery
should NOT routinely include

a. Atropine 3mg
b. adrenaline 1mg boluses
c. 3 stacked shocks
d. amiodarone 300
e. 1L fluid

A

b. adrenaline 1mg boluses

The risk of administering adrenaline in conventional doses is with profound hypertension, bleeding, or tearing of vessel anastomoses on return of spontaneous circulation (ROSC), which can precipitate catastrophic harm or further cardiac arrest.
Adrenaline remains a useful drug in peri-arrest situations in smaller doses.

How well did you know this?
1
Not at all
2
3
4
5
Perfectly
64
Q

bonus neuro radiology questions

A 23 year-old man developed the abrupt onset of an explosive headache followed by nausea and vomiting. On exam, he had a right third nerve palsy.
He undergoes cerebral angiography.
His cerebral aneurysm is in the

a. Anterior Cerebral Artery
b. Anterior communicating artery
c. MCA
d. PCA
e. Basilar artery

A

d. PCA

https://case.edu/med/neurology/NR/SubarachnoidHemorrhag3%20Pcom3.htm

How well did you know this?
1
Not at all
2
3
4
5
Perfectly
65
Q

22.2 The sensor on a NIM (Nerve Integrity Monitor) endotracheal tube used for thyroid surgery directly records

a. Electromyography of internal laryngeal muscles
b. Recurrent laryngeal nerve action potential
c. Movement of the vocal cords on the endotracheal tube
d. Pressure of the vocal cords on the endotracheal tube
e. Recurrent laryngeal nerve action potential

A

a. Electromyography of internal laryngeal muscles

How well did you know this?
1
Not at all
2
3
4
5
Perfectly
66
Q

23.1 A patient is undergoing a posterior spinal fusion with somatosensory evokedpotential (SSEP) monitoring. Ischaemia is suggested by

a. Increased amplitude, increased latency
b. Increased amplitude, decreased latency
c. Decreased amplitude, increased latency
d. Decreased amplitude, decreased latency

A

c. Decreased amplitude, increased latency

How well did you know this?
1
Not at all
2
3
4
5
Perfectly
67
Q

22.1 The most common cause of maternal mortality in women with preeclampsia is

a. Renal failure
b. Hepatic failure
c. Intracranial haemorrhage

A

Intracranial haemorrhage

AHA
https://www.ahajournals.org/doi/epub/10.1161/HYPERTENSIONAHA.118.11513

How well did you know this?
1
Not at all
2
3
4
5
Perfectly
68
Q

22.1 Red man syndrome as a consequence of vancomycin administration is caused by

a. Type II hypersensitivity reaction
b. IgE sensitivity
c. Vasodilation of vessels
d. Mast cell degranulation

A

Mast cell degranulation - anaphylactoid reaction

How well did you know this?
1
Not at all
2
3
4
5
Perfectly
69
Q

20.1 You want to position a internal jugular CVL with a CXR at the caval-atrial junction. Where is this?

a) 2 vertebral bodies superior to carina
b) 1 vertebral body superior to carina
c) At the carina
d) 1 vertebral body inferior to carina
e) 2 vertebral bodies inferior to carina

A

e) 2 vertebral bodies inferior to carina

How well did you know this?
1
Not at all
2
3
4
5
Perfectly
70
Q

21.1 A neonate born by emergency caesarean section is limp, pale, has a weak grimace and weak cry, and a heart rate of 60 beats per minute. The Apgar Score is

A. 3
B. 4
C. 5
D. 6
E. 7

A

3

How well did you know this?
1
Not at all
2
3
4
5
Perfectly
71
Q

20.2 During spinal surgery, the anaesthetic agent that is least likely to decrease motor evoked potentials is

A. Non-depolarising muscle relaxants
B. Nitrous oxide
C. Opioids
D. Propofol
E. Volatiles

A

C. Opioids

A. Non-depolarising muscle relaxants - false - NMBDs abolish MEPs
B. Nitrous oxide - false - N2O can completely abolish MEPs
D. Propofol - false - PPF has less of an effect than volatiles, but still affects MEPs
E. Volatiles - false - volatiles are the most likely

NMBDs > volatiles > N2O > PPF > opioids

https://www.uptodate.com/contents/anesthesia-for-elective-spine-surgery-in-adults
While neurologic injury can cause changes in recorded potentials, other factors can interfere with interpretation. Confounding factors that can occur during surgery include inhalational anesthetics, hypothermia, hypotension, hypoxia, anemia, and preexisting neurologic lesions. Inhaled anesthetics such as isoflurane, sevoflurane, and nitrous oxide can reduce the amplitude and prolong the latency of SSEP and can completely abolish MEP. Neuromuscular blocking agents (NMBAs) also abolish motor evoked potentials and cannot be used when monitoring. Intravenous anesthetics such as propofol, barbiturates, and opioids have less of an effect on monitoring, though very deep anesthesia, even with propofol, can affect waveforms.

https://www.uptodate.com/contents/neuromonitoring-in-surgery-and-anesthesia
Evoked potentials — Evoked potential monitoring is used to assess the integrity of the tested neural pathway. Somatosensory, visual, and brainstem auditory evoked potentials monitor neurologic structures between peripheral sites where specific stimulations are applied, and responses are recorded from central locations. Motor evoked potentials monitor such structures by stimulating the motor cortex and recording from the epidural space (D-wave) or, more commonly, from distal muscles. Changes in evoked responses can result from technical, positional, pharmacologic, physiologic, or surgical causes.

For spine surgery, both MEPs and SSEPs are used to monitor spinal cord function to increase sensitivity. Motor and sensory tracts are anatomically distinct and have different vascular supply in areas of the cortex, brainstem, and spinal cord.

Motor evoked potentials (MEPs) – MEP responses are affected by even very low concentrations of volatile anesthetic agents. In general, total intravenous anesthesia (TIVA) facilitates MEP monitoring. However, inhalation agents at 0.5 MAC or less can be used in many patients, especially during intracranial surgery

Opioids – IV opioids cause small, dose-dependent depression of SSEP and MEP responses, though even at very high doses of opioids, evoked potentials can be recorded [76-78]. Infusions of remifentanil, fentanyl, or sufentanil are commonly used as part of TIVA during neuromonitoring. Opioids tend to produce high-amplitude slow waves in the EEG.

Balanced anesthetic approach — When SSEPs and MEPs are monitored, a balanced anesthetic using both a low-dose inhalation anesthetic (up to 0.5-MAC isoflurane, sevoflurane, or desflurane) and low- to medium-dose propofol (eg, propofol, 40 to 75 mcg/kg/min IV) with a relatively high-dose opioid (eg, remifentanil 0.1 to 0.4 mcg/kg/min) offers several advantages:

●Movement with motor stimulation is reduced, which is particularly important during intracranial aneurysm surgery.
●The addition of a 0.3 to 0.5 MAC inhalation agent may reduce the chance of awareness under anesthesia.
●Compared with TIVA, the addition of a 0.5 MAC inhalation agent allows reduction of the dose of propofol infusion, facilitating more rapid wakeup and earlier neurologic examination.
●Compared with TIVA, the chance of accidental interruption of the anesthetic for mechanical reasons (ie, kinked or infiltrated IV catheter or tubing such that IV agents no longer infuse) is reduced.

How well did you know this?
1
Not at all
2
3
4
5
Perfectly
72
Q

21.2 A patient has numbness and weakness in her hand postoperatively. You are trying to distinguish between an ulnar nerve lesion and a C8-T1 radiculopathy. You can diagnose a C8-T1 radiculopathy if she has weakness

A. Parasthesia in little finger
B. Parasthesia in the distribution of the interscalene nerve
C. Weakness in adductor digiti minimi
D. Weakness in abductor pollicis brevis
E. Weakness in lateral interosseus

A

D. Weakness in abductor pollicis brevis

Muscles weak in C8-T1 radiculopathy but intact in ulnar neuropathy

  1. flexor pollicis brevis
  2. abductor pollicis brevis
  3. opponens pollicis
  4. lateral lumbricals

AbOF the Law may be useful—the abductor (Ab) and flexor (F) pollicis brevis, opponens pollicis (O), and lateral lumbricals (Law) are “above the law” that intrinsic hand muscles are ulnar-innervated

How well did you know this?
1
Not at all
2
3
4
5
Perfectly
73
Q

22.1 A 57-year-old female smoker presents for a laparotomy with the following pulmonary function tests
(normal FEV1 FVC, low RV and FRC only, normal DLCO)
They are consistent with a diagnosis of

a. Obesity
b. PE
c. Pulmonary fibrosis
d. COPD

A

a. Obesity

Obesity and pulmonary function testing
https://www.jacionline.org/article/S0091-6749(05)00164-8/fulltext

  • Full pulmonary function tests are often necessary to better characterize the spirometric abnormalities seen in the obese patient
  • The most sensitive indicator of obesity is a low expiratory reserve volume (ERV) and functional residual capacity
  • Restriction is seen in more severe obesity, with reductions in TLC and FVC.
  • However, residual volume is often preserved because of the relative high closing volume in relation to ERV.
How well did you know this?
1
Not at all
2
3
4
5
Perfectly
74
Q

20.1 What is the arrow pointing to?

a. Ilioinguinal
b. Psoas
c. Iliacus
d. Lateral cutaneous nerve of thigh
e. Obturator

A

b. Psoas

How well did you know this?
1
Not at all
2
3
4
5
Perfectly
75
Q

20.2 You are seeing a 48 year-old woman in your pre-operative clinic for assessment for laparoscopic sleeve gastrectomy. Her co-morbidities include obesity (BMI is 65 kg/m2), hypertension, type 2 diabetes mellitus and polycystic ovary syndrome. Her neck circumference is 38 cm. Her husband states that she snores loudly, but he has never observed her having any apnoeic episodes and she reports no excessive tiredness during the day. Her score using the STOP-BANG questionnaire is

a. 3
b. 4
c. 5
d. 6
e. 7

A

a. 3 (snoring, BMI, Htn)

Snoring loudly
Tiredness during day time
Observed Apnoea
Pressure: Htn

BMI > 35
Age > 50
Neck circumference >40cm (43cms male)
Gender: Male

How well did you know this?
1
Not at all
2
3
4
5
Perfectly
76
Q

23.1 The following is a chest X-ray from a patient complaining of dyspnoea after thoracic surgery. The diagnosis is

(not the image from the exam)

A. Dextracardia
B. Cardiac hernation
C. LLL collapse
D. Tension Pneumohorax

A

B. Cardiac hernation

https://www.ahajournals.org/doi/10.1161/CIRCULATIONAHA.109.896829

How well did you know this?
1
Not at all
2
3
4
5
Perfectly
77
Q

21.1 During trauma resuscitation in adults, contraindications to blind nasogastric tube insertion include all of
the following EXCEPT

a) High C-spine injury
b) Recent nasal surgery
c) Oesophageal fracture
d) Base of skull fracture
e) oesophageal varices

A

High C-Spine injury

How well did you know this?
1
Not at all
2
3
4
5
Perfectly
78
Q

22.2 All of the following conditions are associated with acromegaly EXCEPT

a) cardiac arrhythmias
b) cardiac failure
c) OSA
d) aortic dilation

A

d) aortic dilation

Osteoarthritis
nerve compression syndrome due to bony overgrowth, and carpal tunnel syndrome
Hypertension
Diabetes mellitus
Cardiomyopathy/HF
Colorectal cancer
Sleep Apnea
Thyroid nodules and thyroid cancer
Hypogonadism
Compression of the optic chiasm

Source: BJA

How well did you know this?
1
Not at all
2
3
4
5
Perfectly
79
Q

21.1 A woman is having a potentially curative primary breast cancer resection. Compared with a sevoflurane and opioid technique, using a regional anaesthesia-analgesia technique with paravertebral block and a propofol infusion will result in

a. Decreased cancer recurrence
b. Decreased chronic pain and recurrence
c. Decreased incision pain at 6 months
d. Decreased CPSP pain at 6 months
e. Decreased CPSP pain at 12 months

A

Fuck this question
e. Decreased CPSP pain at 12 months

or it could be updated with an option that says makes no difference
most likely they will just remove the question and this is a big waste of time

https://pubs.asahq.org/anesthesiology/article/135/6/1091/117748/Preoperative-Paravertebral-Block-and-Chronic-Pain
—>This says it makes no difference in 2021

https://www.cochranelibrary.com/cdsr/doi/10.1002/14651858.CD007105.pub4/full
—-> this says weak evidence but it helps prevent persistent post surgical pain at 3-12months in 2018
—-> ANZCA pain book references this article

ANZCA pain book

https://www.bjaed.org/article/S2058-5349(18)30101-X/fulltext

A recent review showed that, whilst there was little effect on intra- and postoperative opioid consumption and PONV, patients receiving either both single-shot injections or placement of paravertebral catheters had less acute pain in the first 72 h after surgery.

There is also a suggestion that the use of TPVB for acute postsurgical pain may protect against the development of chronic postsurgical pain after breast surgery at 6 months.

For breast cancer surgery any form of regional anaesthesia (18 RCTs, n=1,297) reduces CPSP 3 to 12 months after surgery compared with systemic analgesia (OR 0.43; 95%CI 0.28 to 0.68) (NNT 7); specifically paravertebral block (PVB) (6 RCTs, n=419) is effective (OR 0.61; 95%CI 0.39 to 0.97) (NNT 11).

In our study population, regional anaesthesia-analgesia (paravertebral block and propofol) did not reduce breast cancer recurrence after potentially curative surgery compared with volatile anaesthesia (sevoflurane) and opioids. The frequency and severity of persistent incisional breast pain was unaffected by anaesthetic technique. Clinicians can use regional or general anaesthesia with respect to breast cancer recurrence and persistent incisional pain.

https://www.thelancet.com/pdfs/journals/lancet/PIIS0140-6736(19)32313-X.

How well did you know this?
1
Not at all
2
3
4
5
Perfectly
80
Q

22.1 The recommended filter grade of a needle to be effective in excluding microorganisms is

A

0.20 um

How well did you know this?
1
Not at all
2
3
4
5
Perfectly
81
Q

20.1, 22.2 Your patient underwent a stellate ganglion block 2 hours ago. Prior to discharge you are asked to review the patient in recovery because of a droopy upper eyelid. The patient would also be expected to have ipsilateral

a) Pupillary constriction and reaction to light
b) Pupillary constriction and no response to light
c) Pupillary dilation and response to light
d) Pupillary dilation and no response to light

A

a) Pupillary constriction and reaction to light

Stellate ganglion block causes ipsilateral Horner’s Syndrome:
Ptosis (eyelid droop)
Miosis (constricted pupils)
Anhydrosis (loss of sweating)
Enophthalmos (sinking of eyeball into the bony cavity that protects the eye)
*Pupillary constriction in response to light is controlled by the Edinger-Westphal nucleus of CN3, which will remain intact.

How well did you know this?
1
Not at all
2
3
4
5
Perfectly
82
Q

20.2 You are anaesthetising a 35 year old woman undergoing a laparoscopic appendicectomy. She uses a levonorgestrel-secreting intrauterine device (MirenaTM) for contraception and you have used sugammadex for reversal of neuromuscular blockade at the end of the procedure. Your post-operative
advice to her regarding contraception should state that

a. Barrier protection for a week
b. Barrier protection until the next period.
c. The mirena is sufficient
d. OCP for a week
e. OCP until next period

A

a. Barrier protection for a week

In the case of non-oral hormonal contraceptives, the patient must use an additional non hormonal contraceptive method for the next 7 days

How well did you know this?
1
Not at all
2
3
4
5
Perfectly
83
Q

23.1 A healthy woman is admitted to the obstetric unit with threatened preterm labour at 29 weeks gestation. Her blood pressure is 140/80 mmHg. A magnesium sulfate infusion is indicated for the purpose of

A. Maternal seizure prevention
B. Fetal lung development
C. Foetal neuroprotection

A

C. Foetal neuroprotection

https://www.kemh.health.wa.gov.au/~/media/HSPs/NMHS/Hospitals/WNHS/Documents/Clinical-guidelines/Obs-Gyn-Guidelines/Preterm-Labour-Magnesium-Sulphate-for-Neuroprotection-of-the-Fetus.pdf?thn=0#:~:text=MgSO4%20is%20only%20given%20to,4%20hours%20prior%20to%20delivery.

How well did you know this?
1
Not at all
2
3
4
5
Perfectly
84
Q

22.2 For a 70-year-old patient on rivaroxaban with normal renal function a major guideline recommends proceeding with hip fracture surgery after two half-lives of the drug. This equates to

a. 12 hours
b. 24 hours
c. 48 hours
d. 72 hours
e.

A

b. 24 hours

ASA guidelines

-If creatinine clearance >/=30 ml.min-1 (Cockcroft-Gault), proceed with surgery after two half lives (24 h) since the last dose, under general anaesthesia (or spinal anaesthesia if indicated)
- If creatinine clearance < 30 ml.min-1, proceed with surgery after four half lives (48 h) since the last dose, under general anaesthesia (or spinal anaesthesia if indicated)

How well did you know this?
1
Not at all
2
3
4
5
Perfectly
85
Q

21.1 A normal 75 kg term parturient may be expected to have a total blood volume of

a. 5250
b. 6000
c. 6750
d. 7500

A

d. 7500

100ml/kg blood volume in term parturient
7.5L (Average increase around 48%)

BJAed

How well did you know this?
1
Not at all
2
3
4
5
Perfectly
86
Q

22.1 A normal sized six-year-old girl has a haemoglobin of 70 g/L following surgery. The volume of packed red blood cells that you would plan to infuse to raise her haemoglobin to 80 g/L is

a. 80ml
b. 100ml
c. 120ml
d. 180ml
e. 200ml

A

b. 100ml

Paediatric weight estimation:
Luscombe: Weight (kg) = (age x 3) + 7
RCH: Weight (kg) = (age + 4) x 2

Formula for calculating transfusion volume (mL)
Children <20 kg:
PRBC (mL) = wt (kg) x Hb (g/L) rise (desired Hb – actual Hb) x 0.5 (transfusion factor)

Children >20 kg: 1 unit PRBC

Example:
6 + 4 x 2 = 20kg

20kg x 10g/l x 0.5 = 100ml

How well did you know this?
1
Not at all
2
3
4
5
Perfectly
87
Q

21.1 A patient has numbness and weakness in her hand postoperatively. You are trying to distinguish between an ulnar nerve lesion and a C8-T1 radiculopathy.

You can diagnose a C8-T1 radiculopathy if she has weakness

a) Thumb adduction
b) Thumb abduction
c) Fingers adduction
d) Fingers Abduction
e) Little finger flexion

A

b) Thumb abduction
(flexor pollicis brevis)

D. Paraesthesia/sensory loss over medial forearm
(medial antebrachial cutaneous)

Severing Ulnar nerve alone results in numbness of the 4th (ring) and 5th (little) fingers alone

C8 and T1 supply the medial antebrachial cutaneous nerve

Muscles weak in C8-T1 radiculopathy but intact in ulnar neuropathy

  1. flexor pollicis brevis
  2. abductor pollicis brevis
  3. opponens pollicis
  4. lateral lumbricals

AbOF the Law may be useful—the abductor (Ab) and flexor (F) pollicis brevis, opponens pollicis (O), and lateral lumbricals (Law) are “AbOF the Law” that intrinsic hand muscles are ulnar-innervated

How well did you know this?
1
Not at all
2
3
4
5
Perfectly
88
Q

21.2 A respiratory effect of high flow nasal oxygen therapy is

a) Increased deadspace
b) Reduced minute volume
c) Increased work of breathing
d) Reduced RR

A

d) Reduced RR

BJA: HFNP oxygen therapy
https://www.bjanaesthesia.org/article/S0007-0912(17)53999-9/fulltext
- reduced RR
- increased minute volume
- reduced WOB, reduced Vd, reduced AWR
- provides CPAP 3-7 cmH20 (mouth closed)

How well did you know this?
1
Not at all
2
3
4
5
Perfectly
89
Q

23.1 According to Australian and New Zealand Anaesthetic Allergy Group (ANZAAG) anaphylaxis guidelines for adults, cardiopulmonary resuscitation should commence at a systolic blood pressure of less than

a. 70
b. 60
c. 50
d. 40

A

c) 50mmHg

How well did you know this?
1
Not at all
2
3
4
5
Perfectly
90
Q

The initial management for a seizure during an awake craniotomy is

a. Cold saline irrigation
b. Midazolam
c. Propofol

A

a) Cold Saline Irrigation

How well did you know this?
1
Not at all
2
3
4
5
Perfectly
91
Q

20.2 A 56 year old patient presents with exertional syncope. The most likely diagnosis is

a) HOCM
b) Long QT
c) CCF
d) Myocardial ischaemia

A

HOCM if these remembered options are correct

Alternative is Aortic Stenosis which is more common than HOCM in this age group

How well did you know this?
1
Not at all
2
3
4
5
Perfectly
92
Q

22.1 The gauge pressure on a gas cylinder does NOT necessarily represent the contents remaining if the cylinder is filled with

A

Nitrous oxide

Nitrous oxide boiling point -88.6C, critical temperature +36C -> so is below critical temp at room temp, therefore exists as a vapour in equilibrium with its liquid phase and is dependent upon pressure applied to it. Pressure gauge not informative – will always read ~52 bar (the pressure at which N2O liquefies at 20C). As vapour is drawn off, N2O moves from liquid to vapour phase, maintaining the equilibrium and same vapour pressure within the cylinder.
To determine contents: cylinder must be weighed and weight of empty cylinder subtracted, then number of moles of N2O in cylinder calculated using Avogadro’s number.

How well did you know this?
1
Not at all
2
3
4
5
Perfectly
93
Q

21.1 Of the following classes of medication for diabetes mellitus, the most likely to cause hypoglycaemia in the fasted patient are the

A. Biguanides (metformin)
B. Sulphonylureas (gliclazide)
C. Acarbose
D. SGLT2 inhibitors (empaglaflozin)
E. DPP4 inhibitors (sitagliptin)

A

Absolute most = Insulin, but probably not an option.

Sulphonylureas most likely

How well did you know this?
1
Not at all
2
3
4
5
Perfectly
94
Q

20.1 Perioperative overheating is most likely to cause worsening of symptoms of

A) Duchenne Muscular dystrophy
B) Myasthenia gravis
C) Multiple sclerosis
D) Myotonica dystrophia
E) Eaton Lambert syndrome

A

Answer: c) MS

CEACCP 2012 Neuromuscular disorders and anaesthesia. Part 2: specific neuromuscular disorders

Multiple sclerosis
This is the most frequently occurring demyelinating neuromuscular disorder. It is a chronic relapsing condition characterized by the formation of plaques within the brain and spinal cord. These plaques cause demyelination around the axons, resulting in weakness and spasticity as well as sensory dysfunction.
Anaesthetic considerations. Local anaesthetics may exacerbate symptoms due to the increased sensitivity of demyelinated axons to local anaesthetic toxicity.
Non-depolarizing neuromuscular blocking agents may be used in normal doses. Caution should be exercised when using depolar- izing neuromuscular blocking agents if the patient is debilitated. Temperature maintenance is important as symptoms can deteriorate with an increase in temperature, as demyelinated axons are also more sensitive to heat.

BJA: Perioperative management of myasthenia gravis (2021 - written after this MCQ):

Several factors, many associated with surgery and anaesthesia, may exacerbate myasthenia or lead to a myasthenic crisis, a life-threatening condition in which severe respiratory muscle insufficiency leads to respiratory failure.
Crises are most commonly precipitated by infection. Other precipitants include surgery, residual neuromuscular block, pain, many drugs, hypo- and hyperthermia, reduction or withdrawal of treatment, pregnancy, stress and sleep deprivation.

How well did you know this?
1
Not at all
2
3
4
5
Perfectly
95
Q

21.2 Regarding healthcare research, the PICO framework describes

a) Critical appraisal
b) Meta-analysis
c) Observational study
d) Systematic review

A

a) Critical appraisal

PICO is a mnemonic used to describe the four elements of a good clinical foreground question:

P = Population/Patient/Problem - How would I describe the problem or a group of patients similar to mine?

I = Intervention - What main intervention, prognostic factor or exposure am I considering?

C = Comparison - Is there an alternative to compare with the intervention?

O = Outcome - What do I hope to accomplish, measure, improve or affect?

How well did you know this?
1
Not at all
2
3
4
5
Perfectly
96
Q

23.1 The neurosurgical registrar has telephoned about a patient with a spinal cord tumour who is on the list for tomorrow. The registrar tells you the patient has Brown-Séquard syndrome (hemisection of the spinal cord). On clinical examination, below the level of the lesion, you would expect to find all EXCEPT ipsilateral

A. Hyperreflexia
B. Loss of tactile stimulation
C. Paralysis
D. Loss of pain/temperature
E. Loss of vibration/proprioception

A

d) loss of pain and temperature

Brown sequard. Loss of pain and temp on contralateral side.
Paralysis and loss of proprioreception on ipsilateral side

How well did you know this?
1
Not at all
2
3
4
5
Perfectly
97
Q

22.2 A patient presents with sepsis-induced hypoperfusion or septic shock. The minimum suggested volume of intravenous crystalloid to be administered over the first three hours as outlined in the Surviving Sepsis Guideline is

a) 10ml/kg
b) 20ml/kg
c) 30ml/kg
d) 40ml/kg
e) 50ml/kg

A

30ml/kg

https://journals.lww.com/ccmjournal/Fulltext/2021/11000/Executive_Summary__Surviving_Sepsis_Campaign_.14.aspx

How well did you know this?
1
Not at all
2
3
4
5
Perfectly
98
Q

20.2 A patient presents with a serum sodium of 110mmol/L. A feature NOT consistent with a diagnosis of syndrome of inappropriate antidiuretic hormone (SIADH) is

a. urinary sodium >40
b. Euvolemia
c. Increased cortisol
d. Urine osmolarity <100
e. Serum Na <145

A

d. Urine osmolarity <100

DIAGNOSTIC CRITERIA
>hypotonic hyponatraemia
>urine osmolality > plasma osmolality (<275mOsm/kg) (i.e. concentrated urine despite hypotonic blood)
>urinary Na+ > 20mmol/L
>normal renal, hepatic, cardiac, pituitary, adrenal and thyroid function
>euvolaemia (absence of hypotension, hypovolaemia, and oedema)
correction by water restriction

CAUSES (MAD CHOP)

Major Surgery
>abdominal
>thoracic
>transsphenoidal pituitary surgery (6-7 days post op)

ADH production by tumours (Ectopic)
>small cell bronchogenic carcinoma
>adenocarcinoma of pancreas/duodenum
>leukaemia
>lymphoma
>thymoma

Drugs
>antidepressants (e.g. SSRI, TCAs, MAOIs)
>psychotropics (e.g. haloperidol, chlorpromazine), carbamazepine, Na+ valproate)
>anaesthetic drugs (barbiturates, inhalational agents, oxytocin, opioids)
>ADH analogues (vasopressin, DDAVP)
>chemotherapy (e.g.Vinca alkaloids, Melphalan, Methotrexate and cyclophosphamide)
>others (e.g. NSAIDs, amiodarone, ciprofloxacin, morphine, MDMA, proton pump inhibitors)

CNS Disorders
>cerebral trauma
>brain tumour (primary or metastases)
>meningitis/encephalitis
>brain abscess
>SAH
>acute intermittent porphyria
>SLE

Hormone deficiency
>hypothyroidism
>adrenal insufficiency

Others
>Guillain-Barre Syndrome
>HIV infection (early symptomatic or AIDS)
>hereditary SIADH
>giant cell arteritis
>idiopathic (occult small cell or olfactory neuroblastoma)

Pulmonary Disorders
>pneumonia (viral, fungal, bacterial)
>TB
>lung abscess

MANAGEMENT
1. see hyponatraemia
2. fluid restrict
3. incremental increase in Na+ if indicated to avoid central pontine myelinolysis
4. medications to decrease ADH secretion
>Demeclocycline
>Tolvaptan / Conivaptan

How well did you know this?
1
Not at all
2
3
4
5
Perfectly
99
Q

20.2 The composition of Plasma-Lyte 148 (in mmol/l) includes

a Na 140 Mg 1.0 K 5.0 acetate 27 lactate 0
b Na 140 Mg 1.5 K 5.0 acetate 0 lactate 27
c Na 140 Mg 1.0 K 4.0 acetate 24 lactate 0
d Na 140 Mg 1.0 K 4.0 acetate 0 lactate 24
e Na 140 Mg 1.5 K 5.0 acetate 27 lactate 0

A

e Na 140 Mg 1.5 K 5.0 acetate 27 lactate 0

How well did you know this?
1
Not at all
2
3
4
5
Perfectly
100
Q

22.2 Which is least likely to cause inaccuracies in pulse oximetry
a) Anaemia
b) Vasoconstriction
c) AF
d) Methaemoglobin
e) Carboxyhaemoglobin

A

a) Anaemia

No effect
- Fetal haemoglobin (HbF)
- SulphHb
- Bilirubin (absorption peaks are 460, 560 and 600 nm)
- dark skin

Falsely low reading
1. Methaemoglobin (MetHb). The presence of MetHb will prevent the oximeter from working accurately and the readings will tend towards 85%, regardless of the true saturation.
2. Methylene blue. When methylene blue is used in surgery (e.g. parathyroidectomy or to treat methaemoglobinaemia), a short-lived reduction in saturation estimations is seen. Readings may fall by 65% at a concentration of 2-5 mg/kg for between 10 and 60 minutes.
3. Indocyanine green. Use of this dye (e.g. in cardiac output studies) may cause a transient reduction in recorded saturations.
4. A reduction in peripheral pulsatile blood flow produced by peripheral vasoconstriction results in an inadequate signal for analysis.
5. Venous congestion, which may be caused by tricuspid regurgitation, high airway pressures and the Valsalva manoeuvre, may produce venous pulsations which can produce low readings.
6. Venous congestion of the limb may affect readings, as can a badly positioned probe.
7. External fluorescent light in the operating theatre may cause the oximeter to be inaccurate, and the signal may be interrupted by surgical diathermy. Shivering may cause difficulties in picking up an adequate signal.
8. Nail varnish may cause falsely low readings.

Falsely high reading
1. Carboxyhaemoglobin (CoHb). CoHb (haemoglobin combined with carbon monoxide) is registered as 90% oxygenated haemoglobin and 10% desaturated haemoglobin - therefore the oximeter will overestimate the saturation.

Calibration
- Oximeters are calibrated during manufacture and automatically check their internal circuits when they are turned on.
- They are accurate in the range of oxygen saturations of 70% to 100% (+/-2%), but are less accurate under 70%. Below the saturation of 70%, readings are extrapolated.
- The data for calibration came from human volunteer studies, hence it was unethical to allow the saturations to fall below 70%. Due to the shape of the oxyhaemoglobin curve, the saturation starts to fall rapidly at 90%.

Limitations
- The oximeter averages its readings every 10-20 seconds. Hence, they cannot detect acute desaturation. The finger probe has a response time of approximately 60 seconds, whereas the ear probe has a response time of 10-15 seconds.
- The site of application should be checked at regular intervals, as pressure sores and burns have been reported.
- The pulse oximeter only provides information about oxygenation. It does not give any indication of the patient’s carbon dioxide elimination.

How well did you know this?
1
Not at all
2
3
4
5
Perfectly
101
Q

20.2 Cardiovascular effects of hyperthyroidism include

a) Decreased CO
b) Increased PVR
c) Increased DBP
d) Decreased SVR

A

Decreased SVR

Hyperthyroidism:
increases HR
increases cardiac contractility.
increases LVEF
increases diastolic relaxation
increases CO

SVR decreases
>T3 induces systemic vasodilation.

How well did you know this?
1
Not at all
2
3
4
5
Perfectly
102
Q

22.1 In comparison with fresh frozen plasma, cryoprecipitate contains an increased concentration of factor

a. II
b. VII
c. XI
d. XIII

A

d. XIII

But Fibrinogen (I) is the most significant factor that

How well did you know this?
1
Not at all
2
3
4
5
Perfectly
103
Q

21.2 Suxamethonium causes a sustained contraction of the extraocular muscles for up to

a) 2 minutes
b) 3 minutes
c) 5 minutes
d) 10 minutes
e) 20 minutes

A

d) 10 minutes
- best answer; one of those shit questions that depends on your source.

Morgan & Mikhail’s (chapter 36: anaesthesia for ophthalmic surgery):
“ Succinylcholine increases IOP by 5-10mmHg for 5-10 minutes”.
- due to prolonged contracture of the EOM

BARASH:
Succinylcholine increases IOP 7 to 10 mmHg reaching a peak pressure 1 to 2 minutes after IV administration and returns to the baseline in 5 to 7 minutes. This increase may be attenuated by pretreatment with anesthetics, although none completely eliminates the increase in IOP. In the presence of a lacerated globe, this increase in IOP may increase the extrusion of intraocular contents although greater increases in IOP may occur during crying and coughing.

Yao & Artusio’s:
- also quotes same information: increases IOP 7 to 10mmHg, returning to baseline in 5 - 7 minutes.

Stoelting’s:
Intraoccular pressure peaks at 2-4 minutes after administration and returns to normal by 6 minutes

How well did you know this?
1
Not at all
2
3
4
5
Perfectly
104
Q

22.2 During spinal surgery, the anaesthetic agent that is least likely to decrease motor evoked potentials is

a Ketamine
b Precedex
c Propofol
d Volatiles
e Remifentanil

A

Ketamine

How well did you know this?
1
Not at all
2
3
4
5
Perfectly
105
Q

22.1 A patient has undergone a multilevel cervical spine fusion and upon emergence from anaesthesia reports complete visual loss. Fundoscopic examination shows a pale optic disc with haemorrhages. This supports a diagnosis of

a. CRAO
b. AION
c. PION

A

Ischaemic optic neuropathy (anterior)

https://www.researchgate.net/figure/Top-Funduscopic-examination-revealed-pale-and-swollen-discs-with-small-hemorrhages-on_fig2_6759964

At the onset of PION, the optic disc often appears normal, pallor is only seen after 6–8 weeks. The absence of optic disc edema is a key feature distinguishing PION from AION.

Perioperative PION patients may have profound bilateral vision loss (70% of cases), and worse visual outcomes. Prexisting factors that have been related to Perioperative PION include: male sex, obesity, obstructive sleep apnea and amiodarone or PDE-5 inhibitor use.[4]

How well did you know this?
1
Not at all
2
3
4
5
Perfectly
106
Q

22.2 The piece of airway equipment shown is a

a. bullard laryngoscope
b. CMAC video stylet
c. lightwand
d. flexible bougie

A

CMAC video stylet

see image for alternative equipment images

How well did you know this?
1
Not at all
2
3
4
5
Perfectly
107
Q

22.2 The initial management for a seizure during an awake craniotomy is

a. GA and tube
b. Cold saline irrigation of brain
c. IV keppra
d. IV propofol
e. IV midazolam

A

b. Cold saline irrigation of brain

Seizures, either focal or generalized, are most likely to occur during cortical mapping. They are treated by irrigating the brain tissue with ice-cold saline. They usually cease with this treatment alone, but occasionally benzodiazepines, anti-epileptic drugs, or re-sedation with airway control are required.

An emergency plan for airway control has to be in place at all times and this can be challenging as the patient’s head is fixed in head pins and often away from the ventilator. The options include the insertion of an LMA which may be easier than oro-tracheal intubation.

Awake craniotomy is generally a well-tolerated procedure with a low rate of conversion to general anaesthesia and a low rate of complications. One of the most frequent complications is patient intolerance of the procedure, often because of the urinary catheter or prolonged positioning and intra-operative seizures.

How well did you know this?
1
Not at all
2
3
4
5
Perfectly
108
Q

21.1 The equipment shown in the picture is a (airway device shown)

a) Arndt bronchial blocker
b) Cohen bronchial
blocker
c) Microlaryngoscopy tube
d) Hunsaker tube
e) Parker flex ETT

A

Hunsaker Mon-jet ventilation tube for microlarnygeal surgery

Description:
-Laser-safe
-fluoroplastic
-self-centring catheter

Uses:
-subglottic ventilation during microlaryngeal surgery

Components:
- proximal end for attaching to jet insufflation system
-proximal end allows passage of stylet to aid insertion
-Side port at proximal end for monitopring airway pressure and ETCO2
-Outer diameter 4.3mm for maintaining good surgical access
-Green basket to keep the centre port at its tip away from tracheal mucosa and avoiding potential damage from jet ventilation

How well did you know this?
1
Not at all
2
3
4
5
Perfectly
109
Q

22.1 Propofol infusion syndrome is characterised by all of the following EXCEPT

a. Splenomegaly
b. ST elevation
c. Hepatomegaly
d. Rhabdomyolysis
e. Metabolic acidosis

A

a. Splenomegaly

Associated with high doses >4mg/kg/hr and prolonged use (>48hrs)
Safe doses of propofol infusion for sedation in ICU are considered to be 1-4mg/kg/hr
-> fatal Cases pf PRIS have been reported after infusion doses as low as 1.9-2.6mg/kg/hr

Risk factors:
i. Young age
ii. Critical illness
iii. High fat and low Carbohydrate intake
iv. Inborn errors of mitochondrial fatty acid oxidation
v. Catecholamine infusion/ High catecholamine and glucocorticoid levels
vi. Steroid therapy
vii. Severe head injuries

Characteristics:
i. Bradycardia
ii. Severe metabolic acidosis
iii. Cardiovascular collapse
iv. Rhabdomyolysis
v. Hyperlipidaemia
vi. Renal failure
vii. Hepatomegaly

Management:
- Routine monitoring of CK and triglycerides should be performed for the at risk population
○ Daily CK and triglyceridees after 48hrs of propofol infusion
○ Increasing CK in the absence of other pathology triggers suspiscion of PRIS
- Propofol immediately stopped and alternative (midazolam and alfentanil) are used
- PRIS is difficult to treat once it occurs
- CVS support provided as needed
- Renal replacement therapy may be required to treat lactic acidosis, clear propofol and its metabolites from the patient rapidly
- Catecholamine resistant shock has been reported
- Pacing has been used with limited success
ECMO has been reported and successfully used in the CVS support of PRIS

How well did you know this?
1
Not at all
2
3
4
5
Perfectly
110
Q

20.2 According to National Audit Project (NAP) 5, the incidence of awareness during general anaesthesia for cardiac surgery is

a) 1:400
b) 1:800
c) 1:8000
d) 1: 12000
e) 1:20000

A

c) 1:8000

Awareness rates
GA with no muscle relaxant = 1:136,000
GA with muscle relaxation = 1/8,000
CTS 1/8,600
E-LSCS = 1/670
Overall 1:19,000

How well did you know this?
1
Not at all
2
3
4
5
Perfectly
111
Q

22.2 The nerve labelled by the arrow marked H in the diagram is the

  1. Ulnar Nerve
  2. Axillary Nerve
  3. Median Nerve
  4. Medial Cutaneous nerve of the forearm
  5. Long Thoracic Nerve
  6. Dorsal Scapular Nerve
  7. Radial Nerve
  8. Suprascapular nerve
  9. Musculocutaneous Nerve
A
  1. Median Nerve
How well did you know this?
1
Not at all
2
3
4
5
Perfectly
112
Q

22.1 A patient is undergoing treatment for a malignant hyperthermia crisis. Active cooling should be ceased when the patient’s core temperature has dropped to

a. 35
b. 36
c. 37
d. 38

A

38

How well did you know this?
1
Not at all
2
3
4
5
Perfectly
113
Q

23.1 A patient has an acute attack of shingles (herpes zoster). The development of post-
herpetic neuralgia can best be reduced by the administration of

A. Ibuprofen
B. Gabapentin
C. Aciclovir
D. Amitriptyline
E. Oxycodone

A

D. Amitriptyline

Amitriptyline (used in low doses for 90 days from onset of the herpes zoster rash) reduces the incidence of postherpetic neuralgia

N.B
Antiviral agents started within 72 hours of onset of the herpes zoster rash accelerate the resolution of acute pain (U) (Level I) but do not reduce the incidence, severity and duration of postherpetic neuralgia

UTD
Both Gabapentinoids and TCAs are effective at TREATING postherpetic neuralgia. The former have lower risk of discontinuation due to adverse side effects.
For moderate or severe pain, use gabapentinoids.

How well did you know this?
1
Not at all
2
3
4
5
Perfectly
114
Q

22.2 According to the ANZICS Statement on Death and Organ Donation 2021, circulatory determination of death in the context of organ donation requires the absence of evidence of circulation for at least

a. 2min
b. 3min
c. 5 min
d. 10 min

A

c. 5 min

Circulatory determination of death in the context of organ donation

12 Circulatory determination of death in the context of organ donation requires the absence of spontaneous movement, breathing and circulation. Absence of circulation is evidenced by absent arterial pulsatility for 5 minutes, using intra-arterial pressure monitoring and confirmed by clinical examination (absent heart sounds and/or absent central pulse). In cases without an arterial line, electrical asystole should be observed for 5 minutes on the electrocardiogram and confirmed by clinical examination.

13 For the purposes of organ donation, circulatory determination of death should be documented using a specific form (see Appendix E) to demonstrate explicitly that all criteria set out in this Statement are met. The same criteria should be listed in local hospital forms

How well did you know this?
1
Not at all
2
3
4
5
Perfectly
115
Q

21.1 A respiratory effect of high flow nasal oxygen therapy is

A. Reduced RR
B. Reduced MV
C. Increased work of breathing

A

A. Reduced RR

BJA HFNOT

It has been demonstrated that patients with acute hypoxaemic respiratory failure experience improved comfort and tolerance with HFNOT compared with humidified oxygen via a facemask, and traditional non-invasive ventilation masks. Subjective feelings of dyspnoea AND RESPRIATORY RATES are REDUCED as is airway dryness.

How well did you know this?
1
Not at all
2
3
4
5
Perfectly
116
Q

23.1 In order to minimise the risk of cardiac arrhythmia, surgical diathermy has been designed to operate with

A. High frequency
B. High amplitude
C. Low frequency
D. Low amplitude
E. Using EES

A

A. High frequency

How well did you know this?
1
Not at all
2
3
4
5
Perfectly
117
Q

21.1 A patient has numbness and weakness in her hand postoperatively. You are trying to distinguish between an ulnar nerve lesion and a C8-T1 radiculopathy.

You can diagnose a C8-T1 radiculopathy if she has

A. Paraethesia of the 5th digit
B. Paraesthesia over index finger
C. Flexor carpi ulnaris function
D. Paraesthesia/sensory loss over medial forearm
E. Adductor pollicis function

A

Remembered answers don’t help differentiate.
[A. Paraethesia of the 5th digit - can be ulnar only
B. Paraesthesia over index finger - will be median only
C. Flexor carpi ulnaris function - can be ulnar only
D. Paraesthesia/sensory loss over medial forearm - can be ulnar only
E. Adductor pollicis function - can be ulnar only

C8-T1 radiculopathy
Will cause:
Loss of Thumb and finger abduction
(flexor pollicis brevis - suppled by both ulnar deep branch (C8-T1) and median nerve lateral terminal branch C6-T1)

Severing Ulnar nerve alone results in numbness of the 4th (ring) and 5th (little) fingers alone, and potentially medial forearm sensation (C8 and T1 supply the medial antebrachial cutaneous nerve), although loss of forearm sensation is more common in C6 radiculopathies.

All intrinsic muscles of the hand are innervated by the ulnar nerve, except for 4 muscles supplied by the median nerve. These muscles may be weak in C8-T1 radiculopathy but intact in ulnar neuropathy.

  1. flexor pollicis brevis
  2. abductor pollicis brevis, in part.
  3. opponens pollicis
  4. lateral lumbricals

AbOF the Law
may be useful—the abductor (Ab) and flexor (F) pollicis brevis, opponens pollicis (O), and lateral lumbricals (Law) are “above the law”
Or
LOAF

How well did you know this?
1
Not at all
2
3
4
5
Perfectly
118
Q

22.2 Based on this ECG tracing, the mode in which this pacemaker is operating is

a) VAI with intermittent failure to capture
b) AAI with intermittent failure to sense
c) DDD
d) VVI with intermittent failure to capture
e) VVI with intermittent failure to sense

A

e) VVI with intermittent failure to sense

How well did you know this?
1
Not at all
2
3
4
5
Perfectly
119
Q

20.2 Repeated unreasonable behaviour directed towards a person or group that creates a risk to health and safety is best defined as

a. Bullying
b. Harassment
c. Percipience
d. Discrimination
e. Antagonism

A

a. Bullying

Bullying is unreasonable behaviour that creates a risk to health and safety. It is behaviour that is repeated over time or occurs as part of a pattern of behaviour. “Unreasonable behaviour” is behaviour that a reasonable person, having regard to all the circumstances, would expect to victimise, humiliate, undermine or threaten the person to whom the behaviour is directed.

  1. Direct bullying – behaviour that is overt and usually involves conduct directed at a person to belittle or demean them. Examples include:
    > Aggressive and intimidating behaviour.
    > Belittling, degrading or humiliating comments.
    > Spreading misinformation or malicious rumours.
    > Interfering with a person’s property or work equipment.
    > Displaying offensive material (for example pornography).
  2. Indirect bullying – behaviour that excludes or removes benefits from a person. Examples include:
    > Assigning meaningless tasks unrelated to the job.
    > Setting tasks that are unreasonably below or beyond a person’s skill level.
    > Deliberately changing work rosters to inconvenience particular employees.
    > Deliberately withholding information that is vital for effective work performance.

Harassment is any type of unwanted behaviour that offends, humiliates or intimidates a person, and targets them on the basis of a characteristic covered by anti-discrimination law, for example gender, race, ethnicity or disability, etc. In general, harassment is any behaviour that is:
Unwelcome, not asked for and not returned.
Likely to humiliate (put someone down), seriously embarrass, offend or intimidate (threaten or scare) someone.
Based on a personal characteristic (or family or friend’s characteristic) protected by law.
Discrimination means treating a person with an identified attribute or personal characteristic as set out in legislation less favourably than a person who does not have the attribute or personal characteristic.

> Gender.
Transgender, gender history and trans-sexual status.
Pregnancy and potential pregnancy.
Childbirth or breastfeeding.
Marital status.
Sexual orientation.
Lawful sexual activity.
Disability or impairment.
Race (including colour, nationality, descent and origin).
Physical features.
Age.
Carer status and family responsibilities.
Religious belief or activity.
Political belief or activity.
Trade union membership and industrial activity.
Associated with a person who is identified by reference to any of these attributes

Sexual Harassment
Sexual harassment is against the law. Sexual harassment is unwelcome sexual behaviour, which could be expected to make a person feel offended, humiliated or intimidated. It can be physical, verbal or written.

Victimisation
Victimisation is unlawful. Victimisation occurs when a person is treated unfairly due to that person having made a complaint of sexual harassment. Victimisation is behaviour that makes a person suffer a detriment including feeling uncomfortable, isolated, insecure or
intimidated.

https://www.anzca.edu.au/resources/corporate-documents/anzca-policy-on-bullying-discrimination-and-harass.pdf

How well did you know this?
1
Not at all
2
3
4
5
Perfectly
120
Q

20.1According to the ANZCA PS 50 “Recommendation on Practice Re-entry for a Specialist Anaesthetist” it is recommended that after an absence of more than 12 month from practicing clinical anaesthesia a re-entry program should be offered. The duration of the program for every year of absence would usually be at least

A) 2 weeks per year off
B) 3 weeks per year off
C) 4 weeks per year off
D) 6 weeks per year off
E) 8 weeks per year off

A

c) 4 weeks

ANZCA PS

How well did you know this?
1
Not at all
2
3
4
5
Perfectly
121
Q

21.1 You are performing a regional block for analgesia following knee surgery. You have an ultrasound probe scanning the anterior mid-thigh. The muscle indicated by the arrow in the ultrasound image below is the

A: Sartorius
B: Vastus Medialis
C: Adductor Longus
D: Gracilis
E: Rectus femoris

A

A: Sartorius

How well did you know this?
1
Not at all
2
3
4
5
Perfectly
122
Q

20.1 Prolonged block post mivacurium

A)Sugammadex 4mg/kg
B)Neostigmine 100microg/kg
C)FFP 20ml/kg
D)Pralidoxime
E)Wait for it to wear off

A

E)Wait for it to wear off

> Neostigmine inhibits plasma cholinesterases (that could slow mivacurium metabolism), but effects are less than the inhibition of acetylcholinesterases, resulting in a “net” reversal of nondepolarizing block. Dose in stem inappropriate though.

> Administration of whole blood or FFP is not recommended unless there is another primary indication for the transfusion.

> In patients with homozygous pseudocholinesterase deficiency, will result in prolonged NMB; monitor and await.

How well did you know this?
1
Not at all
2
3
4
5
Perfectly
123
Q

20.2 You are performing a regional block for analgesia following knee surgery. You have an ultrasound probe scanning the anterior mid-thigh. The muscle indicated by the arrow in the ultrasound image below is the

A. biceps femoris
B. Sartorius
C. Gracillis
D. Adductor longus
E. Adductor magnus

A

Sartorius

repeat

How well did you know this?
1
Not at all
2
3
4
5
Perfectly
124
Q

22.2 The Pin Index System positions on a C size cylinder of medical oxygen are
a) 1,5
b) 2,5
c) 3,5
d) 1,6
e) these options are made up

A

b) 2,5

Air: 1, 5
Oxygen: 2, 5
N2O: 3,5
CO2: 2, 6
He: 2, 4

Cyclopropane 3, 6
Entonox 7

How well did you know this?
1
Not at all
2
3
4
5
Perfectly
125
Q

22.1 The most clinically useful indicator of effective ventilation during neonatal resuscitation is an improvement in

a. HR increases
b. Grimace
c. Resp rate

A

a. HR increases

How well did you know this?
1
Not at all
2
3
4
5
Perfectly
126
Q

22.2 The diabetic medication that, as part of its therapeutic effect, significantly prolongs gastric emptying is
a) dulaglutide
b) sitagliptin
c) metformin
d) gliclazide
e) acarbose

A

a) dulaglutide

The primary mechanism of action of dulaglutide, as an incretin mimetic hormone or an analogue of human glucagon-like peptide-1, is to increase insulin secretion when glucose levels are elevated, decrease glucagon secretion, and delay gastric emptying in an effort to lower postprandial glucose level.

Acarbose:
Acarbose is a complex oligosaccharide that acts as a competitive, reversible inhibitor of pancreatic alpha-amylase and membrane-bound intestinal alpha-glucoside hydrolase.

Pancreatic alpha-amylase hydrolyzes complex carbohydrates to oligosaccharides in the small intestine

By delaying the digestion of carbohydrates, acarbose slows glucose absorption, resulting in a reduction of postprandial glucose blood concentrations.
-> causes delayed gastric emptying but is not necessarily a part of its therapeutic effect

How well did you know this?
1
Not at all
2
3
4
5
Perfectly
127
Q

20.1 You are asked to review a previously well 48-year-old woman two hours after hysteroscopic myomectomy and endometrial ablation under general anaesthesia. Her observations are: Heart rate 70 /minute, blood pressure 130/80 mmHg, SpO2 98% on 2 litres per minute of oxygen via nasal prongs. She is drowsy but rousable, oriented to person but not to time and place. Her electrolytes show: (List of electrolytes given) The most appropriate treatment is

Na 118, K 3.0, Cr 56, Ur normal.

What is your management?

A. 500ml 0.9% NaCl
B. 3% NaCl 100ml
C. 10mmol KCl
D. Fluid restriction

A

a) 3% saline 100ml

100ml bolus of 3% saline (should raise serum Na by 2-3
meq/L). If no improvement in neurological symptoms, can
repeat bolus 1-2 more times at 10 minute intervals.
Frusemide only recommended if APO

How well did you know this?
1
Not at all
2
3
4
5
Perfectly
128
Q

22.1 A 74-year-old man presents for a femoral popliteal artery bypass procedure for peripheral limb ischaemia. Regarding its role in modifying his perioperative cardiovascular risk, clonidine

a. Increased stroke
b. No change in complications
c. Increased death
d. Increased non fatal MI
e. Increased risk of non fatal cardiac arrest

A

e. Increased risk of non fatal cardiac arrest

POISE II
* clonidine 200mcg per day - did not reduce the rate of composite outcome of death or nonfatal MI - but it increased the risk of clinically important hypotension and nonfatal cardiac arrest
* aspirin initiation or continuation – no significant effect on rate of composite of death or non fatal MI but increased risk of major bleeding

How well did you know this?
1
Not at all
2
3
4
5
Perfectly
129
Q

20.1 The Brain Trauma Foundation guideline for management of severe head trauma recommend the treatment of intracranial pressures greater than

a. 5mmHg
b. 10
c. 15
d. 22
e. 25

A

22

How well did you know this?
1
Not at all
2
3
4
5
Perfectly
130
Q

20.1 Compared to a normothermic patient, a patient with mild intraoperative hypothermia (35.0o C) will have

a. increased bleeding and normal aptt and inr
b. Increased bleeding and decreased inr
c. Increased bleeding and decreased aptt
d. Decreased bleeding

A

a. increased bleeding and normal aptt and inr

Bleeding because cold = we know this

Haemtology analyzer in labs warms blood to 37.2 degrees (fixes hypothermia on sample)

How well did you know this?
1
Not at all
2
3
4
5
Perfectly
131
Q

22.2 The medication most strongly associated with an acute primary hypotensive reaction following transfusion of blood products is
a. aspirin
b. celecoxib
c. hydralazine
d. metoprolol
e. labetalol
f. perindopril

A

f. perindopril

Hypotensive transfusion reactions, which account for almost 3% of all transfusion reactions, are associated with patients treated with angiotensin-converting enzyme inhibitors. The current hypothesis suggests that they are caused by bradykinin-induced vasodilation in the absence of allergic, hemolytic, or septic mechanisms. The hypotension observed frequently is unresponsive to conventional therapy with catecholamines. The suggested intraoperative management includes cessation of transfusion and washing red blood cells before blood replacement.

Hypotensive reactions to transfusion may not always be recognized. To prevent these reactions, clinicians have several options: they may discontinue the ACE inhibitor (elective transfusion), not use a leukoreduction filter (if the patient has no absolute requirement for leukoreduced blood components), use washed cellular components, or use components that have undergone leukoreduction at the collection facility or the hospital blood bank before transfusion (since bradykinin is degraded during storage).

How well did you know this?
1
Not at all
2
3
4
5
Perfectly
132
Q

22.1 The abnormality shown in this image (image of shoulder shown) is LEAST likely to be caused by
an injury to the

a. Accessory nerve N
b. Long thoracic N
c. Dorsal scapular N
d. Suprascapular N

A

Picture in Q shows medial winging of right shoulder

Answer = b

Can be caused by injury to long thoracic nerve (serratus ant’) or serratus itself

a. Accessory nerve (Trapezius paralysis, causing lateral winging)
b. Long thoracic N- (Serratus anterior paralysis, causing medial winging)
c. Dorsal scapular N (Rhomboids paralysis, causing lateral winging)
d. Suprascapular nerve (Infra and supraspinatus – doesn’t affect scapula)

In addition, here is an example of lateral winging

How well did you know this?
1
Not at all
2
3
4
5
Perfectly
133
Q

20.1 Cardiovascular effects of hyperthyroidism include

a) decreased diastolic relaxation
b) decreased SVR
c) decreased PVR
d) increased diastolic BP

A

Decreased SVR
- increased CO, increased SBP and decreased DBP with widened PP

Up to Date
Cardiovascular - Patients with hyperthyroidism have an increase in cardiac output, due both to increased peripheral oxygen needs and increased cardiac contractility. Heart rate is increased, pulse pressure is widened, and peripheral vascular resistance is decreased

How well did you know this?
1
Not at all
2
3
4
5
Perfectly
134
Q

22.2 A 45-year-old male received a heart transplant one month ago. He develops a new supraventricular tachyarrhythmia without hypotension during a gastroscopy. The most appropriate therapy is

a) Adenosine
b) Amiodarone
c) Digoxin
d) Esmolol
e) Verapamil

A

d) Esmolol

Management of Arrhythmias After Heart Transplant
https://www.ahajournals.org/doi/10.1161/CIRCEP.120.007954

In asymptomatic patients, additional cardiac monitoring such as 24-Holter or an event monitor can be useful to assess the SVT burden, and a trial of atrioventricular nodal blockers (β-blockers preferably) can be attempted with caution in view of potential risk of bradycardia. Calcium channel blockers such as diltiazem and verapamil are contraindicated in patients taking immunosuppression such as tacrolimus and cyclosporine as it can impair the metabolism CYP3A, which increases the levels of these drugs potentially causing renal toxicity.

The use of adenosine in the management of SVT has remained a subject of controversy for over a quarter century. In the past, adenosine was contraindicated in patients post-OHT due to its supersensitivity and presumed risk of prolonged atrioventricular block.

Thus, based on the aforementioned data, in patients with OHT, adenosine is feasible and safe at reduced doses (starting at 1.5 mg for patients ≥60 kg) as long as patients are closely monitored, with dose escalation as needed. Furthermore, the 2010 American Heart Association guidelines on advanced cardiovascular life support also recommended lowering the initial dose of adenosine to 3 mg for the acute management of SVT in patients with OHT.

How well did you know this?
1
Not at all
2
3
4
5
Perfectly
135
Q

21.1 The advantage of the Mapleson E circuit in paediatric anaesthesia is due to its

A. Can use low gas flows
B. Feel compliance
C. Assess tidal volume
D. Can rapidly change levels of CPAP
E. Low resistance

A

low resistance

MAPLESON E
- Derived from the Ayre T-piece used in Mapleson D circuit and functions on the same principle as Mapleson D
- The primary difference is in the length of the tubing that is increased to be greater than the patient’s tidal volume
- For spontaneous ventilation, the expiratory limb is open to the atmosphere
- It has no valves so there is no resistance to airflow nor points for possible mechanical failure
- Rebreathing is dependent on the fresh gas flow, patients minute volume and capacity of the expiratory limb
- Its main use is in paediatric patients

How well did you know this?
1
Not at all
2
3
4
5
Perfectly
136
Q

21.2 Analysis of variance (ANOVA) is a statistical test to determine

a) comparisons of means between two groups in normally distributed data
b) comparisons of means between two groups in non-normally distributed data
c) comparisons of means between three groups (unpaired) in normally distributed data
d) comparisons of means between three groups (unpaired) in non-normally distributed data

A

c) comparisons of means between three groups in normally distributed data

ANOVA (analysis of variance): comparisons of means between more than two groups or between several measurements in the same group is called analysis of variance and is frequently cited by the acronym ANOVA

How well did you know this?
1
Not at all
2
3
4
5
Perfectly
137
Q

22.1 A 74-year old man complains of chest pain. An electrocardiograph is performed and displayed here. The occluded coronary artery could be the

a) RCA or LCx
b) RCA
c) LAD

A

RCA or LCx

https://litfl.com/mi-localization-ecg-library/

How well did you know this?
1
Not at all
2
3
4
5
Perfectly
138
Q

21.2 The Vortex Approach to airway management does all of the following EXCEPT

a) At least 1 attempt by the most experienced clinician
b) Maximum 3 attempts at each lifeline (unless gamechanger)
c) CICO status escalates with unsuccessful best effort at any lifeline
d) Trigger for initiating CICO Rescue is SpO2 <90%

A

d) Trigger for initiating CICO Rescue is SpO2 <90%

  • According to the Vortex Approach the trigger for initiating CICO Rescue is the inability to confirm adequate alveolar oxygen delivery following best efforts at all three upper airway lifelines.

Trigger for Initiating CICO Rescue VORTEX APPROACH
The trigger for initiating CICO Rescue is the inability to confirm adequate alveolar oxygen delivery following best efforts at all three upper airway lifelines.

Note that this trigger is independent of the oxygen saturations since, even in the unusual situation where the oxygen saturations remain high following best efforts at all three lifelines, the inability to confirm alveolar oxygen delivery means that eventual desaturation is inevitable.

Rather than being a deterrent to its performance, recognition of the need for CICO Rescue while the oxygen saturations remain high should be viewed as advantageous – providing increased time to perform this confronting procedure in a more controlled manner, thereby increasing the chance of success.

Conversely, a critically low oxygen saturation is not in itself a trigger to initiate CICO Rescue if best efforts at all three lifelines have not yet been completed.

While legitimate opportunities to enter the Green Zone in a timely fashion via the familiar upper airway lifelines remain, these should be given priority, as they are more likely to be successful than resorting to an unfamiliar and more traumatic technique.

Oxygen saturations are therefore not a relevant consideration in deciding the trigger for CICO Rescue – this is always “the inability to confirm adequate alveolar oxygen delivery following best efforts at all three upper airway lifelines”.

They are, however, a relevant consideration in making the context dependent decision of what constitutes a best effort at each lifeline in a particular situation.

This is because the oxygen saturations impact on how much time it is reasonable to invest in optimising each of the upper airway lifelines before declaring a best effort.

When the oxygen saturations are critically low it might be reasonable to have only one attempt at each lifeline before declaring a best effort, even though this means leaving some potential optimisation interventions untried.

This is because the incremental benefit of repeated attempts to optimise a lifeline that has already failed is typically low relative to untried alternative lifelines.

Thus the time expended on such low yield interventions cannot be justified when the patient is already critically hypoxaemic and alternatives (including CICO Rescue) with a substantially higher likelihood of success remain.

How well did you know this?
1
Not at all
2
3
4
5
Perfectly
139
Q

22.1 A test for a condition which has a prevalence of 1 in 1000 has a sensitivity of 100% and a specificity of 90%.
The probability of a patient who receives a positive result actually having the condition is

a. 1%
b. 10%
c. 50%
d. 100%

A

a. 1%

i.e. what is the positive predictive value (PPV) for this test

PPV= TP/ TP +FP
Negative Predictive Value = TN / TN + FN

Prevalence of 1/1000
Sensitivity of 100%
Specificity of 90%

Of patients that are disease positive in population of 1000
TP = 1
FN = 0
-> 100% sensitivity

Of patients that are disease negative in population of 1000
FP = 99
TN = 900
-> 90% Specificity

PPV= 1/ 1 + 99
= 1/100
=1%

NPV= 900/ 900 + 0
= 1/1
= 100%

How well did you know this?
1
Not at all
2
3
4
5
Perfectly
140
Q

23.1 The glucagon-like peptide-1 receptor (GLP-1) agonist semaglutide is associated with

A. delayed gastric emptying
B. hypoglycaemia
C. hyperlactataemia

A

a) delayed gastric emptying

How well did you know this?
1
Not at all
2
3
4
5
Perfectly
141
Q

22.1 A risk factor for the development of torsade de pointes is

a. hyperkalaemia
b. hypermagnasaemia
c. tachycardia
d. Female

A

d. Female

How well did you know this?
1
Not at all
2
3
4
5
Perfectly
142
Q

23.1 A 24-year-old man has been brought into the emergency department with a traumatic fracture of the femur. His observations are: heart rate 90 beats per minute; blood
pressure 120/80 mmHg; respiratory rate 25 breaths per minute. A peripheral VENOUS blood gas sample shows a pH of 7.29. The arterial blood pH can be estimated to be

A. 7.29
B. 7.32
C. 7.35
D. 7.4

A

B. 7.32

https://emj.bmj.com/content/18/5/340

The values of pH on arterial and venous samples were highly correlated (r=0.92) with an average difference between the samples of −0.4 units. There was also a high level of agreement between the methods with the 95% limits of agreement being −0.11 to +0.04 units.

https://litfl.com/vbg-versus-abg/

pH
- Good correlation
- pooled mean difference: +0.035 pH units

How well did you know this?
1
Not at all
2
3
4
5
Perfectly
143
Q

20.1 This type of tracheal tube is best described as a (picture of airway device shown)

a) Mini tracheostomy tube
b) South facing RAE
c) Laser tube
d) Laryngectomy tube
e) Fenestrated tracheostomy tube

A

laryngectomy tube

Rusch Larygoflex Reinforced Laryngectomy tube -

How well did you know this?
1
Not at all
2
3
4
5
Perfectly
144
Q

22.2 A patient is anaesthetised from the awake state to a state of surgical anaesthesia with propofol or a volatile anaesthetic. As the depth of anaesthesia increases, the patient’s electroencephalogram (EEG) will show oscillations that are of

a. low frequency low amplitude
b. low frequency high amplitude
c. high frequency low amplitude
d. high frequency high amplitude

A

b. low frequency high amplitude

Changes in the electroencephalogram during anaesthesia and their physiological basis
https://academic.oup.com/bja/article/115/suppl_1/i27/234261

Figure 1 shows raw EEG waveforms during isoflurane anaesthesia.
During light anaesthesia:
-amplitude is shallow and frequency is high.
When a higher concentration is administered:
-amplitude deepens and EEG frequency slows.

During deep anaesthesia:
- a ‘burst and suppression’ pattern becomes apparent, characterized by extreme activity, represented by high-frequency, large-amplitude waves (bursts), alternating with flat traces (suppression).
- This pattern, excluding brain ischaemia or other factors, indicates that anaesthesia is too deep. Beyond this, flat traces become dominant and, eventually waveforms are no longer apparent.

During isoflurane, sevoflurane or propofol anaesthesia, this sequence of changes in pattern is almost identical.
The major difference in EEG between the volatile agents (isoflurane or sevoflurane) and propofol is apparent in power in the theta range.
During propofol anaesthesia, theta power remains low regardless of concentration, but during isoflurane or sevoflurane anaesthesia, it increases at surgical concentrations of anaesthesia.

How well did you know this?
1
Not at all
2
3
4
5
Perfectly
145
Q

22.1 A drug which does NOT increase the defibrillation threshold in a patient with an implanted cardioverter defibrillator is

a. Amiodarone
b. Atropine
c. B-blocker
d. Flecainide
e. Sotalol

A

e. Sotalol

Drugs that INCREASE defibrillation threshold:
+ Amiodarone (Chronic)
+ Atropine
+ lignocaine
+ Diltiazem
+ Flecainide
+ Verapamil
+ Venlafaxine
+ Anaesthetic agents.

Drugs that DECREASE defibrillation threshold:
- Sotalol
- Amiodarone (acute)
- Nifekalant

Drugs with No Change in DFT
= B- blocker
= Disopyramide
= Procainamide
= Propafenone

https://www.ncbi.nlm.nih.gov/pmc/articles/PMC6304797/

How well did you know this?
1
Not at all
2
3
4
5
Perfectly
146
Q

22.1 Venous air embolism during frontal craniotomy is most likely to arise from the

a. Transverse sinus
b. Sigmoid sinus
c. Superior sagittal
d. Straight

A

c. Superior sagittal

Risk factors for venous air embolism include sitting craniotomy, posterior fossa surgery and procedures near the superior sagittal sinus. In these situations, the surgical site is often above the level of the right atrium and hence venous air entrainment is facilitated, or there is a large risk of venous exposure through which air may be entrained. Depending on the volume of air entrained, reduced end-tidal carbon dioxide, arrhythmias or right heart failure and cardiovascular collapse are all possible. However, changes in clinical parameters often occur late and are nonspecific for small volumes of entrainment. Specific monitoring for detection of venous air embolism includes non-invasive means such as end-tidal nitrogen, precordial Doppler or stethoscope and transcranial Doppler. Invasive methods include transoesophageal echocardiography, oesophageal stethoscope, pulmonary artery catheter and central venous pressure monitoring.

https://resources.wfsahq.org/atotw/anaesthesia-for-craniotomy-and-brain-tumour-resection/

How well did you know this?
1
Not at all
2
3
4
5
Perfectly
147
Q

22.2 The nerve labelled by the arrow marked B in the diagram is the

  1. Ulnar Nerve
  2. Axillary Nerve
  3. Median Nerve
  4. Medial Cutaneous nerve of the forearm
  5. Long Thoracic Nerve
  6. Dorsal Scapular Nerve
  7. Radial Nerve
  8. Suprascapular nerve
  9. Musculocutaneous Nerve
A
  1. Suprascapular nerve
How well did you know this?
1
Not at all
2
3
4
5
Perfectly
148
Q

21.1, 21.2 In maternal cardiac arrest the most common arrhythmia is

a) PEA
b) VT
c) VF
d) Asystole
e) SVT

A

a) PEA

I couldn’t find a great article on this anywhere. BJAED hasn’t got much either

How well did you know this?
1
Not at all
2
3
4
5
Perfectly
149
Q

21.1, 22.2 Regarding healthcare research, the SQUIRE guidelines describe

a) Standards for RCTs
b) Standards for meta-analysis
c) Standards for observational studies
d) Standards for systematic reviews
e) Standards of quality improvement

A

e) Standards of quality improvement

Quality Improvement

(Standards for QUality Imporvement and Reporting Excellence)

CONSORT: randomised trials
PRISMA: systematic reviews and meta-analysis (Preferred Reporting Items for Systematic reviews and meta-analysis).
STROBE: observational studies

How well did you know this?
1
Not at all
2
3
4
5
Perfectly
150
Q

21.1 Sensory innervation of the cornea is by the

A. Ophthalmic division of the Trigeminal nerve
B. Nasocilliary Nerve
C. Frontal Nerve
D. Oculomotor

A

B. Nasocilliary Nerve
a branch of Ophthalmic division of trigeminal

How well did you know this?
1
Not at all
2
3
4
5
Perfectly
151
Q

23.1 During standard diagnostic nocturnal polysomnography for investigation of obstructive sleep apnoea, apnoea is defined as cessation of airflow for

A. 10 sec
B. 20 sec
C. 30 sec
D. 10 sec with 3% desat
E. 20 sec with 3 % desat

A

A

Apnea is defined as the cessation of airflow for ten or more seconds.

Hypopnea is defined as a recognizable, transient reduction, but not a complete cessation of, breathing for ten or more seconds.

Hypopnea requires a 4% fall in SpO2

https://www.ncbi.nlm.nih.gov/books/NBK441909/#:~:text=Obstructive%20Sleep%20Apnea%20(OSA)%2C,for%20ten%20or%20more%20seconds.

How well did you know this?
1
Not at all
2
3
4
5
Perfectly
152
Q

20.1 You are planning to perform an adductor canal block for a patient prior to a total knee arthroplasty. The principal advantage of this approach compared to a conventional femoral nerve block below the inguinal ligament is :

a) better block of infrapatellar nerve
b) better analgesia
c) lower dose of LA needed for same analgesia
d) less motor block to quads

A

d) less motor block to quads

APMSE 5th edition:
Other regional and local analgesic techniques

“Adductor canal block results in similar postoperative pain outcomes following total knee arthroplasty versus femoral nerve block with less quadriceps weakness, earlier mobilisation and better functional recovery

How well did you know this?
1
Not at all
2
3
4
5
Perfectly
153
Q

23.1 Findings associated with massive pericardial tamponade include

a. Electrical alternans
b. Exaggerated collapsible IVC on ECHO during respiratory cycle
c. Pulses alternans
d. Kussmaul breathing

A

a) electrical alternans

Physical findings in Tamponade:
- A number of findings may be present on physical examination, depending upon the type and severity of cardiac tamponade
- None of the findings alone are highly sensitive or specific for the diagnosis.

Beck’s triad
1. Low arterial blood pressure
2. Dilated neck veins
3. Muffled heart sounds
- Are present in only a minority of cases of acute cardiac tamponade.

Diagnosis:
Clinical diagnosis is usually suspected based on the history and physical examination findings, which may include:
●Chest pain
●Syncope or presyncope
●Dyspnea and tachypnea
●Hypotension
●Tachycardia
●Peripheral edema
●Elevated jugular venous pressure
●Pulsus paradoxus

How well did you know this?
1
Not at all
2
3
4
5
Perfectly
154
Q

22.1 The mechanism of action of tranexamic acid is to inhibit the formation of

a. Plasminogen
b. Plasmin
c. Fibrin
d. fibrinogen

A

b. Plasmin

Plasminogen activation results in increased conversion of plasminogen to plasmin, the latter an enzyme that breakdowns the fibrinogen in blood clots.

Tranexamic acid is a synthetic derivative of lysine that exerts antifibrinolytic effects by blocking lysine binding sites on plasminogen molecules, inhibiting the interaction of plasminogen with formed plasmin and fibrin.

As a result, inhibition of plasminogen activation results in stabilization of the preformed fibrin meshwork produced by secondary hemostasis.

How well did you know this?
1
Not at all
2
3
4
5
Perfectly
155
Q

A 35-year-old male, three days post laparoscopic sleeve gastrectomy has ongoing nausea and vomiting. His arterial blood gas measurement is as follows: (ABG shown) The best initial therapeutic option would be

Blood gas given:
hypokalaemia
hypochloraemia
alkalosis
normal lactate

a Laparoscopy
b IV fluids and KCL
c 4% albumin
d HCl infusion
e Acetazolamide

A

b IV fluids and KCL

UTD Stricture post Lap Sleeve Gastrectomy management

Although sleeve strictures have been reported in 0.26 to 4 percent of LSG operations, <1 percent result in symptoms that require endoscopic or surgical intervention

A stricture can manifest acutely, early after surgery, or more chronically.

Although strictures can occur anywhere along the long staple line, they are most often located at the level of the incisura angularis for anatomic reasons.

The etiologies of post-LSG strictures are either mechanical or functional. Mechanical strictures usually derive from the use of small bougies, stapling too close to the bougie (especially at the incisura angularis), twisting of the staple line creating a “spiral” sleeve, or aggressive imbrication of the staple line.

Functional stenoses derive from edema or hematomas at the staple line. As a result, functional stenoses are transient, which present immediately following LSG and resolve spontaneously with expectant treatment.

Patients who present with obstructive symptoms during the early postoperative period should be resuscitated with hydration and antiemetic medications and studied with an upper gastrointestinal (UGI) series.

Stable patients with a stricture can be observed to allow postsurgical mucosal edema to resolve, typically in 24 to 48 hours. Patients who cannot handle their own secretions require nasogastric tube decompression, preferably placed under fluoroscopic guidance.

Patients with an acute stricture who do not respond to conservative management require early surgical reintervention. Laparoscopy could demonstrate kinking of the gastric tube, a tight suture, or a compressing hematoma.

●Endoscopy is a good initial treatment for short-segment strictures, most of which can be dilated with balloons. Multiple treatments in four- to six-week intervals are sometimes needed to treat the stricture and improve patient symptoms. Stents have also been tried but are not effective for post-LSG strictures.

●Laparoscopic seromyotomy is a treatment option for long-segment strictures . In a small retrospective study, patients treated with laparoscopic seromyotomy had good symptomatic relief.

●Conversion to an RYGB is the last option for patients with a refractory stricture who have failed all other treatments.

How well did you know this?
1
Not at all
2
3
4
5
Perfectly
156
Q

A 65-year-old woman has presented with a grade 2 subarachnoid haemorrhage equally suitable for treatment with surgical clipping or endovascular coiling. The factor shown to most effectively reduce mortality in early subarachnoid haemorrhage treatment is

a) Nimodipine
b) Tranexamic acid
c) Early repair
d) Atorvastatin
e) EVD placement

A

c) Early repair

Coil within 24 hours

Early repair - the outcome in terms of survival free of disability at 1 year is significantly better with endovascular coiling. Preferably within 24 hours

How well did you know this?
1
Not at all
2
3
4
5
Perfectly
157
Q

23.1 In subarachnoid block for caesarean section, hyperbaric local anaesthetic compared to regular local anaesthetic has been shown to reduce the

a. Risk of total spinal
b. Analgesic properties
c. Onset of anaesthetic
d. Offset of anaesthetic
e. Chance of inadequate anaesthetic

A

reduce onset time

c) faster onset of anaesthetic

https://pubmed.ncbi.nlm.nih.gov/28708665/ agrees with faster onset but for non obstetric surgery

UTD
hyperbaric bupivacaine because of its rapid onset and the option to modify the spinal level by changing the position of the operating table. Plain bupivacaine (ie, slightly hypobaric, prepared in saline) may also be used for spinal anesthesia for CD. The literature comparing safety and efficacy of hyperbaric with isobaric bupivacaine for CD is inconclusive

How well did you know this?
1
Not at all
2
3
4
5
Perfectly
158
Q

20.1 A patient with multiple co-morbidities has severe symptomatic aortic stenosis and is considered for an aortic valve replacement. Compared to an open surgical approach, a transcatheter aortic valve implantation (TAVI) has

a) Reduced vascular injury
b) Reduced mean valve gradient
c) Reduced paravalvular leak
d) Reduced complete heart block
e) Reduced reintervention

A

b) Reduced mean valve gradient

TAVI decreased:
- AKI
- AF
- Transfusion
- Mean prosthetic valve gradient

TAVI increased:
- Major vascular complications
- Permanent pacemaker implantation
- Paravalvular regurgitation
- Need for re-intervention

How well did you know this?
1
Not at all
2
3
4
5
Perfectly
159
Q

20.1 IgE-related penicillin anaphylaxis crossover rate with cephazolin

a. 0.1%
b. 1%
c. 5%
d. 10%

A

1%

BJA ED

How well did you know this?
1
Not at all
2
3
4
5
Perfectly
160
Q

23.1 According to the Australian and New Zealand Anaesthetic Allergy Group (ANZAAG) guidelines for the investigation of a suspected anaphylactic reaction, serum tryptase should be measured at

a. 0, 4, 12
b. 0, 2, 4, 24
c. 0, 1, 4, 24
d. 0, 4 , 6, 24
e. 1, 6, 24

A

c) 0, 1, 4, 24

Serum tryptase levels are recommended to be collected as soon as possible after the onset of symptoms and then at 1 hour, 4 hours and after 24 hours.

https://www.anzca.edu.au/resources/professional-documents/endorsed-guidelines/anaphylaxis-guideline-2022.pdf

How well did you know this?
1
Not at all
2
3
4
5
Perfectly
161
Q

22.2 For a skewed distribution of data the best measure of dispersion of data is the

a) range
b) mode
c) standard deviation
d) variance
e) Interquartile Range
f) median

A

e) Interquartile Range

https://statisticsbyjim.com/basics/skewed-distribution/

https://statisticsbyjim.com/basics/variability-range-interquartile-variance-standard-deviation/

A measure of variability is a summary statistic that represents the amount of dispersion in a dataset. How spread out are the values? While a measure of central tendency describes the typical value, measures of variability define how far away the data points tend to fall from the center.

In statistics, variability, dispersion, and spread are synonyms that denote the width of the distribution. Just as there are multiple measures of central tendency, there are several measures of variability.

When a distribution has lower variability, the values in a dataset are more consistent. However, when the variability is higher, the data points are more dissimilar and extreme values become more likely. Consequently, understanding variability helps you grasp the likelihood of unusual events.

> Range is easy to understand, it is based on only the two most extreme values in the dataset, which makes it very susceptible to outliers. If one of those numbers is unusually high or low, it affects the entire range even if it is atypical.

> The interquartile range is the middle half of the data. To visualize it, think about the median value that splits the dataset in half. The interquartile range is the middle half of the data that is in between the upper and lower quartiles. In other words, the interquartile range includes the 50% of data points that fall between Q1 and Q3

> The interquartile range is a robust measure of variability in a similar manner that the median is a robust measure of central tendency. Neither measure is influenced dramatically by outliers because they don’t depend on every value. Additionally, the interquartile range is excellent for skewed distributions, just like the median.

> when you have a normal distribution, the standard deviation tells you the percentage of observations that fall specific distances from the mean. However, this doesn’t work for skewed distributions, and the IQR is a great alternative.

> Variance is the average squared difference of the values from the mean. Unlike the previous measures of variability, the variance includes all values in the calculation by comparing each value to the mean. To calculate this statistic, you calculate a set of squared differences between the data points and the mean, sum them, and then divide by the number of observations. Hence, it’s the average squared difference.

> While higher values of the variance indicate greater variability, there is no intuitive interpretation for specific values. Despite this limitation, various statistical tests use the variance in their calculations. For an example, read my post about the F-test and ANOVA. While it is difficult to interpret the variance itself, the standard deviation resolves this problem!

> The standard deviation is the standard or typical difference between each data point and the mean. When the values in a dataset are grouped closer together, you have a smaller standard deviation. On the other hand, when the values are spread out more, the standard deviation is larger because the standard distance is greater

> The standard deviation is just the square root of the variance. Recall that the variance is in squared units. Hence, the square root returns the value to the natural units. The symbol for the standard deviation as a population parameter is σ while s represents it as a sample estimate. To calculate the standard deviation, calculate the variance as shown above, and then take the square root of it. Voila! You have the standard deviation!

> People often confuse the standard deviation with the standard error of the mean. Both measures assess variability, but they have extremely different purposes.

> When you have normally distributed data, or approximately so, the standard deviation becomes particularly valuable. You can use it to determine the proportion of the values that fall within a specified number of standard deviations from the mean. For example, in a normal distribution, 68% of the values will fall within +/- 1 standard deviation from the mean. This property is part of the Empirical Rule. This rule describes the percentage of the data that fall within specific numbers of standard deviations from the mean for bell-shaped curves.

How well did you know this?
1
Not at all
2
3
4
5
Perfectly
162
Q

20.1 A 64-year-old man presenting for elective surgery is on thyroxine 100 mcg daily. His thyroid function tests are: (Thyroid function tests shown). These results are most consistent with

TFTs thryoxine TSH < .05 T4 and T3 completely normal

a) Hypophysectomy
b) Subclinical Hyperthyoirdism
c) Sick euthyroid
d) Toxic Multinodular goitre

A

b) Subclinical Hyperthyoirdism

Subclinical hyperthyroidism: low TSH, normal T3 + T4
Clinical hyperthyroidism: low TSH, high T3, high/normal T4

Subclinical hypothyroidism: high TSH, normal T3 + T4
Clinical hypothyroidism: high TSH, low/normal T3, i T4

Amiodarone: high/normal TSH, low T3 (2o to inhibition of pituitary T4 to T3 conversion)

Sick euthyroid: low TSH, low T3

Hypophysectomy (central hypothyroidism): low/normal TSH/T3/T4

Compliant on thyroxine: normal TSH, high/normal T3, low T4
Non-compliant w thyroxine (pt taking several tabs prior to Dr’s appointment): high TSH, normal T4

How well did you know this?
1
Not at all
2
3
4
5
Perfectly
163
Q

20.1 What is the arrow pointing to?

a. Ilioinguinal
b. Iliohypogastric
c. Genitofemoral
d. Accessory Obturator
e. Obturator

A

e. Obturator

How well did you know this?
1
Not at all
2
3
4
5
Perfectly
164
Q

21.1 A 25-year-old ASA I patient develops ongoing seizures five minutes after receiving a brachial plexus block with ropivacaine. Of the following, the most suitable initial intravenous treatment is

a) Midazolam
b) Intralipid
c) Propofol
d) Levetiracetam
e) Phenytoin

A

Control seizures first
a) Midazolam if an option
or
c) propofol
or

treat seizures 1st followedLAST
- ABCD
- Intralipid 1.5mL/kg

How well did you know this?
1
Not at all
2
3
4
5
Perfectly
165
Q

23.1 A 60-year-old woman presents for thrombectomy with left lower leg ischaemia. She has not received any medications since presentation and takes none at home. The sole abnormality on laboratory testing is an activated partial thromboplastin time (APTT) of 52 seconds. The most likely cause of the raised APTT is

a. Cold agglutinins
b. Erroneous reading
c. Lupus anticoagulant
d. Factor VII deficiency
e. Haemophilia A

A

c. Lupus anticoagulant
(normal PT, raised APTT)

Lupus anticoagulant (more likely to be associated with thrombosis than bleeding)

https://www.uptodate.com/contents/image?imageKey=HEME%2F79969

How well did you know this?
1
Not at all
2
3
4
5
Perfectly
166
Q

21.1 The ANZCA Choosing Wisely recommendations advise avoiding all of the following EXCEPT

a) Doing an epidural on a patient who is labouring normally with a normal pregnancy and no comorbidities
b) Giving blood transfusion on a healthy 20yo male with Hb > 70g/L, except when severe and symptomatic
c) Giving an anaesthetic to a high risk patient with severe comorbidities without risk stratifying them and taking an anaesthetic history and assessment
d) Routinely performing preoperative blood investigations, chest X-ray or spirometry prior to surgery, but instead, ordering in response to patient factors, symptoms and signs, disease, or planned surgery.
e) Ordering cardiac stress testing for asymptomatic patients prior to undergoing low to intermediate risk non-cardiac surgery.

A

doing an epidural on a patient who is labouring with normal pregnancy and no comorbidities

  1. Avoid routinely performing preoperative blood investigations, chest X-ray or spirometry prior to surgery, but instead order in response to patient factors, symptoms and signs, disease, or planned surgery.
  2. Avoid ordering cardiac stress testing for asymptomatic patients prior to undergoing low to intermediate risk non-cardiac surgery.
  3. Avoid administering packed red blood cells (blood transfusion) to a young healthy patient with a haemoglobin of ≥70g/L who does not have on-going blood loss, unless the patient is symptomatic or haemodynamically unstable.
  4. Avoid initiating anaesthesia for patients with limited life expectancy, at high risk of death or severely impaired functional recovery, without discussing expected outcomes and goals of care.
  5. Avoid initiating anaesthesia for patients with significant co-morbidities without adequate, timely preoperative assessment and postoperative facilities to meet their needs.
  6. Avoid routine prescription of slow-release opioids in the management of acute pain unless there is a demonstrated need, close monitoring is available and a cessation plan is in place
How well did you know this?
1
Not at all
2
3
4
5
Perfectly
167
Q

20.2, 22.2 The modified Aldrete scoring system is used for determining the

a) Predicts difficulty of bag mask ventilation
b) Safety of day surgery
c) Discharge from recovery
d) Modification of recovery criteria
e) Discharge from hospital

A

c) Discharge from recovery

Aldrete score, which includes five elements (activity, respiration, circulation, consciousness, oxygen saturation) [16].

The original scoring system was developed before the invention of pulse oximetry and used the patient’s colouration as a surrogate marker of their oxygenation status. A modified Aldrete scoring system was described in 1995 which replaces the assessment of skin colouration with the use of pulse oximetry to measure SpO2.

The Modified Aldrete system includes five additional elements that are particularly useful during the Phase II recovery period prior to discharge to home (dressing, pain, ambulation, feeding, urine output)

How well did you know this?
1
Not at all
2
3
4
5
Perfectly
168
Q

20.2 Idarucizumab reverses the anticoagulant effect of

a) Clopidogrel
b) Rivaroxaban
c) Dabigatran
d) Apixaban
e) Rivaroxaban

A

c) Dabigatran

Idarucizumab (Praxbind) is a monoclonal antibody to dabigatran

Dabigatran bleeding may be treated with:
- idarucizumab
- haemodialysis
- TXA will decrease fibrinolysis and has some effect
- FFP also has some effect

Humanized monoclonal antibody fragment (Fab) indicated in patients treated with dabigatran (Pradaxa) when reversal of the anticoagulant effects are needed for emergency surgery or urgent procedures, or in the event of life-threatening or uncontrolled bleeding
- 5 g IV, provided as 2 separate vials each containing 2.5 g/50 mL (see Administration)
- Limited data support administration of an additional 5 g

Dosage Modifications

Renal impairment: Renal impairment did not impact the reversal effect of idarucizumab; no dosage adjustment required
Hepatic impairment: Not studied
Dosing Considerations

This indication is approved under accelerated approval based on a reduction in unbound dabigatran and normalization of coagulation parameters in healthy volunteers; continued approval for this indication may be contingent upon the results of an ongoing cohort case series study

How well did you know this?
1
Not at all
2
3
4
5
Perfectly
169
Q

22.2 When used for prolonged analgesia in a healthy adult, the recommended maximum dose of ropivacaine via continuous infusion or bolus dosing in a 24-hour period is

a) 450mg
b) 600mg
c) 770mg
d) 1200mg

A

c) 770mg

Product info: Fresenius-Kabi

When prolonged epidural blocks are used, either by continuous infusion or repeated bolus administration, the risks of reaching a toxic plasma concentration or inducing local neural injury must be considered. Cumulative doses of up to 800 mg ropivacaine for surgery and postoperative analgesiaadministered over 24 hours were well tolerated in adults, as were postoperative continuous epidural infusions at rates up to 28 mg/hour for 72 hours.

product info: pfizer

When prolonged blocks are used, either through continuous infusion or through repeated bolus administration, the risks of reaching a toxic plasma concentration or inducing local neural injury must be considered. Experience to date indicates that a cumulative dose of up to 770 mg ropivacaine hydrochloride administered over 24 hours is well tolerated in adults when used for postoperative pain management: i.e., 2016 mg. Caution should be exercised when administering ropivacaine for prolonged periods of time, e.g., > 70 hours in debilitated patients

How well did you know this?
1
Not at all
2
3
4
5
Perfectly
170
Q

20.2 The most common cause of post operative visual loss after spinal surgery is

a) Corneal abrasion
b) Retinal artery occlusion
c) Central retinal vein occlusion
d) Ischaemic optic neuropathy
e) Occipital infarct

A

a) Ischaemic optic neuropathy

Postoperative visual loss (POVL) occurs in 1/60 000–1/125 000 operations. Spinal surgery has the highest incidence of POVL.

American Society of Anesthesiologists (ASA) Post Operative Visual Loss Registry, spinal surgery accounted for 93/131 (70%) of all cases of visual loss after non-ophthalmic surgery.
Of these:
> 83 were attributable to ischaemic optic atrophy (ION)
> 10 were caused by central retinal artery occlusion (CRAO).

CRAO
- caused by direct pressure on the globe causing raised intraocular pressure and compromising retinal perfusion.
- visual loss is usually unilateral and associated with other signs of pressure (e.g. ophthalmoplegia, ptosis, or altered sensation in the territory of the supraorbital nerve).
- Initial careful positioning of the head and regular checks throughout the procedure in case of movement minimizes the risk
- documentation of eye checks should occur every 30mins and horseshoe shaped head rests should be avoided in prone patients

ION
> associated with:
- male gender
- obesity
- increasing blood loss
- operative procedures >6 hrs in length.
- The use of the Wilson frame has also been implicated.
> final common pathway is thought to be hypoperfusion of the optic nerve, there is no clear association with either intraoperative systemic hypotension or with the presence of peripheral vascular disease or diabetes.
> recently updated ASA practice advisory for POVL associated with spinal surgery recommends regular intraoperative testing of haemoglobin concentration. However, it was unable to suggest a transfusion threshold that would prevent POVL.

Other possible causes of POVL:
1. Cortical ischaemia
2. Haemorrhage into a cerebral tumour.

In high-risk cases, assessment of vision should be performed as soon as possible in PACU and an early ophthalmic opinion sought if there is a suggestion of visual compromise.

Initial management
1. optimization of arterial pressure
2. oxygenation
3. correction of anaemia.

Treatment with agents such as acetazolamide has not been beneficial and there is rarely any useful improvement in vision with either injury, so attention should be focused on preventative measures:
1. Careful positioning with the head at the same level as the heart
2. Meticulous haemostasis,
3. Possibly staging prolonged procedures should be considered.

Because of the devastating nature of this complication, patients should be informed of an increased incidence of visual loss after spinal operations that are expected to be of prolonged duration and associated with significant blood loss.

How well did you know this?
1
Not at all
2
3
4
5
Perfectly
171
Q

22.1 When using an endotracheal tube in an adult, the highest recommended cuff pressure to avoid mucosal ischaemia is

a. 10cmH2O
b. 20
c. 30
d. 40
e. 50

A

c. 30cmH2O

How well did you know this?
1
Not at all
2
3
4
5
Perfectly
172
Q

21.2 Of the following drugs, the least likely to cause pulmonary vasodilation when used at low
doses in patients with chronic pulmonary hypertension is

a) Dopamine
b) Dobutamine
c) Vasopressin
d) Milrinone

A

dopamine

  • least likely to cause pulmonary vasodilation (all the others do to my knowledge)
  • From UP TO DATE:
    > At low doses of 1 to 3 mcg/kg per min, dopamine acts primarily on dopamine-1 receptors to dilate the renal and mesenteric artery beds
    > At 3 to 10 mcg/kg per min (and perhaps also at lower doses), dopamine also stimulates beta-1 adrenergic receptors and increases cardiac output, predominantly by increasing stroke volume with variable effects on heart rate.
    > At medium-to-high doses, dopamine also stimulates alpha-adrenergic receptors, although a small study suggested that renal arterial vasodilation and improvement in cardiac output may persist as the dopamine dose is titrated up to 10 mcg/kg per min
    *clinically, the haemodynamic effects of dopamine demonstrate individual variability

Dobutamine (inodilator):
- selective β1-agonist that increases cardiac contractility and reduces pulmonary vascular and systemic vascular resistances

Vasopressin:
- vasopressin may have pulmonary vasodilatory effects in addition to a systemic vasoconstrictive effect

Milrinone (inodilator):
- the phosphodiesterase-3 inhibitors, milrinone and enxoimone, have positive inotropic effects combined with the capacity to reduce RV afterload (‘inodilators’) without significant chronotropic effect, but they can be associated with significant systemic hypotension

How well did you know this?
1
Not at all
2
3
4
5
Perfectly
173
Q

22.2 According to ANZCA PS54(A), an anaesthetic machine requiring electrical power must, in the event of mains power failure, be able to operate under battery backup power for a minimum of

a) 30 min
b) 60 min
c) 120 min
d) 240 min

A

a) 30 min

If the anaesthesia machine requires electrical power for normal operation, a backup power supply must be a part of the machine and permit normal operation for at least 30 minutes after a mains power supply failure. An alarm must be activated at the time of the mains failure and the state of the reserve power supply must be indicated while it is in use.

https://www.anzca.edu.au/getattachment/f05e02ec-2023-4c50-b57f-9549ea0c4183/PS54(A)-Position-statement-on-the-minimum-safety-requirements-for-anaesthesia-machines-and-workstations-for-clinical-practice-2021#page=

How well did you know this?
1
Not at all
2
3
4
5
Perfectly
174
Q

23.1 In patients with primary adrenal insufficiency, a markedly elevated renin is most likely due to

A Insufficient corticosteroid replacement
B Insufficient fludrocortisone replacement
C Excessive corticosteroid replacement
D Excessive fludrocortisone replacement

A

b. Insufficient fludrocortisone replacement

In Primary Adrenal Insufficency, cortisol deficiency results in decreased feedback to the HPA axis, leading to increased secretion of ACTH to stimulate the adrenal cortex. Simultaneously, MCs deficiency causes increased release of renin by the juxtaglomerular apparatus of the kidneys.

How well did you know this?
1
Not at all
2
3
4
5
Perfectly
175
Q

22.1 A 30-year-old parturient presents in labour. She has a history of Addison’s disease from autoimmune adrenalitis and has been taking prednisolone 6 mg daily for ten years. On presentation the patient is given hydrocortisone 100 mg intravenously. The most appropriate steroid replacement regimen the patient should receive during labour is

a. 25mg TDS hydrocortisone
b. 8mg/hr hydrocortisone
c. 6mg PO prednisone

A

8mg/hr

Guidelines for mx of glucocorticoids during the perioperative period for patients with adrenal insufficiency

https://associationofanaesthetists-publications.onlinelibrary.wiley.com/doi/10.1111/anae.14963

How well did you know this?
1
Not at all
2
3
4
5
Perfectly
176
Q

23.1 The next patient on your endoscopy list is a 50-year-old woman who has been scheduled for gastroscopy and colonoscopy under sedation, after unsatisfactory
proceduralist-supervised midazolam and fentanyl sedation in the past. She states that she has egg anaphylaxis and carries an adrenaline (epinephrine) auto-injector.
The most appropriate agent to use for her sedation is

A. Propofol
B. Ketamine
C. Remifentanil
D. Sevofluarane

A

A

The situation in adults is straightforward: there is convincing evidence that propofol is safe in patients who are allergic to peanut and/or soy and/or egg.

BJA Ed
https://academic.oup.com/bja/article/116/1/11/2566111

How well did you know this?
1
Not at all
2
3
4
5
Perfectly
177
Q

21.2 The oral morphine equivalent of tapentadol 50 mg (immediate release) is

a) 5mg
b) 10mg
c) 15mg
d) 20mg
e) 25mg

A

c) 15mg

Oral Tapentadol 25mg = 8mg Oral Morphine

Oral Oxycodone 5mg = 8mg Oral Morphine

Oral Tramadol 25mg = Oral Morphine 5mg

Oral Hydromorphone 4mg = Oral Morphine 20mg

S/L Buprenorphine 200mcg = 8mg Oral Morphine

IV Oxycodone 5mg = Oral Morphine 15mg

IV Morphine 5mg = Oral Morphine 15mg

IV Hydromorphone 1mg = Oral Morphine 15mg

How well did you know this?
1
Not at all
2
3
4
5
Perfectly
178
Q

20.1 The substance that should be avoided in a patient with history of anaphylaxis to MMR vaccine is

a) Protamine
b) Penicillin
c) Sulphonamides
d) Gelofusine

A

gelofusin

Anaphylaxis after vaccination is probably due to anaphylactic sensitivity to gelatin or neomycin, not an egg allergy

How well did you know this?
1
Not at all
2
3
4
5
Perfectly
179
Q

21.1 The implemention of comprehensive multidisciplinary geriatric assessments in the peri-operative period has been shown to

a) Reduce mortality
b) Reduce AKI
c) Reduce periop risk of MACE
d) Reduce length of stay
e) Increase cancellation for surgery

A

d) Reduce length of stay

less time in aged care and reduced mortality

Blue book 2019:
“A referral to a geriatrician for further assessment and management may also be warranted in the preoperative period.
Indeed, a meta-analysis of perioperative interventions to reduce delirium found that a geriatrics consultation before surgery was one of only two perioperative interventions that were associated with a reduction in delirium.”

Association of anaesthetists: The impact of pre-operative comprehensive geriatric assessment on postoperative outcomes in older patients undergoing scheduled surgery: a systematic review

  • reduced medical complications
    > reduced postop delirium,
    > reduced pneumonia
    > reduced pressure sores
  • fewer cancellations
  • reduced length of stay
How well did you know this?
1
Not at all
2
3
4
5
Perfectly
180
Q

22.2 A patient presents for endoscopic retrograde cholangiopancreatography (ERCP) with a history of previous post-ERCP pancreatitis. The management most likely to reduce the likelihood of pancreatitis is

a) Gentamicin
b) PR indomethacin
c) Creon post op
d) Preop smoking cessation

A

b) PR indomethacin

APMSE 5th edition 8.6.1.3: Only rectal NSAIDs are effective for reducing post ERCP pancreatitis, particularly indomethacin. Epidural > PCA for severe acute pancreatitis

A Randomized Trial of Rectal Indomethacin to Prevent Post-ERCP Pancreatitis

https://www.nejm.org/doi/full/10.1056/NEJMoa1111103

Nonsteroidal antiinflammatory drugs (NSAIDs) are potent inhibitors of phospholipase A2, cyclooxygenase, and neutrophil–endothelial interactions, all believed to play an important role in the pathogenesis of acute pancreatitis. NSAIDs are inexpensive and easily administered and have a favorable risk profile when given as a single dose, making them an attractive option in the prevention of post-ERCP pancreatitis. Preliminary studies evaluating the protective effects of single-dose rectal indomethacin or diclofenac in post-ERCP pancreatitis have been conducted, and a meta-analysis suggests benefit.

Results
A total of 602 patients were enrolled and completed follow-up. The majority of patients (82%) had a clinical suspicion of sphincter of Oddi dysfunction. Post-ERCP pancreatitis developed in 27 of 295 patients (9.2%) in the indomethacin group and in 52 of 307 patients (16.9%) in the placebo group (P=0.005). Moderate-to-severe pancreatitis developed in 13 patients (4.4%) in the indomethacin group and in 27 patients (8.8%) in the placebo group (P=0.03).

Conclusions
Among patients at high risk for post-ERCP pancreatitis, rectal indomethacin significantly reduced the incidence of the condition.

How well did you know this?
1
Not at all
2
3
4
5
Perfectly
181
Q

The amount of intravenous potassium chloride required to raise the plasma potassium level from 2.8 mmol/L to 3.8 mmol/L in a normal adult is approximately

a. 10mmol
b. 20mmol
c. 30mmol
d. 100mmol
e. 200mmol

A

e. 200mmol

K+ < 3.0 mmol/L: 200-400 mmol of potassium are required to raise it by 1 mmol/L
K+ > 3.0 mmol/L: 100-200 mmol of potassium are required to raise it by 1 mmol/L

Hypokalaemia P. GLOVER
https://www.cicm.org.au/CICM_Media/CICMSite/CICM-Website/Resources/Publications/CCR Journal/Previous Editions/September 1999/05-Sept_1999_Hypokalaemia.pdf

If the serum potassium level is greater than 3 mmol/L, 100-200 mmol of potassium are required to raise it by 1 mmol/L; 200 - 400 mmol are required to raise the serum potassium level by 1 mmol/L when the potassium concentration is less than 3mmol/L, assuming a normal distribution between cells and the intracellular space, and a linear relationship between plasma potassium and body deficit (which has been described, i.e. 0.27 mmol/L/100 mmol deficit/70 kg), exists. The rate of administration of potassium will be influenced by the presence and seriousness of the pathophysiological changes caused by hypokalaemia. The underlying disorder should also be treated simultaneously.

How well did you know this?
1
Not at all
2
3
4
5
Perfectly
182
Q

22.2 A 72-year-old patient is undergoing resection of an anterior skull based tumour using a combined endoscopic and frontal craniotomy approach. Seven hours into the procedure she has a large diuresis of pale urine and you suspect she may have developed diabetes insipidus. The most appropriate test result to confirm your diagnosis in this setting is a

a. Low serum ADH levels
b. Sequentially increasing Na levels
c. Serum osmolality <260
d. Urine Na >40
e. Urine specific gravity > something

A

b. Sequentially increasing Na levels

How well did you know this?
1
Not at all
2
3
4
5
Perfectly
183
Q

20.2 You are called to assist in the resuscitation of a 75-year-old female patient in the emergency department who is hypotensive and hypoxaemic in extremis. The image shown is of a focused transthoracic echocardiogram, parasternal short axis view. The most likely diagnosis is

a) Pulmonary embolism
b) Anterior MI
c) Cardiac tamponade
d) Pneumothorax

A

a) Pulmonary embolism

A bit about the RV in PE:

The right ventricle drapes around the LV. In response to an acute Pulmonary Embolus (PE) it first dilates. The RV can’t generate much force without training, sowhen the Pulmonary Vascular Resistance (PVR) first rises with a PE, thepulmonary arterypressures don’t actually rise substantially because the RV can’t generate largepressures.

Looking at the ventricle in short axis, the septum maybow towardstheLV which will form aD shape indiastole,producing a“volumeoverloaded right ventricle” appearance.

Only later whenthe RV has beentrainedwill it be able togenerate higher pressures. If the LV is D shaped insystole, this is a “pressureoverloaded right ventricle”.

Acute cor pulmonale with bothpressureANDvolumeoverload (D shape insystoleANDdiastole)is often absent.

How well did you know this?
1
Not at all
2
3
4
5
Perfectly
184
Q

22.2 The nerve labelled by the arrow marked A in the diagram is the

  1. Ulnar Nerve
  2. Axillary Nerve
  3. Median Nerve
  4. Medial Cutaneous nerve of the forearm
  5. Long Thoracic Nerve
  6. Dorsal Scapular Nerve
  7. Radial Nerve
  8. Suprascapular nerve
  9. Musculocutaneous Nerve
A
  1. Dorsal Scapular Nerve
How well did you know this?
1
Not at all
2
3
4
5
Perfectly
185
Q

21.1 A five-year-old child weighing 25 kg is to be strictly nil by mouth overnight following a laparotomy. The most appropriate fluid prescription is

a. 65ml/hr N Saline
b. 45ml/hr N saline
c. 45ml/hr N Saline w 5% dex
d. 65ml/hr .45% saline w 2.5% dex
e. 65ml/hr .45% saline w 5% dex

A

b. 45ml/hr N saline w 5% dextrose
Nsaline + 5% dextrose is fluid of choice

A guide to paediatric anaesthesia fluid management
-421 rule overestimates fluid resus
-due to stress response from ADH release
-post-op fluid maintenance is 2/3rds calculated due to increased ADH
-never use hypotonic fluids

https://www.rch.org.au/clinicalguide/guideline_index/Intravenous_fluids/

How well did you know this?
1
Not at all
2
3
4
5
Perfectly
186
Q

23.1 A level two check of the inhalational anaesthesia delivery device does NOT include checking the

A. Accurate delivery of volatile concentration from vaporiser
B. Connection of vaporiser and seating
C. Secure vaporiser cap
D. Adequate filling of vaporizers
E. Power to vaporiser

A

a) Accurate delivery of volatile concentration from vaporiser

PS31

Level two check should be performed at the start of each anaesthetic list.

4.2.3.2 Inhalational anaesthesia delivery devices (vapouriser)

4.2.3.2.1 Ensure electricity is connected to vapourisers that require it.

4.2.3.2.2 Check the anaesthetic liquid level is within marked limits.

4.2.3.2.3 Ensure all filling ports are sealed.

4.2.3.2.4 Check correct seating, locking and interlocking of detachable vapourisers or casettes.

4.2.3.2.5 Test for circuit leaks with a cassette installed or for each vapouriser in the “on” and “off” state.

4.2.3.3 Check for machine leaks upstream from the common gas outlet or breathing system, using a protocol appropriate for the anaesthesia delivery system.

How well did you know this?
1
Not at all
2
3
4
5
Perfectly
187
Q

22.2 A 48-year-old man is day two post-laparoscopic high anterior resection. He has used 42 mg of intravenous morphine in the past 24 hours. You wish to start him on oral tapentadol immediate release. The most appropriate equianalgesic dosage would be

a) 50mg six times a day
b) 100mg six times a day
c) 200mg six times a day
d) 300 mg six times a day

A

a) 50mg six times a day

42mg IV Morphine = 126mg Oral Morphine

126/8= 15.75
15.75 x 25 = 393.75 (*400mg/day Tapentadol)

Option 50mg 6 times a day = 300mg
As direct OME to tapentadol conversion is 400mg, a 300mg dose represents a 25% dose reduction, which is line with a 25-50% dose reduction due to incomplete cross-tolerance during opioid rotation.

Oral Tapentadol 25mg = 8mg Oral Morphine

Oral Oxycodone 5mg = 8mg Oral Morphine

Oral Tramadol 25mg = Oral Morphine 5mg

Oral Hydromorphone 4mg = Oral Morphine 20mg

S/L Buprenorphine 200mcg = 8mg Oral Morphine

IV Oxycodone 5mg = Oral Morphine 15mg

IV Morphine 5mg = Oral Morphine 15mg

IV Hydromorphone 1mg = Oral Morphine 15mg

How well did you know this?
1
Not at all
2
3
4
5
Perfectly
188
Q

20.1 You are inserting a pulmonary artery catheter in an intubated patient prior to cardiac surgery, and a significant amount of blood appears in the endotracheal tube. The most appropriate specific initial management is to:

A. Remove PAC and insert DLT
B. Wedge PAC and insert DLT
C. Wedge PAC and insert bronchial blocker
D. Withdraw PAC 2 cm and insert DLT
E. Withdraw PAC and insert bronchial blocker
F. Inflate balloon

A

D. Withdraw PAC 2 cm and insert DLT

LITFL: Pulmonary haemorrhage after PAOP measurement

a life threatening time-critical emergency
pulmonary artery rupture caused by inflation of the pulmonary artery catheter (PAC) balloon during ‘wedging’ (measurement of the pulmonary artery occlusion pressure)
some experts advise against measuring PAWP because of the risk of pulmonary artery rupture
0.2% risk,  30% mortality

RISK FACTORS

pulmonary hypertension
mitral valve disease
anticoagulants
age >60 years

MANAGEMENT

Goals

prevent further pulmonary haemorrhage
stop bleeding
resuscitate

Call for help

ICU consultant
anaesthetist/ OT
cardiothoracic surgeon
interventional radiology

Resuscitation

A
    may have to be emergently intubated if not already
B
    FiO2 1.0
    controlled ventilation
    if able to recognize which lung is haemorrhaging may be able to perform lung isolation (insert single lumen tube into unaffected side, exchange for a double lumen tube or use bronchial blocker with bronchoscopic assistance)
    apply PEEP to tamponade wound
C
    large bore IV cannulae, fluids, blood products, inotropes

Specific therapy

Lay the patient ruptured side down
withdraw pulmonary catheter 2-3 cm with balloon down then refloat PAC with balloon inflated to occlude pulmonary artery (to try to tamponade bleeding)
stop antiplatelet agents and anticoagulants
give reversal agents:
— protamine for heparin
— platelets for anti-platelet agents
give blood products as indicated by FBC, coags and clinical state
interventions
— angiogram or bronchoscopy to isolate pulmonary vessel involved
— if bleeding doesn’t settle will require lobectomy
How well did you know this?
1
Not at all
2
3
4
5
Perfectly
189
Q

21.2, 22.2 A patient has blunt chest trauma. A thoracotomy is indicated if the immediate blood drainage after closed thoracostomy is greater than

a) 500mL
b) 750mL
c) 1L
d) 1.2L
e) 1.5L

A

1,500 mL immediately

OR

200 mL/hr in the first 2-4 hours

How well did you know this?
1
Not at all
2
3
4
5
Perfectly
190
Q

21.2 A 30 year old athlete undergoing a knee arthroscopy under general anaesthesia becomes tachycardic intraoperatively. A 12-lead electrocardiogram (ECG) is obtained. The most likely diagnosis is

a) Atrial fibrillation
b) Atrial flutter
c) Sinus tachycardia
d) WPW

A

d) WPW
Type B pattern

LITFL:
ECG features of WPW in sinus rhythm
-> PR interval < 120ms
-> Delta wave: slurring slow rise of initial portion of the QRS
-> QRS prolongation > 110ms
-> Discordant ST-segment and T-wave changes (i.e. in the opposite direction to the major component of the QRS complex)
-> Pseudo-infarction pattern in up to 70% of patients — due to negatively deflected delta waves in inferior/anterior leads (“pseudo-Q waves”), or prominent R waves in V1-3 (mimicking posterior infarction

Can be left-sided (Type A) or right-sided (Type B), and ECG features will vary depending on this:

Left-sided AP:
produces a positive delta wave in all precordial leads, with R/S > 1 in V1.
(Dominant R Wave in V1)
Sometimes referred to as a type A WPW pattern

Right-sided AP:
produces a negative delta wave in leads V1 and V2.
Sometimes referred to as a type B WPW pattern

Tachyarrhythmias in WPW

There are only two main forms of tachyarrhythmias that occur in patients with WPW

  1. Atrial fibrillation or flutter.
    -> Due to direct conduction from atria to ventricles via an AP, bypassing the AV node
  2. Atrioventricular re-entry tachycardia (AVRT).
    -> Due to formation of a re-entry circuit involving the AP

Breakdown of Type A example:
- Sinus rhythm with a very short PR interval (< 120 ms)
- Broad QRS complexes with a slurred upstroke to the QRS complex — the delta wave
- Dominant R wave in V1 suggests a left-sided AP, and is sometimes referred to as “Type A” WPW
- Tall R waves and inverted T waves in V1-3 mimicking right ventricular hypertrophy (RVH) — these changes are due to WPW and do not indicate underlying RVH
- Negative delta wave in aVL simulating the Q waves of lateral infarction — this is referred to as the “pseudo-infarction” pattern

How well did you know this?
1
Not at all
2
3
4
5
Perfectly
191
Q

21.1 A 45-year-old man has the following results on his blood biochemistry testing (Liver function tests shown). The most likely diagnosis is

a. Cholecystitis
b. Metastatic liver disease
c. Hepatitis C
d. Chronic liver disease
e. Paracetamol toxicity

A

a. Cholecystitis

Example and explanation taken from RACGP:
The raised AlP relative to Alt suggests cholestasis and the high GGt confirms liver origin. The mild hyperbilirubinaemia confirms the clinical impression of jaundice. Biliary disease is highly likely with gallstones the most likely differential diagnosis. however, this clinical picture may also occur in drug reactions or infiltrative conditions. After a careful history, abdominal ultrasound is the most appropriate next investigation.

How well did you know this?
1
Not at all
2
3
4
5
Perfectly
192
Q

23.1 The bioavailability of an oral dose of ketamine is approximately

A. 10%
B. 20%
C. 40%
D. 70%
E. 80%

A

B. 20%

25% (a few studies have higher ranges but typically around 20-25%)

https://doi.org/10.1192/bjp.bp.115.165498

How well did you know this?
1
Not at all
2
3
4
5
Perfectly
193
Q

bonus neuro radiology questions

A 24 year-old man developed an explosive headache followed by nausea and vomiting.
He undergoes cerebral angiography.
His cerebral aneurysm is in the

a. Anterior Cerebral Artery
b. Anterior communicating artery
c. MCA
d. PCA
e. Basilar artery

A

b. Anterior communicating artery

https://case.edu/med/neurology/NR/SubarachnoidHemorrhageAComm.htm

How well did you know this?
1
Not at all
2
3
4
5
Perfectly
194
Q

The ANZCA guidelines regarding pre-operative oral intake for infants under 6 months of age having an elective procedure under anaesthesia are

a) Breast milk 2 hours before, clear fluids 1 hour before 3mls/kg
b) Breast milk 2 hours before, clear fluids 1 hour before 5mls/kg
c) Breast milk 3 hours before, clear fluids 1 hour before 3mls/kg
d) Breast milk 3 hours before, clear fluids 1 hour before 5mls/kg
e) Breast milk 4 hours before, clear fluids 1 hour before 3mls/kg

A

c) Breast milk 3 hours before, clear fluids 1 hour before 3mls/kg

PS07 - patient preparation and preanaesthetic consultation appendix 1 2023

Children up to 16 years:

Clear fluids of 3ml/kg up to 1 hour before.

<6 months
Formula 4 hours
Breast 3 hours
Clear 1 hour

> 6 months:
solids and formula wait 6 hours. Breast milk 4 hours.
Clear 1 hour

How well did you know this?
1
Not at all
2
3
4
5
Perfectly
195
Q

22.1 Complex regional pain syndrome is NOT characterised by

a. Vasomotor
b. Sudomotor
c. Pain distal to primary injury
d. Hypoaesthesia
e. Edema

A

Hypoasthesia - Veldman criteria

Pain distal to primary injury – Not mentioned specifically in Budapest criteria, but in Veldman

Hypoaesthesia = reduced sensation to pain

How well did you know this?
1
Not at all
2
3
4
5
Perfectly
196
Q

20.1 The anti-emetic action of aprepitant is via receptors for

A. Serotonin
B. Neurokinin-A
C. Dopamine
D. Substance P
E. Glycine

A

D. Substance P

Development of aprepitant, the first neurokinin-1 receptor antagonist for the prevention of chemotherapy-induced nausea and vomiting (2011)
https://www.ncbi.nlm.nih.gov/pubmed/21434941

Aprepitant acts centrally at NK-1 receptors in vomiting centres within the central nervous system to block their activation by substance P released as an unwanted consequence of chemotherapy.

How well did you know this?
1
Not at all
2
3
4
5
Perfectly
197
Q

21.2, 22.2 You are involved in the care of a two-year-old child who ingested a button battery within the last 4 hours. You should consider giving

a. milk,
b. sodium bicarbonate
c. Pantoprazole
d. sucralfate

A

Honey (or sucralfate) - 10 mL every 10 minutes (maximum 6 times) while awaiting surgical retrieval

Source QCH guidelines

How well did you know this?
1
Not at all
2
3
4
5
Perfectly
198
Q

21.2 The image below shows results from non inferiority trials. The trial labelled ‘B’ is best
described as

a) Non-inferiority is not demonstrated
b) Non-inferiority is demonstrated
c) Superiority is demonstrated
d) Inferiority is demonstrated

A

a) Non-inferiority is not demonstrated

Possible outcomes in a non-inferiority trial.
In A (blue), non-inferiority is demonstrated.
In B (green), non-inferiority is not demonstrated, and the trial is inconclusive.
In C (red), the new treatment is inferior.

How well did you know this?
1
Not at all
2
3
4
5
Perfectly
199
Q

23.1 Of the following drugs, the LEAST suitable for managing atrial arrhythmias in a patient with a left ventricular assist device is

A. Metoprolol
B. Amiodarone
C. Digoxin
D. Diltiazem

A

d) diltiazem

Nondihydropyridine calcium channel blockers should be used cautiously in patients with HFrEF because of their negative inotropic effects, and the role of these agents in LVAD recipients remains unclear

https://www.ahajournals.org/doi/10.1161/CIR.0000000000000673
Should also avoid sotolol

How well did you know this?
1
Not at all
2
3
4
5
Perfectly
200
Q

20.2 A 55 year old man with no past history of ischaemic heart disease is 3 days post total hip replacement surgery. He has an episode of chest pain that sounds ischaemic, began at rest and lasts thirty minutes before resolving fully. There are no ECG changes nor troponin rise. The diagnosis is

a. No diagnosis made
b. Unstable angina
c. STEMI
d. NSTEMI
e. MINS

A

b. Unstable angina

Not a Repeat, no Tropnin rise in this question making the answer unstable angina as opposed to NSTEMI

UTD:

Unstable angina (UA) and acute non-ST elevation myocardial infarction (NSTEMI) differ primarily in whether the ischemia is severe enough to cause sufficient myocardial damage to release detectable quantities of a marker of myocardial injury (troponins):

●UA is considered to be present in patients with ischemic symptoms suggestive of an ACS and no elevation in troponins, with or without electrocardiogram changes indicative of ischemia (eg, ST segment depression or transient elevation or new T wave inversion).

●NSTEMI is considered to be present in patients having the same manifestations as those in UA, but in whom an elevation in troponins is present.

MINS: Myocardial injury after non-cardiac surgery (up to 30 days post-op):
1. Elevated postop troponin
2. Resulting from myocardial ischaemia (i.e. no evidence of a non-ischaemic aetiology), not requiring an ischaemic feature (i.e. no chest pain, no ECG change)

VISION studies (Vascular Events in Noncardiac Surgery Patients Cohort Evaluation) demonstrated that severity of MINS strongly associated with 30-day mortality after NCS.

hs-cTnT
<20ng/L ~ 0.5% 30 day mortality
20-64ng/L ~3% 30 day mortality
65-999 ng/L ~9% 30 day mortality
>1000ng/L ~30% 30 day mortality

Whilst VISION trial identified MINS in at risk patients, the question now becomes what interventions are available to prevent this complication?

How well did you know this?
1
Not at all
2
3
4
5
Perfectly
201
Q

20.1 Best resolution US probe for median nerve visualisation:
d) 5-10mHz
e) 6-13mHz

A

High frequency probe at 90 degrees to the skin
- to best visualise superficial structures have the probe at 90 degrees to the skin with a high frequency transducer

it is best to use high-frequency transducers (up to 10–15 MHz range) to image superficial structures (such as for stellate ganglion blocks) and low-frequency transducers (typically 2–5 MHz) for imaging the lumbar neuraxial structures that are deep in most adults.

How well did you know this?
1
Not at all
2
3
4
5
Perfectly
202
Q

22.2 A test for a condition which has a prevalence of 1 in 1,000 has a sensitivity of 100% and a specificity of 90%. The probability of a patient who receives a positive result actually having the condition is
–50% was not an option

a. 1%
b. 10%
c. 90%
d. 100%

A

a. 1%

i.e. what is the positive predictive value (PPV) for this test

PPV= TP/ TP +FP
Negative Predictive Value = TN / TN + FN

Prevalence of 1/1000
Sensitivity of 100%
Specificity of 90%

Of patients that are disease positive in population of 1000
TP = 1
FN = 0
-> 100% sensitivity

Of patients that are disease negative in population of 1000
FP = 99
TN = 900
-> 90% Specificity

PPV= TP/ TP + FP
= 1/ 1 + 99
= 1/100
=1%

NPV= TN/ TN + FN
=900/ 900 + 0
= 1/1
= 100%

How well did you know this?
1
Not at all
2
3
4
5
Perfectly
203
Q

22.1 A patient is anaesthetised from the awake state to a state of surgical anaesthesia with propofol or a volatile anaesthetic. As the depth of anaesthesia increases, the patient’s electroencephalogram (EEG) will show oscillations that are of

A

Dominant EEG frequency decreases, and amplitude increases with increasing concentrations of anaesthetic. End result is burst suppression

https://academic.oup.com/bja/article/115/suppl_ 1/i27/234261
Figure 1 shows raw EEG waveforms during isoflurane anaesthesia.
During light anaesthesia:
-amplitude is shallow and frequency is high.
When a higher concentration is administered:
-amplitude deepens and EEG frequency slows.

During deep anaesthesia:
- a ‘burst and suppression’ pattern becomes apparent, characterized by extreme activity, represented by high-frequency, large-amplitude waves (bursts), alternating with flat traces (suppression).
- This pattern, excluding brain ischaemia or other factors, indicates that anaesthesia is too deep. Beyond this, flat traces become dominant and, eventually waveforms are no longer apparent.

During isoflurane, sevoflurane or propofol anaesthesia, this sequence of changes in pattern is almost identical.
The major difference in EEG between the volatile agents (isoflurane or sevoflurane) and propofol is apparent in power in the theta range.
During propofol anaesthesia, theta power remains low regardless of concentration, but during isoflurane or sevoflurane anaesthesia, it increases at surgical concentrations of anaesthesia.

How well did you know this?
1
Not at all
2
3
4
5
Perfectly
204
Q

23.1 A five-year-old child weighing 25 kg is to be strictly nil by mouth overnight following a laparotomy. The most appropriate fluid prescription is

a) 45ml/hr 0.9% NS 2.5% dextrose
b) 65ml/hr 0.9% NS 5% dextrose
c) 45ml/hr 0.45% saline with 2.5% dextrose
d) 65ml/hr 0.45% saline with 5% dextrose
e) 45ml/hr 0.9% NS 5% dextrose

A

e. 45ml/hr 0.9% NS 5% dextrose

REPEAT
2/3rd standard full maintenance as unwell

How well did you know this?
1
Not at all
2
3
4
5
Perfectly
205
Q

23.1 The nerve labelled with the arrow in the diagram is the (diagram of the brachial
plexus shown)

a. Musculocutaneous
b. Median
c. Radial
d. Ulnar
e. Axillary

A

a) muscolocutaneous

How well did you know this?
1
Not at all
2
3
4
5
Perfectly
206
Q

21.1 The 12 lead ECG shown (ECG with ST depression V1-V5, perhaps 1mm ste in lead 3) is most consistent with acute total occlusion of the

A. Posterior descending
B. RCA
C. LAD
D. OM

Note this is not the exact ECG with lead I changes too
A

C. LAD

How well did you know this?
1
Not at all
2
3
4
5
Perfectly
207
Q

20.1 Dental damage risk to be determined in your department. 100 cases reviewed, zero cases of dental damage. What is the 95% confidence interval?

a) 0/100
b) 1/100
c) 3/100
d) 5/100
e) 9/100

A

Answer: 3/100

In statistical analysis, the rule of three states that if a certain event did not occur in a sample with n subjects, the interval from 0 to 3/n is a 95% confidence interval for the rate of occurrences in the population. When n is greater than 30, this is a good approximation of results from more sensitive tests. For example, a pain-relief drug is tested on 1500 human subjects, and no adverse event is recorded. From the rule of three, it can be concluded with 95% confidence that fewer than 1 person in 500 (or 3/1500) will experience an adverse event. By symmetry, for only successes, the 95% confidence interval is [1−3/n,1].

The rule is useful in the interpretation of clinical trials generally, particularly in phase II and phase III where often there are limitations in duration or statistical power. The rule of three applies well beyond medical research, to any trial done n times. If 300 parachutes are randomly tested and all open successfully, then it is concluded with 95% confidence that fewer than 1 in 100 parachutes with the same characteristics (3/300) will fail.

How well did you know this?
1
Not at all
2
3
4
5
Perfectly
208
Q

21.1 The main advantage of using norepinephrine (noradrenaline) over phenylephrine for the prevention of
hypotension as a result of spinal anaesthesia for elective caesarean section is

A. Better APGAR
B. Better foetal acid/base
C. Less nausea/vomiting
D. Less maternal bradycardia

A

less maternal bradycardia

How well did you know this?
1
Not at all
2
3
4
5
Perfectly
209
Q

21.1 The muscle or muscle group with the greatest sensitivity to the action of non-depolarising neuromuscular blocking agents is/are the

a. Abdominal muscles
b. Adductor pollicis
c. Pharyngeal muscles
d. Diaphragm

A

c. Pharyngeal muscles

Millers Anaesthesia:
Reference artyicle from Millers: https://pubs.asahq.org/anesthesiology/article/92/4/977/710/The-Incidence-and-Mechanisms-of-Pharyngeal-and

An adductor pollicis TOF ratio of 0.90 or less was associated with impaired pharyngeal function and airway protection, resulting in a four- to fivefold increase in the incidence of pharyngeal dysfunction causing misdirected swallowing. Moreover, pharyngeal function and airway protection may be impaired, even if the adductor pollicis muscle has recovered to a TOF ratio of more than 0.90.

How well did you know this?
1
Not at all
2
3
4
5
Perfectly
210
Q

22.1 Complications of severe anorexia nervosa (body weight < 40% ideal) include all of the following EXCEPT

a. HypoK
b. Cl abnormality
c. Delayed gastric emptying
d. Hypercalcaemia
e. Cardiomyopathy

A

Hypercalcaemia

How well did you know this?
1
Not at all
2
3
4
5
Perfectly
211
Q

21.2 The nerve labelled by the arrow in the diagram below is the

a) Obturator
b) Accessory obturator
c) Genitofemoral
d) Ilioinguinal
e) Iliohypogastric

A

c) Genitofemoral

How well did you know this?
1
Not at all
2
3
4
5
Perfectly
212
Q

22.1 A 45-year-old man presents on the day of surgery for an elective inguinal hernia repair. He is well
but is noted to be mildly jaundiced. He takes simvastatin for hyperlipidaemia and has no other medical history. He consumes about three standard drinks of alcohol per day and does not smoke. He briefly experimented with illicit drugs more than ten years ago. His laboratory results show: (supplied) The most likely diagnosis is

Normal electrolytes
ALP 85 N
ALT 31 N
AST 31 N
GGT 15 N
Urea 10 [4-9]
Creatinine 103 N
Total protein 74 N
Albumin 40 N
BSL 4.2 N
Bilirubin 29 [0-20]
Conjugated 5
Unconjugated 24

A. Fatty liver
B. Hepatitis
C. Cholestasis
D. Gilbert syndrome
E. Drug induced

A

Gilberts

Gilbert’s syndrome is a benign genetic condition that commonly presents as incidental
hyperbilirubinaemia or painless jaundice.

It is relatively common with a population frequency of approximately 2–10%.

Gilbert’s syndrome is caused by defective bilirubin clearance by the hepatic conjugating enzyme UDP-glucuronosyltransferase

https://www.rcpa.edu.au/getattachment/8b9a8acf-f7f5-4088-951c-3f65f0c2f8fe/Interpreting-liver-function-tests.aspx

How well did you know this?
1
Not at all
2
3
4
5
Perfectly
213
Q

20.2 A 46-year old man collapses unexpectedly and fractures his femur. He is booked for acute theatre. A pre-operative electrocardiogram is performed. Of the following, the most appropriate peri-operative medical management is (ECG shown)
ECG = WPW

a) Flecainide
b) Aspirin
c) Digoxin
d) Magnesium
e) Verapamil

A

a) Flecanide

WPW ECG = short PR, wide QRS, delta wave at start of QRS
If WPW, need to prolong refractor period of accessory pathway with agents such as procainamide/flecainide/amiodarone/sotalol.
Avoid verapamil (increases ventricular rate).
Avoid beta blockers (don’t affect accessory pathway).
https://litfl.com/wolff-parkinson-white-syndrome-ccc/

How well did you know this?
1
Not at all
2
3
4
5
Perfectly
214
Q

22.2 Predictors of difficult sedation (agitation or inability to complete the procedure) of patients undergoing gastroscopy do NOT include

A

Unknown options but…

Factors associated WITH difficulty during Gastroscopy were younger age, procedure indication, male sex, presence of a trainee, psychiatric history and benzodiazepine and opioid use. Factors associated with difficulty during COLONOSCOPY were younger age, female sex, BMI <25, procedure indication, tobacco, benzodiazepine, opioid and other psychoactive medication use

How well did you know this?
1
Not at all
2
3
4
5
Perfectly
215
Q

21.2 The condition in which volatile anaesthesia is least appropriate is

a) Multiple sclerosis
b) Myasthenia gravis
c) Lambert-Eaton syndrome
d) Guillain-Barre syndrome
e) Muscular dystrophy

A

e) Muscular dystrophy
- rhabdomyolysis risk if given to patients with Duchenne or Becker’s muscular dystrophy
- volatiles safe in all above, and also safe in patient’s with myotonic dystrophy

Malignant hyperthermia
- high mortality uncoupling regulation of RyR1 to SR
Duschenne muscular dystrophy
- fatal rhabdo (hyperkalaemia)

How well did you know this?
1
Not at all
2
3
4
5
Perfectly
216
Q

23.1 A patient with severe abdominal trauma develops acute respiratory distress syndrome. A diagnosis of abdominal compartment syndrome is confirmed if the patient also has a sustained intraabdominal pressure greater than

A. 10mmHg
B. 16mmHg
C. 20mmHg
D. 24mmHg

A

c) 20mmHg

Intra-abdominal hypertension is defined as a sustained intra-abdominal pressure of >12 mm Hg, and abdominal compartment syndrome occurs at a pressure >20 mm Hg in association with new organ dysfunction.

Intra-abdominal hypertension is graded as follows: Grade 1=12–15 mm Hg; Grade 2=16–20 mm Hg; Grade 3=21–25 mm Hg; and Grade 4 >25 mm Hg.

https://academic.oup.com/bjaed/article/12/3/110/258792

How well did you know this?
1
Not at all
2
3
4
5
Perfectly
217
Q

21.2 An electrocardiogram (ECG) abnormality which is NOT usually associated with severe anorexia nervosa is

a) Sinus tachycardia
b) Wandering atrial pacemakers
c) ST depression
d) T wave inversion
e) Prolonged QT

A

a) Sinus tachycardia

BJA: Anorexia nervosa: perioperative implications
https://academic.oup.com/bjaed/article/9/2/61/299563

Cardiovascular

Typically anorexic patients are hypotensive and bradycardic. These physiological markers may be used as indications for hospitalization.

Bradycardia reflects the decrease in basal metabolic rate that arises as an adaptive response to starvation. Although patients are usually in sinus rhythm, electrocardiographic abnormalities are common and may be found in >80% of strict dieters. These include: atrioventricular block, ST depression, T wave inversion, and QT prolongation.

QT prolongation may be caused by hypocalcaemia, hypomagnesaemia, drugs, or directly by starvation itself. Electrolyte disturbances have a significant causal role in ECG abnormalities.

Other factors, for example, atypical antipsychotics, may also contribute.

Associated arrhythmias include: sinus arrest, wandering atrial pacemakers, nodal escape beats, supraventricular tachycardia, and ventricular tachycardia.

The reported incidence of arrhythmias under anaesthesia is 16–62%.

With respect to myocardial contractility, left ventricular function has been demonstrated to be impaired in a proportion of patients. Echocardiographic studies have also demonstrated a higher incidence of mitral valve prolapse in anorexic patients. The reasons for this are not entirely clear. It is postulated that the loss of left ventricular volume and mass leads to abnormal mitral valve motion.

In addition to starvation-induced myocardial impairment, the myocardium may be specifically damaged by pharmacological agents.

For example, emetogenic ipecac syrup is directly cardiomyotoxic and produces inflammatory changes and myocardial fibre degeneration when used long term.

Rarely, antipsychotic drugs, for example, olanzapine, may cause cardiomyopathy.

Myocardial impairment can be caused by hypophosphataemia which also reduces the threshold for arrhythmias.

Compromised myocardial function requires judicious use of fluids perioperatively as there is an increased risk of congestive cardiac failure. Echocardiography along with invasive perioperative monitoring (central venous catheter) should be considered to prevent fluid overload.

How well did you know this?
1
Not at all
2
3
4
5
Perfectly
218
Q

23.1 Features of hypocalcaemia include all of the following EXCEPT

a. Polydipsia
b. Circumoral tingling
c. Long QTc
d. Laryngospasm
e. Hallucinations

A

a) polydipsia

Hypocalcemia varies from a mild asymptomatic biochemical abnormality to a life-threatening disorder. Acute hypocalcemia can lead to paresthesia, tetany, and seizures (characteristic physical signs may be observed, including Chvostek sign, which is poorly sensitive and specific of hypocalcemia, and Trousseau sign).

https://bestpractice.bmj.com/topics/en-us/160

How well did you know this?
1
Not at all
2
3
4
5
Perfectly
219
Q

20.1Of the following, the LEAST appropriate treatment in the management of severe acute respiratory distress syndrome (ARDS) is

a) High PEEP
b) Recruitment maneuvers
c) Neuromuscular blockade
d) Prone
e) Negative fluid balance

A

Muscle relaxation or Recruitment maneuvers

A, D and E are all appropriate for ARDS.
Muscle relaxation and lung recruitment are controversial.
Best answer is probably A muscle relaxation (not recommended unless there is dyssynchrony).
Recruitment and higher PEEP are conditional.

UP TO DATE: RE: Muscule relaxation: “ Until a clear benefit is demonstrated, we suggest not routinely administering NMBs to patients with moderate to severe ARDS, unless other indications are present (eg, severe ventilator dyssynchrony, particularly if it leads to double triggering, or unwanted motor movement refractory to ventilator adjustment and sedation). “

Recruitment manoeuvres – no positive influence on survival.
https://derangedphysiology.com/main/required-reading/respiratory-medicine-and-ventilation/Chapter%20512/ventilation-strategies-ards

Does this strategy improve survival? Probably not, according to this Cochrane review. In fact, in the ART trial (Cavalcanti et al, 2017) they probably caused harm. But, they can improve oxygenation temporarily.

One can see the benefit of recruitment manoeuvres in patients who have accidentally become disconnected from the ventilator.

The 2017 ATS guidelines were published in May of 20117, whereas the ART trial came out in September that year, and so the ATS guidelines still recommend recruitment manoeuvres whereas the rest of the world probably does not.

In fact, in their answer to Question 8 from the first paper of 2018 the college remarked that if any trainee who confesses to the routine use of recruitment manouvres, “they were should be marked down” by the examiners.

How well did you know this?
1
Not at all
2
3
4
5
Perfectly
220
Q

22.1 A man has symptomatic carbon monoxide poisoning. His pulse oximetry (SpO2) and arterial blood gas (PaO2) would be expected to show

a. Normal SpO2, Normal PaO2
b. Normal SpO2, reduced PaO2
c. Reduced SpO2, normal PaO2
d. Reduced SpO2, reduced PaO2

A

a. Normal SpO2, Normal PaO2

ABG

HbCO (elevated levels are significant, but low levels do not rule out exposure)
lactate (tissue hypoxia)
PaO2 should be normal, SpO2 only accurate if measured (not calculated from PaO2)
MetHb (exclude)

https://litfl.com/carbon-monoxide-poisoning/

How well did you know this?
1
Not at all
2
3
4
5
Perfectly
221
Q

21.2 A 69-year-old woman has a recent onset of dyspnoea and undergoes a right heart catheterisation, with results displayed below. Her pulmonary capillary wedge pressure is 10 mmHg. The most likely diagnosis is

a) Mitral stenosis
b) Mitral regurgitation
c) Aortic stenosis
d) Pulmonary embolism
e) COPD

A

d) Pulmonary embolism
- fits with history of acute dyspnoea
- PCWP normal, therefore precapillary PH
- thus left heart disease unlikely to be the cause of elevated RVSP (clinical group 2)
- COPD possible if cor pulmonale, but this is an unlikely cause of acute dyspnoea given history

Normal PCWP excludes left heart disease as cause of pulmonary HTN (so not MR, MS or AS). The causes of pre-capillary pulm HTN are pulmonary arterial hypertension, pulmHTN secondary to lung disease, chronic thromboembolic pulmonaryHTN, pulmHTN with unclear/multifactorial mechanisms.
Normal pulmonary capillary wedge pressure = 8- 12mmHg
Normal PASP: 15-25mmHg
Normal PADP: 8-15mmHg
Pulmonary HTN is mPAP ≥25mmHg at rest. mPAP = PADP + (PASP-PADP/3)
mPAP in this image is 43 mmHg Transpulmonary gradient = mPAP – PAWP
20B
20B

How well did you know this?
1
Not at all
2
3
4
5
Perfectly
222
Q

23.1 A patient with an acute subarachnoid haemorrhage arrives in the emergency department. Her Glasgow Coma Scale score is 10 and she has no motor deficit. A CT brain shows diffuse subarachnoid haemorrhage with no localised areas of blood greater than 1mm thick, and no intracerebral or intraventricular blood. Her World Federation of Neurosurgical Societies (WFNS) grade of subarachnoid haemorrhage is

A. 1
B. 2
C. 3
D. 4
E. 5

A

D. 4

  • alternatively her Fisher score is: grade 2 (​diffuse thin (<1 mm) SAH, no clots; which estimates an incidence of symptomatic vasospasm of 25%)

Note the new modified Fischer scale.
G0 No SAH or IVH (0%)
G1 Focal or diffuse thin SAH but no IVH (6-24%)
G2 Focal or diffuse thin SAH with IVH (15-33%)
G3 Thick SAH no IVH (33-35%)
G4 Thicc SAH with IVH (34-40%)

The main differences between the Fisher scale and modified Fisher scale are:
1) Fisher scale, no SAH is grade 1, but 0 in modified Fisher scale
2) Fisher scale, thin SAH & no IVH is grade 2, but 1 in modified Fisher scale
3) Fisher scale, thick SAH with no IVH is grade 3 and the same 3 in modified Fisher scale
4) Fisher scale, any IVH is grade 4, irrespective of the presence of SAH but in modified Fisher scale it is either 2 if thin or no SAH, or grade 4 if thick SAH

Modified Fisher Scale:
grade 0
- no subarachnoid hemorrhage (SAH)
- no intraventricular hemorrhage (IVH)
- incidence of symptomatic vasospasm: 0% 3

grade 1
- focal or diffuse, thin SAH
- no IVH
- the incidence of symptomatic vasospasm: 24%

grade 2
- focal or diffuse, thin SAH
- IVH present
- the incidence of symptomatic vasospasm: 33%

grade 3
- thick SAH
- no IVH
- the incidence of symptomatic vasospasm: 33%

grade 4
- thick SAH
- IVH present
- the incidence of symptomatic vasospasm: 40%

Note: the original study did not include a specified measurement or criteria to define thick vs thin hemorrhage.

REPEAT

How well did you know this?
1
Not at all
2
3
4
5
Perfectly
223
Q

20.1 The radial artery pressure trace shown below is from a patient who has an intra-aortic balloon pump in situ. The device has been switched to 1:2 augmentation to assess the timing. The trace shows an augmented beat followed by an un-augmented beat. With respect to the augmentation, the trace shows

a. Correct timing
b. Early inflation
c. Late inflation
d. Early deflation
e. Late deflation

A

Bonus question

b. Early inflation

Waveform features:
> Diastolic augmentation (peak B) encroaches on the peak corresponding to unassisted systole (peak A) – the two peaks have merged and are barely distinguishable.
> There is no ‘sharp V’ or dicrotic notch between peaks A and B.

Early IAB inflation may result in:
> Premature closure of the aortic valve and possible aortic regurgitation, thus impairing left ventricular emptying. There may be an increase in LVEDV, LVEDP and PCWP.
> Increased left ventricular wall stress (afterload) and increased myocardial oxygen consumption will occur.

how to correct:
Delay the onset of IAB inflation, so that it inflates at the dicrotic notch resulting in a ‘sharp V’ (see the normal pressure waveform).

How well did you know this?
1
Not at all
2
3
4
5
Perfectly
224
Q

21.1 A 30-year-old woman, gravida 2, parity 1, undergoes an elective lower segment caesarean section for breech presentation. The international consensus statement on the use of uterotonic agents recommends that the first line uterotonic management is
a) 1unit
b) 1 unit followed by infusion 2.5-7.5 Units/hr
c) 3 units
d) 3 units followed by infusion

A

Bolus 1 IU oxytocin; start oxytocin infusion at 2.5–7.5IU.h (0.04–0.125 IU.min)

EmLSCS; 3 IU oxytocinover≥30 s; start oxytocininfusion at 7.5–15 IU.h (0.125–0.25 IU.min).

https://associationofanaesthetists-publications.onlinelibrary.wiley.com/doi/10.1111/anae.14757

How well did you know this?
1
Not at all
2
3
4
5
Perfectly
225
Q

23.1 A risk factor for the development of chronic postsurgical pain is having

a. Age >65
b. Male
c. Pain at site 1 month prior to surgery
d. Higher SES

A

c. Pain at site 1 month prior to surgery

Pain itself is a risk factor: the strongest predictors of CPSP are chronic preoperative pain and the severity of acute postoperative pain

https://www.ncbi.nlm.nih.gov/pmc/articles/PMC5741327/#:~:text=Pain%20its

How well did you know this?
1
Not at all
2
3
4
5
Perfectly
226
Q

22.1 The first-line drug recommended by both the Australian Resuscitation Council and the New Zealand Resuscitation Council to treat severe cyanide poisoning is

a. Methylene blue
b. Hydroxycobalamine
c. Sodium thiosulfate

A

hydroxycobalamin

How well did you know this?
1
Not at all
2
3
4
5
Perfectly
227
Q

During a routine preoperative examination of a patient’s heart, you note exaggerated splitting of the second heart sound with inspiration. This is characteristically heard in

A. Aortic Reguritation
B. HOCM
C. Left bundle branch block
D. Mitral Stenosis
E. Pulmonary Stenosis

A

E. Pulmonary Stenosis

DERANGED PHYSIOLOGY:

Splitting of the first heart sound
Right bundle branch block can produce a split first heart sound - because the contraction of the right ventricle is delayed- the conduction occurs via the left ventricle rather than the bundle of His- and thefore the closure of the tricuspid valve occurs after a substantial delay.
Atrial septal defect can result in a fixed split of the first heart sound

Splitting of the second heart sound

It is normal for this sound to be split. The high pressure in the systemic circulation slams the aortic valve shut rather abruptly, almost angrily. In contrast, low pressure of the pulmonary circulation tends to close the pulmonary valve gently, and therefore the pulmonary component of the second heart sound (P2) is usually delayed by about 20-30 milliseconds.

It is also normal for increased right ventricular filling to cause a widening of the split. The more blood in the RV, the longer it takes to eject, and therefore the greater the delay until pulmonary valve closure.

n the spontaneously breathing patient, the delay is greatest during inspiration. Naturally, in the patient ventilated with positive pressure the delay is greatest during expiration (positive pressure being a barrier to diastolic filling).

Increased normal splitting of S2

Anything that delays the end of right ventricular systole can cause this sort of picture.

Right bundle branch block - the delay in conduction via the left ventricle causes a delay in right ventricular contraction, and therefore a delay in pulmonary valve closure. The S1 will also be split.
Ventricular septal defect - because the right ventricle receives a large volume load directly from the left ventricle, and therefore takes longer to complete its systolic contraction.

Pulmonary valve stenosis - because the right ventricle takes longer to empty though a narrowed valve

Mitral regurgitation- not because right ventricular contraction is delayed, but because left ventricular contraction is shortened (as the LV empties in both the aortic and the atrial directuion, systole is over very quickly).

Fixed splitting of S2

Atrial septal defect - the atria, joined by a gaping hole in their seput, act as one atrium. The result is a reasonably equal distribution in volume betweent the right and left atrium. This way, both sides of the circulation share the same diastolic filling pressure. Dragging more volume into the right atrium with respiratory activity will not cause an inequality of ventricular filling (between the right and left ventricles) because the venous return will be “shared”.

Reversed splitting of S2

In this situation, P2 occurs before A2, and splitting widens during expiration (or inspiration in the mechanically ventilated patient). This only happens if the conduction to the left ventricle is delayed, or if the left ventricle is massively volume overload (and the right ventricle is not).
Left bundle branch block - the left ventricle depolarises after the right ventricle, and A2 is delayed
Aortic stenosis - the left ventricle empties slowly though a narrow valve
Large patent ductus arteriosus - the left ventricle receives a backflow of blood from the aorta, which causes it to become volume-overloaded

How well did you know this?
1
Not at all
2
3
4
5
Perfectly
228
Q

21.1 A patient with C6 tetraplegia is undergoing removal of bladder stones under general anaesthesia. The blood pressure rises to 166/88 mmHg. The appropriate response is to

a. Clonidine
b. Hydralazine
c. Decompress the bladder
d. Fentanyl
e. Deepen your anaesthetic

A

decompress the bladder

Autonomic Dysreflexia:
- medical emergency characterised by severe hypertension,
- brought on by stimulation below the level of the lesion

Factors affecting the development of ADR:
1. Level of spinal injury
2. Duration of injury
3. Whether injury is complete or incomplete

Pathology:
Stimuli arise from caudal roots below the level of the lesion leading to uncontrolled sympathetic activation below the level of the lesion
○ 80% being due to bladder distension
○ Other triggers include
§ bowel distension
§ acute abdo pathology
§ activation of pain fibres
§ sexual activity
§ uterine contractions

How well did you know this?
1
Not at all
2
3
4
5
Perfectly
229
Q

21.1 A 100 kg 32-year-old male presents two hours after suffering a 30% total body surface area electrical burn. He has had no resuscitation fluids. The infusion rate of isotonic crystalloid resuscitation fluid required for this man for the next six hours is

a. 500 ml/hr
b. 750 ml/hr
c. 1000 ml/hr
d. 1200 ml/hr

A

c. 1000 ml/hr

30% x 100kg x 4ml= 12000ml
50% in first 8hrs = 6000ml
pt presents 2hrs late = 6000ml/6hrs = 1000ml/hr

or

30% x 100kg x 3ml= 9000ml
50% in 1st 8 hrs= 4500ml
pt presents 2hrs late= 4500ml/6hrs= 750ml/hr

750mls/hr

EMSB recommends 3ml/kg, BJA recommends 4ml/kg
-> go with 4ml/kg because electrical burn and more likely to require increased fluids anyway

How well did you know this?
1
Not at all
2
3
4
5
Perfectly
230
Q

23.1 According to the categorisation system used in Australia and New Zealand for prescribing medicines safely in pregnancy, category X denotes drugs which are

a. Drugs that absolutely must not be used for pregnancy. (absolute contraindication)
b. Untested drugs in pregnancy
c. Drugs safe in pregnancy

A

a. Drugs that absolutely must not be used for pregnancy. (absolute contraindication)

https://www.tga.gov.au/australian-categorisation-system-prescribing-medicines-pregnancy

How well did you know this?
1
Not at all
2
3
4
5
Perfectly
231
Q

21.1 Considering emergency front-of-neck airway access, the major blood vessel that is most likely to lie anterior to the trachea above the sternal notch is the

a) Brachiocephalic artery
b) Brachiocephalic Vein
c) Superior thyroid artery
d) Inferior thyroid artery
e) Carotid artery

A

a) Brachiocephalic artery

Major vessels, most commonly the brachiocephalic artery, traverse the anterior tracheal wall in up to 53% patients at the suprasternal notch

How well did you know this?
1
Not at all
2
3
4
5
Perfectly
232
Q

23.1 Self-report of pain in children is usually possible by the age of

a. 2 yo
b. 4 yo
c. 6 yo
d. 8 yo

A

b) 4yo

4 yo = wong baker faces score 3-18.
8 yo = Visual analogue scale.

https://www.rch.org.au/rchcpg/hospital_clinical_guideline_index/Pain_assessment_and_measur ement/

APMSE 5 also

How well did you know this?
1
Not at all
2
3
4
5
Perfectly
233
Q

22.1 Preperitoneal pelvic packing is a surgical treatment of haemorrhage from a/an

a) pelvic fracture

A

Haemodynamically unstable pelvic fracture

How well did you know this?
1
Not at all
2
3
4
5
Perfectly
234
Q

22.1 Following the initial subarachnoid haemorrhage from a ruptured aneurysm, the patient is at greatest risk of rebleeding during the following

a. 1-3 days
b. 3-5 days
c. 5-7 days
d. 7-10 days

A

a. 1-3 days

How well did you know this?
1
Not at all
2
3
4
5
Perfectly
235
Q

22.2 A child with well controlled dysrhythmias has an ASA (American Society of Anesthesiologists) Physical Status classification of at least

a) I
b) II
c) III
d) IV
e) V

A

B II

ASA II Paediatric examples: Asymptomatic congenital cardiac disease, well controlled dysrhythmias, asthma without exacerbation, well controlled epilepsy, non-insulin dependent diabetes mellitus, abnormal BMI percentile for age, mild/moderate OSA, oncologic state in remission, autism with mild limitations

How well did you know this?
1
Not at all
2
3
4
5
Perfectly
236
Q

20.1 Interventions that reduce the risk of agitation following electroconvulsive therapy include all of the following EXCEPT

a Low dose of propofol following the seizure
b Low dose of midazolam following the seizure
c Premedication with olanzapine
d Premedication with dexmedetomidine
e Induction with remifentanil

A

e) Remifentanil bolus

Induction agents:

Propofol:
-0.75-2.5mg/kg
- shortest seizure duration
- improved CVS stability, less PONV, quicker emergence
- pain on injection

Etomidate
- 0.15-0.3mg/kg
- Prolonged seizure activity, may reduce seizure threshold
- Useful in resistant seizures
- Hyperdynamic response more pronounced compared with propofol, increased PONV, longer emergence time

Methohexital
- 0.5-1.5 mg/kg
- “gold standard” for ECT seizure quality
- long history of use
- reduced availability; lack of familiarity with sue

Thiopental
- 2-5mg/kg
- Seizure duration reduced but better than propofol
- need to reconstitute, has increased dysrhythmias

Ketamine
- 0.7-2.8mg/kg
- unclear effect on seizures: reduced and prolongesd in different studies
- usefull in resistant seizures
- emergence phenomena, reduced CVS stability and increases ICP

Sevoflurane
-6-8% inspired concentration; MAC1-2
- reduced seizure duration compared to methohexital
- useful if difficult IV access, reduces uterine contractions in pregnancy
- extra equipment needed; more time consuming

Induction agents in the descending order of CMRO2 reducing ability:
Propofol > sevoflurane > thiopental and methohexital > etomidate > ketamine.

Induction agents in the descending order of CBF and ICP reducing ability:
Propofol > thiopental and methohexital > etomidate > ketamine.

Induction agents in the descending order of emergence time:
Ketamine > etomidate > barbiturates > propofol > sevoflurane.

Emergence time is the time from drug administration for general anaesthesia till eye opening or following commands. The differences in emergence time among induction agents suitable for ECT are small, and these small variations in emergence should not govern drug choice.

Induction agents in descending order of seizure threshold reducing property are:
Etomidate > ketamine > methohexital > thiopental > propofol.

Opioid:
- Alfentanil (10-20mcg/kg) or remifentanil (1mcg/kg) can be used along with the induction agent to increase the seizure duration and reduce haemodynamic response.
- It is unclear if the effect on seizure duration is an inherent effect of the opioid or as a result of its dose sparing effect.

NMB:
-Neuromuscular blocking agents reduce muscular convulsions and decrease the risk of serious injury.
- Sux at 0.5mg/kg most commonly used, larger doses upto 1.5mg/kg nay be required

Adjuncts:
- used to reduce dose of induction agent, or mitigate cardiovascular response to ECT in high risk patients

  • To treat adverse PNS effects
    Glycopyrolate:
  • superior anti-sialogogue effect
  • no adverse CNS effects
  • less post ECT tachycardia
    Atropine
  • routine atropine pre-medicattion is not recommended due to adverse effects of increased myocardial work and O2 demand

To treat Adverse SNS effects:
- B-blockers: atenolol (pre-ECT) or labetalol and esmolol (intra-ECT), this may reduce seizure duration
-CCB: sublingual nifedapine and IV nicardipine for Htn but may reduce seizure duration
- a-2 agonists: Dexmedetomidine blunts the hyperdynamic rsponse as does GTN and should be considered in patients at high risk of ischaemia
- Dexmedetomidine reduces the incidence of post-ECT adverse effects such as headache, agitation, postictal delirium, or pain associated with propofol injection
-IV lignocaine is not effective

Emergence agitation:
- Small doses of midazolam may be useful if simple measures such as a secluded, calm recovery environment do not help
-However, we avoid administration of any benzodiazepine such as midazolam before performing an ECT procedure, due to known anticonvulsant properties that would make seizure induction more difficult
- In patients with a history of severe postictal agitation, intravenous (IV) benzodiazepines or propofol may be administered at the end of the seizure Dexmedetomidine may be useful in the treatment of refractory cases

How well did you know this?
1
Not at all
2
3
4
5
Perfectly
237
Q

22.2 A 45-year-old man is ventilated in the intensive care unit and is in a critical state. His pulmonary artery wedge pressure is 26 mmHg, cardiac index is 1.7 L/minute/m2 and his PaO2/FiO2 ratio is 200 mmHg. A decision is made to place him on extracorporeal membrane oxygenation. The most appropriate mode is

a) VV ECMO
b) VA ECMO
c) Atrio-aorto ECMO
d) Ventriculo-atrial ECMO

A

b) VA ECMO

PaO2/FiO2 ratio
Mild: 200-300 = mortality 27%
Moderate = 100-200 mortality 32%
Severe < 100 = Mortality 45%

Cardiac Index
Normal: 2.5-4.2l/min

PAWP:
Normal 4-12mmHg

CI is low, PaO2/FiO2 ratio is mild, PAWP is high

How well did you know this?
1
Not at all
2
3
4
5
Perfectly
238
Q

22.1 The sensory innervation to the larynx above the vocal cords is provided by the

a) External SLN
b) Internal SLN
c) RLN

A

b) Internal SLN

How well did you know this?
1
Not at all
2
3
4
5
Perfectly
239
Q

23.1 A third heart sound at the apex may be heard in

a) pulmonary stenosis
b) pulmonary hypertension
c) pericarditis
d) pregnancy

A

d. pregnancy

How well did you know this?
1
Not at all
2
3
4
5
Perfectly
240
Q

22.2 A five-month-old child is to undergo routine elective morning surgery. Current ANZCA guidelines advise minimum fasting intervals prior to anaesthesia of

A. 4 hours for breast milk, 2 hours clear fluids
B. 4 hours for formula, 1 hour clear fluids
C. 3 hours for breast milk, 1 hour for clear fluids
D. 6 hours for formula, 2 hours clear fluids
E. 8 hours for solids, 4 hours for all fluids

A

C 3 hours for breast milk, 1 hour for clear fluids

also

B. 4 hours for formula, 1 hour clear fluids

0-6mo 4/3/1
Children > 6mo 6/4/1

Clear fluids 3mL/kg max

How well did you know this?
1
Not at all
2
3
4
5
Perfectly
241
Q

22.2 The most likely side effect observed in the post anaesthetic care unit after the use of dexmedetomidine is

a. Bradycardia
b. hypotension
c. shivering
d. cough
e. sedation

A

b. hypotension

The use of dexmedetomidine did not increase the duration of PACU LOS but was associated with reduced emergence agitation, cough, pain, postoperative nausea and vomiting, and shivering in PACU. There was an increased incidence of hypotension but not residual sedation or bradycardia in PACU.

https://pubmed.ncbi.nlm.nih.gov/35085107/#:~:text=Conclusions%3A%20The%20use%20of%20dexmedetomidine,sedation%20or%20bradycardia%20in%20PACU

How well did you know this?
1
Not at all
2
3
4
5
Perfectly
242
Q

22.2 The nerve labelled by the arrow marked F in the diagram is the

  1. Ulnar Nerve
  2. Axillary Nerve
  3. Median Nerve
  4. Medial Cutaneous nerve of the forearm
  5. Long Thoracic Nerve
  6. Dorsal Scapular Nerve
  7. Radial Nerve
  8. Suprascapular nerve
  9. Musculocutaneous Nerve
A
  1. Axillary Nerve
How well did you know this?
1
Not at all
2
3
4
5
Perfectly
243
Q

20.1 Perioperative hypothermia down to 35degrees - effect on bleeding:

a) More bleeding with normal INR and APTT
b) More bleeding with normal INR and raised APTT
c) More bleeding with raised INR and normal APTT
d) Unchanged bleeding and normal INR and APTT
e) Unchanged bleeding and elevated INR and APTT

A

More bleeding with normal INR and APPT

https://academic.oup.com/bja/article/117/suppl_3/iii18/2664400
Bleeding observed at reduced temperatures (33 – 37 °C) often occurs because of defects in platelet adhesion, while at temperatures below 33 °C, both reduced platelet function and coagulation enzyme activity contribute

Also lab INR and APTT are not temperature corrected

How well did you know this?
1
Not at all
2
3
4
5
Perfectly
244
Q

23.1 The success rate of stopping smoking before surgery is NOT improved by

a) Bupropion
b) Clonidine
c) Nortroptyline
d) Varencicline
e) SSRI

A

E - SSRIs

ANZCA PG12 Background Paper

How well did you know this?
1
Not at all
2
3
4
5
Perfectly
245
Q

20.1 What is the level below which we need to replace fibrinogen in a pregnant patient with a PPH

A. <1 g/L
B. <1.5 g/L
C. <2 g/L
D. <2.5 g/L
E. <3 g/L

A

<2g/L

How well did you know this?
1
Not at all
2
3
4
5
Perfectly
246
Q

22.2 The nerve labelled by the arrow marked E in the diagram is the

  1. Ulnar Nerve
  2. Axillary Nerve
  3. Median Nerve
  4. Medial Cutaneous nerve of the forearm
  5. Long Thoracic Nerve
  6. Dorsal Scapular Nerve
  7. Radial Nerve
  8. Suprascapular nerve
  9. Musculocutaneous Nerve
A
  1. Musculocutaneous Nerve
How well did you know this?
1
Not at all
2
3
4
5
Perfectly
247
Q

21.2 A patient presents for a trans-urethral resection of the prostate (TURP). He had a single drug-eluting coronary stent for angina pectoris inserted six months ago and is taking clopidogrel and aspirin. The most appropriate preoperative management of his medications is to

a) Cease aspirin, continue clopidogrel
b) Cease aspirin for 10 days, cease clopidogrel for 5 days
c) Cease clopidogrel for 5 days, continue aspirin
d) Cease clopidogrel for 10 days, continue aspirin
e) Continue both aspirin and clopidogrel

A

c) Cease clopidogrel for 5 days, continue aspirin
- prostatic surgery, the risk of major bleeding may be greater than the risk of stent thrombosis
- For clopidogrel, we stop five days before surgery
- Clopidogrel, if stopped, should be restarted with a loading dose of 300 mg as soon as possible after surgery, perhaps later in the day if postoperative bleeding has stopped. Some experts recommend a higher loading dose of 600 mg to decrease time to effectiveness in the higher-risk postoperative setting
- suggest that surgery be performed in centers with 24-hour interventional cardiology coverage

UP TO DATE: Noncardiac surgery after PCI

Nonemergency noncardiac surgery — For patients who have undergone previous stenting with either BMS or DES and who will need cessation of one or both antiplatelet agents, we prefer to defer planned nonemergency, nonurgent noncardiac surgery until at least six months after stent implantation. The risks of noncardiac surgery before six months are increased after both BMS and DES.
For patients whose surgery requires cessation of one or both antiplatelet agents and cannot wait six months, and where the risks of delaying surgery outweigh the benefits, our recommended minimal duration of DAPT is four to six weeks, depending on the urgency of surgery and risk of thrombotic complication. This is based in part on evidence suggesting that the increased risk of MI and cardiac death is highest within the first month after stent placement and no clear difference in risk between BMS and DES. Although we prefer to wait at least six weeks when possible, in patients for whom earlier surgery is in their best interest after weighing risks and benefits, we sometimes refer patients as early as four weeks after stent placement.

The proinflammatory and prothrombotic risks of surgery may increase the baseline risk of stent thrombosis even in the presence of DAPT and regardless of stent type during this early period after stenting. We believe this risk to be higher prior to the minimum duration of DAPT recommended above, but the final decision to continue or discontinue antiplatelet therapy in the perioperative period should be made only after an informed discussion among the surgeon, managing cardiologist (and other health care providers), and patient has taken place. In many cases, DAPT can be continued in the perioperative period, although for some surgeries, such as neurosurgery, posterior eye surgery, or prostatic surgery, the risk of major bleeding may be greater than the risk of stent thrombosis.

In these patients who undergo noncardiac surgery before the recommended minimum duration of DAPT, a platelet P2Y12 receptor blocker should be discontinued for as brief a period as possible. Aspirin should be continued through the perioperative period, since the risk of stent thrombosis is further increased with the cessation of both aspirin and clopidogrel and surgery can usually be safely performed on aspirin. The rationale to continue aspirin comes in part from the POISE-2 trial (PCI subgroup analysis), which is discussed separately. However, as many neurosurgical patients, for whom bleeding might be life threatening or lead to severe adverse outcomes, were not enrolled in POISE-2, the optimal strategy is not known.

●Minor surgical and dental procedures usually do not require cessation of antiplatelet therapy.
●With regard to stopping P2Y12 inhibitor prior to noncardiac surgery, we generally follow recommendations found in the manufacturer’s package insert for each drug.
- For clopidogrel, we stop five days before surgery; that is, the last dose is taken on the sixth day before surgery.
- For prasugrel, we stop seven days before surgery.
- For ticagrelor, we stop three to five days before surgery.
- Some experts are willing to recommend shorter discontinuation periods for procedures less likely to be associated with major bleeding.
●Clopidogrel, if stopped, should be restarted with a loading dose of 300 mg as soon as possible after surgery, perhaps later in the day if postoperative bleeding has stopped. Some experts recommend a higher loading dose of 600 mg to decrease time to effectiveness in the higher-risk postoperative setting.
●We suggest that surgery be performed in centers with 24-hour interventional cardiology coverage

How well did you know this?
1
Not at all
2
3
4
5
Perfectly
248
Q

23.1 A patient presents for a transurethral resection of the prostate (TURP). He had a single drug-eluting coronary stent for angina pectoris inserted six months ago and is taking clopidogrel and aspirin. The most appropriate preoperative management of his medications is to

a) Cease aspirin, continue clopidogrel
b) Cease aspirin for 10 days, cease clopidogrel for 5 days
c) Cease clopidogrel for 5 days, continue aspirin
d) Cease clopidogrel for 10 days, continue aspirin
e) Continue both aspirin and clopidogrel

A

C) Cease clopidogrel for 5 days, continue aspirin

WFSA update document
https://resources.wfsahq.org/wp-content/uploads/uia29-Perioperative-management-of-patients-with-coronary-stents-for-non-cardiac-surgery.pdf

Dual antiplatelet therapy should be continued in all patients with coronary stents presenting for surgery.

However, if there is a high risk of surgical bleeding then clopidogrel should be stopped 5-7 days before surgery and monotherapy with aspirin should be continued.

Clopidogrel should be restarted as soon as possible post surgery. Cessation of aspirin therapy may be considered during intracranial surgery and transuretheral resection of prostrate as these procedures are associated with an increased risk of bleeding, but only after contemplating the risk-benefit ratio.

2014 AHA/ACC guidelines on perioperative medicine don’t give a firm answer except: > 180 days since insertion = proceed (Level II b evidence)

How well did you know this?
1
Not at all
2
3
4
5
Perfectly
249
Q

22.1 The manufacturer’s instructions for use of the i-gel supraglottic airway device recommend a minimum patient weight of

a. 1 kg
b. 2 kg
c. 3 kg
d. 5 kg
e. 10 kg

A

b. 2 kg

How well did you know this?
1
Not at all
2
3
4
5
Perfectly
250
Q

22.2 A 50-year-old man has the following pulmonary function test result: (provided). The most consistent diagnosis is
FEV1 68%, FVC 68%, DLCO 91%

a. Pulmonary hypertension
b. pulmonary fibrosis
c. myasthenia gravis
d. sarcoidosis

A

c. myasthenia gravis

How well did you know this?
1
Not at all
2
3
4
5
Perfectly
251
Q

21.2 A 25-year-old male has continued post operative bleeding after an extraction of an impacted third molar tooth under a general anaesthetic. The patient mentions that his father bruises quite easily. His coagulation screen reveals: (Coagulation tests provided). The most likely diagnosis is

His coagulation screen reveals: Prolonged APTT, Normal PT.

a) Factor V Leiden
b) Haemophilia A
c) Haemophilia B
d) Von willebrand disease

A

d) Von willebrand disease
- autosomal dominant inheritance
- may have normal or prolonged APTT, PT is normal

*Haem A: X-linked recessive disorder; would expect prolonged aPTT, and normal PT
*Haem B: X-linked recessive disorder; would expect normal aPTT and normal PT

Up to date:
Inheritance patterns — Most cases of VWD are transmitted as an autosomal dominant trait; this includes types 1 and 2B, and most types 2A and 2M.

Baseline hemostasis assessment —
Most patients will have a complete blood count (CBC) with platelet count and coagulation studies during the initial evaluation for excessive bleeding or bruising.
●Individuals with VWD generally have a normal CBC and a normal platelet count, with the exception of those with type 2B VWD, most of whom will have mild thrombocytopenia (eg, platelet count 100,000 to 140,000/microL).
●Individuals with VWD may have a normal or prolonged activated partial thromboplastin time (aPTT), depending on the degree of reduction of the factor VIII level. The prothrombin time (PT) is normal in VWD.

Up to date:
●Hemophilia A – Inherited deficiency of factor VIII (factor 8 [F8]); an X-linked recessive disorder.
●Hemophilia B – Inherited deficiency of factor IX (factor 9 [F9]); also called Christmas disease; an X-linked recessive disorder.

Laboratory findings —
Hemophilia is characterized by a prolonged activated partial thromboplastin time (aPTT).
However, the aPTT may be normal in individuals with milder factor deficiencies (eg, factor activity level >15 percent), especially in hemophilia B (factor IX deficiency), where even individuals with moderate disease may have a normal aPTT.
In some individuals with hemophilia A, factor VIII levels may increase with stress, leading to a normalization of the aPTT or mis-categorization of factor levels and disease severity.
In patients with hemophilia, the aPTT corrects in mixing studies, unless an inhibitor is present, which only applies to individuals who have received factor infusions or who have an autoantibody such as a lupus anticoagulant or an acquired factor inhibitor.
Mixing studies that do not show correction of a prolonged aPTT suggest an alternative diagnosis such as an acquired factor inhibitor.
The platelet count and prothrombin time (PT) are normal in hemophilia.
Thrombocytopenia and/or prolonged PT suggest another diagnosis instead of (or in addition to) hemophilia.
Measurement of the factor activity level (factor VIII in hemophilia A; factor IX in hemophilia B) shows a reduced level compared with controls (generally <40 percent).
One exception is an individual with mild hemophilia A who undergoes testing when stressed or pregnant and has a falsely elevated factor level. If this is suspected, factor activity testing should be repeated under conditions of low stress.
The plasma von Willebrand factor antigen (VWF:Ag) is normal in hemophilia.
If VWF:Ag is reduced, this suggests the possibility of von Willebrand disease (VWD) rather than (or in addition to) hemophilia.
Urinalysis is not done routinely, but if performed it may sometimes (but not always) show microscopic or macroscopic hematuria.

How well did you know this?
1
Not at all
2
3
4
5
Perfectly
252
Q

22.1 An anaesthetised patient is ventilated and has standard monitoring plus a central venous line. As surgery commences, the line isolation monitor alarms, indicating a potential leakage current of greater than 5 mA from one of the power circuits in use. The most appropriate action is to

a) Ignore it
b) Disconnect non-essential
equipment one by one to identify fault

A

Line isolation monitor alarms when single fault in system. If the alarm is going off, the last piece of equipment plugged in is usually suspect and should be unplugged.

How well did you know this?
1
Not at all
2
3
4
5
Perfectly
253
Q

21.1 The atmospheric lifetime of nitrous oxide (in years) is approximately

A. 1yr
B. 10 yr
C. 50 yrs
D. 100years

A

100 years
Desflurane: 10yrs
Sevoflurane 1yr
ISO quoted about 3 to 5

How well did you know this?
1
Not at all
2
3
4
5
Perfectly
254
Q

23.1 Of the following, the drug that is LEAST likely to provide effective analgesia following paediatric tonsillectomy is

A. Inhalational anesthesia
B. Remifentanil at end of case
C. Dexamethasone
D. Intranasal ketamine

or

a. Ketamine
b. Clonidine
c. NSAIDs
d. Paracetamol
e. Dexamethasone

A

A. Inhalational anesthesia

or

b. Clonidine
Prospect: two studies focused on tonsillectomy, and those did not show any additional analgesic effect of clonidine when used on top of adequate baseline medication after tonsillectomy.

PROSPECT
https://associationofanaesthetists-publications.onlinelibrary.wiley.com/doi/10.1111/anae.15299#:~:text=The%20basic%20analgesic%20regimen%20should,analgesic%20and%20anti%2Demetic%20effects.

How well did you know this?
1
Not at all
2
3
4
5
Perfectly
255
Q

23.1 In a patient with glucose-6-phosphate dehydrogenase deficiency (G6PD), the
intravenous agent that should be avoided is

a. Methylene blue
b. Indocyanine green (ICG)
c. Iodine
d. Dextrose

A

a) methylene blue

Drugs to avoid:

Antibiotics
Sulphonamides (check with your doctor)
Co-trimoxazole (Bactrim, Septrin)
Dapsone
Chloramphenicol
Nitrofurantoin
Nalidixic acid

Antimalarials
Chloroquine
Hydroxychloroquine
Primaquine
Quinine
Mepacrine

Chemicals
Moth balls (naphthalene)
Methylene blue

Foods
Fava beans (also called broad beans)

Other drugs
Sulphasalazine
Methyldopa
Large doses of vitamin C
Hydralazine
Procainamide
Quinidine
Some anti-cancer drugs

How well did you know this?
1
Not at all
2
3
4
5
Perfectly
256
Q

21.2 A patient with a history of restless leg syndrome is agitated in the post-anaesthesia care unit.
After excluding other causes, the best treatment of the agitation in this patient is

a) Pethidine
b) Clonidine
c) Droperidol
d) Haloperidol
e) Midazolam

A

midazolam

  • Opioids, benzodiazepines and pregabalin may also be used to alleviate symptoms.

Perioperative treatment of symptoms
If RLS symptoms occur perioperatively, patients should be allowed to walk or move their legs in bed as soon as possible.
If prolonged bed rest is required, the frequency of RLS medications may be increased to three times a day.
If oral intake is feasible, a patient’s usual oral medication may be given.
Levodopa (a dopamine agonist) may be administered by nasogastric tube.
Alternatively, parenteral apomorphine or a rotigotine patch may be used.
Apomorphine (1 milligram) may be injected subcutaneously on an hourly basis.
Nausea is a common side effect so it may need to be given with an antiemetic.
Rotigotine patches may be used every 24 hours.
Opioids, benzodiazepines and pregabalin may also be used to alleviate symptoms.
Patients should be proactively investigated and treated for iron deficiency, targeting ferritin level greater than 300 micrograms/ litre in adults, and 50 micrograms/litre in children.

How well did you know this?
1
Not at all
2
3
4
5
Perfectly
257
Q

21.1 Unsupported ventilation in a non-anaesthetised patient with long-standing tetraplegia is improved when

a) Trendelenberg
b) Reverse Trendelenberg
c) Supine
d) Left lateral
e) Right lateral

A

C) supine

Vital capacity is increased in the supine position as abdominal wall paralysis permits greater displacement of abdominal contents during caudad diaphragmatic excursion. Patients will benefit from being recovered in the supine position.

Effect of the level of the lesion

Lesions above C3: complete dependence on mechanical ventilation because of phrenic nerve denervation causing complete diaphragmatic paralysis.

Lesions between C3 and C5: variable dependence on ventilatory support because of variable effect on diaphragmatic and accessory muscle function.

Lesions between C6 and C8: they may require intermittent non-invasive ventilatory support. Intact diaphragmatic function and accessory neck muscles enable adequate inspiratory effort. However, intercostals and abdominal wall muscles remain paralysed. Exhalation occurs via passive recoil of the chest wall, and cough is impaired. There is an increased risk of pneumonia because of poor mobilization of lung secretions.

Thoracic injuries: little respiratory compromise; the main problems are attributable to an inefficient cough.

How well did you know this?
1
Not at all
2
3
4
5
Perfectly
258
Q

20.2 When providing anaesthesia for endovascular treatment of acute ischaemic stroke, the Society of NeuroInterventional Surgery and the Neurocritical Care Society recommend

a) General anaesthesia
b) Hypervolaemia
c) Maintain temp >35
d) Maintain BGL 8-12
e) Maintain SBP 140-180

A

e) Maintain SBP 140-180

  1. Tight control of BP, preferentially with IABP
    > goal of >140/90 mmHg and <180/105 mmHg.
  2. Oxygen supplementation to maintain SpO2 >92%.
  3. Maintenance of eucapnia to avoid cerebral vasoconstriction
    > (ETCO2 35- 45 mmHg)
  4. Temperature maintained 35-37c
  5. Euglycaemia (BGL 70-140 mg/dL (4-8 mmol/L)) and hourly monitoring
How well did you know this?
1
Not at all
2
3
4
5
Perfectly
259
Q

21.1 Of the following, allergy based on cross reaction to penicillin sensitivity is most likely with

A) Cephazolin
B) ceftriaxone
C) cefapime
D) cefaclor
E) cefoxatin

A

D) Cefaclor

  1. Cephalexin? More so than Cephazolin (no B-lactam)
  2. Cefaclor

Source: UpToDate

How well did you know this?
1
Not at all
2
3
4
5
Perfectly
260
Q

21.1 21.2 The breast does NOT receive sensory innervation from the

a. Long thoracic
b. Thoracodorsal
c. Anterior intercostals
d. Posterior intercostals
e. Supraclavicular

A

b. Thoracodorsal

Thoracodorsal nerve (C6-C8) is a branch of the posterior cord of the brachial plexus. Its primary function is motor innervation of the latissimus dorsi muscle. Its blockade is relevant in more extensive breast reconstruction procedures.

The Pecs I, Pecs II and Serratus Plane blocks are superficial thoracic wall blocks which through blockade of the
1. Pectoral N.
2. Intercostal N.
3. Thoracodorsal N.
3. Long thoracic N.
It can be used to provide analgesia for breast surgery and other procedures/surgery involving the anterior chest wall.

How well did you know this?
1
Not at all
2
3
4
5
Perfectly
261
Q

22.2 1 MAC of sevoflurane affects the sensory evoked potential signal for spinal surgery by

a) increased latency, increased conduction speed, increased amplitude
b) increased latency, decreased conduction speed, decreased amplitude
c) decrease latency, increased conduction speed, decreased amplitude
d) increased latency, increased conduction speed, decreased aptitude

A

Increased latency, decreased conduction speed, decreased amplitude

How well did you know this?
1
Not at all
2
3
4
5
Perfectly
262
Q

20.1 The neurosurgical registrar has telephoned about a patient with a spinal cord tumour who is on the list for tomorrow. The registrar tells you the patient has Brown-Séquard syndrome (hemisection of the spinal cord). On clinical examination, below the level of the lesion, you would expect to find all EXCEPT ipsilateral

A. Hyperreflexia
B. Loss of tactile stimulation
C. Paralysis
D. Loss of pain/temperature
E. Loss of vibration/proprioception

A

D. Loss of pain/temperature

Brown-Sequard syndrome:
- Also known as Lateral hemi-section syndrome
- Causes
○ Common
§ Knife or bullet injuries
§ Demyelination
○ Rare
§ Spinal cord tumours
§ Disc herniation
§ Infarction
§ infection
Ipsilateral:
- Motor weakness
- Loss of vibration sensation
- Loss of proprioception sensation
Contralateral:
- Loss of pain sensation
- Loss of temperature

Segmental Syndrome:
- Pathologies that affect all functions of the spinal cord at one or more levels
- Total cord transection:
○ Cessation of function in all ascending and descending spinal cord pathways
○ Loss of all types of sensation below the level of the lesion
○ Loss of movement below the level of the lesion
- Acute transection:
○ Spinal shock
○ Flaccid paralysis
○ Urinary retention
○ Diminished tendon reflexes
○ This is usually temporary followed by:
§ Increased tone
§ Spasticity
§ Hyperrelfexia
§ supervene days or weeks after the event
- Transverse injuries above C3 involve sensation of respiration and are often fatal if acute
- Lesions above L2 will cause impotence and spastic paralysis of bladder
- Causes:
○ Myelopathies
§ Traumatic injury
§ Spinal cord haemorrhage
○ Epidural or intramedullary abscesses or tumours and transverse myelitis may have a more subacute presentation

Dorsal (posterior) cord syndrome:
- Bilateral involvement of:
○ Dorsal Columns
○ Corticospinal tracts
○ Descending central autonomic tracts to bladder control centres in the sacral cord
- Symptoms/signs:
○ Gait Ataxia (DC)
○ Paraesthesias (DC)
○ Weakness (CST)
§ Acute
□ Muscle flaccidity
□ Hyporeflexia
§ Chronic
□ Muscle hypertonia
□ Hyperreflexia
○ Extensor plantar response
○ Urinary incontinence (Auto)
- Causes:
○ MS
○ Tabes dorsalis
○ Friedrich ataxia
○ Sub-acute combined degeneration
○ Vascular malformations
○ Epidural and intradural extrameduallry tumours
○ Atlantoaxial subluxation

Ventral (anterior) cord syndrome
- Involves cords in the anterior 2/3rds of the spinal cord
○ Corticospinal tract
○ Spinothalamic tract
○ Descending autonomic tracts to the sacral centers for bladder control
- Signs/symptoms
○ Weakness (CST)
○ Reflex changes (CST)
○ B/L temp and pain sensation (Spino)
○ Tactile and vibratory sense are normal
○ Urinary incontinence (Auto)
- Causes:
○ Spinal cord infarction
○ Intervertebral disc herniation
○ Radiation myelopathy

How well did you know this?
1
Not at all
2
3
4
5
Perfectly
263
Q

21.1 The risk of a perioperative respiratory adverse event in a child is least likely to be increased by

A. Asthma
B. infection 3 weeks ago,
C. history of eczema,
D. passive smoking

A

history of eczema

How well did you know this?
1
Not at all
2
3
4
5
Perfectly
264
Q

21.2 The number of segments in the lower lobe of the left lung is
a) 3
b) 4
c) 5
d) 10
e) 12

A

b) 4

Right lung:
RUL: APA
RML: LM
RLL: SMALP

Left lung:
LUL: ASIA (S&I form the lingular lobe)
LLL: ALPS

Subsegments (total of 42)
Left: 10 + 10
Right: 6 + 4 + 12

How well did you know this?
1
Not at all
2
3
4
5
Perfectly
265
Q

21.2 Methylene blue may be used in the treatment of all of the following conditions EXCEPT

a) Methemoglobinemia
b) Priapism
c) Hepatopulmonary syndrome
d) G6PD deficiency
e) Sepsis

A

d) G6PD deficiency
(contraindicated)

Methylene blue PI:

PROVEBLUE® is indicated:
* for the treatment of drug-induced methaemoglobinaemia (e.g. prilocaine)
* for the treatment of idiopathic methaemoglobinaemia (in which structural abnormality of haemoglobin is not present)
* as a bacteriological stain
* as a dye in diagnostic procedures such as fistula detection
* for the delineation of certain body tissues during surgery.

Contraindications:
PROVEBLUE® is contraindicated in the following circumstances:
* known hypersensitivity to the drug or any other thiazide dyes
* patients with severe renal impairment
* patients with glucose-6-phosphate dehydrogenase deficiency
* methaemoglobinaemia due to chlorate poisoning
* methaemoglobinaemia during treatment of cyanide poisoning

Intrathecal and subcutaneous injection of methylene blue are also contraindicated as they can result in neural damage (intrathecal administration) and necrotic abscess (subcutaneous administration).

Precautions:
Methylene blue is a potent monoamine oxidase inhibitor.
Serotonin syndrome.

Dose:

Adults and children: In the treatment of methaemoglobinaemia, methylene blue is administered intravenously as the 0.5 % solution in doses of 1 to 2 mg per kg bodyweight injected over a period of 5 minutes.
A repeat dose may be given after one hour if required.
A maximum dose of 7mg/kg bodyweight is recommended.
The use of methylene blue is not recommended in infants under 4 months of age.

STAT PEARLS :Methylene blue
https://www.ncbi.nlm.nih.gov/books/NBK557593/

“Methylene blue is a safe drug at a therapeutic dose of <2 mg/kg; however, when levels are >7 mg/kg, many of the adverse effects it exhibits will occur. Serotonin syndrome has been found to occur when combining serotonergic agents with methylene blue at a dose of 5 mg/kg.”

Methylene blue: caution serotonin syndrome, G6PD deficiency
Indications: vasoplegic syndrome, plasmodium falciparum, methaemoglobinaemia, diagnostic purposes.

Safe at doses <2mg/kg. (used in vasoplegic syndrome on CPB at 3mg/kg - Up to date)
Serotonin syndrome at >5m/kg
Other adverse effects at >7mg/kg.

How well did you know this?
1
Not at all
2
3
4
5
Perfectly
266
Q

21.2 A patient presents with a serum sodium of 110 mmol/L. A feature NOT consistent with a
diagnosis of syndrome of inappropriate antiduretic hormone (SIADH) is

a) Urine osmolality <100mOsm/kg
b) Euvolaemic state
c) Urine Na >40 mmol/L
d) Increased cortisol

A

a) Urine osmolality <100mOsm/kg

DIAGNOSTIC CRITERIA

hypotonic hyponatraemia

urine osmolality > plasma osmolality (<275mOsm/kg) (i.e. concentrated urine despite hypotonic blood)

urinary Na+ > 20mmol/L

normal renal, hepatic, cardiac, pituitary, adrenal and thyroid function

euvolaemia (absence of hypotension, hypovolaemia, and oedema)

correction by water restriction

Source LITFL

How well did you know this?
1
Not at all
2
3
4
5
Perfectly
267
Q

The power of a two sample (two group) randomised controlled trial is NOT affected by (the)

a) Sample size
b) Statistical test
c) P value
d) Standard deviation/ variance in sample
e) Effect size

? same?

The power of a statistical test is not affected by the:

a) sample size
b) population variability
c) alpha error
d) effect size
e) robustness of the statistics

A

c) p-value
e) robustness of the statistics

The power of a study is determined by:
1. Frequency of outcome being studied
2. magnitude of effect
3. study design
4. sample size.

The greater the error variance (or standard deviation) the less the power

BARASH: Experimental Medicine: Statistical Analysis

POWER:
The error of failing to reject a false null hypothesis (false-negative) is called a type II or β-error.

(The power of a test is 1−β.)
The probability of a type II error depends on four factors.

  1. Unfortunately, the smaller the α, the greater the chance of a false-negative conclusion; this fact keeps the experimenter from automatically choosing a very small α.
  2. The more variability there is in the populations being compared, the greater the chance of a type II error. This is analogous to listening to a noisy radio broadcast; the more static there is, the harder it will be to discriminate between words.
  3. Increasing the number of subjects will lower the probability of a type II error.
  4. The most important factor is the magnitude of the difference between the two experimental conditions. The probability of a type II error goes from very high, when there is only a small difference, to extremely low, when the two conditions produce large differences in population parameters.

There are four options for decreasing type II error (increasing statistical power):
(1) raise α,
(2) reduce population variability,
(3) make the sample bigger, and
(4) make the difference between the conditions greater.

What is Alpha:

Alpha is also known as the level of significance. This represents the probability of obtaining your results due to chance. The smaller this value is, the more “unusual” the results, indicating that the sample is from a different population than it’s being compared to, for example. Commonly, this value is set to .05 (or 5%), but can take on any value chosen by the research not exceeding .05.

Alpha also represents your chance of making a Type I Error. What’s that? The chance that you reject the null hypothesis when in reality, you should fail to reject the null hypothesis. In other words, your sample data indicates that there is a difference when in reality, there is not. Like a false positive.

How well did you know this?
1
Not at all
2
3
4
5
Perfectly
268
Q

23.1 The odds ratio is the measure of choice for a

a. Case control
b. Cohort
c. RCT
d. Epidemiological study

A

a) case control

https://www.cdc.gov/csels/dsepd/ss1978/lesson3/section5.html

How well did you know this?
1
Not at all
2
3
4
5
Perfectly
269
Q

23.1 A patient with long-term severe anorexia nervosa is commenced on a normal diet. Three days later she develops cardiac failure and exhibits a decreased level of consciousness. The most important parameter to assay and normalise is the plasma

a. Phosphate
b. Potassium
c. Magnesium
d. Sodium
e. Calcium

A

a) Phosphate

hypophosphate: Clinical symptoms range from muscle weakness and paraesthesia to severe cardiac failure, seizures and diaphragmatic paralysis

Refeeding malnourished patients with anorexia nervosa can be associated with hypophosphatemia, cardiac arrhythmia and delirium. Phosphorus repletion should be started early with and serum levels maintained above 3 mg/dL

weakness and fatigue, in the context of a recent history of starting a regular diet while in a state of chronic malnutrition, are concerning for refeeding syndrome, which typically occurs 2 to 5 days after beginning nutritional repletion. Depleted phosphate stores due to prolonged starvation, hypocalcemia, and hypokalemia can lead to impaired muscle contractility and subsequently weakness, myalgia, and tetany.

https://www.ncbi.nlm.nih.gov/pmc/articles/PMC4168120/

How well did you know this?
1
Not at all
2
3
4
5
Perfectly
270
Q

21.1 Of the following, the device that delivers the greatest flow when using ‘Level 1® Fast Flow Fluid Warmer’ rapid fluid infuser system is a (list of intravascular catheters)

a. 6.5 Fr sheath
b. 8.5 Fr Multilumen line
c. 8.5 Fr Swan Ganz Sheath
d. Multilumen something 14G cannula (50mm?)
e. Peripheral RICC line, 8.5 Fr

A

e. Peripheral RICC line, 8.5 Fr

How well did you know this?
1
Not at all
2
3
4
5
Perfectly
271
Q

21.1 The risk of major bleeding in patients taking direct oral anticoagulants (DOACs) is NOT significantly increased by commencing administration of

a) Atorvastatin
b) Amiodarone
c) Digoxin
d) Diltiazem
e) Fluconazole

A

1st a) Atorvastatin
2nd c) Digoxin

source of Atorvastatin > Digoxin
https://www.ncbi.nlm.nih.gov/pmc/articles/PMC5818856/

All of the DOACs are avid substrates for the excretory P-gp system of the gastrointestinal epithelial cells, and drugs that inhibit or induce the P-gp system may affect plasma DOAC levels

Dabigatran and edoxaban are substrates for P-glycoprotein (P-gp)

Apixaban and rivaroxaban are metabolised by cytochrome P450 enzyme CYP3A4 and are substrates for P-gp

There is study evidence that among patients taking DOACs for non-valvular atrial fibrillation, concurrent use of amiodarone, fluconazole, rifampicin, and phenytoin compared with the use of DOACs alone, was associated with increased risk of major bleeding

It is unlikely that clinically significant interactions occur between dabigatran and other drugs that are merely substrates for P-gp-mediated excretion. When dabigatran was coadministered with digoxin neither digoxin nor dabigatran plasma levels were significantly altered

Rivaroxaban and apixaban are metabolised to an extent of 40–50 % in the liver to variable degrees by CYP3A4 and may interact with drugs that inhibit this enzyme.

The metabolism of Apixaban and rivaroxaban can be decreased when combined with Atorvastatin which is also metabolised by CYP3A4

How well did you know this?
1
Not at all
2
3
4
5
Perfectly
272
Q

23.1 Rotational thromboelastometry (ROTEM) is performed on a bleeding patient with the
following series of graphs produced. The most appropriate therapy to be
administered is

a. TXA
b. Fibrinogen
c. Cryo
d. FFP

A

a) TXA

Hyperfibrinolysis

https://derangedphysiology.com/main/required-reading/haematology-and-oncology/Chapter%201.2.0.1/intepretation-abnormal-rotem-data

How well did you know this?
1
Not at all
2
3
4
5
Perfectly
273
Q

22.1 This posteroanterior chest X-ray shows enlargement of the
(everyone seems to be unsure of answer, no image supplied)

a. Aorta
b. RA
c. RV
d. LA
e. LV

A
How well did you know this?
1
Not at all
2
3
4
5
Perfectly
274
Q

22.2 Blockade of the superficial cervical plexus includes the
a. C1 dermatome
b. C5
c. phrenic nerve
d. transverse cervical
e. greater occipital

A

d. transverse cervical

Supraclavicular nerve block. An initial injection of 3 mL local anesthetic is deposited at the midpoint of the sternocleidomastoid muscle, followed by 7 mL injected subcutaneously in a caudad and cephalad direction along the posterior border of the muscle.

complications:
1.Infection
2.Hematoma
3.Phrenic nerve block
4.Local anesthetic toxicity
5.Nerve injury

https://www.nysora.com/techniques/head-and-neck-blocks/cervical/cervical-plexus-block/

How well did you know this?
1
Not at all
2
3
4
5
Perfectly
275
Q

22.1 A 26-year-old patient presents with exertional syncope. The most likely diagnosis is

a. HOCM
b. Long QT syndrome
c. CCF
d. IHD

A

HOCM: pathopneumonic

A person who has syncope during exertion is more likely to have an obstruction to blood flow (aortic stenosis or hypertrophic cardiomyopathy) or ventricular tachycardia as a cause. On the other hand, syncope after completion of exercise is more likely of reflex origin, such as the common faint.

https://www.uptodate.com/contents/syncope-fainting-beyond-the-basics#:~:text=A%20person%20who%20has%20syncope,such%20as%20the%20common%20faint

How well did you know this?
1
Not at all
2
3
4
5
Perfectly
276
Q

21.1 Local anaesthetic-induced myotoxicity is most likely to be associated with

A. Biers
B. Interscalene
C. Sciatic
D. Adductor Canal

A

D. Adductor Canal

unclear phenomonenon
prolonged exposure and high concentrations of local anaesthetic

How well did you know this?
1
Not at all
2
3
4
5
Perfectly
277
Q

23.1 You are called to assist in the resuscitation of a 75-year-old patient in the emergency
department who is in extremis with severe hypotension and hypoxaemia. The image
shown is of a focused transthoracic echocardiogram, parasternal short axis view.
The most likely diagnosis is

a. PE
b. Tamponade

A

a) PE

D-shaped left ventricle

How well did you know this?
1
Not at all
2
3
4
5
Perfectly
278
Q

22.2 A thoracic regional technique that will NOT provide analgesia for sternal fractures is a
a. PECS I
b. PECS II
c. Parasternal intercostal nerve block?
d. Transfascial muscle block (can’t remember wording)
e. transverse thoracic plane block

A

b. PECS I

(PECS II Covers SA and will extend to the sternum)

How well did you know this?
1
Not at all
2
3
4
5
Perfectly
279
Q

21.2 With regard to the risk of postoperative surgical-site infection, 8 mg dexamethasone administered intraoperatively has

a) No increased risk of surgical wound infection
b) Increased surgical wound infection in diabetics
c) Increased surgical wound infection in non-diabetics
d) Decreased surgical wound infection

A

a) No increased risk of surgical wound infection

  • Now, the Perioperative Administration of Dexamethasone and Infection Trial (PADDI), led by Professor Tomás Corcoran, Director of Research in the Department of Anaesthesia and Pain Medicine, Royal Perth Hospital has found that administering a low-dose of dexamethasone during anaesthesia for surgical operations does not increase the risk of surgical wound infections.
How well did you know this?
1
Not at all
2
3
4
5
Perfectly
280
Q

22.1 The train-of-four (TOF) ratio above which the majority of anaesthetists will NOT be able to visually detect fade on TOF stimulation is

a. 0.2
b. 0.4
c. 0.6
d. 0.7
e. 0.9

A

b. 0.4

BJA: Monitoring of neuromuscular block
https://academic.oup.com/bjaed/article/6/1/7/347026

When neuromuscular monitoring is used, visual or tactile evaluation of the degree of neuromuscular block is unreliable.

Even experienced anaesthetists are unable to detect fade when the TOF ratio is >0.4.

It is now thought that significant residual curarization is still present if the TOF ratio is <0.97 (not 0.7 as previously suggested8).

How well did you know this?
1
Not at all
2
3
4
5
Perfectly
281
Q

22.2 You are inserting a pulmonary artery catheter in an intubated patient prior to cardiac surgery and a significant amount of blood appears in the endotracheal tube. The most appropriate specific initial management is to

a. Remove PAC and insert DLT
b. Wedge PAC and insert BB
c. Wedge PAC and insert DLT
d. Withdraw PAC 2cm and insert DLT

A

d. Withdraw PAC 2cm and insert DLT

Pulmonary rupture

Miller:
- Position pt with bleeding lung dependent
- Perform endotracheal intubation, oxygenation, airway toilet
- Isolate lung by endobronchial DLT or SLT or bronchial blocker
- Withdraw PAC several centimetres, leaving it in the main PA. Do not inflate the balloon (except with fluoroscopic guidance)
- Position pt with isolated bleeding lung nondependent. Administer PEEP to the bleeding lung if possible
- Transport the patient to medical imaging for diagnosis and embolisation if feasible

How well did you know this?
1
Not at all
2
3
4
5
Perfectly
282
Q

21.1, 20.1 The drug which has the LEAST impact on somatosensory evoked potentials (SSEPs) monitored in a 15-year-old patient undergoing scoliosis surgery is

A) propofol
B) fentanyl
C) desflurane
D) Midazolam
E) sevoflurane

A

B) fentanyl

Drugs which have the least impact on SSEPs
1. Ketamine
2. Opioids
3. Dexmedetomidine

Article in Anaesthesiology
https://pubs.asahq.org/anesthesiology/article/99/3/716/40407/Pharmacologic-and-Physiologic-Influences-Affecting

o SSEPs = small amplitude potentials measured over the sensory cortex or via epidural electrodes from stimuli applied to the posterior tibial nerves. SSEPs are transmitted via the posterior columns of the spinal cord in the territory of the posterior spinal arteries which supply the posterior 1/3 of the cord. As they are low amplitude they are affected by basal muscle tremor and the signal-to-noise ratio is improved by increasing the depth of muscle relaxation. Their use is not significantly affected by therapeutic concentrations of anaesthetic vapours

o MEPs = series of short-duration constant current stimuli of 300-700 V applied to the motor cortex and measured via needle electrodes inserted into tibialis anterior, abductor halluces and vastus medialis muscles along with selected small muscles of the hands for reference. MEPs rely on corticospinal tract integrity which lies in the territory of the anterior spinal artery. MEPs therefore complement SSEPs in their assessment of spinal cord function. MEPs are large amplitude potentials and are incompatible with profound muscle relaxation. Neuromuscular blocking agents are therefore best avoided or given by infusion and dose optomised with discussion with the technicians (or just give remi).

o All anaesthetic vapours reduce MEP amplitude in a dose-dependent manner, and more than 0.5 MAC are not compatible with reliable monitoring. Thus Propofol TIVA is preferred.

  • Remifentanil is commonly used due to low context sensitive half life and negligible effect on intraop evoked responses
How well did you know this?
1
Not at all
2
3
4
5
Perfectly
283
Q

22.1 Created by the Global Initiative for Chronic Obstructive Lung Disease, the alphabetical GOLD groups A to D are tools for the assessment of chronic obstructive pulmonary disease. These classes are based on

a. Symptoms and exacerbations
b. FEV1
c. FEV1 and exacerbations
d. FEV1/FVC and exacerbations
e. FEV1 and symptoms

A

Sx and exacerbations

FEV1

GOLD ABE assessment tool

How well did you know this?
1
Not at all
2
3
4
5
Perfectly
284
Q

23.1 A newborn baby is pale, limp, grimacing with stimulation, gasping weakly, and has a pulse rate of 90 beats per minute. This corresponds to an Apgar score of

A. 1
B. 2
C. 3
D. 4
E. 5

A

C. 3

Subject repeat but different stem

How well did you know this?
1
Not at all
2
3
4
5
Perfectly
285
Q

21.1 The independent predictors for severe bone cement implantation syndrome (BCIS) in cemented
hemiarthroplasty for hip fracture do NOT include

a. Male
b. GA
c. severe cardiopulmonary disease
d. Diuretic use
e. Age

A

b. GA

Independent predictors for severe BCIS were:
ASA grade III—IV
chronic obstructive pulmonary disease
medication with diuretics or warfarin

Source: BJA 2014 Article
https://academic.oup.com/bja/article/113/5/800/2920080

How well did you know this?
1
Not at all
2
3
4
5
Perfectly
286
Q

21.2 A factor that is NOT used to calculate the Child-Pugh score is

a) Albumin
b) Bilirubin
c) INR
d) Creatinine
e) Ascites

A

d) Creatinine

  • Originally devised to predict outcomes in Cirrhotic patients undergoing portosystemic Surgery
  • Assess perioperative risk for patients with liver disease who undergo hepatic or non-hepatic surgery
  • Factors include:
    o Encepahlopathy
    § None +1
    § Mild to moerate + 2
    § Severe +3
    o Ascites
    § None +1
    § Mild to moderate (diuretic responsive) +2
    § Severe (diuretic refractory) +3
    o Bilirubin
    § <2 mg/dl +1
    § 2-3mg/dl +2
    § >3 mg/dl +3
    o Albumin
    § >3.5g/dl +1
    § 2.8-3.5g/dl +2
    § <2.8g/dl +3
    o INR
    § <1.7 +1
    § 1.7-2.3 +2
    § >2.3 +3
  • Class A 5-6 points
    o 1-5yr survival rate 95%
  • Class B 7-9 points
    o 1-5 year survival rate 75%
  • Class C 10-15 points
    1-5 yr survival rate 50%

Original study Mortality rates in patients who undergo abdominal surgeries:
- Class A 10%
- Class B 82%
- Class C 82%

Newer Study mortality rates after surgery:
- Class A 2%
- Class B 12%
- Class C 12%

Drawbacks:
- Subjective measurement of:
o ascites
o encephalopathy
- Does not consider
o Pre-op infection
o Aetiology of cirrhosis
o Surgery type

How well did you know this?
1
Not at all
2
3
4
5
Perfectly
287
Q

21.1 A patient with a purely metabolic acidosis has a serum bicarbonate of 14 mmol/L and a lactate of 3.8 mmol/L. The expected PaCO2 is

a. 24
b. 29
c. 35
d. 40

A

B. 29

PaCO2= 1.5 x 14 + 8
PaCO2= 21 + 8
PaCO2= 29

Winter’s formula: expected PaCO2 = [1.5 x (serum HCO3)] + [8±2]
if PaCO2 lower, there is a concomitant primary respiratory alkalosis
if PaCO2 higher, there is a concomitant primary respiratory acidosis

How well did you know this?
1
Not at all
2
3
4
5
Perfectly
288
Q

A 50-year-old man has the following pulmonary function test result. The most consistent diagnosis is

FEV1 98% predicted
FVC 98% predicted
DLCO 48% predicted

a) Asthma
b) Obesity
c) Sarcoidosis
d) Pulmonary hypertension

A

d) Pulmonary hypertension
Normal spirometry + low DLCO

Asthma: obstructive pattern and normal DLCO
Obesity: restrictive pattern and normal DLCO
Sarcoid: restrictive pattern and low DLCO

How well did you know this?
1
Not at all
2
3
4
5
Perfectly
289
Q

The maximum warm ischaemic time acceptable for procuring the kidney following donation after cardiac death is

A. 30 mins
B. 60 mins
C. 90 mins
D. 120 mins
E. 180 mins

A

Warm ischaemia time:
- Time from treatment withdrawal to the start of cold perfusion of the donated organs
- Significance is the impact on graft function
- Most important phase of WIT begins when the systolic BP is < 60mmHg
- This includes the waiting period from the absence of circulation to the declaration of death and the time before initiating the flow of cold perfusate through the cannula

Maximum WARM Ischaemia time
- Heart 30 mins
- Liver 30 mins
- Pancreas 30 mins
- Kidney 60 mins
- Lungs 90 mins

Maximum COLD Ischaemia time:
- Heart = 4 hrs
- Lungs = 6-8hrs
- Liver/Pancreas = 12hrs (DBD)/6 hrs (DCD)
- Kidneys = 18hrs (DBD)/ 12 hrs (DCD)

How well did you know this?
1
Not at all
2
3
4
5
Perfectly
290
Q

22.2 A 47-year-old man is anaesthetised for an elective laparoscopic cholecystectomy. Three minutes after induction, he is noted to have a heart rate of 130 bpm and systolic blood pressure of 60 mmHg. The most appropriate initial dose of adrenaline is
a) 100mcg IM Adr
b) 200mcg IM Adr
c) 20mcg Adr IV
d) 100mcg Adr IV
e) 50mcg Adr IV

A

c) 20mcg Adr IV

ANZAAG use Ring and Mesmer scale for anaphylactic reactions as a base for classifying anaphylaxis grade (see image)

From sunny coast QH document
With PAGS ‘Life Threatening Anaphylaxis’ can be distinguished from
‘Moderate Anaphylaxis’ in an adult by the presence of any
one of these signs:
* systolic blood pressure of <60 mmHg
* life-threatening tachy- or bradyarrhythmia
* oxygen saturation <90%
* inspiratory pressures of >40 cmH2

Life-threatening anaphylaxis

How well did you know this?
1
Not at all
2
3
4
5
Perfectly
291
Q

21.2 Cardiovascular effects of hyperthyroidism include

a) Increased DBP
b) Narrow pulse pressure
c) Reduced diastolic relaxation
d) Decreased CO
e) Decreased SVR

A

e) Decreased SVR
- increased CO, increased SBP and decreased DBP with widened PP

UP TO DATE: Cardiovascular effects of hyperthyroidism:

  • Thyroid hormone has important effects on cardiac muscle, the peripheral circulation, and the sympathetic nervous system that alter cardiovascular hemodynamics in a predictable way in patients with hyperthyroidism.
  • The main changes are :
    ●Increases in heart rate, cardiac contractility, systolic and mean pulmonary artery pressure, cardiac output, diastolic relaxation, and myocardial oxygen consumption
    ●Reductions in systemic vascular resistance and diastolic pressure
How well did you know this?
1
Not at all
2
3
4
5
Perfectly
292
Q

20.2 When performing an infraclavicular block of the brachial plexus under ultrasound guidance, the structure indicated by the arrow is the (ultrasound image shown

a) Musculocutaneous nerve
b) Lateral cord
c) Medial cord
d) Superior trunk
e) Inferior trunk

A

c) Medial cord

How well did you know this?
1
Not at all
2
3
4
5
Perfectly
293
Q

20.1 Abuse of nitrous oxide may lead to

a. Anaemia due to decreased erythropoietin
b. Anaemia due to glutathione deficiency
c. Neurological damage due to methionine deficit
d. Pulmonary HTN

A

C

Methionine Synthetase Inhibitor

How well did you know this?
1
Not at all
2
3
4
5
Perfectly
294
Q

22.1 A patient in atrial fibrillation with a CHA2DS2-VASc score of 2 has presented for elective hip surgery. Warfarin had been ceased for four days preoperatively and on the day before surgery the international normalized ratio (INR) was 2.1. The best course of action at this point is to

a) Postpone surgery
b) Vitamin K 3mg IV
c) Prothrombinex 25IU/kg
d) Cell saver intraop
e) Proceed with surgery

A

Give 3mg of Vitamin K and re-check on day of surgery proceed if INR <1.5 on DOS

How well did you know this?
1
Not at all
2
3
4
5
Perfectly
295
Q

23.1 A patient is suffering an acute myocardial infarction. Australian and New Zealand
guidelines recommend the threshold for the use of supplemental oxygen is when the
SpO2 falls below

a. 88%
b. 90%
c. 93%
d. 97%
e. 100%

A

c) 93%

ANZCOR suggests against the routine administration of oxygen in persons with chest pain.13 [2015 COSTR, weak recommendation, very-low certainty evidence]
For persons with heart attack, routine use of oxygen is not recommended if the oxygen saturation is >93% [National Heart Foundation of Australia & Cardiac Society of Australia and New Zealand: practice advice].9

How well did you know this?
1
Not at all
2
3
4
5
Perfectly
296
Q

21.2 Cryoprecipitate contains all of the following EXCEPT

a) Factor I
b) Factor VII
c) Factor VIII
d) VWF
e) Fibronectin

A

b) Factor VII

Redcross:
Cryoprecipitate contains most of the following found in fresh frozen plasma:
1. factor VIII
2. fibrinogen
3. factor XIII
4. von Willebrand factor
5. fibronectin

Prothrombinex-VF® is a lyophilised concentrate of human coagulation factors it contains:

Factors:
II
IX
X
small amount of factor VII.

Also contains:
plasma proteins (human)
Antithrombin III (human)
Heparin sodium (porcine)
Sodium
Phosphate
Citrate
Chloride

How well did you know this?
1
Not at all
2
3
4
5
Perfectly
297
Q

20.2 During the 21st century, the dominant ozone-depleting substance emitted as a result of medical usage to date has been

a) Desflurane
b) Nitrous oxide
c) CO2
d) Isoflurane
e) CFCs

A

Nitrous oxide

Halothane & isoflurane cause catalytic destruction of ozone, but halothane hardly used and isoflurane has short atmospheric lifetime.

Desflurane + sevoflurane don’t cause ozone depletion.

How well did you know this?
1
Not at all
2
3
4
5
Perfectly
298
Q

21.2 The size 5 i-gel® supraglottic airway is recommended for patients who weigh over

a) 50kg
b) 60kg
c) 70kg
d) 80kg
e) 90kg

A

e) 90kg

How well did you know this?
1
Not at all
2
3
4
5
Perfectly
299
Q

21.2, 22.2 Of the following, the procedure that is most commonly associated with chronic pain after surgery is

a) Amputation
b) Mastectomy
c) Thoracotomy
d) TKR
e) Hernia repair

A

a) Amputation

Top 10 Rank order:
1. Amputation 30-85%
2. Thoracotomy 5-67%
3. Mastectomy 11-57%
4. Inguinal hernia repair 0-63%
5. Sternotomy 28-56%
6. Cholecystectomy 3-56%
7. Knee arthroplasty 19-43%
8. Breast Augmentation 13-38%
9. Vasectomy 0-37%
10. Radical prostatectomy 35%

How well did you know this?
1
Not at all
2
3
4
5
Perfectly
300
Q

21.1 Local anaesthetic systemic toxicity does NOT manifest as

a) hypoxaemia
b) severe agitation
c) sinus bradycardia
d) VF
e) seizures

A

a) hypoxaemia

After LA administration, any abnormal cardiovascular or neurological symptoms and signs, including isolated cardiac arrest, should raise suspicion of LAST

Presenting features of LAST vary widely. Cardiovascular collapse may occur without preceding neurological changes.

Clinical features of LAST:

CNS
- 2 stage process of excitatory phase followed by a depressive phase
- early signs:
1. perioral tingling
2. tinnitus
3. slurred speech
4. lightheadedness
5. tremor
6. change in mental state: confusion and agitation

  • excitatory phase culminates in generalised convulsions
    -Depressive phase:
    1. Coma
    2. Respiratory depression

CVS
- 3 phases:
- initial phase:
Htn and tachycardia

  • intermediate phase:
    myocardial depression and hypotension
  • terminal phase:
    peripheral vasodialtion
    severe hypotension

arrhythmias:
1. sinus bradycardia
2. conduction blocks
3. VT
4. Asystole

How well did you know this?
1
Not at all
2
3
4
5
Perfectly
301
Q

21.2 The risk of postoperative respiratory failure in myasthenia gravis is increased by the
administration of

a) Teicoplanin
b) Flucloxacillin
c) Cephazolin
d) Gentamicin
e) Vancomycin

A

d) Gentamicin

Drugs in the anaesthetic trolley that may unmask or worsen MG:
- NMBs
- gentamicin
- beta blockers (metoprolol)
- magnesium

Anaesthetic drugs to be cautious with:
- dexamethasone
- antipsychotics
- anticonvulsants
- antibiotics (vancomycin, metronidazole)

How well did you know this?
1
Not at all
2
3
4
5
Perfectly
302
Q

20.2 The flow volume loop is most consistent with (Flow-volume loop shown)

a) Variable intra-thoracic obstruction
b) Variable extra-thoracic obstruction
c) Lower airway obstruction
d) Fixed upper Airway obstruction
e) Mixed pattern

A

d) Fixed upper Airway obstruction

Fixed upper airway obstruction (can be intrathoracic or extrathoracic): flow limitation and flattening are noted in both the inspiratory and expiratory limbs of the flow-volume loop.

How well did you know this?
1
Not at all
2
3
4
5
Perfectly
303
Q

22.2 The nerve labelled by the arrow marked G in the diagram is the

  1. Ulnar Nerve
  2. Axillary Nerve
  3. Median Nerve
  4. Medial Cutaneous nerve of the forearm
  5. Long Thoracic Nerve
  6. Dorsal Scapular Nerve
  7. Radial Nerve
  8. Suprascapular nerve
  9. Musculocutaneous Nerve
A
  1. Radial Nerve
How well did you know this?
1
Not at all
2
3
4
5
Perfectly
304
Q

23.1 A patient requiring an elective major joint replacement has had a recent stroke. The minimum recommended duration between the stroke and surgery is

a) 3 months
b) 6 months
c) 9 months
d) 12 months

A

c. 9
AHA guidelines

12 Months
But 12 weeks minimum

Although the evidence between surgical timing and stroke risk is limited to only these 2 studies, we suggest that elective noncardiac surgery be deferred at least 6 months after a prior stroke, and possibly as long as 9 months to reduce the risk of perioperative stroke in patients undergoing noncardiac surgery.

Alternatively, patients who stand to gain significant improvements in quality of life with elective surgery may consider waiting only 6 months after a prior stroke

REPEAT

How well did you know this?
1
Not at all
2
3
4
5
Perfectly
305
Q

20.1 What is the arrow pointing to?

a. Psoas
b. Femoral
c. Genitofemoral
d. Lumbosacral trunk
e. Obturator

A

d. Lumbosacral trunk

How well did you know this?
1
Not at all
2
3
4
5
Perfectly
306
Q

20.1 A 25-year-old man suffers a 30% total body surface area burn. A physiological change expected within the first 24 hours is

a) Increase PVR
b) Decreased SVR
c) Increased cardiac index
d) Increased stroke volume

A

a) Increase PVR

UTD:
Low cardiac output “ebb,” phase (resuscitative phase) –

In the first 48 hours after a major burn, cardiac output (CO) is reduced up to 60 percent from baseline due to:
1. hypovolemia from permeability-induced plasma loss
2. reduced myocardial response to catecholamines
3. increased systemic vascular resistance due to elevated vasopressin levels
4. depressed myocardial contractility
5. possible myocardial ischemia due to decreased coronary blood flow

The large volumes that these patients require can sometimes result in over-resuscitation, leading to pulmonary edema and right heart failure.

High cardiac output “flow,” phase (recovery phase)
During the recovery phase 72 to 96 hours postburn, hyperdynamic and hypermetabolic responses result in:
1. increased CO
2. Tachycardia
3. increased myocardial oxygen consumption
4. decreased systemic vascular resistance

Elevation of catecholamines in major burns produces:
1. hyperdynamic circulation
2. augments energy expenditure
3. romotes protein catabolism in skeletal muscle.

Nonselective beta blocker therapy is sometimes used to block catecholamine receptors, treat cardiac dysfunction, and modulate the hyperdynamic response during this phase

How well did you know this?
1
Not at all
2
3
4
5
Perfectly
307
Q

23.1 Cryoprecipitate contains coagulation factors

A. 2, 8, 13, von willebrands
B. 1, 7, 13 , von willebrands.
C. 1, 8, 13, von willebrands.
D. 2, 7, 13, von willebrands.

A

C.

Cryoprecipitate contains Factor VIII, XIII, fibrinogen (factor I), fibronectin, vWF

https://www.lifeblood.com.au/health-professionals/products/blood-components/cryoprecipitate

How well did you know this?
1
Not at all
2
3
4
5
Perfectly
308
Q

21.1 A 40-year-old man suffers a hydrofluoric acid burn to 60% of his total body surface area in an industrial accident. An expected electrolyte disturbance is:
a. Hypocalcemia
b. Hyponatremia
c. Hypophosphatemia
d. Hypomagnesemia

A

hypoCALCEMIA

Hydrofluoric acid: highly corrosive inorganic aacid of element fluorine
- used in glass etching, electronics industry and cleaning solutions for metals, stone and marble
- dilute solutions can rapidly penetrate the skin and exposure of even a small area can be fatal (2%TBSA)

Fluoride ions bind with calcium and magnesium ions in the tissues
-> significant hypocalcaemia and hypomagnesaemia
-> Hyperkalaemia may also be seen
Direct cardiotoxic effects of fluoride ions can lead to cardiac arrhythmias that are difficult to treat

How well did you know this?
1
Not at all
2
3
4
5
Perfectly
309
Q

20.1 The radial artery pressure trace shown below is from a patient who has an intra-aortic balloon pump in situ. The device has been switched to 1:2 augmentation to assess the timing. The trace shows an augmented beat followed by an un-augmented beat. With respect to the augmentation, the trace shows

a. Correct timing
b. Early inflation
c. Late inflation
d. Early deflation
e. Late deflation

A

d. Early deflation

https://litfl.com/intra-aortic-balloon-pump-trouble-shooting/

Waveform features:
> There is a sharp drop in pressure immediately following the peak of diastolic augmentation (peak B).
> Diastolic augmentation may be suboptimal but it is difficult to confirm in the absence of a pressure scale or comparison to an unassisted waveform.
> Assisted aortic end-diastolic BP may be sub-optimally increased (trough C), but it is difficult to say based on the information given. With early deflation a widened U-shaped trough is typically seen.
> Early deflation can lead to an assisted aortic end-diastolic BP that equals or exceeds the unassisted aortic end-diastolic BP (trough F), although this is not the case in this scenario.
> Assisted SBP (peak D) is the same or higher than the unassisted SBP (peak A) – it should be slightly less.

Early IAB deflation may result in:
> inadequate coronary perfusion, with the potential for retrograde coronary blood flow. This may result in angina due to decreased myocardial oxygen supply.
> suboptimal afterload reduction and increased myocardial oxygen demand

How to correct:
Prolong the IAB inflation time, so that it deflates at the end of diastole, just before the onset of isovolumetric systolic contraction.

How well did you know this?
1
Not at all
2
3
4
5
Perfectly
310
Q

22.1 When fresh frozen plasma is administered to treat hypofibrinogenaemia in a bleeding patient, the volume required to achieve an increase in plasma fibrinogen concentration of one gram per litre is

A. 5 ml/kg
B. 10 ml/kg
C. 20 ml/kg
D. 30 ml/kg
E. 50 ml/kg

A

D. 30 ml/kg

Identification and Management of Obstetric Hemorrhage
Anesthesiology Clinics - Obstetric Anesthesia (2017)
https://www.anesthesiology.theclinics.com/article/S1932-2275(16)30074-X/fulltext

Although FFP, cryoprecipitate, and fibrinogen concentrates can all be used to increase fibrinogen levels, the optimal strategy for managing hypofibrinogenemia in obstetric hemorrhage is unclear. The relatively low concentration of fibrinogen in FFP limits its usefulness in the treatment of significant hypofibrinogenemia. To increase fibrinogen plasma level by 1 g/L, 30 mL/kg of FFP is necessary, increasing the risk of pulmonary edema and other hypervolemic complications. Cryoprecipitate, which is a concentrated source of fibrinogen, factor VIII, fibronectin, von Willebrand factor (vWF), and factor XIII, will increase fibrinogen levels by ~0.7 to 1 g/L for every 100 mL given. Although cryoprecipitate is associated with a lower transfusion volume, the standard “dose” (10 U) is typically prepared by pooling concentrates from multiple donors. Given the risk of infectious disease transmission and/or an immunologic reaction from exposure to multiple donors, several countries preferentially use purified, pasteurized fibrinogen concentrate for the treatment of congenital and/ or acquired hypofibrinogenemia. Fibrinogen concentrates are also prepared from large donor pools, but subsequent processing removes or inactivates potentially contaminating viruses, antibodies, and antigens. Studies comparing cryoprecipitate and fibrinogen concentrates utilization in hemorrhage resuscitation suggest fibrinogen concentrates are associated with lower blood loss, decreased RBC transfusion, and greater increases in plasma fibrinogen levels. Although the most appropriate method of fibrinogen replacement is somewhat controversial, the critical role of fibrinogen in reversing the coagulopathy accompanying obstetric hemorrhage is clear. As such, close monitoring and replacement of fibrinogen are crucial in the management of the bleeding parturient.

How well did you know this?
1
Not at all
2
3
4
5
Perfectly
311
Q

23.1 In patients without other comorbidities, bariatric weight loss surgery is indicated when
the body mass index (kg/m2) is greater than

a. 35
b. 40
c. 45
d. 50

A

a. 35

Major updates (2022) to 1991 National Institutes of Health guidelines for bariatric surgery

Metabolic and bariatric surgery (MBS) is recommended for individuals with a body mass index (BMI) 35 kg/m2 , regardless of presence, absence, or severity of co-morbidities.

MBS should be considered for individuals with metabolic disease and BMI of 30-34.9 kg/m2

BMI thresholds should be adjusted in the Asian population such that a BMI 25 kg/m2 suggests clinical obesity, and individuals with BMI 27.5 kg/m2 should be offered MBS.

Long-term results of MBS consistently demonstrate safety and efficacy.

Appropriately selected children and adolescents should be considered for MBS.

https://www.soard.org/article/S1550-7289(22)00641-4/fulltext#:~:text=The%201991%20NIH%20Consensus%20Statement,surgery%20that%20is%20applied%20universally.

How well did you know this?
1
Not at all
2
3
4
5
Perfectly
312
Q

23.1 According to the ANZICS Statement on Death and Organ Donation (2021), for the diagnosis of brain death after resuscitation and return of circulation following cardiorespiratory arrest, clinical testing should be delayed for at least

a. 12hr
b. 24hr
c. 36hr
d. 48hr
e. 72hr

A

b) 24 hrs

How well did you know this?
1
Not at all
2
3
4
5
Perfectly
313
Q

20.1 To reduce the risk of ?re-bleed, Neuroradiology society recommend:(uncertain source of this question)

a. Coiling <24hrs
b. Coiling >24hrs
c. Clipping <24hrs
d. Clipping >24hrs

A

A or D

International subarachnoid aneurysm trial (ISAT) of neurosurgical clipping versus endovascular coiling in 2143 patients with ruptured intracranial aneurysms: a randomised comparison of effects on survival, dependency, seizures, rebleeding, subgroups, and aneurysm occlusion

Findings:
In patients with ruptured intracranial aneurysms suitable for both treatments, endovascular coiling is more likely to result in independent survival at 1 year than neurosurgical clipping; the survival benefit continues for at least 7 years. The risk of late rebleeding is low, but is more common after endovascular coiling than after neurosurgical clipping.

How well did you know this?
1
Not at all
2
3
4
5
Perfectly
314
Q

20.2 You are called to assist with a patient in the intensive care unit who has had cardiac surgery three days ago and is now in cardiac arrest. External cardiac massage should aim for a systolic blood pressure of

a. 40
b. 60
c. 80
d. 100
e. 120

A

b. 60

BJA Article - ​Management of cardiac arrest following cardiac surgery - BJA Education

In the CICU, the effectiveness of ECC is confirmed by monitoring the arterial pressure trace with a target compression rate and depth to achieve a systolic impulse of > 60 mm Hg to maintain a mean perfusion pressure, preventing ventricular distension, LV wall stress, and ischaemia.

How well did you know this?
1
Not at all
2
3
4
5
Perfectly
315
Q

22.2 Normal (0.9%) saline has the physical properties of
a. Na 140, 280 mOsm/L
b. Na 148, 296 mOsm/L
c. Na 150, 300 mOsm/L
d. Na 154, 308 mOsm/L

A

D Na 154, 308 mOsm/L

How well did you know this?
1
Not at all
2
3
4
5
Perfectly
316
Q

21.2 The most common complication of extracorporeal membrane oxygenation (ECMO) in adults is

a) Bleeding
b) Vascular damage
c) Embolism
d) Inadvertently decannulate

A

a) Bleeding

ECMO complications:
- patient complications: bleeding & coagulopathy most common
- mechanical complications: access insufficiency common

Blue book 2017

How well did you know this?
1
Not at all
2
3
4
5
Perfectly
317
Q

20.1 What is the abnormality in this CXR?

a. Pneumonectomy
b. Pleural effusion
c. Pneumonia
d. Unilateral pulmonary oedema

A

c. Pneumonia

Complete white-out of left lung with air bronchogram sign consistent with total consolidation. Patchy areas of ground glass opacities in right lung. There is silhouetting of the left heart border and left hemi-diaphragm. There is no rib crowding to suggest atelectasis.

Complete white-out of the hemithorax with air bronchograms and trachea in a central position is consistent with consolidation secondary to pneumonia. The patient is on linezolid, micafungin, and piperacillin-tazobactam pending culture and susceptibility studies.

Differential diagnosis of hemithorax white-out with a midline trachea include:
- consolidation
- pulmonary edema/ARDS
- pleural mass
- chest wall mass

How well did you know this?
1
Not at all
2
3
4
5
Perfectly
318
Q

22.2 Of the following, the congenital condition LEAST commonly associated with obstructive sleep apnoea in children is

A

Hypoplastic mandible (micrognathia) – difficult intubation
§ Pierre Robin sequence
§ Treacher Collins
§ Hemifacial microsomia (Goldenhar syndrome)

Midface hypoplasia – difficult bag-mask ventilation
§ Apert syndrome
§ Crouzon syndrome
§ Pfeiffer syndrome
§ Saethre-Chotzen syndrome

Macroglossia – difficult bag-mask ventilation AND difficult intubation
§ Hurler’s/Hunter’s syndrome (mucopolysaccharidoses)
§ Beckwith-Wiedemann syndrome
§ Down’s syndrome

https://www.frca.co.uk/Documents/250%20The%20Difficult%20Paediatric%20Ai

Mucopolysaccharidoses, Down syndrome, muscular dystrophies, and other neurologic disorders have been associated with obstructive sleep apnea

Prevalence of OSAS.
Genetic Disorder Prevalence of OSAS
Neuromuscular diseases 69.2%
Prader–Willi syndrome 94.7%
Arnold–Chiari syndrome 80%
Achondroplasia 100%
Crouzon syndrome 100%
https://www.ncbi.nlm.nih.gov/pmc/articles/PMC8156845/

https://www.frca.co.uk/Documents/250%20The%20Difficult%20Paediatric%20Ai

How well did you know this?
1
Not at all
2
3
4
5
Perfectly
319
Q

(

21.1 A 10-year-old boy (weight 30 kg) has a displaced distal forearm fracture that requires manipulation and application of plaster. The volume of 0.5% lidocaine (lignocaine) that should be used for intravenous regional anaesthesia (Bier block) is

a. 12 ml
b. 18ml
c. 30 ml
d. 42 ml

A

b. 18ml

3mg/kg max dose as per RCH guidelines
3mg x 30kg = 90mg
90mg/5mg/ml = 18ml

or

0.6ml/kg of 0.5% Lignocaine
0.6ml x 30kg = 18ml

https://www.rch.org.au/clinicalguide/guideline_index/Bier_block/

How well did you know this?
1
Not at all
2
3
4
5
Perfectly
320
Q

20.1 A 55-year-old lady scheduled for a transphenoidal hypophysectomy undergoes an oral glucose tolerance test with the following results:

GH normal <10
Time 0, BSL 5.5, GH 30, IGF-1 790 (elevated)
Time 30, BSL 7.6, GH 24
Time 60, BSL 7.2, GH 28
Time 90, BSL 6.5, GH 26
Time 120, BSL 5.8, GH 29

These results are most consistent with a diagnosis of

A. Prolactinoma
B. Acromegaly
C. Cushing’s
D. MEN 2
E. Normal

A

Acromegaly

IGF-2 is consistently elevated

GH should be suppressed by glucose load in healthy
pt.

The continued elevation of GH despite glucose is
suggestive of acromegaly

How well did you know this?
1
Not at all
2
3
4
5
Perfectly
321
Q

23.1 In children, severe sleep apnoea is suggested by an apnoea-hypopnoea index
greater than

a. 10
b. 15
c. 20
d. 30
e. 40

A

a) 10

How well did you know this?
1
Not at all
2
3
4
5
Perfectly
322
Q

21.1 You are performing a regional block for analgesia following knee surgery. You have an ultrasound probe scanning the anterior mid-thigh. The muscle indicated by the arrow in the ultrasound image below is the

A: Sartorius
B: Vastus Medialis
C: Adductor Longus
D: Gracilis
E: Rectus femoris

A

B: Vastus Medialis

How well did you know this?
1
Not at all
2
3
4
5
Perfectly
323
Q

20.2 Elimination of remifentanil occurs following breakdown mainly by

a Plasma cholinesterase
b RBC esterases
c Hoffman degradation
d Hepatic Metabolism
e Plasma esterases

A

e Plasma esterases

Plasma esterases (not cholinesterase)

Esmolol metabolism is via RBC esterases.

How well did you know this?
1
Not at all
2
3
4
5
Perfectly
324
Q

22.1 You are planning to perform a peribulbar block and wish to check the axial length of the eye prior to proceeding. The average axial length of the globe in adults as measured by ultrasound is

a. 20mm
b. 23mm
c. 26mm
d. 29mm
e. 32mm

A

B 23mm

How well did you know this?
1
Not at all
2
3
4
5
Perfectly
325
Q

422.1 The current ANZCA guidelines for preoperative fasting of adult patients state that studies have shown that it is safe to administer

a) unlimited clear fluid 2 hours prior
b) 200ml clear fluid 2 hours prior
c) 300ml clear fluid 2 hours prior
d) 400ml clear fluid 2 hours prior

A

400mls of clear fluids pre op

Safe upper limit - definitely has not not been identified and will vary from patient to patient.

Clear fluids
Water / CHO rich fluids / pulp free fruit juice / clear cordial / black tea and coffee

How well did you know this?
1
Not at all
2
3
4
5
Perfectly
326
Q

20.2 The structure labelled A shows (gastric ultrasound image shown)

a. Empty stomach
b. Full stomach with Solids
c. Full stomach with liquids and Air
d. Gall Bladder
e. Abdominal Aorta

A

c. Full stomach with liquids and Air

How well did you know this?
1
Not at all
2
3
4
5
Perfectly
327
Q

22.1 An asymptomatic 65-year-old male with squamous cell carcinoma of the left lung has been referred for assessment of suitability for lung resection. There is no evidence of spread on computerised tomography scanning. PaCO2, electrocardiogram, full blood count and electrolytes are normal. His SpO2 on room air is 95%. His forced expiratory volume in one second is 2.3 litres (predicted 3.3 litres) and forced vital capacity is 3.4 litres (predicted 4.4 litres). The most appropriate course of action is to

a. Proceed with lobectomy or pneumonectomy
b. Proceed with lobectomy only
c. DLCO testing
d. Lung V/Q scan
e. CPET

A

a. Proceed with lobectomy or pneumonectomy

FEV1 surgical suitability:
- >80% or >2l pneumonectomy
○ no further testing required
- >80% or >1.5l lobectomy
○ no further testing required
- <80% or <2l for pneumonectomy
○ -> calculate ppoFEV1
- <80% or <1.5l for lobectomy
○ -> perform DLCO and express as % of predicted DLCO
○ Saturations on air
- ppoFEV1 < 40% and DLCO <40% = High Risk
- ppoFEV1 >40% and DLCO >40% and SaO2 >90% = Average risk (no further testing)

How well did you know this?
1
Not at all
2
3
4
5
Perfectly
328
Q

21.1 The composition of blood returned to the patient from intraoperative cell salvage shows

A. No evidence of haemolysis
B. Normal 2,3 DPG
C. Nil evidence of bone cement or some embolism type
D. Normal levels of coagulation factors

A

B. Normal 2,3 DPG

higher Hct-60%
No immunimodulation
require reinfusion within 6hrs
pause with sement, caution metal fragments

How well did you know this?
1
Not at all
2
3
4
5
Perfectly
329
Q

22.2 The nerve labelled by the arrow marked J in the diagram is the

  1. Ulnar Nerve
  2. Axillary Nerve
  3. Median Nerve
  4. Medial Cutaneous nerve of the forearm
  5. Long Thoracic Nerve
  6. Dorsal Scapular Nerve
  7. Radial Nerve
  8. Suprascapular nerve
  9. Musculocutaneous Nerve
A
  1. Medial Cutaneous nerve of the forearm
How well did you know this?
1
Not at all
2
3
4
5
Perfectly
330
Q

21.2, 20.1 The muscle or muscle group with the greatest resistance to the action of non-depolarising neuromuscular blocking agents is the

a) Adductor pollicis
b) Diaphragm
c) Orbicularis oculi
d) Pharyngeal

A

b) Diaphragm

How well did you know this?
1
Not at all
2
3
4
5
Perfectly
331
Q

22.2 The normal axial length of the globe of an adult eye is
a. 20mm
b. 23mm
c. 26mm
d. 29mm
e. 32mm

A

23mm

How well did you know this?
1
Not at all
2
3
4
5
Perfectly
332
Q

22.1 The nerve labelled by the arrow in the diagram (image of brachial plexus given) is the

a. Median nerve
b. MC nerve
c. Radial nerve
d. Ulnar nerve

A

Extensive Brachial plexus anatomy questions produced in 2022.2 paper on Brainscape

How well did you know this?
1
Not at all
2
3
4
5
Perfectly
333
Q

21.1 Toxicity of methylene blue is likely to be seen after single bolus dose (in mg/kg) greater than

a. 1mg/kg
b. 2mg/kg
c. 5mg/kg
d. 0.5mg/kg
e. 0.1mg/kg

A

c. 5mg/kg

Methylene blue due to its monoamine oxidase(MAO) inhibiting property may precipitate potentially fatal serotonin toxicity at doses >5mg/kg.

Source: STAT PEARLS - Methylene blue https://www.ncbi.nlm.nih.gov/books/NBK557593/

How well did you know this?
1
Not at all
2
3
4
5
Perfectly
334
Q

20.1 RFTS: Normal ratio, low FVC, low FEV1, Normal DLCO:
a) Sarcoid
b) Myasthenia Gravis
c) Asthma
d) Emphysema

A

b) Myasthenia Gravis

How well did you know this?
1
Not at all
2
3
4
5
Perfectly
335
Q

20.2 Of the following, the agent that causes the LEAST prolongation of the Thrombin Clotting Time (or Thrombin Time) is

a) Heparin
b) LMWH
c) Bivalirudin
d) Warfarin
e) Dabigatran

A

d) Warfarin

Warfarin – no effect on thrombin time
Heparin - causes considerable prolongation of TT.

LMWH, fondaparinux or direct factor Xa inhibitors have no effect on TT as the predominantly inhibit factor Xa.
-> However LMWH in very high concentration can affect TT.

Dabigatran, Bivalirudin and other direct thrombin inhibitors prolong TT considerably.

The thrombin time (TT), also known as the thrombin clotting time (TCT) is a blood test that measures the time it takes for a clot to form in the plasma of a blood sample containing anticoagulant, after an excess of thrombin has been added. Warfarin prevents thrombin synthesis but does not inhibit it, therefore no effect on TT.

How well did you know this?
1
Not at all
2
3
4
5
Perfectly
336
Q

23.1 Following the insertion of a peripherally inserted central catheter (PICC) into the cephalic vein in the upper arm, the patient complains of numbness in their forearm. It is likely that during insertion the operator has injured the

A. Median cutaneous antebrachial
B. Median antebrachial
C. Lateral antebrachial
D. Posterior brachial
E. Posterior cutaneous nerve (of the forearm)

A

c) lateral antebrachial

https://anatomytool.org/content/radiopaedia-drawing-contents-superficial-cubital-fossa-english-labels

double check - no reference

How well did you know this?
1
Not at all
2
3
4
5
Perfectly
337
Q

22.2 In a previously normal patient with cardiac failure secondary to acute pulmonary embolism, the best choice of vasoactive agent for initial treatment is

a. Dobutamine
b. Milrinone
c. Isoprenaline
d. Noradrenaline

A

d. Noradrenaline

Supportive Management of Massive PE

Coexisting left ventricular systolic dysfunction and diastolic dysfunction complicate the management of heart failure patients with massive PE. Although a common strategy in response to systemic arterial hypotension is to prescribe a fluid bolus, volume loading may worsen biventricular failure, pulmonary edema, and hypoxemia. An initial trial of volume expansion, limited to 250 to 500 mL, may be attempted in those heart failure patients without evidence of increased right-sided filling pressures or pulmonary edema.6

Although non–heart failure patients generally respond well to pure vasopressors for hemodynamic support in massive PE, many heart failure patients will not tolerate the isolated increase in systemic vascular resistance. PE patients with heart failure may require an agent with mixed vasopressor and inotropic properties such as norepinephrine, epinephrine, or dopamine. Whereas LV function often becomes hyperdynamic to compensate for RV failure, the presence of underlying LV systolic dysfunction in heart failure patients may limit the patient’s ability to maintain normal systemic cardiac output and may necessitate the addition of inotropes.

https://www.ahajournals.org/doi/10.1161/CIRCULATIONAHA.108.803965

How well did you know this?
1
Not at all
2
3
4
5
Perfectly
338
Q

21.2 A patient has return of spontaneous circulation (ROSC) but remains unresponsive after
cardiac arrest. ANZCOR Guidelines recommend all the following measures EXCEPT

a) Aiming SpO2 94%
b) Treating hyperglycaemia >10mmol/L
c) Normothermia
d) Cardiac catherisation
e) Amiodarone infusion

A

c) Normothermia
- TTM to 32-36 degrees is recommended for those that remain unresponsive

How well did you know this?
1
Not at all
2
3
4
5
Perfectly
339
Q

21.2 A 65 year old woman is dyspnoeic after a total hip replacement. A lung ultrasound is performed in the post-anaesthesia care unit, with a still image shown below. The likely cause of the dyspnoea is

a) Effusion
b) PE
c) Pneumothorax
d) Pneumonia

A

PTx

Commented from example image:
The stratosphere sign. Absent lung sliding on M-mode in a patient with a pneumothorax. Notice the absence of T lines (the lung pulse)

LITFL

The ultrasound appearance of pneumothorax
1. Loss of lung sliding and the movement artefact deep to the pleural line
- A pneumothorax lies deep to the smooth parietal pleural surface.
- The gas interface creates a highly reflective surface reflecting all ultrasound energy.
- This prevents imaging of structures lying below the pneumothorax. The movement of the lung, deep to the pneumothorax is completely hidden – lung sliding is lost.

  1. Loss of characteristic B-lines
    - B-lines (vertical short path reverberation artefacts) are created by alveolar and interstitial fluid or fibrosis at the lung surface.
    - In the same way that pneumothorax hides lung sliding it also hides any B-lines lying below.
  2. Increased clarity of A-lines
    - A-lines (horizontal long path reverberation artefacts) are echogenic horizontal artifactual lines deep to the pleural surface that are characteristic of pneumothorax.
    - The mirror like, flat parietal pleura overlying the pneumothorax reflects the ultrasound which often then reverberates between the pleural surface and other horizontal reflecting surfaces above. These include fascial planes and the transducer surface itself.
    - Multiple reflections cause horizontal linear artefacts mirroring the flat surfaces above the pleural surface, deep to the pleural surface.
  3. Lung Point
    - Pneumothorax separates the visceral and parietal pleural surfaces.
    - The point at which these surfaces meet is known as the lung point
How well did you know this?
1
Not at all
2
3
4
5
Perfectly
340
Q

22.2 A 76-year-old man requires an emergency thoracotomy to treat an expanding haemothorax. He is mildly hypotensive and is not fasted. His plasma electrolytes and haemoglobin are below. The most appropriate strategy to employ to intubate him with a double lumen endotracheal tube is to (use)

K 6.3 Ur 7-ish Cr 174

a. Cisatracurium 0.5mg/kg
b. Rocuronium 1.2mg/kg
c. Suxamethonium 1mg/kg
d. Suxamethonium 0.5mg/kg (?was this an option)

A

b. Rocuronium 1.2mg/kg

Cis not appropriate for intubation

Sux with K 6.3 is risky. (I’ve never heard of reduced dose)

How well did you know this?
1
Not at all
2
3
4
5
Perfectly
341
Q

21.2 In a patient with anaemia of chronic disease, of the following the most likely to be elevated is

a. MCV
b. transferrin saturation
c. Increased soluble Transferrin Receptor
d. Ferritin
e. Total iron binding capacity

A

d. Ferritin

ANZCA blue book:

ACD caused primarily by inflammation

Mechanism:
1. Iron
- Inflammation reduces Iron availabilty as a protective mechanism whereby Iron is sequestered and stored in macrophages to limit availability to microbial pathogens
- Hepcidin expression is increased, this prevents the release of Iron by reticuloendothelial system resulting in “functional iron deficiency” with reduced tissue availability of iron, despite apparently normal total body iron stores. (hence increased Ferritin)

  1. Response to erythropoietin
    - mechanism not clear suspect blunting of response to erythropoietin
  2. Therapeutic agents
    chemotherapies that impair bone marrow response to erythropoiesis
    65% of patients with lung and gynae cancer treated with platinum based drug develop anaemia

RCPA advice on interpretation of Soluble Transferrin Receptor:

Soluble transferrin receptor levels in plasma are elevated if there is increased iron demand due to Iron deficiency, increased erythropoiesis (eg, Haemolysis) or dyserythropoiesis (eg, Megaloblastic anaemia), regardless of other, coexistent states.

Thus, it can be used to demonstrate iron deficiency in patients who also have an acute phase response and it can distinguish Iron deficiency from the Anaemia of chronic disease.

Patients with an acute phase response have reduced plasma iron and transferrin with elevation of Ferritin, making these usual indicators unreliable.

How well did you know this?
1
Not at all
2
3
4
5
Perfectly
342
Q

21.1 You have been asked to anaesthetise a patient with a history of severe depression which has been
well controlled on moclobemide. The most appropriate medications in combination with propofol are

a. Sevoflurane, morphine, phenylephrine
b. Sevoflurane, pethidine, phenylephrine
c. Midazolam, fentanyl, ephedrine
d. sevoflurane, oxycodone, ephedrine

A

a. Sevoflurane, morphine, phenylephrine

Moclobemide = MAOi

How well did you know this?
1
Not at all
2
3
4
5
Perfectly
343
Q

23.1 Desufflation after surgical pneumoperitoneum is NOT associated with an increase in

a) SVR
b) CI
c) EF
d) preload
e) LV work

A

a) SVR

How well did you know this?
1
Not at all
2
3
4
5
Perfectly
344
Q

When providing anaesthesia for endovascular treatment of acute ischaemic stroke, the Society of NeuroInterventional Surgery and the Neurocritical Care Society recommend

A. General anaesthesia
B. Hypervolaemia
C. Maintain temp <35
D. Maintain BGL 8-12
E. Maintain sBP 140-180

A

E. Maintain sBP 140-180

https://journals.lww.com/jnsa/Fulltext/2014/04000/Society_for_Neuroscience_in_Anesthesiology_and.1.aspx

We recommend that hemodynamic monitoring and management, as outlined below, should be started as soon as diagnosis of AIS has been made (class IIa, level of evidence C).

Heart rate and cardiac rhythm should be monitored continuously and blood pressure should be monitored continuously or measured at least once every 3 minutes.

We recommend that systolic blood pressure should be maintained >140mm Hg (fluids and vaso- pressors) and <180mm Hg (with or without IV tPA), and diastolic blood pressure <105 mm Hg (class IIa, level of evidence B).
Cause of hypotension should be investigated (volume depletion, myocardial infarction, cardiac arrhythmia, blood loss, retroperitoneal hemorrhage, and aortic dissection) and treated if possible.

We also recommend that blood pressure targets may be adjusted (lowered) in communication with the neuro- interventionalists and neurologists following successful recanalization of occluded vessel(s) (class IIb, level of evidence C), as reperfused brain often lacks autor- egulation leading to high risk of hyperperfusion leading potentially to hemorrhagic conversion.

How well did you know this?
1
Not at all
2
3
4
5
Perfectly
345
Q

20.1 Infrarenal aortic cross clamp will cause a(an):

a) Increased by 40% renal blood flow
b) Increased by 20% renal blood flow
c) Unchanged renal blood flow
d) Decreased 20% renal blood flow
e) Decreased 40%renal blood flow

A

e) Decreased 40%

Infra-renal aortic cross-clamping leads to a reduction in renal blood flow by up to 40%, as a result of an increase in renal vascular resistance of up to 75%.

The mechanism underlying this increased resistance is uncertain but may, in part, be a result of the associated decrease in cardiac output during aortic cross-clamping, as well as because of humoral mechanisms, which lead to increased release of renin. After declamping, there is a maldistribution of renal blood flow away from the cortex for at least 60 min.

How well did you know this?
1
Not at all
2
3
4
5
Perfectly
346
Q

21.2 A bleeding patient has ROTEM results including (results displayed) . The most appropriate treatment is

a) Cryoprecipitate
b) FFP
c) Platelets
d) TXA
e) Protamine

A

e) Protamine

The interpretation of this graph is not especially laborious. The cardinal abnormality is the massively prolonged CT and CF of the INTEM graph, which suggests that something has killed the intrinsic pathway of the clotting cascade. The CT returns to normal in the HEPTEM graph, which is essentially just an INTEM test with adde heparinase. The presence of heparinase seems to have reversed all of the coagulopathy - the CFT, alpha-angle and MCF have all returned to normal. Therefore, this patient has no coagulation problems other than the heparin.

https://derangedphysiology.com/main/required-reading/haematology-and-oncology/Chapter 1.2.0.1/intepretation-abnormal-rotem-data

How well did you know this?
1
Not at all
2
3
4
5
Perfectly
347
Q

22.1 Of the following, the drug most likely to cause pulmonary arterial vasodilation with systemic arterial vasoconstriction when used in low doses is

a) Adrenaline
b) Noradrenaline
c) Vasopressin
d) Dopamine
e) Dobutamine

A

c) Vasopressin

https://emcrit.org/ibcc/pressors/

  • From UP TO DATE:
    > At low doses of 1 to 3 mcg/kg per min, dopamine acts primarily on dopamine-1 receptors to dilate the renal and mesenteric artery beds
    > At 3 to 10 mcg/kg per min (and perhaps also at lower doses), dopamine also stimulates beta-1 adrenergic receptors and increases cardiac output, predominantly by increasing stroke volume with variable effects on heart rate.
    > At medium-to-high doses, dopamine also stimulates alpha-adrenergic receptors, although a small study suggested that renal arterial vasodilation and improvement in cardiac output may persist as the dopamine dose is titrated up to 10 mcg/kg per min
    *clinically, the haemodynamic effects of dopamine demonstrate individual variability

Dobutamine (inodilator):
- selective β1-agonist that increases cardiac contractility and reduces pulmonary vascular and systemic vascular resistances

Vasopressin:
- vasopressin may have pulmonary vasodilatory effects in addition to a systemic vasoconstrictive effect

Milrinone (inodilator):
- the phosphodiesterase-3 inhibitors, milrinone and enxoimone, have positive inotropic effects combined with the capacity to reduce RV afterload (‘inodilators’) without significant chronotropic effect, but they can be associated with significant systemic hypotension

How well did you know this?
1
Not at all
2
3
4
5
Perfectly
348
Q

bonus neuro radiology questions

A 34 year-old woman developed the lightning onset of an explosive headache followed by nausea and vomiting.
She undergoes cerebral angiography.
Her cerebral aneurysm is in the

a. Anterior Cerebral Artery
b. Anterior communicating artery
c. MCA
d. PCA
e. Basilar artery

A

d. PCA

https://case.edu/med/neurology/NR/SubarachnoidHemorrhag3%20Pcom.htm

How well did you know this?
1
Not at all
2
3
4
5
Perfectly
349
Q

23.1 A nerve that does NOT provide sensory innervation to the shoulder joint is the

A. Axillary
B. Lateral pectoral
C. Subscapular
D. Supraclavicular
E. Suprascapular

A

d) Supraclavicular

Axillary nerve innervates skin to inferior deltoid (regimental badge)+ motor to terres minor and deltoid.

Lateral pectoral nerve innervates the anterosuperior part of the glenohumeral joint.

Subscapular nerves - upper subscapular nerve serves the upper portion of the subscapularis muscle; the middle subscapular nerve (thoracodorsal nerve) innervates latissiumus dorsi; lower subscapular nerve innervates subscapularis and terres major.

Supraclavicular nerve - sensory only and innervates skin across entire shoulder and trapezius in a ‘cape-like’ fashion - sometimes missed in interscalene block.

Suprascapular nerve sensory innervation to glenohumeral joint and acromiovlavicular joint + motor to supraspinatus/infraspinatous (rotator cuff)

https://pubmed.ncbi.nlm.nih.gov/32712453/

How well did you know this?
1
Not at all
2
3
4
5
Perfectly
350
Q

20.2 A patient with a purely metabolic acidosis has a serum bicarbonate of 14 mmol/L and a lactate of 3.8 mmol/L. The expected PaCO2 is

A

29mmHg

How well did you know this?
1
Not at all
2
3
4
5
Perfectly
351
Q

20.1 Patient with Fontan circulation and peritonism having induction for laparotomy. Drops sats on induction. Best move?

a. Decrease volatile
b. Reverse Trendelenberg
c. Decrease FiO2
d. Increase PEEP
e. Increase tidal volume

A 22-year-old man with a Fontan circulation is on your emergency list for an appendicectomy. He has had abdominal pain and vomiting for 3 days, and has a peritonitic abdomen. His preoperative arterial oxygen saturation is 95%. Shortly after induction he becomes hypotensive BP 80/45, and saturations fall to 75%. His condition is most likely to be improved by:

A. Increasing the inspiratory time.
B. Decreasing the ventilator tidal volumes.
C. Adding positive end-expiratory pressure (PEEP).
D. Positioning reverse trendelenberg.

A

A

Couldn’t find a clear source but we know;

A - will decrease venoplegia and improve venous return
B - Would not help, decrease VR
C - Don’t drop FiO2 when desatting…
D - increases PVR (unless below FRC) and reduces pulmonary flow
E - Same as above, increased PVR and reduces flow through pulmonary circuit

B. Decreasing the ventilator tidal volumes.

Patients who have undergone the Fontan procedure depend on blood flow through the pulmonary circulation without the assistance of the right ventricle. The difference between central venous pressure and systemic ventricular end-diastolic pressure (termed the “transpulmonary gradient”) is the primary force promoting pulmonary blood flow and, more importantly, cardiac output.

Circulation in the Fontan patient is promoted by low pulmonary vascular resistance. Positive-pressure ventilation with increased tidal volumes, as described above, can result in excessive intrathoracic pressures, leading to decreased venous return to the heart and increased pulmonary vascular resistance.
In periods of low oxygen saturation, 100% inspiratory oxygen is appropriate.
The addition of PEEP will increase intrathoracic pressure, reducing venous return.
Trendelenberg positioning would increase CVP and therefore bloodflow through pulmonary circulation.

How well did you know this?
1
Not at all
2
3
4
5
Perfectly
352
Q

22.2 Suxamethonium may be safely given to patients with (list of neuromuscular diseases given)

a. Becker muscular dystrophy
b. Myaesthenia gravis (new option)
c. Guillain Barre
d. Hypokalaemic periodic paralysis (new option)
e. Duchenne muscular dystrophy

A

b. Myaesthenia gravis

ED95 is 0.8mg/kg in a MG patient

How well did you know this?
1
Not at all
2
3
4
5
Perfectly
353
Q

20.1 The flow volume loop is most consistent with

a) Variable intra-thoracic obstruction
b) Variable extra-thoracic obstruction
c) Fixed large airway
d) Restrictive lung pattern
e) Mixed pattern

A

b) Variable extra-thoracic obstruction

How well did you know this?
1
Not at all
2
3
4
5
Perfectly
354
Q

21.1 A common electrolyte disturbance following the administration of ferric carboxymaltose is

a. hypophosphatemia
b. hypocalicaemia
c. hypokalaemia
d. hypercalicaemia
e. hypernatraemia

A

Hypophosphataemia

Ferric carboxymaltose (Ferinject) for iron-deficiency anaemia
https://www.nps.org.au/radar/articles/ferric-carboxymaltose-ferinject-for-iron-deficiency-anaemia

In this set of patients administered FCM (n = 5799), treatment-related side effects that occurred in more than 1% of the group included:
- nausea (3.1%)
- hypophosphataemia (1.9%)
- injection-site reactions (1.6%)
- headache (1.4%)
- hypertension (1.3%)
- dizziness (1.2%)

How well did you know this?
1
Not at all
2
3
4
5
Perfectly
355
Q

22.2 You will anaesthetise a 39-year-old woman for a laparoscopic cholecystectomy. She has a history of mastocytosis and has never had an anaesthetic in the past. A drug which you should avoid is
a. fentanyl
b. morphine
c. remifentanil
d. tramadol

A

B Morphine

Histamine-releasing

How well did you know this?
1
Not at all
2
3
4
5
Perfectly
356
Q

21.1 A four-year-old boy with a history of waddling gait, larger than normal calves and frequent falls receives a spontaneously breathing volatile-based anaesthetic with sevoflurane.

One hour into the case he develops peaked T waves and then the end-tidal CO2 begins to rise. The most appropriate immediate treatment is to

a. Temp probe, and go from there
b. Cool + dantrolene
c. Stop volatile, cool + dantrolene
d. Stop volatile, calcium
e. Stop volatile

A

d. Stop volatile, calcium

?Duchenne muscular dystrophy?
This patient most likely has Anaesthesia Induced Rhabdomyolysis (AIR) given the peaked Twaves and slow rise in ETCO2

Immediate MH Management:
Stop administering Sevo, flush machine (or new), charcoal filters. Dantrolene.

How well did you know this?
1
Not at all
2
3
4
5
Perfectly
357
Q

23.1 Expected features of Guillain-Barré syndrome include

A. Descending paralysis
B. Flaccid paralysis
C. Unilateral leg weakness

A

b) flaccid paralysis

Guillain–Barré syndrome (GBS) is an inflammatory disease of the PNS and is the most common cause of acute flaccid paralysis

How well did you know this?
1
Not at all
2
3
4
5
Perfectly
358
Q

22.1 When using cardioversion to revert a patient in atrial fibrillation to sinus rhythm, the direct current shock is synchronised with the ECG to coincide with the

a. Start of R wave
b. Start of Q wave
c. Middle of T wave
d. peak of R wave

A

R-wave peak

How well did you know this?
1
Not at all
2
3
4
5
Perfectly
359
Q

21.1 A 25-year-old man suffers a 30% total body surface area burn. A cardiovascular physiological change expected within the first 24 hours is

a. Decreased PVR
b. Increased SVR
c. Decreased SVR
d. Reduced PA pressure
e. Increased hepatic blood flow

A

increased SVR

EMSB handbook
CO is reduced after Burn injury 2ry to:
- myocardial depressant mediators
- decreased blood volume
- reduced venous return
- increased pulmonary and systemic vascular resistance due to increased levels of catecholamines

In the first 24hrs reduced cardiac output persists even after restoration of blood volume

Between 24-48hrs post burn a hyperdynamic state develops with reduced peripheral resistance, increased oxygen consumption and increased cardiac output

How well did you know this?
1
Not at all
2
3
4
5
Perfectly
360
Q

20.2 The part of the lung that is typically divided into superior and inferior segments is the

a) RUL
b) RML
c) RLL
d) LUL
e) Left lingula

A

e) Left lingula

LEFT LUNG: ASIA ALPS
Apical Posterior
Superior lingula
Inferior lingula
Apical Anterior

Anterior basal
Lateral basal
Posterior basal
Superior

RIGHT LUNG: A PALM Seed Makes Another little Palm

RUL:
Apical
Posterior
Anterior

RML:
Lateral
Medial

RLL
Superior
Medial basal
Anterior basal
Lateral basal
Posterior basal

How well did you know this?
1
Not at all
2
3
4
5
Perfectly
361
Q

20.2 The ANZCA guidelines regarding pre-operative oral intake for infants under 6 months of age having an elective procedure under anaesthesia are

a) Breast milk 2 hours before, clear fluids 1 hour before to max 3ml/kg
b) Breast milk 2 hours, clear fluids 1 hour before to max 5ml/kg
c) Breast milk 3 hours, clear fluids 1 hour to max 3ml/kg
d) Breast milk 3 hours, clear fluid 1 hour to max 5ml/kg
e) Breast milk 4 hours, clear fluids 1 hour to max 3ml/kg

A

Repeat

c) Breast milk 3 hours, clear fluids 1 hour to max 3ml/kg

Infants <6 months having elective procedure
* 4 hours for formula
* 3 hours for breast milk
* 1 hour for clear fluids (≤3 ml/kg/hr)

Children > 6 months having elective procedure
* 6 hours for limited solid food or formula
* 4 hours for breast milk
* 1 hour for clear fluids (≤ 3ml/kg/hr)

How well did you know this?
1
Not at all
2
3
4
5
Perfectly
362
Q

20.2 This lung ultrasound shows

a) Normal lungs
b) Pulmonary odema
c) Pneumothorax
d) Pleural effusion
e) Pneumonia

A

b) Pulmonary oedema

B-lines

> Vertical echogenic short path reverberation artefacts originating at the pleural line and extending to the deepest part of the ultrasound image.
They interrupt any horizontal A-lines.
Occasional B-lines are considered normal.
More than 3 B-lines in any single view is considered pathological.
Where there are numerous B-lines in close proximity they become confluent.
B-lines move with lung movement.
They are caused by ultrasound energy reverberating in a fluid filled focus that is surrounded by air. These foci may be interstitial or alveolar.
Cardiogenic and noncardiogenic oedema may have very similar appearances.
Interstitial thickening due to fibrosis or lymphangitis can also create the sonographic appearance of diffuse B-lines.

How well did you know this?
1
Not at all
2
3
4
5
Perfectly
363
Q

23.1 Pulse pressure variation is defined as

a. 100x SBP max - SBP min / SBP min
b. 100 x PPmax - PPmax / PPmin
c. 100x SBP max - SBP min/ SBP mean
d. 100 x PPmax - PPmin / PPmean

A

d) 100 x PPmax - PPmin / PPmean

How well did you know this?
1
Not at all
2
3
4
5
Perfectly
364
Q

20.1 What is the arrow pointing to?

a. Ilioinguinal
b. Iliohypogastric
c. Iliacus
d. Lateral Femoral Cutaneous
e. Obturator

A

d. Lateral Femoral Cutaneous

How well did you know this?
1
Not at all
2
3
4
5
Perfectly
365
Q

21.1 Perioperative overheating is most likely to cause worsening of symptoms of

a) Duchenne Muscular dystrophy
b) Myasthenia gravis
c) Multiple sclerosis
d) Myotonica dystrophia
e) Eaton Lambert syndrome

A

multiple sclerosis.

https://academic.oup.com/bjaed/article/11/4/119/266998
Anaesthetic considerations.
- Local anaesthetics may exacerbate symptoms due to the increased sensitivity of demyelinated axons to local anaesthetic toxicity.
- Non-depolarizing neuromuscular blocking agents may be used in normal doses.
Caution should be exercised when using depolarizing neuromuscular blocking agents if the patient is debilitated.
- Temperature maintenance is important as symptoms can deteriorate with an increase in temperature, as demyelinated axons are also more sensitive to heat.

How well did you know this?
1
Not at all
2
3
4
5
Perfectly
366
Q

22.2 The 2012 Berlin definition of the acute respiratory distress syndrome (ARDS) defines moderate disease as one with a PaO2 / FiO2 ratio (in mmHg) of

a) 50-100
b) 100-200
c) 200-300
d) 300-400

A

a) 100-200

2012 BERLIN DEFINITION OF ARDS

ARDS is an acute diffuse, inflammatory lung injury, leading to increased pulmonary vascular permeability, increased lung weight, and loss of aerated lung tissue…[with] hypoxemia and bilateral radiographic opacities, associated with increased venous admixture, increased physiological dead space and decreased lung compliance.

Key components
- acute, meaning onset over 1 week or less
- bilateral opacities consistent with pulmonary edema must be present and may be detected on CT or chest radiograph
- PF ratio <300mmHg with a minimum of 5 cmH20 PEEP (or CPAP)
- “must not be fully explained by cardiac failure or fluid overload,” in the physician’s best estimation using available information — an “objective assessment“ (e.g. echocardiogram) should be performed in most cases if there is no clear cause such as trauma or sepsis.

Severity
- ARDS is categorized as being mild, moderate, or severe:

How well did you know this?
1
Not at all
2
3
4
5
Perfectly
367
Q

20.2 When performing an infraclavicular block of the brachial plexus under ultrasound guidance, the structure indicated by the arrow is the (ultrasound image shown

a) Posterior Cord
b) Lateral cord
c) Median nerve
d) Superior trunk
e) Inferior trunk

A

a) Posterior Cord

How well did you know this?
1
Not at all
2
3
4
5
Perfectly
368
Q

20.2 This ECG rhythm strip represents

A
How well did you know this?
1
Not at all
2
3
4
5
Perfectly
369
Q

22.1 A patient presents for endoscopic retrograde cholangiopancreatography (ERCP) with a history of previous post-ERCP pancreatitis. The management most likely to reduce the likelihood of pancreatitis is

a. Gentamicin
b. PR indomethacin
c. Creon post op
d. Preop smoking cessation

A

Rectal indomethacin

APMSE 5th edition 8.6.1.3: Only rectal NSAIDs are effective for reducing post ERCP pancreatitis, particularly indomethacin. Epidural > PCA for severe acute pancreatitis

How well did you know this?
1
Not at all
2
3
4
5
Perfectly
370
Q

22.2 Of the following, all are useful for the treatment of status epilepticus EXCEPT

a. Calcium
b. isoflurane
c. ketamine
d. propofol
e. phenytoin

A

a. Calcium
(unless hyppocalcaemia is causing your seizures)

Deranged Physiology:
First line agents
- Benzodiazepines: boluses every 2-5 minutes
- Phenytoin: 20mg/kg loading dose
Phenytoin on its own is useless. Or rather, it is inferior to benzodiazepines as a solitary agent. Always, both must be used simultaneously.

Second line agents
- Midazolam infusion
- Phenytoin (well, rather, the American study recommends fosphenytoin)
- Phenobarbital and levetiracetam are also in this second line of attack

Third line agents: for refractory status epilepticus
- Propofol infusion, or midazolam infusion, or thiopentone infusion.
- At this stage, continuous EEG monitoring becomes mandatory
- The role of traditional antiepileptic drugs is also exhausted at this stage, as there will probably be no benefit from adding them into a situation where a constantly observed burst suppression is already achieved by high dose anaesthetic infusion.

Fourth line agents: for these, there is little evidence.
- Volatile anaesthetic agents
- Desflurane and Isoflurane
- Ketamine
- Lignocaine
- Magnesium
- Pyridoxine

Fifth line therapies:
- Hypothermia
- Ketogenic diet
- Deep brain stimulation
- Surgical management

How well did you know this?
1
Not at all
2
3
4
5
Perfectly
371
Q

20.1 A 22-year-old patient is scheduled for resection of a large extra-adrenal paraganglionoma. The tumour is secreting metanephrine. The most likely therapy to be commenced at the preassessment clinic prior to surgery is

a) Prazocin
b) Phentolamine
c) Magnesium
d) Phenoxybenzamine
e) Ca channel blocker

A

Phenoxybenzamine

UpToDate
Phenoxybenzamine​ is the preferred drug for preoperative preparation to control blood pressure and arrhythmia in most centers in the United States. It is an irreversible, long-acting, nonspecific alpha-adrenergic blocking agent.
With their more favorable side-effect profiles and lower financial cost, selective alpha-1-adrenergic blocking agents (eg, ​prazosin​, t​ erazosin​, or d​ oxazosin​) are utilized in many centers or are preferred to ​phenoxybenzamine​ when long-term pharmacologic treatment is indicated (eg, for metastatic pheochromocytoma).

How well did you know this?
1
Not at all
2
3
4
5
Perfectly
372
Q

22.2 AA 15-year-old patient with a known prolonged QT interval has a ventricular tachyarrhythmia while being monitored postoperatively in the postanaesthesia care unit. The patient is alert, orientated and without chest pain but feels unwell. The best initial management is

A. Magnesium
B. Synchronised shock
C. Amiodarone
D. Adenosine
E. Metoprolol

A

A. Magnesium TdP

UTD - BB specifically propanolol or nadolol
if Hx of syncope / seizures or resus SCA

https://www.uptodate.com/contents/congenital-long-qt-syndrome-treatment

How well did you know this?
1
Not at all
2
3
4
5
Perfectly
373
Q

23.1 A technique which is NOT effective in providing analgesia for a sternal fracture is a

A. Pecs 1
B. Pecs 2
C. Thoracic transversus plane block
D. Subpectoral fascial plane block

A

A. Pecs 1

https://www.nysora.com/topics/regional-anesthesia-for-specific-surgical-procedures/thorax/pectoralis-serratus-plane-blocks/

How well did you know this?
1
Not at all
2
3
4
5
Perfectly
374
Q

21.2 The power board on the back of the anaesthesia machine has caught fire during an elective case. This should be extinguished with

a) CO2
b) Fire blanket
c) Wet chemical powder
d) Foam
e) Water

A

a) CO2

How well did you know this?
1
Not at all
2
3
4
5
Perfectly
375
Q

22.2 Non-anaesthetist practitioners wishing to provide procedural sedation should have training in sedation and/or anaesthesia for a minimum of

a) 6 weeks
b) 3 months
c) 6 months
d) 12 months

A

b) 3 months

ANZCA PS09 2014

NB: PG09 was updated in 2022 and no longer states a minimum timeframe, so this is unlikley to return as an MCQ

How well did you know this?
1
Not at all
2
3
4
5
Perfectly
376
Q

20.1 Which drug not metabolised by CYP2D6?
a) Oxycodone
b) Tramadol
c) Amitryptiline
d) Codeine
e) Hydromorphone

A

e) Hydromorphone

How well did you know this?
1
Not at all
2
3
4
5
Perfectly
377
Q

22.2 In Australia and New Zealand, the proportion of blood donors who are cytomegalovirus (CMV) seropositive is
(rough numbers in the options, can’t remember exactly)

a. 65 to 85 per million
b. 650 to 850 per million
c. 6.5 to 8.5 per hundred
d. 65 to 85 per hundred

A

d. 65 to 85 per hundred

85% of australians are CMV positive by the age of 40

https://www.blood.gov.au/system/files/documents/cmv-blood-components.pdf

How well did you know this?
1
Not at all
2
3
4
5
Perfectly
378
Q

22.1 The underlying trigger for the development of acute traumatic coagulopathy is

a. Acidosis
b. Hypothermia
c. Endothelial damage from ischaemia
d. Dilution of coagulation factors from resuscitation
e. Activation of fibrinolysis

A

Endothelial damage due to ischaemia

How well did you know this?
1
Not at all
2
3
4
5
Perfectly
379
Q

21.1 Major international guidelines recommend maintaining the core body temperature between 32°C and
36°C in comatose patients after

A. SAH
B. Stroke
C. Cardiac Arrest

A

Cardiac Arrest

Source: LITFL

How well did you know this?
1
Not at all
2
3
4
5
Perfectly
380
Q

23.1 For a woman who has a history of preeclampsia in a previous pregnancy, the intervention with the best evidence for prevention of preeclampsia during future pregnancies is

A. Aspirin 150mg daily (option was definitely 150mg not 100mg)
B. Mg
C. Heparin subcut
D. Ca

A

A. Aspirin 150mg daily (option was definitely 150mg not 100mg)

or

D. Ca

Aspirin should be given at a dose between 75 and 150 mg per day, started preferably before 16 weeks, possibly taken at night, and continued until delivery;

https://www.somanz.org/content/uploads/2023/06/SOMANZ_Hypertension_in_Pregnancy_Guideline_2023.pdf

Calcium supplementation (1.5g/day) should therefore be offered to women with moderate to high risk of preeclampsia, particularly those with a low dietary calcium intake (247)

How well did you know this?
1
Not at all
2
3
4
5
Perfectly
381
Q

22.1 A two-year-old boy with a history of respiratory tract infection one week previously has just undergone squint surgery. His airway was managed with a size 4.5 mm cuffed endotracheal tube.
The surgery was unremarkable. Twenty minutes after extubation he is awake and appears anxious, with stridor and a visible tracheal tug. His oxygen saturation is 96% on room air. The best initial management of this child is to administer

a) Dexamethasone 0.6mg/kg
b) Adrenaline nebulised 1:1000 - 0.5mL/kg
c) CPAP + T piece
d) Drugs for re-intubation

A

Nebulised Adrenaline
1mg
0.5ml/kg of 1:1000 Adrenaline nebulised
once adrenaline given consider dose of Steroid dexamethasone or hydrocortisone

How well did you know this?
1
Not at all
2
3
4
5
Perfectly
382
Q

22.1 Abnormal Q waves are NOT a feature of the electrocardiogram in

A. Digitalis toxicity
B. LBBB
C. Recent transmural MI
D. Wolff-Parkinson-White
E. Previous MI

A

A. Digitalis toxicity

Miller’s
The ECG made easy
http://lifeinthefastlane.com/ecg-library/pmi/

Normal Q waves
- Due to depolarisation of the interventricular septum from left to right
- Seen in the left-sided leads (I, aVL, V5, V6)

Pathological Q waves
- > 1 mm depth
- > 1 mm (= 40 ms) across

Digoxin ECG changes
- Therapeutic: prolonged PR interval (AV nodal delay), shortened QTC intervals (rapid ventricular repolarisation), ST depression (↓ slope of phase 3), T wave inversion
- Toxic: atrial or ventricular arrhythmias (↑ automaticity), prolonged PR interval → heart block, SA node inhibition → sinus arrest
- Atrial tachycardia with block = most common arrhythmia attributed to digoxin toxicity
- VF = most frequent cause of death
- QRS = normal!

Q waves in MI
- Occur with transmural infarctions, and are less likely with subendocardial infarctions
- Develops days after the onset of AMI, and is usually permanent
- Indicates the part of the heart that has been damaged

LBBB ECG changes
- Wide QRS
- Wide QS complex in lead V1
- Wide R wave in lead V6 with slight notching at the peak and TWI
- The axis is highly variable: can be normal or deviated to the left or right

Wolff-Parkinson-White syndrome
- Due to the presence of an accessory bundle between the atrium and ventricle, which has no AV node to delay conduction
- Short PR interval
- Early slurred upstroke of the QRS complex due to delta wave
- Reciprocal leads will have Q wave like appearance

How well did you know this?
1
Not at all
2
3
4
5
Perfectly
383
Q

23.1 The parameter that changes most with increasing age in the otherwise normal lung is the

a. Closing capacity
b. Residual volume
c. FRC
d. Lung capacity.

A

a) Closing capacity

see graph in Millers

How well did you know this?
1
Not at all
2
3
4
5
Perfectly
384
Q

21.2 The equipment shown in the picture below is a

a) Parker flex tip
b) Hunsaker
c) Laryngectomy tube
d) NIM tube
e) Reinforced tube

A

NIM tube: Neural Integrity Monitor Electromyogram Tracheal Tube

How well did you know this?
1
Not at all
2
3
4
5
Perfectly
385
Q

22.2 When using an endotracheal tube in an adult, the highest recommended cuff pressure to avoid mucosal ischaemia is

a. 10cmH2O
b. 20
c. 30
d. 40
e. 50

A

c. 30cmH2O

paeds 20cmH20

How well did you know this?
1
Not at all
2
3
4
5
Perfectly
386
Q

22.1 A patient with a haemopneumothorax has a chest drain in situ, which is attached to a three-bottle underwater seal drain apparatus. The system is attached to wall suction at -80 cmH20. This will cause

a) Failure of underwater seal
b) Water in suction chamber will enter drainage chamber
c) Reexpansion of haemopneumothorax
d) Oscillation in tube will diminish
e) Inability for stuff to drain into first bottle

A

Oscillations in the tube will be diminished

How well did you know this?
1
Not at all
2
3
4
5
Perfectly
387
Q

20.1 According to the Australian and New Zealand Resuscitation Guidelines the immediate treatment for an adult conscious victim with a severe airway obstruction due to a foreign body inhalation is

a) a single back blow
b) two back blow
c) up to 5 blows to back, then up to 5 chest thrusts
d) up to 5 chest thrusts
e) sweep mouth

A

c) 5 back, 5 chest

4.1 Assess Severity
The simplest way to assess severity of a FBAO is to assess for effective cough.

4.2 Effective Cough (Mild Airway Obstruction)
The person with an effective cough should be given reassurance and encouragement to keep coughing to expel the foreign material. If the obstruction is not relieved the rescuer should call an ambulance.

4.3 Ineffective Cough (Severe Airway Obstruction) Conscious person
If the person is conscious send for an ambulance and perform up to five sharp, back blows with the heel of one hand in the middle of the back between the shoulder blades. Check to see if each back blow has relieved the airway obstruction. The aim is to relieve the obstruction with each blow rather than to give all five blows. An infant may be placed in a head downwards position prior to delivering back blows, i.e. across the rescuer’s lap [Class B; LOE IV].1,2
If back blows are unsuccessful the rescuer should perform up to five chest thrusts. To perform chest thrusts, identify the same compression point as for CPR and give up to five chest thrusts. These are similar to chest compressions but sharper and delivered at a slower rate. The infant should be placed in a head downwards and on their back across the rescuer’s thigh, while children and adults may be treated in the sitting or standing position [Class B; LOE IV].1,2
With each chest thrust, check to see whether the airway obstruction has been relieved. The aim is to relieve the obstruction rather than deliver all five chest thrusts. If the obstruction is still not relieved and the person remains responsive, continue alternating five back blows with five chest thrusts.

Unconscious person
If the person becomes unresponsive a finger sweep can be used if solid material is visible in the airway.1,2 [Class A; LOE IV] Call an ambulance and start CPR.

How well did you know this?
1
Not at all
2
3
4
5
Perfectly
388
Q

22.2 Which of the following risk factors for preeclampsia in isolation would be sufficient to recommend commencing low-dose aspirin?

a. Age >40
b. >10 years since last pregnancy
c. Family hx of pre eclampsia
d. autoimmune disease
e. BMI >35

A

d. autoimmune disease (with potential vascular complications)

RANZCOG

Maternal characteristics that are associated with an increased likelihood of pre-eclampsia include:
- previous pre-eclampsia, particularly when more serious or early onset before 34 weeks
- pre-existing medical conditions (including chronic hypertension, underlying renal disease, or pre-gestational diabetes mellitus),
- underlying antiphospholipid antibody syndrome,
- multiple pregnancy

UTD: Preeclampsia: Prevention
https://www.uptodate.com/contents/preeclampsia-prevention

Based on the available data (see ‘Evidence of efficacy’ above), we recommend low-dose aspirin prophylaxis for women at high risk for preeclampsia. There is no consensus on the exact criteria that confer high risk. It is reasonable to use the US Preventive Services Task Force (USPSTF) high-risk criteria, which are also endorsed by the American College of Obstetricians and Gynecologists (ACOG).

The incidence of preeclampsia is estimated to be at least 8 percent for pregnant women with any one of these high risk factors:
●Previous pregnancy with preeclampsia, especially early onset and with an adverse outcome
●Multifetal gestation
●Chronic hypertension
●Type 1 or 2 diabetes mellitus
●Chronic kidney disease
●Autoimmune disease with potential vascular complications (antiphospholipid syndrome, systemic lupus erythematosus)

How well did you know this?
1
Not at all
2
3
4
5
Perfectly
389
Q

23.1 This Doppler trace obtained by transoesophageal echocardiography of the descending aorta suggests

a. AS
b. AR

A

b. AR

https://litfl.com/oesophageal-doppler/

How well did you know this?
1
Not at all
2
3
4
5
Perfectly
390
Q

20.1 What is the arrow pointing to?

a. Psoas
b. Femoral
c. Genitofemoral
d. Lateral cutaneous nerve of thigh
e. Lumbosacral trunk

A

b. Femoral

How well did you know this?
1
Not at all
2
3
4
5
Perfectly
391
Q

22.2 A patient has return of spontaneous circulation (ROSC) but remains unresponsive after cardiac arrest. ANZCOR Guidelines recommend all the following measures EXCEPT

a) Titrating FiO2 for SpO2 94-98%
b) Treating hyperglycaemia >10mmol/L
c) Targeted temp management at 32-36 degrees
d) Maintaining MAP >70

A

d) Maintaining MAP >70

Recommends maintaining equal or greater than pts usual, or at least a SBP> 100mmHg

https://www.resus.org.nz/assets/Uploads/ANZCOR-Guideline-11.7-Jan16.pdf

How well did you know this?
1
Not at all
2
3
4
5
Perfectly
392
Q

23.1 A multitrauma patient is being managed with a resuscitative endovascular balloon occlusion device of the aorta (REBOA) as part of damage control resuscitation. The
balloon has been inserted for intractable pelvic bleeding. The most appropriate location for the device placement is between the

A. Between artery of adamkiewicz to coeliac artery
B. Between coeliac artery to renal artery
C. Between lowest renal artery to bifurcation of aorta
D. Between coeliac and bifurcation

A

C. Between lowest renal artery to bifurcation of aorta

https://litfl.com/reboa-in-resuscitation/

Anatomy:
The aorta is divided into three separate zones for the purposes of REBOA (aortic length varies between individuals)

Zone I of the aorta extends from the origin of the left subclavian artery to the coeliac artery (approx 20cm long in a young adult male)
Zone II extends from the coeliac artery to the most caudal renal artery (approx 3cm long)
Zone III extends distally from the most caudal renal artery to the aortic bifurcation (approx 10cm long)

REBOA location based on injury:

> suspected or diagnosed intra-abdominal haemorrhage due to blunt trauma or penetrating torso injuries (Zone I REBOA), or

> blunt trauma patients with suspected pelvic fracture and isolated pelvic haemorrhage (Zone III REBOA), or

> patients with penetrating injury to the pelvic or groin area with uncontrolled haemorrhage from a junctional vascular injury (iliac or common femoral vessels) (Zone III REBOA)

Simplistic rendering of aorta. Zone 1 (from left subclavian artery to the upper border of the celiac trunk), Zone 2 (the upper border of the celiac trunk to the lower border of the distal take-off of the renal arteries), and Zone 3 (from the lower border of the lower renal artery to the aortic bifurcation). Zone 1 is occluded in the case of cardiac arrest or life-threatening intra-abdominal hemorrhage; Zone 2 has no current indication; and Zone 3 is occluded in the case of life-threatening pelvic or lower limb haemorrhage7. REBOA Resuscitative Endovascular Balloon Occlusion of the Aorta.

How well did you know this?
1
Not at all
2
3
4
5
Perfectly
393
Q

20.1 The radial artery pressure trace shown below is from a patient who has an intra-aortic balloon pump in situ. The device has been switched to 1:2 augmentation to assess the timing. The trace shows an augmented beat followed by an un-augmented beat. With respect to the augmentation, the trace shows

a. Correct timing
b. Early inflation
c. Late inflation
d. Early deflation
e. Late deflation

A

Bonus question

e. Late deflation

https://litfl.com/intra-aortic-balloon-pump-trouble-shooting/

Waveform features:
> The peak corresponding to diastolic augmentation (peak C) is widened.
> Assisted aortic end-diastolic BP (trough E) is the same as, not lower than, the unassisted aortic end-diastolic BP (trough G).
> The upstroke of assisted systolic BP (peak F) has a gentle gradient resulting in a prolonged rise.

Late deflation of the IAB has these effects:
> There is no afterload reduction. The inflated balloon may actually impede left ventricular ejection and increase the afterload.
> Myocardial oxygen consumption will increase because the left ventricle experiences a longer period of isovolumetric contraction (when most myocardial oxygen consumption occurs) and has to contract against greater resistance (afterload).

How to correct:
> Shorten the IAB inflation time, so that the IAB deflates at the end of diastole – just before isovolumetric contraction of the left ventricle.

How well did you know this?
1
Not at all
2
3
4
5
Perfectly
394
Q

20.2 The equipment shown in the picture is a(n) (picture of an airway device shown)

A

Arndt bronchial blocker
- use with SLT
- 9fr, loop around scope for positioning
- suction to deflate lung

395
Q

21.1 The domains described in the Edmonton Frail Scale do NOT include

a. Cognition
b. Mental illness
c. Weight
d. Age
e. Functional assessment

A

age

Domains:
1. Cognition
2. General health status
3. Functional independance
4. social support
5. Medication use
6. Nutrition
7. Mood ?interpreted as mental illness in stem?
8. Continence
9. Functional performance

Scoring
0-5= Not frail
6-7= Vulnerable
8-9= Mild Frailty
10-11= Moderate Frailty
12-17= Severe Frailty

Easy way to remember is CONFUSION
Cognition
Overall Health
Nutrition
Function performance
Urine incontinence
Social support
Independence
Oral Meds
Nightmares

396
Q

20.2 The flow volume loop is most consistent with (Flow-volume loop shown)

a) Variable intra-thoracic obstruction
b) Variable extra-thoracic obstruction
c) Lower airway obstruction
d) Fixed upper Airway obstruction
e) Mixed pattern

A

a) Variable intra-thoracic obstruction

Dynamic (or variable, nonfixed) intrathoracic obstruction: flow limitation and flattening are noted on the expiratory limb of the loop.

397
Q

20.2 This lung ultrasound shows

a) Normal lungs
b) Pulmonary odema
c) Pneumothorax
d) Pleural effusion
e) Pneumonia

A

a) Normal lungs

M-mode image demonstrating seashore sign seen with normal lung sliding.

398
Q

23.1 An absolute contraindication to skin prick testing for the diagnosis of allergies is

a. Pregnancy
b. Severe dermatographia
c. Concurrent antihistamine use
d. Concurrent beta blocker
e. Asthma

A

b) severe dermatographia

399
Q

23.1 A patient with idiopathic pulmonary hypertension has had a right heart catheter with
the following results The transpulmonary gradient is

(table of numbers from RHC given, including mPAP 40 and PCWP 13)

A

? no recalled ?

MPAP – PCWP = Transpulmonary gradient

27mmHg

TPG = mPAP – PCWP

400
Q

20.2 The next patient on your anaesthetist-supported endoscopy list is a fifty-year old woman who has been scheduled for gastroscopy and colonoscopy under sedation, having failed with proceduralist- supervised midazolam and fentanyl sedation in the past. She states that she has egg anaphylaxis, and carries an EpiPen. The most appropriate agent to use for her sedation is

a) Ketamine
b) Propofol
c) Remifentanil
d) Sevoflurane
e) Thiopentone

A

b) Propofol

BJA: No evidence for contraindications to the use of propofol in adults allergic to egg, soy or peanut

“No connection between allergy to propofol and allergy to egg, soy or peanut was found. The present practice of choosing alternatives to propofol in patients with this kind of food allergy is not evidence based and should be reconsidered.”

401
Q

23.1 An otherwise healthy child with a history of leukaemia four years ago, now in
remission, has an American Society of Anesthesiologists (ASA) classification of at
least

a. 1
b. 2
c. 3
d. 4
e. 5

A

ASA 2

402
Q

21.1, 21.2 You give a dose of intravenous indocyanine green to facilitate videoangiography during cerebral aneurysm surgery. The displayed pulse oximetry (SpO2) and cerebral oxygen tissue saturation (SctO2) changes you expect to see are

a. Increases NIRS , decreases peripheral
b. Decreases NIRS, decreases peripheral
c. No change NIRS, decreases peripheral
d. Increases NIRS and peripheral
e. Decreases NIRS, increases peripheral

A

a. Increases NIRS , decreases peripheral

SctO2 up, SpO2 down.

Source: Korean Journal Anaesthesia
https://www.researchgate.net/publication/274570990_Effects_of_intravenously_administered_indocyanine_green_on_near-infrared_cerebral_oximetry_and_pulse_oximetry_readings

403
Q

23.1 Patients with rheumatoid arthritis and the most common form of atlantoaxial instability have a widened atlantodental interval. This is measured between the

A. distance from posterior surface of dens to anterior surface of posterior arch of atlas
B. distance from anterior surface of dens to anterior surface of posterior arch of atlas
C. distance from posterior surface of dens to anterior surface of anterior arch of atlas
D. distance from posterior surface of dens to posterior surface of posterior arch of atlas
E. distance from anterior surface of dens to posterior surface of anterior arch of atlas

A

E. distance from anterior surface of dens to posterior surface of anterior arch of atlas

The atlantodental interval is used in the diagnosis of atlanto-occipital dissociation injuries and injuries of the atlas and axis.

The anterior atlantodental interval is the horizontal distance between the posterior cortex of the anterior arch of the atlas (C1) and the anterior cortex of the dens in the median (midsagittal) plane

Normal values for anterior atlantodental interval are:
radiographs:
adults:
males: <3 mm
females: <2.5 mm 1 (although most authors describe <3 mm ref)

children:
<5 mm ref

CT: adults: <2 mm

404
Q

22.2 Analysis of variance (ANOVA) is a statistical test to determine

a) The validity between an expected and observed outcome in a population
b) The difference between the means of more than two populations
c) The difference between two populations with non-parametric data
d) The degree of similarity of the median between two or more populations
e) If the variance within a population is likely to be abnormally or normally distributed

A

B) analyse the difference between the means of more than two groups

405
Q

22.2 An analgesic which is a category A drug using the Australian and New Zealand categories for prescribing medicines in pregnancy is

a. codeine
b. morphine
c. fentanyl
d. tramadol
e. oxycodone

A

a. codeine

Oxycodone B
Morphine C
Tramadol C
Fentanyl C

406
Q

22.1 The EXTEM plot from a ROTEM sample is shown. The most appropriate treatment for this bleeding patient is

(EXTEM graph with low amplitude and hyperfibrinolysis)

a. Platelets
b. TXA
c. Fibrinogen
d. Coagulation factors

A

b. TXA

407
Q

21.2 The maximum warm ischaemia time acceptable for procuring the kidney following donation
after circulatory death (DCD) is

a) 30 minutes
b) 60 minutes
c) 90 minutes
d) 120 minutes

A

Warm ischaemia time:
- Time from treatment withdrawal to the start of cold perfusion of the donated organs
- Significance is the impact on graft function
- Most important phase of WIT begins when the systolic BP is < 60mmHg
- This includes the waiting period from the absence of circulation to the declaration of death and the time before initiating the flow of cold perfusate through the cannula

Maximum WARM Ischaemia time
- Heart 30 mins
- Liver 30 mins
- Pancreas 30 mins
- Kidney 60 mins
- Lungs 90 mins

Maximum COLD Ischaemia time:
- Heart = 4 hrs
- Lungs = 6-8hrs
- Liver/Pancreas = 12hrs (DBD)/6 hrs (DCD)
- Kidneys = 18hrs (DBD)/ 12 hrs (DCD)

408
Q

22.2 A non-obese adult patient is administered a target-controlled propofol infusion for more than 15 minutes, with a constant target plasma concentration of 4 μg/ml propofol. Compared to the Marsh model, the propofol dose given by the Schnider model will be a

a) Smaller bolus smaller total dose
b) Smaller bolus larger total dose
c) Larger bolus smaller total dose
d) Larger bolus larger total dose
e) Smaller bolus same total dose

A

a) Smaller bolus smaller total dose

Marsh = more, Schnider = sparing
Marsh based on mass alone = MMA
Schnider includes senescence, sex = SSS

409
Q

23.1 A new antiemetic reduces the risk of postoperative vomiting by 20%. In a population with a baseline risk of postoperative vomiting of 10%, the number needed to treat is

a. 2
b. 5
c. 10
d. 20
e. 50

A

(base rate is 10%, experimental group is 8% (20% below 10%) therefore 100/ 2 = 50
or 1 divided by risk reduction

population risk = 10/100 patients get PONV
population risk + new antiemetic = 8/100 patients get PONV (8/100 as reduction by 20% with new drug)

RR= 0.10-0.08=0.02
NNT= 1/RR
=1/0.02
=50

410
Q

21.1 A 48 year old male is day two post-laparoscopic high anterior resection. He has used 42 mg of intravenous morphine in the past 24 hours. You wish to start him on oral tapentadol immediate release. The most appropriate equianalgesic dosage would be

a. 50 QID
b. 100 QID
c. 150 QID
d. 200 QID

A

b. 100mg QID

42mg IV Morphine = 126mg Oral Morphine

126/8= 15.75
15.75 x 25 = 393.75 (*400mg/day Tapentadol)

Oral Tapentadol 25mg = 8mg Oral Morphine

Oral Oxycodone 5mg = 8mg Oral Morphine

Oral Tramadol 25mg = Oral Morphine 5mg

Oral Hydromorphone 4mg = Oral Morphine 20mg

S/L Buprenorphine 200mcg = 8mg Oral Morphine

IV Oxycodone 5mg = Oral Morphine 15mg

IV Morphine 5mg = Oral Morphine 15mg

IV Hydromorphone 1mg = Oral Morphine 15mg

411
Q

21.1 Cardiovascular effects of hyperthyroidism include

a. Decreased SVR
b. Increased SVR
c. Decreased diastolic relaxation
d. Decreased PVR
e. Increased diastolic blood pressure

A

decreased SVR

Hyperthyroidism causes a hyperdynamic circulation, characterized by increased cardiac contractility and heart rate, increased preload, and decreased systemic vascular resistance (SVR), resulting in significantly increased cardiac output

https://www.sciencedirect.com/science/article/pii/S0735109718333795#:~:text=Hyperthyroidism%20causes%20a%20hyperdynamic%20circulation,in%20significantly%20increased%20cardiac%20output.

412
Q

21.2 A 59-year-old lady presents for elective coronary artery graft surgery. She has a pulmonary artery catheter inserted with the waveforms displayed below. Her cardiac output is 4.5 L/min.

Her mean pulmonary artery pressure is 33 mmHg. The most likely explanation for the waveforms seen is that she has

A

MR?? See UpToDate PAC interpreting

413
Q

21.1 Consideration for same-day discharge in an ex-premature infant after orchidopexy for undescended testis would be suitable at a minimum postmenstrual age of

a) 46 wks
b) 48 wks
c) 50 wks
d) 52 wks
e) 54 wks

A

e) 54

ANZCA PS 29:
1. Ex-preterm infants at risk of post-op apnoea should not be considered for same day discharge unless they are medically fit and have reached a PMA of 54 weeks.
2. Term infants should not be considered for same day discharge unless they are medically fit and have reached a PMA of 46 weeks.

SPANZA PS29 BP:
Apnoea significant if:
- lasts more than 15 seconds
- HR <100 (or <30 from baseline)
- SpO2 <90%
Apnoea risks are: PMA, anaemia, chronic lung disease, pre-operative apnoea of prematurity

Most post-op apnoea occur within first 2 hours
In healthy infants, after 12 apnoea free hours, apnoea risk approaches preop levels in health infants
infants should be monitored for 12 apnoea free hours
high risk infants need to be admitted for a longer period of monitoring

414
Q

23.1 A Laser-Flex tube has a double cuff with two separate pilot balloons. The correct colours of the pilot balloons are that

a. Blue proximal cuff, clear distal cuff
b. Clear proximal cuff, blue distal cuff
c. Blue both
d. Clear both

A

b) Clear Proximal, Blue Distal

https://www.medtronic.com/content/dam/covidien/library/us/en/product/intubation-products/shiley-laser-oral-nasal-tracheal-tube-information-sheet.pdf

415
Q

21.1 A 50-year-old man is seen prior to his hip revision surgery. His blood results are

Hb 110 (130-170 normal range)
Ferritin 31 (30-100 range)
Transferrin saturation 21% (normal 20-80)
CRP 10 (0.1-10 normal)

The most likely diagnosis is

a) iron deficiency anaemia
b) anaemia of chronic disease
c) anaemia of chronic inflammation
d) anaemia of chronic inflammation with iron deficiency
e) megaloblastic anaemia

A

Anaemia of chronic inflamation with iron deficiciency

416
Q

20.2, 22.2 The analgesic drug with the most favourable Number Needed to Treat (NNT) for neuropathic pain is

a) Amitriptyline
b) Gabapentin
c) Tramadol
d) Pregabalin
e) Carbamazepine

A

Tramadol

APMSE 5th edition:

Tramadol is an effective treatment for neuropathic pain with NNT of 4.4 (95%CI 2.9 to 8.8)

Alpha-2-delta ligands (gabapentinoids) are the only anticonvulsants with proven efficacy in the treatment of chronic neuropathic pain.
At doses of 1,800 mg to 3,600 mg/d, gabapentin is effective in treating neuropathic pain, in particular caused by postherpetic neuralgia (NNT 6.7; 95%CI 5.4 to 8.7)

Pregabalin
Postherpetic neuralgia: 300 mg/d pregabalin (NNT 5.3; 95%CI 3.9 to 8.1) (4 RCTs, n=713) and 600 mg/d (NNT 3.9; 95%CI 3.1 to 5.5) (4 RCTs, n=732);
* Painful diabetic neuropathy: 600 mg/d pregabalin (NNT 7.8; 95% CI 5.4 to 14) (5 RCTs, n=1,015);
* Mixed or unclassified post-traumatic neuropathic pain: 600 mg/d pregabalin (NNT 7.2; 95%CI 5.4 to 11) (4 RCTs, n=1,367);
* Central neuropathic pain (mainly SCI): 600 mg/d pregabalin (NNT 9.8; 95%CI 6.0 to 28) (3 RCTs, n=562).

Amitriptyline NNT 4.6 (TCAs are effective in treatment of neuropathic pain (amitrip NNT 4.6))
Amitriptyline

By order of favourable NNT:

  1. TCAs (amitriptyline) NNT: 3.6, NNH: 9
  2. Strong opioids NNT 4.3 NNH 11.7
  3. Tramadol NNT: 4.7, NNH 12.6
  4. SNRIs (duloxetine and venlafaxine) NNT 6.4, NNH 11.8
  5. Gabapentin NNT: 7.2 NNH 25.6
  6. Pregabalin NNT:7.7, NNH 13.9

ANZCA Pain book

Treatment of chronic neuropathic pain after SCI (Guy 2016 GL). These guidelines recommend:

  • First line: pregabalin, gabapentin and amitriptyline;
  • Second line: tramadol and lamotrigine (in incomplete SCI);
  • Third line: Transcranial direct current stimulation (tDCS) alone and combined with visual illusion;
  • Fourth line: TENS, oxycodone and dorsal root entry zone lesions.
417
Q

21.1 The recommended cleaning protocol for a laryngoscope handle which has been used but which has no visible soiling is

a) Disinfect with chlorhex/alcohol
b) Autoclave
c) Wipe with detergent
d) Nothing
e) Sterilise

A

c) Wipe with detergent

Laryngoscope handles:
-non-critical devices
-should be cleaned with
detergent and water between each patient use.
-If contaminated with blood, they should be washed and disinfected.

Laryngoscope blades:
-considered critical equipment because they may penetrate skin or mucous membranes, require sterilisation.

Bougies:
-Re-use of these items has been associated with cross-infection.
-It is preferable that alternative single-use intubation aids are employed when possible

Face Masks:
-In contact with intact skin, these items are frequently contaminated by secretions
-considered semi-critical, requiring cleaning and
thermal disinfection

418
Q

The commonest primary cause of death from anaesthesia airway events in the NAP4 report was

A. Barotrauma
B. Aspiration
C. Tracheostomy dislodged
D. Bleeding post-trache insertion

A

B. Aspiration

Aspiration was the single commonest cause of death in anaesthesia events.
Poor judgement was the likely root cause in many cases which included elements of poor assessment of risk (patient and operation) and failure to use airway devices or techniques that would offer increased protection against aspiration. Several major events occurred when there were clear indications for a rapid sequence induction but this was not performed.

419
Q

22.1 A 50-year-old man with carcinoid syndrome having a resection of a peripheral hepatic metastasis develops a sudden fall in BP from 110/70 mmHg to 85/50 mmHg without significant bleeding. The most appropriate management is

a. Normal saline bolus
b. Octreotide 50mcg bolus
c. Metaraminol 0.5mg
d. Noradrenaline 5mcg bolus
e. Calcium 6.8mmol

A

b. Octreotide 50mcg bolus

Vasoactive hormone release intra-operatively is best treated with intravenous boluses of 20–50 µg of octreotide, titrated to haemodynamic response. Vasopressin as an alternative vasoconstrictor that may be useful if prolonged vasoconstriction is required; however, the evidence base is small.

It must be borne in mind that concomitant fluid losses, especially bleeding, may be responsible for intra-operative instability rather than hormone excess and that fluid resuscitation may be the answer rather than further octreotide therapy

https://academic.oup.com/bjaed/article/11/1/9/285683

420
Q

21.1 A 30-year-old professional athlete who underwent a knee arthroscopy under general anaesthesia becomes tachycardic in the recovery room. His non-invasive systolic blood pressure is 90 mmHg. A 12-lead ECG is obtained. The most appropriate therapy is

a. Adenosine 6mg (or 60mg remembered by other cohort)
b. valsalva
c. 50J
d. 200J

A

b. valsalva

Fluid and magnesium - fixes all.

But could also be conscious VT or something stupid….

421
Q

22.1 Suxamethonium may be safely given to patients with

a. Becker muscular dystrophy
b. Cerebral palsy
c. Guillain Barre
d. Frederich’s ataxia
e. Duchenne muscular dystrophy

A

CP

b. Cerebral palsy
->sux and volatiles are not contraindicated
-> presence of extrajunctional receptors may cause hyperkalaemia

a. Becker muscular dystrophy
-> essentially milder Duchenne’s (see duchenne response to Sux)

b. Cerebral palsy
-> Sux and volatiles not contraindicated
-> reduced MAC requirement
-> increased sensitivity to muscle relaxants

c. Guillain Barre
-> sux contraindicated due to risk of hyperkalaemia
-> increased sensitivity to Non depolarising NB

d. Frederich’s ataxia
-> sux should be avoided due to risk of hyperkalaemia

e. Duchenne muscular dystrophy
-> sux and volatiles contraindicated due to rick of hyperkalaemia and rhabdomyolysis

422
Q

22.2 The use of intraoperative dexamethasone for tonsillectomy

a) Increased oedema
b) Increased post tonsillectomy bleed
c) Increased Analgesic requirement
d) Reduced time to resumption of oral intake

A

d) Reduced time to resumption of oral intake

The effect of preoperative dexamethasone on early oral intake, vomiting and pain after tonsillectomy
https://pubmed.ncbi.nlm.nih.gov/15979735/

Conclusion: Preoperative dexamethasone use significantly reduces early posttonsillectomy pain, improves oral intake and facilitates meeting the discharge criteria while using standard anesthesia technique and sharp dissection tonsillectomy without any significant side effects.

423
Q

21.2 A risk factor which increases the likelihood of developing local anaesthetic systemic toxicity is

a) Hypoxia
b) Alkalaemia
c) High alpha1-acid glycoprotein
d) Hypocarbia
e) Increased carnitine levels

A

hypoxia

Hypoxia
Acidosis
co morbidities:
CVS
ischaemia
arrhythmias
conduction abnormalities
low ejection fraction
Other
extremes of age
frailty
conditions that cause mitochondrial dysfunction (e.g., carnitine deficiency);
liver (delayed onset)
kidney disease (delayed onset)
Smaller patient
Non-US guided blocks

https://www.apsf.org/article/local-anesthetic-systemic-toxicity-last-revisited-a-paradigm-in-evolution/

Understanding factors that increase risk is vital, as identifying patients with an elevated susceptibility to LAST enables clinicians to modify treatment and reduce the risk. Hypoxia and acidosis were recognized decades ago as factors predisposing to LAST.5 More recently identified co-morbidities include pre-existing heart disease (especially ischemia, arrhythmias, conduction abnormalities, and low ejection fraction), extremes of age, frailty, and conditions that cause mitochondrial dysfunction (e.g., carnitine deficiency); liver or kidney disease can also increase the risk of delayed LAST by depressing local anesthetic metabolism or disposition.5 Interestingly, Barrington and Kruger2 examined a registry of ~25,000 peripheral nerve blocks performed in Australia from January 2007 to May 2012 and identified 22 cases of LAST (overall incidence, 0.87 per 1000). They found that ultrasound guidance lowered the risk of LAST (odds ratio, 0.23, CI: 0.088–0.59, p=0.002)—presumably a result of fewer unidentified intravascular injections and possibly lower volumes of the drug used to achieve a block. Nevertheless, no single method can completely eliminate these events and roughly 16% of reported LAST occurred despite the use of ultrasound. Barrington and Kruger also noted that small patient size was a risk factor for LAST. The role of skeletal muscle as a large reservoir compartment for local anesthetic may explain this phenomenon and was confirmed in a rat model by Fettiplace et al.6 It is reasonable to adjust local anesthetic dose in all such “at-risk” patients or possibly avoid peripheral nerve block or local anesthetic infusion entirely if the risk is deemed too consequential. Surprisingly, Barrington and Kruger found 16 cases involving ropivacaine and the remainder were lidocaine-induced; notably, the LAST rate with lidocaine was approximately 5 times greater than that for ropivacaine.

NYSORA: LAST

NYSORA Tips
There is a greater likelihood for LA systemic toxicity in petite patients (small muscle mass), those at the extremes of age, and patients with preexisting heart disease or carnitine deficiency.
Roughly half the cases of LAST are atypical, with no seizures (other CNS symptoms), only CV toxicity or delayed onset.
The incidence of toxicity increases with injections near richly vascular areas. It is highest with paravertebral injections, followed by upper and lower extremity PNBs.
Prevention of LAST-related morbidity requires optimizing a complete system for regional anesthesia: patient selection, nerve block choice, drug and dose, complete monitoring and use of USGRA when possible, and preparing for LAST by having a kit available and practicing with simulation.
Prevention also includes raising awareness and educating our non-anesthesiology colleagues about proper use of LAs and risks, including management of LAST.

424
Q

20.1 Differential hypoxia is seen during what?

a) Ecco2 device
b) VV ecmo
c) VA ecmo
d) Haemodialysis
e) Peritoneal Dialysis

A

VA ecmo

VA - bleeding (large bore arterial puncture)

425
Q

23.1 Tranexamic acid is NOT useful for managing

A. Post cardiac bypass
B. Neurotrauma
C. PPH
D. Trauma
E. Upper GI bleed

A

E. Upper GI bleed

Incompressible sites, large volume blood loss and mortality risk are a few of the things that made GI bleeds seem like a natural fit for TXA administration. Early research seemed promising, but trials were small. The HALT-IT trial examined over 15,000 patients to see if TXA reduced death [14]. Not only did TXA have no effect on mortality, it increased the risk of seizure and thromboembolic events.

Take home: No demonstrated benefit with TXA in GI bleeding

https://www.ems1.com/research-reviews/articles/understanding-txa-AFkqRLajUv46X7xV/

426
Q

20.1 A change in respiratory physiology which occurs during pregnancy is DECREASED

a) Oxygen consumption
b) Diaphragmatic excursion
c) Minute ventilation
d) FEV1
e) ERV

A

e) ERV

e) ERV
- diaphragm displaced by gravid uterus = reduced ERV and RV = decreased FRC

FRC decreases by approximately 20 percent during the latter half of pregnancy, due to a decrease in both ERV and residual volume

Chest wall configuration — Outward flaring of the ribs, which begins early in pregnancy, and the progressively enlarging uterus raise the diaphragm up to 4 cm above its usual resting position [6]. However, diaphragmatic excursion during respiration is not impaired and actually increases by up to 2 cm.

427
Q

21.1 Suxamethonium may be safely given to patients with

a) chronic spinal cord injury
b) Hypokalaemic periodic paralysis
c) muscular dystrophy
d) myasthenia gravis
e) multiple sclerosis

A

d) myasthenia gravis

In contrast to other neuromuscular disorders, succinylcholine may be used in myasthenia gravis. The required dose may need to be increased by up to two-fold, as those with the disease show a relative resistance to the drug.

Sux is not recommended in patients with neuromuscular disease due to:
1. presence of extrajunctional receptors and risk of hyperkalaemia and rhabodmyolysis
2. fasiculations causing temperomandibular muscle spasm preventing intubation

Suxamethonium is
contraindicated in patients with recent burns or
spinal cord trauma causing paraplegia (can be given
immediately after the injury, but should be avoided
from approximately day 10 to day 100 after the injury),

ED95 is 0.8mg/kg in a MG patient

428
Q

20.1 Of the following agents, haemodialysis is most effective in clearing (list of anticoagulant drugs given)

a. Warfarin
b. Clopidogrel
c. Apixaban
d. Dabigatran
e. Rivaroxaban

A

Addendum:

Dabigatran is cleared by dialysis, which leads to a precipitous drop in the drug level, whereas rivaroxaban is not cleared by dialysis because 95% of rivaroxaban is protein bound.

Apixaban is not removed by haemodialysis.4
Rivaroxaban has ~30% renal excretion and is not removed by dialysis

Canberra Health Services
https://www.canberrahealthservices.act.gov.au ›

Hemodialysis does not significantly influence the effects of dual antiplatelet therapy with aspirin or clopidogrel.

Common Dialyzable Drugs
Mnemonics BLISTMED and ISTUMBLED:

B - Barbiturates
L - Lithium
I - Isoniazid
S - Salicylates
T - Theophyline/Caffeine (both are methylxanthines)
M - Methanol, metformin
E - Ethylene glycol
D - Depakote, dabigatran
Others - Carbamezepine

I - INH/isopropyl alcohol
S - Salicylates
T - Theophylline
U - Uremia
M - Methanol
B - Barbiturates
L - Lithium
E - Ethylene glycol
D - Dabigatran, depakote

429
Q

20.1 A new antiemetic drug ‘X’ is being evaluated. The percentage of patients who suffered postoperative nausea and vomiting (PONV) after administration of either the drug ‘X’ or placebo is as follows: percentage of patients with PONV after drug X = 20%; percentage of patients with PONV after placebo = 25%. The number needed to treat (NNT) is

a. 5
b. 20
c. 22.5
d. 25
e. 45

A

B

RR = 0.25-0.2 = 0.05
NNT = 1/RR
= 20

430
Q

21.2 Techniques to improve the speed of onset and spread of a peribulbar block include all of the
following EXCEPT

a) Honan balloon
b) Digital pressure
c) Ocular massage
d) Hyalase

A

c) Ocular massage

Hyalase
Mixing with lignocaine
Higher concentration
Higher volume
Occular pressure (spread and IOP reduction)

Source: 2x BJA Ed articles

431
Q

20.2 The right ventricular systolic pressure (RVSP) can be used to estimate pulmonary arterial pressure. The method used for determining RVSP by echocardiography involves applying the

a) Poiseuille equation, tricuspid regurg peak + RAP
b) Poiseuille equation using mitral peak + LAP
c) Poiseuille equation using mitral peak + CVP
d) Bernoulli equation using tricuspid regurg peak + LAP
e) Bernoulli equation using tricuspid regurg peak + RAP

A

Answer: e) Bernoulli equation using tricuspid peak + RAP
i.e. RVSP = 4v2 + RAP (~PAP in absence of RVOTO)

432
Q

21.1 Effective pharmacotherapy options to support smoking cessation in the perioperative period include all of the following EXCEPT

a) bupropion
b) clonidine
c) nortoptyline
d) Varenicicline
e) fluoxetine

A

Fluoxetine

433
Q

21.2 You are examining the precordium of a patient in the preadmission clinic and hear a fourth heart sound at the apex. This finding is consistent with

a) AR
b) Athlete
c) Normal
d) Hypertension

A

) Hypertension

Talley & O’Connor CVS Exam:
S3: Physiological in pregnancy; sign of LV failure; AR & MR
S4: Never physiological, most often due to systemic hypertension

Atrial gallop - stiff LV
- hypertrophy or ischaemic ventricle

Source CV phys

434
Q

20.1 ECG with infarct what territory

a) PDA
b) Obtuse marginal
c) LAD
d) RCA
e) Left circumflex

A

RCA

Source: LITFL

RCA occlusion is suggested by:
ST elevation in lead III > lead II
Presence of reciprocal ST depression in lead I
Signs of right ventricular infarction: STE in V1 and V4R

435
Q

22.1 A 75-year-old man has a loud ejection systolic murmur detected on clinical examination before a joint replacement. A focused transthoracic echocardiogram (TTE) detects a calcified aortic valve with a peak aortic jet velocity of 3 m/s. The peak gradient across the aortic valve is

a) 36mmHg
b) 44mmHg
c) 48mmHg
d) 56mmHg
e) None of the above

A

= ΔP = 4v2 = 4 x 9 = 36

436
Q

22.2 The most common complication of extracorporeal membrane oxygenation (ECMO) in adults is

a. Bleeding
b. Thrombosis
c. infection
d. gas embolism

A

a. Bleeding

repeat

437
Q

22.1 A 70-year-old man undergoes a stress echocardiogram as part of his preoperative preparation before a total hip replacement. If he has clinically significant coronary artery disease, the earliest indicator during his test is most likely to be

a. Angina
b. ECG changes
c. Hypotension
d. RWMA

A

RWMA

Echocardiographic images recorded during ischemia show abnormalities of wall motion, thus allowing for the detection of significant coronary artery disease
https://www.sciencedirect.com/topics/nursing-and-health-professions/stress-echocardiography

438
Q

21.2 The medical laser LEAST likely to cause eye injury is

a) CO2
b) Nd:YAG
c) Argon
d) Green light

A

CO2

Laser danger is proportional to penetration.
Penetration inversely proportional to the laser wavelength.

CO2 laser has very little penetration (~ 10micrometres), as it has a wavelength of 10 600nm.
Helium-Neon laser also has very little penetration.

Nd:YAG is the most powerful, with a penetration of 2-6mm, as it has a wavelength of 1064nm.

439
Q

23.1 A drug that is NOT useful for the treatment of vasoplegic shock is

A. Hydroxycobalamin
B. Methylene blue
C. Dobutamine
D. vasopressin
E. Dopamine

A

c. dobutamine

UTD

440
Q

21.2 The apical four–chamber view of a transthoracic echocardiogram below shows

a. PFO
b. SAM
c. Dilated RA and RV
d. Dilated LA and LV
e. Tamponade

A

Dilated RA and RV

441
Q

21.1 Non-anaesthetist practitioners wishing to provide procedural sedation should have training in sedation
and/or anaesthesia for a minimum of

6 weeks
3 months
6 months
12 months

A

3 months full time

Source: ANZCA PG 09

442
Q

The manufacturer guidelines suggest the smallest sized endotracheal tube that should be safely passed over an Aintree Intubation Catheter is (internal diameter) size

A. 4.0
B. 5.0
C. 6.0
D. 7.0
E. 8.0

A

Size 7.0

The Tube

The endotracheal tube has a length and diameter. The endotracheal tubes size (“give me a 6.0 tube”) refers to its internal diameter in millimeters (mm). The ETT will typically list both the inner diameter and outer diameter on the tube (for example, a 6.0 endotracheal tube will list both the internal diameter, ID 6.0, and outer diameter, OD 8.8).

443
Q

20.2 A patient has a peripherally inserted central catheter (PICC) inserted. The follow-up chest X-ray shows the tip positioned in the (Chest X-Ray shown)

a) Azygos vein
b) Coronary sinus
c) SVC
d) R atrium
e) L atrium

A

a) Azygos vein

Correct positioning in image

444
Q

20.2 Complications from dural puncture and resultant intracranial hypotension do NOT include

a) Cortical vein thrombosis
b) Seizure
c) Subdural haematoma
d) Encephalitis
e) Stroke

A

d) Encephalitis

Complications of dural puncture include:
1. PDPH
2. hearing loss
3. pneumocephalus (if LOR to air)
4. chronic headache
5. chronic back pain
6. subdural haematoma
7. cerebral vein thrombosis
8. bacterial meningitis
9. diplopia
10. cranial nerve palsy
11. seizures

445
Q

21.1 A man who had successful treatment of a germ cell tumour 10 years ago presents for laparoscopic appendectomy. Your intraoperative management should consider

a) ETCO2 45
b) RR 20
c) MAP 90
d) SpO2 88–92%

A

d) SpO2 88–92%

oxygen administration/ low fio2
assumed bleomycin

Bleomycin
Bleomycin is a particularly important chemotherapy drug for the anaesthetist to be aware of. Bleomycin is often used to treat germ cell tumours and Hodgkin’s disease in a curative setting. The major limitation of bleomycin therapy is the potential for subacute pulmonary damage that can progress to life-threatening pulmonary fibrosis. Pulmonary toxicity occurs in 6–10% patients and can be fatal.2 Exposure to high-inspired concentration oxygen therapy, even for short periods, as experienced during anaesthesia, is often implicated in causing rapidly progressive pulmonary toxicity in patients previously treated with bleomycin.3 These claims have been considered controversial by some, but it is the authors’ recommendation that any patient previously exposed to bleomycin therapy should be treated as high risk, and summary guidance regarding oxygen therapy is shown in Table 4.

Summary guidance—oxygen therapy for patients who have received bleomycin > Patients have a life-long risk of bleomycin-induced lung injury
> Oxygen therapy should be avoided if at all possible
> Clinical procedures (and leisure activities) involving a high should be avoided If a patient is hypoxic
> O2 therapy should be minimized to maintain O2 saturation of 88–92%
> High oxygen concentrations should be used with extreme caution for immediate life-saving indications only (to maintain O2 saturation of 88–92%)

446
Q

22.1 The most common type of perioperative stroke is

a. Embolic
b. Hypotensive
c. Thrombotic

A

Emboli

Blue book repeat

447
Q

22.2 Most consistent risk factor for PONV in children (not on report)
a. Use of N2O
b. Patient anxiety
c. Use of short acting opioids
d. Age >3
E.

A

d. Age >3

448
Q

22.1 A 50-year-old man is admitted with a stroke and undergoes cerebral angiography. The artery
marked by the arrow on the angiogram is the

a. Vertebral
b. Basilar
c. PCA
d. PICA
e. Anterior cerebral artery

A
449
Q

20.2, 21.2 The anion which contributes the most to the anion gap is

a) Albumin
b) Chloride
c) Phosphate
d) Bicarbonate

A

albumin

ALBUMIN AND PHOSPHATE
the normal anion gap depends on serum phosphate and serum albumin
the normal AG = 0.2 x [albumin] (g/L) + 1.5 x [phosphate] (mmol/L)
albumin is the major unmeasured anion and contributes almost the whole of the value of the anion gap.
every 1g/L decrease in albumin will decrease anion gap by 0.25 mmoles
a normally high anion gap acidosis in a patient with hypoalbuminaemia may appear as a normal anion gap acidosis.
this is particularly relevant in ICU patients where lower albumin levels are common
Effects of albumin:
Anion gap may be underesitmated in hypoalbuminaemia, because if albumin decreased by 1g/L then the anion gap decreases by 0.25 mmol
To overcome the effects of the hypoalbuminaemia on the AG, the corrected AG can be used which is AG + (0.25 X (40-albumin) expressed in g/L

450
Q

22.2 The composition of blood returned to the patient from intraoperative cell salvage shows

a) Normal plasma proteins
b) Normal platelets
c) Normal 23 DPG
d) Absence of fat emboli
e) Absence of haemolysed RBC

A

c) Normal 23 DPG

https://www.bjaed.org/article/S2058-5349(20)30157-8/fulltext

Advantages of Cell salvage:
1. reduction in need for donor blood transfusion

  1. no restrictive transfusion triggers
  2. superior oxygen delivery compared to donor blood
    -> red cells retain elliptical profiles and retain deformability
    -> increased concentrations of 2,3 DPG and ATP
    -> evidence supports early transfusion as oxygen carriage and deformability degrade over time
  3. lack of adverse immunolgical effects
    -> no sensitisation to antigens; Kell, duffy or Lutheran
    -> donor blood transfusion causes dose-dependant transfusion related immunosupression (TRIM) this can lead to increased risk of post-op infection and posible increased risk of tumour growth in patients undergoing cancer surgery
  4. Fulfills criteria for certain cultural groups to receive blood transfusion (JW)
  5. Financial benefits despite equipment and staffing costs

Disadvantages:
1. The salvaged blood contains clinically insignificant concentrations of clotting factors and platelets, and when large volumes of blood are processed, the use of clotting factors, platelets, and calcium may be necessary.

  1. High initial cost of equipment and training
  2. Processing of blood requires a few minutes, blood may not be immediately available in time critical scenariois
  3. REinfusion hypotension can occur and can be very marked requiring vasopressors
  4. More labor intensive than donor blood, increased diligence required when collecting blood
  5. May not be appropriate for all situations of operative blood loss
    ->Malignancy: use is controversial but supported in some instances (cystectomy radical prostatectomy, nephrectomy)
    ->Sepsis: not absolute contraindication but colume of contaminated material and pus must be limited
    ->Haemaglobonpathy: relative contraindication in sickle cell trait/disease and thalassaemia due to red cell fragility and potential for haemolysis
451
Q

21.1 The following muscles of the larynx are all innervated by the recurrent laryngeal nerve, EXCEPT

a) Posterior Cricoarytenoid
b) Lateral Cricoarytenoid
c) Interarytenoid
d) Thyroarytenoid
e) Vocalis
f) Cricothyroid

A

f)Cricothyroid

Nerve supply of larynx:
1. Superior laryngeal nerve:
- Cricothyroid muscle
- sensory supply to the interior of larynx down to vocal cords
2. internal laryngeal nerve
-
3. Recurrent laryngeal nervs
- motor supply to the intrinsic muscles of the larynx apart from cricothyroid
- sensory supply to laryngeal mucosa inferior to the cords

Intrinsic muscles of the larynx
a) Posterior Cricoarytenoid
- abducts the cords, opens the glottis
- only muscle to open glottis
b) Lateral Cricoarytenoid
- adducts the cords and closes the glottis
c) Interarytenoid
- only unpaired muscle
- closes the glottis
- continues upwards to form the aryepiglottic muscle which acts as a weak sphincter
d) Thyroarytenoid
- relaxes vocal cords
e) Vocalis
-adjusts tension in the cords
f) Cricothyroid
- only intrinsic muscle that lies outside the cartilagenous framework
- only tensor of the vocal cords

actions of intrinsic laryngeal muscles
1. Abductor of the cords: posterior cricoarytenoids
2. Adductors of the cords: lateral cricoarytenoids, interarytenoids
3. Sphincter to the vestibule: aryepiglottics, thyroepiglotics
4. Tension regulators of the cords: Cricothyroids (tensors), Thyroarytenoids (relaxors), Vocales (fine adjustment)

452
Q

20.1 The radial artery pressure trace shown below is from a patient who has an intra-aortic balloon pump in situ. The device has been switched to 1:2 augmentation to assess the timing. The trace shows an augmented beat followed by an un-augmented beat. With respect to the augmentation, the trace shows

a. Correct timing
b. Early inflation
c. Late inflation
d. Early deflation
e. Late deflation

A

Bonus question

a. Correct timing

https://derangedphysiology.com/main/required-reading/cardiothoracic-intensive-care/Chapter%20634/normal-iabp-waveform

453
Q

20.1 A 34-year-old woman with cystic fibrosis has had a recent transthoracic echocardiogram to evaluate pulmonary pressure and suitability for lung transplantation. Below is a continuous wave Doppler trace through the tricuspid valve.
Peak velocity= 3
Her central venous pressure is 5 mmHg. Her estimated right ventricular systolic pressure (RVSP) is

a) 39
b) 41
c) 45
d) 50
e) 61

A

a) 41mmHg

Answer: RVSP = 4v2 + CVP
4x3x3+5 = 41

454
Q

21.1 A patient had prolonged surgery with a laryngeal mask airway in situ. The following day he reports a problem with his tongue. You examine him and see the following when he protrudes his tongue: The most likely cause of the abnormality is

a. R hypoglossal nerve injury
b. L hypoglossal
c. R glossopharyngeal
d. L glossopharyngeal

A

L hypoglossal

The hypoglossal nerve innervates all the extrinsic and intrinsic muscles of the tongue, except the palatoglossus which is innervated by the vagus nerve.

Injury to the hypoglossal nerve causes ipsilateral tongue deviation (pathognomonic), with dysarthria and dysphagia in severe cases. The tongue deviates towards the side that is affected due to the unopposed action of the contralateral genioglossus

The symptoms and signs of hypoglossal neurapraxia are often self-limiting and 43% of diagnosed patients achieve resolution within 6 weeks of surgery and an additional 40% are symptom free within 6 months after surgery

Nerves injured by SAD
- Lingual nerve (2ry to tube)
- Hypoglossal nerve (2ry to cuff)
- Recurrent laryngeal nerve (2ry to cuff)

Presenting symptoms and signs
Lingual nerve:
- loss of taste and sensation to tip of tongue

Hypoglossal nerve:
- dysphagia
- dysarthria
- tongue deviation in unilateral injury

Recurrent laryngeal nerve:
- altered voice
- rarely: stridor

Risk factors for injury:
- use of nitrous oxide-> over inflation
- selection of SAD that is too small-> over inflation
- LMA maximum inflation pressure 60cmH2O

455
Q

20.1 The catheter type most likely to be associated with bloodstream sepsis per days insertion is:

a) Peripheral venous catheter
b) PICC
c) non-tunneled CVL
d) tunneled CVL
e) Peripheral arterial catheter

A

c) non-tunneled CVL

UTD:
In a systematic review that included 200 studies, the risk of catheter-related blood stream infection per 1000 catheter-days and varied by types of intravascular catheter were as follows [38]:

●Peripherally inserted central catheters (PICCs) – 1.1 (95% CI 0.9-1.3)

●Cuffed and tunneled central venous catheters – 1.6 (95% CI 1.5-1.7)

●Noncuffed central venous catheters

*Nonmedicated and tunneled – 1.7 (95% CI 1.2-2.3)

*Nonmedicated and nontunneled – 2.7 (95% CI 2.6-2.9)

However, there was no adjustment for severity of illness. Therefore, a particular type of catheter could be associated with an increased risk of infection if it was preferentially used in more severely ill or vulnerable patients.

456
Q

22.2 Created by the Global Initiative for Chronic Obstructive Lung Disease (GOLD 2017), the numerical GOLD classes 1 to 4 are classes of severity for chronic obstructive pulmonary disease (COPD). These classes are based on an assessment of the

A. Exertional dyspnoea
B. Exertional dyspnoea and FEV1
C. Exertional dyspnoea and number of exacerbations per year
D. Spirometry FEV1 only
E. Number of exacerbations per year only

A

ALTERED 22.1 QUESTION

D Spirometry FEV1 only

GOLD 1 > 80% Pred
GOLD 2 50-79% Pred
GOLD 3 30-49% Pred
GOLD 4 < 30% Pred

457
Q

22.2 According to the RELIEF study, in major abdominal surgery a liberal fluid strategy (10 mL/kg of crystalloid at induction followed by 8 mL/kg/hour during the case) compared to a restrictive fluid strategy, results in

A. Increased bowel anastomosis breakdown
B. Increased mortality
C. Decreased mortality
D. No difference in wound infection
E. Decreased acute kidney injury

A

E. Decreased acute kidney injury

Restrictive had more AKI
Otherwise no outcome significant statistically

https://www.thebottomline.org.uk/summaries/relief/

458
Q

Indications for the use of hyperbaric oxygen therapy in the treatment of acute carbon monoxide toxicity include all of the following EXCEPT

a. Pregnancy
b. COHb level 10%
c. Difficult to examine patient as likely concomitant drug overdose
d. Myocardial ischaemia
e. Reduced GCS

A

b. COHb level 10%

459
Q

21.1 In elderly patients without diabetes mellitus the use of aspirin in primary prevention of disease

a. Reduced cardiovascular mortality
b. Increased incidence of major bleeding
c. Increased cancer related death
d. Lower all cause mortality
e. Reduced thromboembolic events

A

increased incidence of major bleeding

460
Q

22.1 You review a patient before major bowel surgery. Using the American Heart Association/American College of Cardiology consensus guidelines, you assess him as being at intermediate risk of a perioperative adverse cardiac event. When explaining this to the patient, this best translates to a numerical risk in the range of

a) 1-5%
b) 5-10%
c) 10-15%
d) 15-20%

A

a) <1% low risk for 30 day adverse cardiovascular event. >5% high. 1-5% therefore moderate.

https://www.ahajournals.org/doi/10.1161/circ.105.10.1257

Based on surgery type

c) 10-15% (unlikely this)

Based on patient factors alone, adults can be categorized into low (<5%), borderline (5 to <7.5%), intermediate (≥7.5 to <20%), or high (≥20%) 10-year CVD risk. Source: ACC/AHA Guideline 2019

https://www.acc.org/latest-in-cardiology/ten-points-to-remember/2019/03/07/16/00/2019-acc-aha-guideline-on-primary-prevention-gl-prevention

https://www.jacc.org/doi/epdf/10.1016/j.jacc.2019.03.010

461
Q

20.2 You are performing a regional block for analgesia following knee surgery. You have an ultrasound probe scanning the anterior mid-thigh. The muscle indicated by the arrow in the ultrasound image below is the

A

Vastus Medialis

462
Q

20.1 A 55-year-old man with no past history of ischaemic heart disease is 3 days post total hip replacement surgery. He has an episode of chest pain that sounds ischaemic, began at rest and lasts thirty minutes before resolving fully. There are no ECG changes. 6 hours later there is a troponin rise above the 99th percentile upper reference limit. The diagnosis is

A. STEMI
B. NSTEMI
C. Myocardial injury for non cardiac surgery (MINS)
D. No diagnosis
E. Unstable angina

A

C. MINS (Anthony reckons)

B. NSTEMI

https://resources.wfsahq.org/atotw/perioperative-myocardial-ischaemia-in-non-cardiac-surgery/

MI is defined as myocardial cell death due to prolonged myocardial ischaemia. It is diagnosed by22 a rise of cardiac biomarker value above the 99th percentile limit with at least 1 of the following:
1. Symptoms of ischaemia,
2. New ST-segment T wave changes or new left bundle branch block,
3. New pathological Q waves,
4. Imaging evidence of new loss of viable myocardium or new regional wall motion abnormality
5. Identification of an intracoronary thrombus by angiography or autopsy OR: Cardiac death with symptoms suggestive of myocardial ischaemia.

It is important to note that the above diagnostic criteria have not been created for the perioperative setting. An elevated troponin after noncardiac surgery, even without other features of ischaemia independently increases the risk of 30-day mortality.21 With this in mind, a new perioperative diagnosis has been created—myocardial injury after noncardiac surgery (MINS). MINS is diagnosed by a postoperative peak troponin T of 0.03 ng/mL or greater due to myocardial ischaemia.21 Although a diagnosis of MINS has prognostic significance, the clinical utility remains uncertain.

MINS is defined as myocardial cell injury during the first 30 days after noncardiac surgery due to an ischemic etiology (ie, no evidence of a nonischemic etiology like sepsis, rapid atrial fibrillation, pulmonary embolism, cardioversion, chronically elevated troponin, etc) and is independently associated with mortality. MINS includes MI (both symptomatic and non-symptomatic) and patients with postoperative elevations in troponin but who do not have symptoms, electrocardiographic abnormalities, or other criteria that meet the universal definition described above, and have no evidence of a nonischemic etiology for their troponin elevation)

Circulation https://www.ahajournals.org/doi/10.1161/CIR.0000000000001024
MINS includes myocardial infarction and ischemic myocardial injury that do not fulfill the Universal Definition of Myocardial Infarction (myocardial injury with a rise or fall of cTn above the 99th percentile of the upper reference limit and at least 1 of the following: ischemic symptoms, new ischemic electrocardiographic changes, development of new pathological Q waves on ECG, imaging evidence of myocardial ischemia, or angiographic or autopsy evidence of coronary thrombus).

MINS is defined by at least 1 postoperative cTn concentration that exceeds the 99th percentile upper reference limit of the cTn assay as a result of a presumed ischemic mechanism (ie, supply-demand mismatch or atherothrombosis) in the absence of overt nonischemic causes (eg, pulmonary embolus; Table 1). Such elevations in cTn must be identified within the first 30 days after surgery but nearly always occur within the first 2 postoperative days. Clinical symptoms and electrocardiographic changes are not required to establish a diagnosis of MINS, which includes myocardial infarction and ischemic myocardial injury.

463
Q

21.2 When performing cannulation of the median cubital vein the structure that is LEAST likely to be inadvertently punctured or damaged is the

a) Ulnar artery
b) Radial nerve
c) Median nerve
d) Brachial artery
e) Ulnar nerve

A

E) ulnar nerve

464
Q

21.2 A man who had successful treatment of a germ cell tumour ten years ago presents for laparoscopic appendectomy. Your intraoperative management should consider

a) Lung protective ventilation
b) Oncoanaesthesia
c) Lowest FiO2 possible
d) MAP 60

A

c) Lowest FiO2 possible

Bleomycin

Bleomycin is a particularly important chemotherapy drug for the anaesthetist to be aware of.
Bleomycin is often used to treat germ cell tumours and Hodgkin’s disease in a curative setting.
The major limitation of bleomycin therapy is the potential for subacute pulmonary damage that can progress to life-threatening pulmonary fibrosis.
Pulmonary toxicity occurs in 6–10% patients and can be fatal.
Exposure to high-inspired concentration oxygen therapy, even for short periods, as experienced during anaesthesia, is often implicated in causing rapidly progressive pulmonary toxicity in patients previously treated with bleomycin.
These claims have been considered controversial by some, but it is the authors’ recommendation that any patient previously exposed to bleomycin therapy should be treated as high risk, and summary guidance regarding oxygen therapy is shown:

465
Q

21.2 The domains described in the Edmonton Frail Scale do NOT include

A) Cognition
B) Mental illness
C) Weight
D) Age
E) Functional assessment

A

D) Age

Can Grandma Functionally Support Medication Nutrition Mood Continence Self

i.e.
C (cognition)
G (general health)
F (functional independence)
S (social support)
M (medication use)
N (nutritional status)
M (mood, presence of mental illness)
C (continence)
S (self reported performance)

466
Q

22.1 The 2012 Berlin definition of the Acute Respiratory Distress Syndrome (ARDS) defines moderate disease as one with a PaO2 / FiO2 ratio (in mmHg) of

A

100 < PaO2/FiO2 ≤ 200 with PEEP ≥5 cmH2O

https://link.springer.com/article/10.1007/s00134-012-2682-1/tables/3

467
Q

20.2 You are performing a regional block for analgesia following knee surgery. You have an ultrasound probe scanning the anterior mid-thigh. The muscle indicated by the arrow in the ultrasound image below is the

A

Sartorius

468
Q

23.1 The Sequential Organ Failure Assessment (SOFA) score is used in intensive care for the assessment of sepsis. This score does NOT include the

a. Bilirubin
b. Platelets
c. PaO2/FiO2
d. GCS
e. Hypoglycaemia

A

e) hypoglycaemia

469
Q

22.1 Predictors of successful awake extubation after volatile anaesthesia in infants do NOT include

a. Grimace
b. RR >16
c. TV >5ml/kg
d. Conjugate gaze
e. Eye opening

A

b. RR >16

470
Q

22.2 You are performing femoral venous cannulation in an obese man under ultrasound guidance. The image quality is suboptimal as the vein is deep. The best way to improve the image quality is to

a. Increase USS speed of transmission
b. Decrease USS speed of transmission
c. Use higher frequency probe
d. Use lower frequency probe
e. Increase wavelength

A

d. Use lower frequency probe

471
Q

A patient in the intensive care unit has ventricular fibrillation two hours after her coronary artery bypass graft procedure. Recommended immediate management does NOT include

a) Atropine
b) Adrenaline
c) 3 stacked shocks
d) Amiodarone

A

b) Adrenaline
-You do not use full dose adrenaline (rather, give smaller doses)
-You do three “stacked shocks”
-You try pacing (rate of 90, DDD) in asystole if pacing wires are available
-If they are already paced and in PEA, you turn off the pacing to “unmask” VF.
-These shocks and attempted pacing are all measures you take before starting CPR, which is a departure from the ACLS norms.
-If you can’t control a shockable rhythm with three stacked shocks, you give amiodarone immediately rather than after three cycles.
-Amiodarone is the only drug in the protocol, which makes it easy to remember. -
-After five minutes of unsuccessful resuscitation the chest should be re-opened. —External CPR is pointless in all of the common causes of arrest in this scenario. Therefore, CPR is something you do while waiting to re-open the chest.
-Non-surgical staff are encouraged to re-open the chest in an emergency

https://derangedphysiology.com/main/required-reading/cardiac-arrest-and-resuscitation/Chapter%20221/cardiac-arrest-following-cardiac-surgery

472
Q

21.2 Your patient has been administered 50 mL of oral 5–aminolevulinic acid hydrochloride
(Gliolan) three hours prior to her scheduled craniotomy for resection of a glioblastoma. Care
should be taken perioperatively to avoid the adverse effect of

a) Acute kidney injury
b) Photosensitivity
c) Increased ICP
d) Hypertension
e) Hypokalaemia

A

photosensitivity

Gliolan (PI):

  • Aminolevulinic acid hydrochloride (ALA)
  • Natural precurore of haeme, metabolised into fluorescent prophyrins
  • The fluorescence in certain tissue targets for photodynamic diagnosis
  • Increased fluorescent porphyrin formation by malignant glioma tissue (i.e. GBM)
  • After excitation with blue light (λ=400‑410 nm), PPIX is strongly fluorescent (peak at λ=635 nm) and can be visualised after appropriate modifications to a standard neurosurgical microscope.
  • Avoid exposure of eyes and skin to light sources afterwards (photosensivity).

Contraindications:
- hypersensitivity
- porphyria
- pregnancy

Precautions:
- After administration of Gliolan, exposure of eyes and skin to strong light sources (e.g. operating illumination, direct sunlight or brightly focused indoor light) should be avoided for 24 hours.
- Co-administration with other potentially phototoxic substances (e.g. tetracyclines, sulfonamides, fluoroquinolones, hypericin extracts) should be avoided
- Within 24 hours after administration, other potentially hepatotoxic medicinal products should be avoided.
- In patients with pre-existing cardiovascular disease, Gliolan should be used with caution since literature reports have shown decreased systolic and diastolic blood pressures, pulmonary artery systolic and diastolic pressure as well as pulmonary vascular resistance.

473
Q

20.2 A woman with atrial fibrillation has no valvular heart disease. According to AHA guidelines, oral anticoagulants are definitely recommended if her CHA2DS2-VASc score is greater than or equal to

A. 1
B. 2
C. 3
D. 4
E. 5

A

C. 3

  • if male CHA2DS2-VASc score ≥2 to be recommended chronic OAC (Grade 1A).
  • if female CHA2DS2-VASc score ≥3 to be recommended chronic OAC (Grade 1A).
  • non-sex risk factor also holds bearing:
  • For patients with CHA2DS2-VASc score of 1 in males and 2 in females based on age 65 to 74 years, we recommend chronic OAC (Grade 1A).

Up to date:

Our approach to deciding whether to prescribe anticoagulant therapy for patients with AF (excluding those with rheumatic mitral stenosis that is severe or clinically significant [mitral valve area ≤1.5 cm2], a bioprosthetic valve [surgical or bioprosthetic] within the first three to six months after implantation, or a mechanical heart valve) is as follows:

*For a CHA2DS2-VASc score ≥2 in males or ≥3 in females, we recommend chronic OAC (Grade 1A).

*For a CHA2DS2-VASc score of 1 in males and 2 in females:
-For patients with CHA2DS2-VASc score of 1 in males and 2 in females based on age 65 to 74 years, we recommend chronic OAC (Grade 1A). Age 65 to 74 years is a stronger risk factor than the other factors conferring one CHA2DS2-VASc score point.
-For patients with other risk factors, the decision to anticoagulate is based upon the specific nonsex risk factor and the burden of AF. For patients with very low burden of AF (eg, AF that is well documented as limited to an isolated episode that may have been due to a reversible cause such as recent surgery, heavy alcohol ingestion, or sleep deprivation), it may be reasonable to forgo chronic OAC and institute close surveillance for recurrent AF, although it may not be possible to reliably estimate AF burden from surveying symptoms or infrequent monitoring. The frequency and duration of AF episodes vary widely over time, and episodes are often asymptomatic.

*For patients with a CHA2DS2-VASc of 0 in males or 1 in females, we suggest against OAC (Grade 2C). Patient values and preferences may impact the decision. For example, a patient who is particularly stroke averse and is not at increased risk for bleeding may reasonably choose anticoagulation, particularly if the patient is a candidate for treatment with a direct oral anticoagulant (DOAC).

2019 AHA/ACC/HRS Focused Update of the 2014 AHA/ACC/HRS Guideline

474
Q

20.2 Piped oxygen supply in major hospitals is predominantly sourced from

a VIE
b Cylinders
c Pipeline off site
d Oxygen concentrator on site

A

a VIE

475
Q

22.2 A drug that is contraindicated for a patient with a history of heparin-induced thrombocytopaenia is

a) Bivalirudin
b) Danaparoid
c) Prothrombinex
d) Fib conc

A

c) Prothrombinex

Has factors 2, 9, 10, heparin, ATIII

476
Q

22.1 In the awake term neonate the systolic arterial blood pressure is normally approximately

a. 55mmHg
b. 70mmHg
c. 80mmHg
d. 90mmHg

A

b. 70mmHg

477
Q

21.2 A normal 75 kg term parturient may be expected to have a total blood volume of

a) 5250mls
b) 6000mls
c) 6750mls
d) 7500mls

A

d) 7500mls

Compared with the blood volume (65 to 70 mL/kg) in nonpregnant women, the blood volume in pregnant women at term is increased to 100 mL/kg

478
Q

21.1, 22.2 IIn critically ill patients undergoing mechanical ventilation, energy dense enteral nutrition (1.5 kcal/mL/kg) compared to routine (1 kcal/mL/kg) enteral feeding provides

a) Higher incidence of VAP
b) Lower incidence of AKI
c) Lower all cause 90-day mortality
d) No difference

A

d) No difference

Repeat

Conclusions

In patients undergoing mechanical ventilation, the rate of survival at 90 days associated with the use of an energy-dense formulation for enteral delivery of nutrition was not higher than that with routine enteral nutrition. (Funded by National Health and Medical Research Institute of Australia and the Health Research Council of New Zealand; TARGET ClinicalTrials.gov number, NCT02306746. opens in new tab.)

https://www.nejm.org/doi/full/10.1056/NEJMoa1811687

479
Q

22.2 The recommended antibiotic prophylaxis for surgical termination of pregnancy is

a. Clindamycin 600 mg
b. Cephalexin 500 mg
c. Doxycycline 400 mg
d. Cephazolin 2g
e. Cephazolin 2g and metronidazole

A

c. Doxycycline 400mg

Insertion of Mirena-> no antibiotics
exception is acute PID-> clindamycin

https://ranzcog.edu.au/wp-content/uploads/2022/05/Prophylactic-Antibiotics-in-Obstetrics-and-Gynaecology.pdf

480
Q

Of the following, the agent that has the greatest capacity to absorb infrared radiation in the atmosphere is

a) CO2
b) desflurane
c) sevoflurane
d) nitrous
e) isoflurane

A

b) Desflurane

Atmospheric heat absorbed by a substance compared with CO2 is its GWP
GWP CO2 = 1
GWP N20 = 265 (atmospheric lifetime of 114yrs)
GWP sevo = 130 (atmospheric lifetime of 1.1yrs)
GWP desflurane = 2540 (atmospheric lifetime of 14yrs)

481
Q

21.1, 21.2 The most common cause of cor pulmonale is

a) Shunt
b) COPD
c) Volume overload
d) PE
e) Kyphoscoliosis

A

Chronic obstructive pulmonary disease (COPD) is the most common cause of cor pulmonale

leads to an increase in RV afterload secondary to changes in pulmonary vascular structure and mechanics, and lung hyperinflation.

Patients with COPD who subsequently develop RV dysfunction have an increased risk of admission to hospital and mortality

482
Q

21.2 A forest plot is a commonly used tool in meta-analysis. It presents

a) A qualitative analysis of pooled data from multiple studies
b) A number needed to treat vs number needed to harm
c) The non-inferiority of a study
d) The pooled data from all of the studies

A

d) The pooled data from all of the studies

Forest plots or blobbograms are used in order to show graphically the studies which have been included in the meta-analysis.
They demonstrate the differences between studies and provide an estimate of the overall result.

483
Q

A fasted patient with type 2 diabetes mellitus presents for elective surgery. She has omitted one dose of a sodium-glucose co-transporter-2 (SGLT2) inhibitor. The lowest pinprick ketone level that would support a diagnosis of euglycaemic ketoacidosis is

a) 0.3
b) 0.6
c) 1.0
d) 3.0

A

c) 1.0

Clinicians should consider DKA/euDKA in patients taking SGLT2i who have one or more of:
- symptoms of abdo pain, nausea, vomiting, fatigue or metabolic acidosis
(a normal or only modestedly elevated plasma glucose level does not exclude diagnosis)
- finger prick capillary blood ketone ( or blood beta-hydroxybutyrate) level >1/0mmol/l with or without hyperglycaemia
- Low (negative) Base Excess <-5mmol/l indicating metabolic acidosis on arterial or venous blood gasses

If the blood ketone leve is >1.0mmol/l in an unwell patient on SGLT2i, take arterial or venous blood gases to measure the BE.
If ketones >1.0mmol/l and BE <-5mmol/l the patient has presumed DKA
If the BSL is <14mmol/l it is presumed euDKA

> for a ward patient a MET team should be activated or ICU contacted for review in collaboration with endocrinology services

> management priorities include:
1. Rehydration
2. IV insulin with added dextrose if BSL <15mmol/l
3. hourly monitoring of blood glucose, ketones and blood gases
4. All should be reviewed by an endocrinologist or on-call physician and critical care specialist

484
Q

20.1 The concept of response surface modelling in anaesthesia refers to:

a. The combined effect of two drugs at varying doses on a given response
b. Probability of something
c. Effect of one drug on something
d. Overlap of something

A

None of those?

Mathematical model for plotting responses to 2 varying drugs when used in conjunction (Ie opioid and propofol)

(create a surface that encompass the complete set of isobolograms, concentration-effect curves and the shift of concentration-effect curve in the presence of another drug https://www.sciencedirect.com/science/article/pii/S1875459715000569)

485
Q

21.1 Once a unit of fresh packed red blood cells has been removed from controlled refrigeration the transfusion should be completed within

a) 2.5 hours
b) 3 hours
c) 3.5 hours
d) 4 hours

A

4 hours

Redcross: “Transfusion of each pack should be completed prior to the labelled expiry or within four hours, whichever is sooner. “

Shelf life of platelets: 5 days (Stored at 20-24 degrees, must be agitated gently and continuously)

FFP: Once FFP is thawed, must use within 24 hours.

Albumin administration: At RCH we allow the product to be administered within 6 hours of piercing the bottle. (from RCH.org)

Cryoprecipitate
Thawed cryoprecipitate should be maintained at 20°C to 24°C until transfused.
Once thawed, should be used within six hours if it is a closed single unit, or within four hours if it is an open system or units have been pooled.

486
Q

23.1 A feature of citrate toxicity following massive blood transfusion is

a. Hypotension
b. Metabolic acidosis
c. Hypokalaemia

A

Hypotension

Citrate is the anticoagulant used in blood products. It is usually rapidly metabolised by the liver. Rapid administration of large quantities of stored blood may cause hypocalcaemia and hypomagnesaemia when citrate binds calcium and magnesium. This can result in myocardial depression or coagulopathy. Patients most at risk are those with liver dysfunction or neonates with immature liver function having rapid large volume transfusion

https://litfl.com/citrate-toxicity/

Hypocalcaemia resulting in
long QT,
reduced inotropy,
hypotension
systemic hypocoag

Metabolic
Met alk with HCO3 formation
HAGMA with citrate accumulation
Hypernatraemia from Na citrate
Hypomag due to citrate chelation
Hypokalaemia due to low mag and met alk

487
Q

21.2 An adult with renal failure on regular haemodialysis has an ASA (American Society of
Anesthesiologists) physical status classification of at least

a) 1
b) 2
c) 3
d) 4
e) 5

A

ASA 3

Source: ASA Classification
https://www.asahq.org/standards-and-guidelines/asa-physical-status-classification-system

488
Q

20.2 You are asked to review a patient who underwent upper limb surgery. During the procedure the anaesthetist placed a nerve block. The patient has weakness on external shoulder rotation and atrophy of supraspinatus and infraspinatus muscles. The nerve most likely to have been injured is the

a) Axillary
b) Supraclavicular
c) Subscapular
d) Suprascapular
e) Long thoracic
f) Spinal accessory

A

d) Suprascapular

Suprascapula nerve (C5,6)
- innervates supra and infraspinatus
- comes off superior trunk of the brachial plexus, and is usually anaesthetised by an interscalene block
- sensory innervation to 70% posterior-superior shoulders and portion of the anterior axilla and the ACJ

Supraclavicular nerve (C3,4)
- provides sensory to the ‘cape’ of the shoulder
- component of the cervical plexus block
- lies outside the brachial plexus
- commonly missed during supraclavicular brachial plexus blocks

Subscapular nerve:
- subscapularis
- medial rotation shoulder

Dorsal scapular nerve:
- branch of the brachial plexus
- supplies rhomboid major muscle, rhomboid minor muscle, and levator scapulae muscle
- causes the scapula to be moved medially towards the vertebral column
- Dorsal scapular nerve syndrome can cause a winged scapula, with pain and limited motion

Thoracodorsal nerve:
- thoracodorsal nerve also branches from the posterior division of the brachial plexus
- this nerve innervates the latissimus dorsi muscle.

489
Q

21.2 In a patient with tetraplegia who develops autonomic dysreflexia, the expected haemodynamic response is

a) hypertension from splanchnic vasoconstriction above the level of the lesion
b) hypertension from splanchnic vasoconstriction below the level of the lesion
c) hypotension from uncontrolled vagal tone above the level of the lesion
d) hypotension from reduced sympathetic tone below the level of the lesion

A

b) hypertension from splanchnic vasoconstriction below the level of the lesion

ADR:
- increased SNS below
- increased PSNS above

Hypertension (>25 mmHg increase)

> 40 mmHg increase or SBP > 150 is severe

490
Q

23.1 A 40-year-old woman is administered a nerve block for extraction of her right lower wisdom tooth. The nerve that should be blocked is the

A. Mental
B. Lingual
C. Inferior alveolar

A

c) inf alveolar

The conventional inferior alveolar nerve block is the most commonly used nerve block technique in dentistry

The nerves anesthetized are the inferior alveolar, incisor, mental, and lingual nerves. The mandibular teeth to the midline, the body of the mandible, the lower part of the mandibular ramus, buccal periosteum and mucous membrane to the premolars, anterior 2/3 of the tongue, oral floor, lingual soft tissue, and the periosteum are all anesthetized

https://www.ncbi.nlm.nih.gov/pmc/articles/PMC6218392/

491
Q

23.1 You are using intraoperative cell salvage during a high-risk caesarean section. The salvaged blood has been washed and reinfused through a leukodepletion filter. This process should remove all of the following EXCEPT

A. Vernix
B. Alpha fetoprotein
C. Foetal RBC
D. Amniotic fluid
E. Foetal squamous cell

A

c) Foetal RBC

All others removed with leukodepletion filter

492
Q

23.1 A 36-year-old woman sustains an injury to her left arm and presents with pain. She informs you that she experiences unpleasant intermittent and spontaneous shooting sensations in her arm. This sensation is

a. Dysaesthesia
b. Allodynia
c. Hyperalgesia
d. Hyperaesthesia
e. Paraesthesia

A

a. Dysaesthesia

Chronic pain that may involve itchiness, burning, electric shock, or a general tightening in any part of the body.

Allodynia Pain from stimuli which are not normally painful. The pain may occur other than in the area stimulated.
Hyperalgesia is an abnormally increased sensitivity to pain
Hyperesthesia is a condition that involves an abnormal increase in sensitivity to stimuli of the sense

https://www.iasp-pain.org/resources/terminology/#:~:text=DYSESTHESIA,sen

493
Q

21.1 A 69-year-old woman has a recent onset of dyspnoea and undergoes a right heart catheterisation, with results displayed below. Her pulmonary capillary wedge pressure is 10 mmHg. The most likely 21.1 diagnosis is

a. Mitral stenosis
b. Mitral regurg
c. Emphysema
d. Pulmonary embolus
e. Aortic stenosis

A

PE

494
Q

20.1 A 15-year-old boy undergoes a cardiac procedure for congenital heart disease. The intrathoracic device is a(n) (chest X-Ray shown)

a) AV repair
b) PV repair
c) ASD closure device
d) Parachute device
e) Right atrial appendage closure
device

A

c) ASD closure device
Amplatzer Device

495
Q

23.1 A woman who is to undergo a caesarean section reports that she is allergic to amoxicillin. The reaction is limited to a rash. For surgical antimicrobial prophylaxis, you should administer

A. Cefoxitin
B. Cefazolin
C. Doxycycline
D. Clindamycin

A

Cefazolin

A first-generation cephalosporin is recommended, such as 2g intravenous cefazolin. The dose should be increased to 3g for women weighing over 120kg. Consideration should also be given to a repeat dose if the procedure is prolonged (over 3 hours).

  • For women with a history of immediate or delayed nonsevere hypersensitivity to
    penicillins, cefazolin, as above, remains appropriate.
  • For women with a history of immediate or delayed severe hypersensitivity to penicillins, use Clindamycin 600mg iv plus Gentamicin 2mg/kg iv.
  • For women colonised with Methicillin-resistant Staphylococcus aureas (MRSA) or at increased risk of being colonised with MRSA, add Vancomycin 15mg/kg iv.
  • Azithromycin may be considered at caesarean sections performed during labour or at least four hours after rupture of membranes (2). Administration of azithromycin 500mg has been shown to reduce a composite outcome of endometritis, wound infection or other infection (3). However, a strong recommendation in favour of routine use is not yet warranted given the concerns around the external validity of the paper, inducing resistance to azithromycin and possible effects on the establishment of the indigenous microbiome.
496
Q

21.2 The oculocardiac reflex results in

a) Hypertension
b) Apnoea
c) Junctional rhythm
d) Torsades

A

c) Junctional rhythm

Up to date: Anaesthesia for elective eye surgery

Oculocardiac reflex manifestations —
Manifestations of the oculocardiac reflex commonly occur when pressure is applied to extraocular muscles.
These include bradycardia (a decrease of 10 to 20 percent in the basal heart rate), junctional rhythms, hypotension, and, rarely, asystole.
This reflex can occur during injection of local anesthesia or during the surgical procedure itself.
Management includes stopping the stimulus (eg, release of traction or manipulation of the extraocular muscles).
If this is ineffective, an anticholinergic medication (eg, atropine or glycopyrrolate) is administered.
The risk of inducing this reflex may be reduced by an effective regional anesthetic block or general anesthesia with adequate depth.

497
Q

23.1 The function of the (electrical) earth conductor in operating theatre patient monitoring equipment is to

A. Prevent microshock
B. Prevent electrocution

A

B - prevention of electrocution.

BJA Education

https://academic.oup.com/bjaed/article/11/6/224/263710

498
Q

20.1 A woman who is 35 weeks pregnant presents with nausea and vomiting. Among other blood test abnormalities, her alanine transaminase (ALT) level is 400 IU/l (normal <34) and her International Normalized Ratio (INR) is 2.3. This is most consistent with

a) Hyperemesis gravidarum
b) HELLP syndrome
c) PET with severe features
d) Intrahepatic cholestasis of pregnancy
e) Acute fatty liver of pregnancy

A

a) Acute fatty liver

Key differences between AFLP and HELLP
- Coagulopathy with deranged INR in AFLP Vs. Thrombocytopenia in HELLP
- Thrombocytopaenia in HELLP, Plts tend to be unaffected in AFLP
- Degree of transaminitis in stem consistent with aFLP

Swansea Criteria for AFLP: Atleast 6 or more of the features described:
- Vomiting
- Abdominal pain
- Polydypsia/polyuria
- Encephalopathy
- Elevated Bilirubin
- Hypoglycaemia
- Elevated urate
- Leucocytosis
- Ascites/ Bright liver on US scan
- Elevated transaminases
- Elevated Amonia
- Renal impairment
- Coagulopathy
- Microvesicular streatosis on liver biopsy

AFLP:
Pathophysiology
- Variant of pre-eclampsia
- presents in the late 3rd trimester
- more common in women with multiple pregnancies and lower BMI
- Women with the condition are more likely to have children with disorders of B-fatty oxidation
- Associated with high maternal mortality, on average 1 death per year

Daignosis and presentation:
- Diagnosis by Swansea criteria in absence of another explanation
- Imaging with CT or MRI a liver biopsy or fat stain may support diagnosis
- present with non-specific symptoms of:
Nausea and vomiting
Malaise
Associated Htn and proteinuria
- Renal impairment develops in 90% of patients
- Fulminant hepatic failure can occur
- Patients may develop hepatic encephalopathy, coagulopathy and profound hypoglycaemia
- Metabolic acidosis and elevated lactate are not part of diagnostic criteria but are important features
- Pancreatitis and ARDS are other rare complications

Delivery and Anaesthetic concerns:
- Delivery must be expedited once the patient is stablized due to high foetal mortality
- Specific anaesthetic concerns are top correct hypoglycaemia and coagulopathy before delivery
- Plt function tends to remain stable unlike HELLP syndrome
- Uneventful regional anaesthesia has been reported and may improve hepatic blood flow but is often precluded due to coagulopathy
- Symptoms can deteriorate post-partum with worsening liver, renal function and coagulopathy for 48hrs

499
Q

20.1 Complications of dural puncture with intracranial hypotension do not cause

A) Cortical vein thrombosis
B) Seizure
C) Subdural haematoma
D) Encephalitis
E) Stroke

A

d) Encephalitis

UTD:
OTHER COMPLICATIONS OF DURAL PUNCTURE

PDPH is the most common adverse outcome of dural puncture and is generally self-limited and benign.

-Hearing loss (hypoacusia) may occur after dural puncture, and has been variably reported in up to 10 to 50 percent of patients after spinal anesthesia
- unilateral or bilateral, and may occur even in the absence of headache.
- Hearing loss is usually transient, but there are reported cases of hearing loss lasting for years after spinal anesthesia, unintentional dural puncture (UDP) and diagnostic lumbar puncture (LP).
- thought to relate to intracranial hypotension, with risk factors similar to risk factors for PDPH.
- In small studies, larger needle size and cutting needles have been associated with increased incidence of hearing loss.
- Epidural blood patch (EBP) has been performed with resolution of hearing loss.

  • Injection of air into the subarachnoid space during placement of neuraxial block may result in acute onset of severe headache and other neurologic signs and symptoms.
  • This complication may occur with an UDP if air, rather than saline, is used for loss of resistance to identify the epidural space.
  • A pneumocephalus headache can occur within a few seconds if the epidural is placed with the patient in the sitting position, but may be delayed until the patient sits up if the epidural is placed in the lateral decubitus position.
  • Regardless of onset delay, the headache is usually maximal at onset (ie, “thunderclap”).
  • Treatment of pneumocephalus headache is symptomatic.
  • Limited data suggest that normobaric oxygen therapy leads to more rapid resolution; hyperbaric oxygen therapy may be indicated for more severe cases of pneumocephalus.
  • Dural puncture is rarely associated with long-lasting complications.
  • Cases of persistent headache have been reported, some of which have required surgical repair of the dural rent or fluoroscopically-guided blood patch.
  • Increased chronic back pain has also been reported in patients who have had UDP, with no increased risk conferred by EBP as treatment for the PDPH.
  • PDPH may also be associated with persistent headache, chronic low back pain, bacterial meningitis, and postpartum depression.
  • EBP is not indicated as treatment for any of these complications.
  • In rare cases, dural puncture has been associated with reversible cerebral vasoconstriction syndrome (RCVS) and posterior reversible encephalopathy syndrome (PRES), but causation is uncertain; several of these reports involved obstetric patients with possible preeclampsia or eclampsia, which are also associated with RCVS and PRES.
500
Q

23.1 A 30-year-old woman has her bipolar disorder well controlled with lithium therapy. The analgesic agent LEAST suitable for her is

a. Tramadol
b. Diclofenac
c. Oxycodone
d. Methadone

A

b) diclofenac

LIthium perioperative concerns:
- Prolongation of NMB
- Reduction in anaesthetic agent requirement
- Avoid NSAIDs
- No withdrawl symptoms
- Discontinue 24hrs before surgery

NSAIDs differentially alter lithium concentrations by multiple mechanisms, and one of these is to reduce prostaglandin E2

BJA: perioperative advice for psychotropic drugs

501
Q

22.1 A 24-year-old man with Wolff-Parkinson-White syndrome is having anaesthesia for a knee arthroscopy. During the procedure he develops the following rhythm. His blood pressure is 100/65mmHg.
The most appropriate treatment is

a. Adenosine
b. Procainamide
c. Verapamil

A

b. Procainamide
BJA: Perioperative cardiac arrhythmias
https://academic.oup.com/bja/article/93/1/86/265716

  • Paroxysmal SVT (PSVT) due to re‐entrant circuits that involve accessory pathways (congenital electrical connections between the atrium and ventricle that bypass the AV node, such as Wolff–Parkinson–White Syndrome) pose caveats in the management of SVT.
  • It should be noted that patients with accessory pathways, in addition to PSVT, may also develop atrial fibrillation, and in the latter situation are at increased risk for developing ventricular fibrillation (VF) upon exposure to classic AV‐nodal blocking agents (digoxin, calcium channel blockers, beta blockers, adenosine) because these agents reduce the accessory bundle refractory period.
  • In such cases, i.v. procainamide, which slows conduction over the accessory bundle, is an acceptable option. Flecainide and amiodarone should also be considered, and cardiology consultation may be helpful.2
502
Q

20.2 The recommended maximum cuff pressure for insufflating a classic Laryngeal Mask is

a 15 cm H20
b 30 cm H20
c 40 cm H20
d 60cm H2O

A

d 60cm H2O

503
Q

22.2 The nerve labelled by the arrow marked I in the diagram is the

  1. Ulnar Nerve
  2. Axillary Nerve
  3. Median Nerve
  4. Medial Cutaneous nerve of the forearm
  5. Long Thoracic Nerve
  6. Dorsal Scapular Nerve
  7. Radial Nerve
  8. Suprascapular nerve
  9. Musculocutaneous Nerve
A
  1. Ulnar Nerve
504
Q

20.1 The maximum fraction of inspired oxygen that can be prescribed with a Venturi mask is

a) 30%
b) 40%
c) 50%
d) 60%
e) 70%

A

c) 60%

505
Q

22.2 A 25-year-old ASA (American Society of Anesthesiologists) physical status classification I patient develops seizures five minutes after receiving a brachial plexus block with ropivacaine. Of the following, the most suitable initial intravenous treatment is
a) phenytoin
b) levetiracetam
c) propofol
d) intralipid

A

c) propofol

https://anaesthetists.org/Portals/0/PDFs/Guidelines%20PDFs/Guideline_management_severe_local_anaesthetic_toxicity_v2_2010_final.pdf?ver=2018-07-11-163755-240&ver=2018-07-11-163755-240

506
Q

21.1 21.2 Benztropine ameliorates the side effects of drugs that antagonize

a) Dopamine receptor
b) Nicotinic Ach receptor
c) Muscarinic Ach receptor
d) Serotonin
e) Noradrenaline

A

a) Dopamine receptor

MOA: central acting anticholinergic

507
Q

22.2 The correct blood collection tube for a mast cell tryptase test is a

a. Potassium EDTA
b. serum separating tube
c. sodium citrate
d. sodium oxalate something

A

b. serum separating tube (gold top tube or red)

Potassium EDTA (purple)
-> FBC

sodium citrate (blue)
-> clotting screen/Rotem

sodium oxalate (green)
-> heavy metals (lead copper zinc)

508
Q

23.1 You are called to an airway emergency in the intensive care unit. A 40-year-old woman with morbid obesity and pneumonia had an elective percutaneous tracheostomy inserted eight hours previously. She is sedated, paralysed and ventilated. After being turned for pressure care, she desaturates and there is no clear CO2 trace on capnography.
The tracheostomy tube is still in the neck but you are concerned it has been displaced. Your immediate management should be to:

a. Reintubate from the mouth
b. Bronch via Trache
c. ?

A

a) reintubate from the mouth

? couldn’t find other recalled answers ? Will depend on the remembered answers ?

The key principles of the algorithm are:
1.Waveform capnography has a prominent role at an early stage in emergency management.
2.Oxygenation of the patient is prioritised.
3.Trials of ventilation via a potentially displaced tracheostomy tube to assess patency are avoided.
4.Suction is only attempted after removing a potentially blocked inner tube.
5.Oxygen is applied to both potential airways.
6.Simple methods to oxygenate and ventilate via the stoma are described.
7.A blocked or displaced tracheostomy tube is removed as soon as this is established and not as a ‘last resort’
BJA: Update on management of tracheostomy
https://www.bjaed.org/article/S2058-5349(19)30125-8/fulltext

https://www.tracheostomy.org.uk/storage/files/Patent%20Airway%20Algorithm.pdf

509
Q

20.2 The 12 lead ECG shown is most consistent with acute total occlusion of the

a) RCA
b) LCx
c) LAD
d) PDA

A

c) LAD

LIFL: Myocardial Ischaemia
https://litfl.com/myocardial-ischaemia-ecg-library/

More Myocardial Ischaemia ECG Examples: Example 1

Subendocardial ischaemia:
The most striking abnormality is the widespread ST depression, seen in leads I, II and V5-6. This is consistent with widespread subendocardial ischaemia.
There is also some subtle ST elevation in V1-2 and aVR with small Q waves in V1-2, suggesting that the cause of the widespread ischaemia is a proximal LAD occlusion.

510
Q

21.1 Of the following, the lifestyle modification that is least effective in reducing essential hypertension is

a. Stopping caffeine
b. Low sodium diet
c. Low potassium diet
d. Exercise

A

c. Low potassium diet (high K diet - dec HTN)

511
Q

22.2 In preschool-aged children having tonsillectomy under general anaesthesia, delirium is more likely with the use of

a) Remifentanil at end of case
b) Dexamethasone
c) IN something? ketamine?
d) Inhalational anaesthetic

A

D Inhalational anaesthetic

https://resources.wfsahq.org/atotw/emergence-delirium-in-pediatric-patients/

512
Q

21.1 Chronic recreational use of nitrous oxide may lead to
a. Anaemia due to decreased EPO
b. Anaemia from glutathione deficiency
c. Neurological damage due to methionine deficit
d. Pulmonary hypertension

A

neurological damage due to methionine deficit

513
Q

20.1 During a tracheostomy, what vessel is most at risk beneath tracheostomy and above sternal notch?

a) Superior thyroid artery
b) Brachicephalic Vein
c) Brachiocephalic artery
d) Inferior thyroid artery
e) Carotid artery

A

brachiocephalic artery

BJA: Emergency FONA in airway management

“Major vessels, most commonly the brachiocephalic artery, traverse the anterior tracheal wall in up to 53% patients at the suprasternal notch.”

514
Q

20.2 The muscle or muscle group with the greatest sensitivity to the action of non-depolarising neuromuscular blocking agents is the

a) Abdominal muscles
b) Adductor pollicus
c) Pharyngeal muscles
d) Diaphragm
e) Obbicularis occuli

A

c) pharyngeal muscles

Millers Anaesthesia:
Reference artyicle from Millers: https://pubs.asahq.org/anesthesiology/article/92/4/977/710/The-Incidence-and-Mechanisms-of-Pharyngeal-and

An adductor pollicis TOF ratio of 0.90 or less was associated with impaired pharyngeal function and airway protection, resulting in a four- to fivefold increase in the incidence of pharyngeal dysfunction causing misdirected swallowing. Moreover, pharyngeal function and airway protection may be impaired, even if the adductor pollicis muscle has recovered to a TOF ratio of more than 0.90.

515
Q

21.1 A 30-year-old woman is administered an anaesthetic for a laparoscopic cholecystectomy for acute cholecystitis. She is breastfeeding her six-week-old infant. During anaesthesia she receives the following drugs: propofol, fentanyl, sevoflurane, rocuronium, oxycodone, parecoxib, ondansetron, sugammadex and cefuroxime. The best advice regarding breastfeeding after anaesthesia is to

a) Discard 12 hours post procedure
b) discard 24 hours post procedure
c) discard 1st feed
d) discard first 2 feeds
e) discarding not required

A

e) discarding not required

516
Q

20.2 In the POISE study the use of beta blockers on the day of surgery as a cardio protective strategy in high risk patients has been associated with

a) Increased heart rate
b) Decreased hypotension
c) Increased mortality
d) Increased myocardial infarction

A

c) Increased mortality

Use of perioperative metoprolol was associated with:
* Decreased rate of myocardial infarction
* Decreased rate of revascularisation
* Decreased rate of developing new atrial fibrillation
* INCREASED rate of death
* INCREASED rate of stroke
* INCREASED rate of significant hypotension
INCREASED rate of significant bradycardia

517
Q

22.1 Under the NEXUS criteria, requirements to clear the cervical spine of trauma patients without radiographic imaging include all of the following EXCEPT

a. No midline tenderness
b. No distracting injury
c. No altered level of consciousness
d. Able to turn head 45 deg
e. No focal neurological deficit

A

d. Able to turn head 45 deg

NEXUS criteria:

One easy mnemonic for these criteria is NSAID:

Neuro Deficit
Spinal Tenderness (Midline)
Altered Mental Status/Level of Consciousness
Intoxication
Distracting Injury
518
Q

22.2 An eight-year-old-child with sickle cell disease is scheduled for emergency fixation of a fractured radius. Her haemoglobin is 80 g/L. The most appropriate management is

a. Blood type and screen
b. Exchange transfusion for HbSS <30%
c. transfuse for Hb >100
d. careful haemostasis and monitor Hb

A

c. transfuse for Hb >100
Emergency fixation means there is no time for an exchange transfusion

perioperative goals:
- planning and optimisation
- ensuring adequate O2 delivery
- hydration
- analgesia
- performed at a centre with a multidisciplinary sickle cell team

Children presenting for high-risk surgery (for example neurosurgical, cardiothoracic, or complex orthopaedic surgery) or high-risk children (previous stroke, acute CS, or end-organ damage), who were not included in this study, commonly receive an exchange transfusion or top-up transfusion, aiming for a preoperative haemoglobin concentration of 10 g dl−1 and Hb SS <30%. There is less evidence available for the role of transfusion in children with other forms of SCD.

Exchange transfusion vs. top-up transfusion
Exchange Transfusion:
- slowly removing the person’s blood and replacing with fresh donor blood or plasma
- Performed in cycles lasting a few minutes with slow removal of 5-20ml of blood and an equal amount of fresh pre-warmed blood or plasma flows into the person’s body
- in sickle cell disease blood is removed and replaced with donor blood to achieve a specific concentration of HbSS blood with a usual target of <30%
- Exchange transfusion removes HbS and increases HBA

Top-up transfusion:
- standard transfusion process of giving donor blood
- advantages of simple top-up include:
1. Increase oxygen carrying capacity
2. Decrease proportion of sickle haemoglobin HbS relative to Haemoglobin A (HBA)
3. Prevent or reverse complications of vast-occlusion
4. Can be given acutely
- disadvantages include:
1. Hyperviscosity if the Hb is increased to significantly over the patients baseline (target Hb should be 100g/l)
2. HbS is not removed, only diluted

519
Q

22.1 A 78-year-old man is undergoing left heart catheter angiography. A graph displaying pressures in the aorta (Ao) and left ventricle (LV) as well as electrocardiography trace over time is demonstrated below. These pressure recordings are characteristic of

a. MR
b. MS
c. AR
d. AS
e. TR

A

Aortic stenosis

520
Q

22.1 In a 5-year-old child with severe life-threatening anaphylaxis and no intravenous access, the recommended initial dose of intramuscular adrenaline is

a. 100mcg
b. 150mcg
c. 300mcg
d. 500mcg
e. 600mcg

A

150mcg IM

Then commence adrenaline infusion 0.1mcg/kg/min to 2mcg/kg/min

Refractory management:
Additional IV fluid 20-40ml/kg,
Noradrenaline infusion 0.1- 2mcg/kg/min
Vasopressin infusion 0.02-0.06 units/kg/hr, glucagon 40mcg/kg IV

521
Q

23.1 In a 21-year-old man with an isolated acute severe traumatic brain injury, systolic blood pressure should be maintained at a level equal to or greater than

a) 90
b) 100
c) 110
d) 120
e) 140

A

c) 110

Brain trauma foundation
Level III recommendation.
To decrease mortality and improve outcomes:

Maintain SBP at >100mmHg for patients 50 - 69
Maintain SBP at >110 for patients 15 - 49
Maintain SBP at >110 for patients 70 or older

522
Q

Piped oxygen supply in major hospitals is predominantly sourced from

A) Onsite oxygen concentrator
B) Onsite oxygen cylinder bank
C) Onsite oxygen liquid evaporator
D) Offsite pipeline supply

A

C) Onsite oxygen liquid evaporator (VIE)

VIE Pros and Cons
Pros
- Cheapest option for oxygen delivery and storage
- Storing oxygen as a liquid is much more efficient than as a gas
- Does not require power

Cons
- Set-up costs are expensive
- Requires a back-up setup
- Will waste large volumes of oxygen if not being used continuously
- Fire and explosion risk

523
Q

21.2 Performing a superficial cervical plexus block will block all of the following nerves EXCEPT the

a) Greater occipital
b) Greater auricular
c) Lesser occipital
d) Supraclavicular
e) Transverse cervical

A

a) Greater occipital

524
Q

21.2 In pulmonary function testing the presence of airflow limitation is defined by a post- bronchodilator FEV1/FVC ratio less than
a) 0.5
b) 0.6
c) 0.7
d) 0.8

A

c) 0.7

525
Q

21.2 Of the following, the LEAST likely cause of high anion gap metabolic acidosis is

a) Pancreatic fistula
b) DKA
c) Cardiac failure
d) Anti-retroviral
e) Methanol

A

pancreatic fistula

-> should cause NAGMA

HAGMA:
Lactate
Toxins
Ketones
Renal failure

NAGMA
Chloride
Addison’s, adrenal insuffiency, acetazolamide
GI loss (pancreatic fistula)
Extra: RTA

Anion gap:
- Anion Gap = Na+ – (Cl- + HCO3-)
- The Anion Gap (AG) is a derived variable primarily used for the evaluation of metabolic acidosis to determine the presence of unmeasured anions
- The normal anion gap is assumed to be 12, and the normal HCO3 is assumed to be 24

Delta ratio:
- can check delta ratio in the presence of a high anion gap metabolic acidosis (HAGMA) to determine if it is a ‘pure’ HAGMA or if there is coexistant normal anion gap metabolic acidosis (NAGMA) or metabolic alkalosis.

526
Q

22.1 A 65-year-old man presents to the preadmission clinic two weeks prior to his total knee replacement. His blood results include haemoglobin 100 g/L, ferritin 20 μg/L and normal C-reactive protein. The best course of action is to

a. Proceed
b. EPO and iron
c. Iron tablet and delay 3 months
d. Iron transfusion and proceed
e. PRBC

A

Postpone 3 months and give oral iron

527
Q

21.2 Postdural puncture headache in obstetric anaesthesia is associated with a greater likelihood of all of the following EXCEPT

a) Postpartum depression
b) Bacterial meningitis
c) Chronic back pain
d) Cerebral vein thrombosis
e) Posterior reversible encephalopathy syndrome (PRES)

A

No answer provided. ?encephalitis
These are all complications of dural puncture.

Encephalitis most likely answer

https://www.uptodate.com/contents/post-dural-puncture-headache

Complications of PDPH
1. Chronic Back pain
2. Hearing loss
3. Acute onset headache consider pneumopcephalus headache
4. Persistent headache
5. Increased risk of subdural haematoma
6. postpartum depression
7. bacterial meningitis
8. Reversible cerebral vasoconstriction syndrome (RCVS)
9. Posterior reversible encephalopathy syndrome (PRES)

528
Q

20.1 A patient with von Willebrand deficiency Type 1 presents with mild but persistent epistaxis.

First-line medical therapy should include:

a) Factor VII
b) Factor VIII
c) Recombinant von Willebrand factor
d) TXA
e) FFP

A

d) TXA

VWD Types:
1 - quantitative - minor effect on bleeding - DDAVP useful
2 - qualitative - spectrum of effects on bleeding - (2a,2b,2m,2n) - DDAVP may be useful in consult with haem
3 - absence - major bleeding - no effect of DDAVP

factors not recommended in Type 1
TXA and DDAVP are recommended but DDAVP not in list
TXA 10mg/kg IV q8h
DDAVP 300mcg intranasal 90-120 mins preop
(DDAVP increases factor VIII levels 2-5x via release of VWF which binds VIII and prevents its clearance)

Treatment of bleeding in an individual with von Willebrand disease (VWD) depends on:
1. Severity of bleeding
2, Site of bleeding
3. the type of VWD
4. the previous responses to therapy.

The two main approaches:
1. Increasing the level of normal von Willebrand factor (VWF) activity via DDAVP
2. Replacing the defective VWF with VWF concentrates

VWF concentrates have been demonstrated to provide excellent to good hemostasis in a number of patient populations and a number of bleeding types.

DDAVP is only effective in some individuals, produces a smaller increase in VWF activity, and has a later onset and shorter duration of action.

529
Q

22.2 You have diagnosed anaphylaxis in an eight-year-old girl having an appendicectomy. She weighs 20 kg and has refractory bronchospasm despite an adrenaline (epinephrine) infusion running at 15 mcg/min. The recommended initial dose of salbutamol (100 mcg/puff) via metered dose inhaler is

a) 1 puff
b) 3 puffs
c) 6 puffs
d) 10 puffs
e) 12 puffs

A

12 puffs
6puffs< 6yrs
12 puffs> 6 yrs

530
Q

22.2 A 56-year-old patient presents with exertional syncope. The most likely diagnosis is
(previously this was a 26yo)

a) HOCM
b) Aortic stenosis
c) Long QT syndrome

A

b) Aortic Stenosis

531
Q

20.1 What order to you remove your PPE?
a) Gloves, gown, goggles, mask, wash hands
b) Gloves, gown, wash hands, goggle, mask
c) Gown, goggles, mask, glove, wash hands
d) Goggles, mask, gown, glove, wash hands

A

Gloves, goggles, gown, mask, hand hygiene

CDC

532
Q

22.2 In a burns patient, the blood concentration of propofol is

a) Increased due to reduced cardiac output
b) Increased due to dehydration and reduced circulating volume
c) Reduced due to increased volume of distribution and clearance
d) Increased due to reduced renal clearance
e) Reduced due to increased inflammatory cytokines

A

c) Reduced due to increased volume of distribution and clearance

2010 Paper on major burns

The pharmacokinetic characteristics of a propofol bolus administered in patients with major burns were enhanced clearance and expanded volume of distribution.

BURN and WT were the important covariates. For sedation or anesthesia induction, a higher than recommended dose of propofol may be required to maintain therapeutic plasma drug concentrations in patients with severe burns.

Vigilance regarding the burned individual and careful titration of hypnotics to the desired effect cannot be overemphasized.

https://pubmed.ncbi.nlm.nih.gov/20510522/

533
Q

21.1 A man presents with a fractured tibia. He has increasing pain in his leg, loss of sensation on the plantar surface of his foot and weakness of toe flexion. This is most consistent with a compartment syndrome of the leg in the

A. Anterior
B. Lateral
C. Superficial Posterior
D. Deep posterior

A

Deep Posterior Compartment

Source: UpToDate

534
Q

21.1 Of the following, the incidence of venous air embolism is considered highest for

a) LUSCS
b) Prostatectomy
c) Coronary artery surgery
d) Spinal surgery
e) Gastric endoscopy

A

a) LUSCS

Rates of VAE by surgical procedure:
LUSCS: 10%-97%

Neurosurgery:
Posterior Fossa: 76%
Cervical Laminectomy: 7-25%
Lateral/Prone Neurosurgery: 15-25
%
Total Hip Replacement: 30%
Lap Cholecystectomy: 69%

535
Q

23.1 The nerve most likely to be inadequately anaesthetised with an incomplete interscalene brachial plexus block is the

A. Medial brachial cutaneous nerve
B. Median…
C. Supraclavicular
D. Musculocutaneous nerve

A

a. medial cutanous brachial nerve

C8/T1 roots are often missed. Therefore, interscalene blocks tend to fail on the ulnar side of the arm

Medial brachial cutaneous nerve (C8-T1, arises from the medial cord of the brachial plexus): upper medial arm

NYSORA

536
Q

23.1 In cardiac surgery a low-normal central venous pressure and a low blood pressure with a hyperdynamic heart is suggestive of

a. Hypovolaemia
b. Vasoplegia
c. Left ventricular dysfunction

A

b) vasoplegia

537
Q

A 50-year-old man is admitted with a stroke and undergoes cerebral angiography. The artery marked on angiography is the

a) Anterior Cerebral Artery
b) Middle Cerebral Artery
c) Posterior Cerebral Artery
d) Basillar Artery
e) Superior Cerebellar Artery

A

Answer: This time thought to be posterior cerebral (previously middle cerebral)

538
Q

22.2 Unsupported ventilation in a non-anaesthetised patient with long-standing tetraplegia is improved when

a) Left lateral
b) Right lateral
c) Supine
d) Trendelenberg
e) Reverse Trendelenberg

A

d) Trendelenberg c) Supine

Moving from upright to supine affects the respiratory function of the tetraplegic and high paraplegic individual differently to the able-bodied person.

The increase in abdominal girth when sitting in tetraplegia is secondary to decreased abdominal muscle strength and the associated increased abdominal wall compliance.

In the seated position, the abdominal contents are less supported by the decreased abdominal wall muscle tone and fall forward, increasing the waist size and lowering the diaphragm.

In able-bodied subjects, the FVC is reduced in the supine position, whereas in tetraplegia it is increased.

Postural changes are associated with symptoms; patients with an acute, high SCI report less breathlessness when supine compared to sitting.

In the supine position, the weight of the abdominal contents forces the diaphragm to a higher resting level so that contraction produces greater absolute excursion of the diaphragm; an effect that can be increased when the person with tetraplegia is tipped 15° head down from supine such that the vital capacity rises by a further 6%

539
Q

20.2 The flow volume loop is most consistent with (Flow-volume loop shown)

a) Variable intra-thoracic obstruction
b) Variable extra-thoracic obstruction
c) Mixed Pattern
d) Fixed upper Airway obstruction
e) Normal

A

e) Normal

Normal flow-volume loop: the expiratory portion of the flow-volume curve is characterized by a rapid rise to the peak flow rate, followed by a nearly linear fall in flow. The inspiratory curve is a relatively symmetrical, saddle-shaped curve.

540
Q

22.1 The effect of a drop in patient core temperature from 37 C to 34 C is to

a. Increased k time
b. Decreased viscosity
c. Decreased platelet function

A

c. Decreased platelet function

541
Q

20.1 The risk of major bleeding in patients taking non-vitamin K oral anticoagulants (NOACs) is significantly increased by commencing administration of

a) Diltiazem
b) Clarithromycin
c) Atorvastatin
e) Fluconazole

A

e) Fluconazole

Among patients taking NOACs for nonvalvular atrial fibrillation, concurrent use of amiodarone, fluconazole, rifampin, and
phenytoin compared with the use of NOACs alone, was associated with increased risk of major bleeding

JAMA 2017 ACC/AHA

542
Q

23.1 Three-factor prothrombin complex concentrate reverses warfarin therapy within

A. 5 mins
B. 15 mins
C. 60 mins
D. 120 mins

A

a) 15 mins

50UI/kg,
Prothrombinex-VF is able to completely reverse a supratherapeutic INR within 15 minutes however, vitamin K is also required to sustain the reversal effect as the half-lives of the infused clotting factors are similar to endogenous factors.

https://www.mja.com.au/journal/2013/198/4/update-consensus-guidelines-warfarin-reversal#:~:text=Prothrombinex%2DVF%20is%20able%20to,similar%20to%20endogenous%20clotting%20factors.

543
Q

22.1 This image is an apical four chamber view obtained by transthoracic echocardiography. The artery that supplies the area indicated by the arrow is the

a. RCA
b. LAD
c. CCx

A
544
Q

A 63-year-old man has undergone a right pneumonectomy for malignancy. Twelve hours postoperatively he suddenly develops profound hypotension and shock. Clinical examination reveals a raised central venous pressure. The most useful IMMEDIATE action would be to

a. Turn left lateral
b. Re-insert chest drain on operative site
c. Tamponade

A

a) turn left lateral

UTD:
Cardiac herniation is usually seen within three days of surgery, presenting as sudden onset of hypotension and shock, cyanosis, chest pain, and superior vena cava syndrome. The acute event is usually preceded immediately by coughing, moving the patient, vomiting, or extubation.

Treatment involves emergent surgery to reposition the heart and close the pericardial defect to prevent recurrence.

?bleeding Rapid filling of the PPS with blood can occur within 24 hours of surgery. This complication is more common after pleuropneumonectomy or pneumonectomy for suppurative lung disease. The clinical presentation may be with hypotension and shock due to the loss of intravascular blood volume. The mainstay of treatment is surgical reexploration and control of bleeding sources.

545
Q

21.2 The most common type of perioperative stroke is

a) Thrombotic
b) Ischaemic
c) Hypotension
d) Embolic
e) Haemorrhagic

A

embolic

546
Q

21.2 ANZCA fasting guidelines classify all of the following as clear fluids EXCEPT

a) clear cordial
b) black coffee
c) strained broth
d) pulp free fruit juice

A

strained broth

ANZCA PS07:
“Clear fluids are regarded as water, carbohydrate rich fluids, specifically developed for perioperative use, pulp free fruit juice, clear cordial, black tea and coffee. It excludes fluids containing particulate matter, soluble fibre, milk-based drinks and jelly”

547
Q

20.2 The therapy most likely to decrease mortality in neonates with congenital diaphragmatic hernia is

a) Early surgical intervention - within 6 hours
b) HFOV
c) Lung protective ventilation
d) Nitric oxide
e) thoracoscopic vs open approach?

A

c) Lung protective ventilation

A congenital diaphragmatic hernia (CDH) occurs when a defect in the diaphragm allows abdominal organs to protrude into the thoracic cavity (Fig. 1). It affects approximately 1 in 3600 registered births and is a potentially life-threatening condition, the severity of which is primarily related to the extent of lung hypoplasia and pulmonary hypertension.

Advances in management strategies include protective ventilation, careful timing of surgery, the judicious use of extracorporeal membrane oxygenation (ECMO) and the introduction of both thoracoscopic and fetal intervention, but it remains a challenging condition to treat successfully with overall mortality rates still around 30%

A validated scoring system has been proposed based on various measurements, including:

(i) birth weight <1.5 kg,
(ii) Apgar score at 5 mins <7,
(iii) presence of chromosomal abnormality,
(iv) presence of major cardiac abnormality, and
(v) suprasystemic pulmonary hypertension on echocardiography.

This scoring system enables patients to be stratified into low (<10%), intermediate (∼25%) or high risk (∼50%) mortality groups.
Cardiac defects have been shown to worsen outcome regardless of the severity of the hernia
The presence of a small contralateral lung or a bilateral CDH are also poor prognostic signs.

Ventilation
Historically, aggressive ventilation was used to induce hypocapnia and alkalosis and thereby reduce pulmonary hypertension; however, protective ventilation strategies that avoid further injury to damaged lung tissue have reduced mortality in CDH. The CDH EURO Consortium advocates aiming for the limitation of peak inspiratory pressures to 25 cm H2O with PEEP kept at 3–5 cm H2O and allowing permissive hypercapnia.

High-frequency oscillatory ventilation
High frequency oscillatory ventilation (HFOV) is classically used as a rescue strategy when hypoxia and severe hypercapnia persist despite maximal conventional ventilation. The VICI trial (2016) randomised 171 neonates to conventional ventilation or HFOV as the initial mode of ventilation and found no significant difference in mortality, but those who had conventional ventilation were ventilated for shorter periods, needed less nitric oxide, sildenafil and ECMO, and had lower requirements for inotropic drugs.

Timing of surgery
Historically, CDH repair was treated as a surgical emergency. However, the degree of pulmonary hypoplasia is the major influence on prognosis and emergency surgery therefore confers little benefit. There is much debate but little consensus within the literature regarding the optimal timing of surgery.
Recommendations from the CDH EURO Consortium state that the following physiological parameters should be met before surgery:

(i) mean arterial pressure normal for gestation,
(ii) preductal oxygen saturation consistently 85–95% on FiO2 <0.5,
(iii) lactate below 3 mmol litre−1, and
(iv) urine output more than 1 ml kg−1 h−1 12

These recommendations do, however, acknowledge that repair on ECMO is a viable treatment strategy in the context of appropriate patient selection.

Little evidence Thorascopic approach has improved mortality compared to open approach

BJA Education Article - ​Anaesthetic management of patients with a congenital diaphragmatic hernia
https://www.bjaed.org/article/S2058-5349(18)30013-1/fulltext

548
Q

22.1 A patient’s glomerular filtration rate is estimated at 35 mL/min/1.73m2. The patient’s chronic kidney disease can be classified as Stage

a. 5
b. 4
c. 3a
d. 3b
e. 2

A

3b

Category GFR
ml/min/1.73 m2 Terms
G1 ≥90 Normal or high
G2 60-89 Mildly decreased*
G3a 45-59 Mildly to moderately decreased
G3b 30-44 Moderately to severely decreased
G4 15-29 Severely decreased
G5 <15 Kidney failure

Assign Albuminuria category as follows:
Albuminuria categories in CKD
Category ACR (mg/g) Terms
A1 <30 Normal to mildly increased
A2 30-300 Moderately increased*
A3 >300 Severely increased**
Abbreviations: ACR, albumin-to-creatinine ratio; CKD, chronic kidney disease.
*Relative to young adult level.
**Including nephrotic syndrome (albumin excretion ACR >2220 mg/g)

**Collectively referred to as “CGA Staging”

549
Q

21.1 A 50 year old man has the following pulmonary function test result. The most consistent diagnosis is

FEV1 - test result - predicted - % predicted 68%
FVC - test result - predicted - % predicted 68%
DLCO 46%

a) Asthma
b) Myasthenia Gravis
c) Emphysema
d) Sarcoidosis
e) Pulmonary Hypertension

A

d) Sarcoidosis

Pulmonary hypertension: Normal spirometry + low DLCO
Asthma: obstructive pattern and normal DLCO
Obesity: restrictive pattern and normal DLCO
Sarcoid: restrictive pattern and low DLCO

550
Q

20.2 Application of a pacemaker magnet to a ventricular implanted pacemaker would be expected to convert the operating mode to

a. DOO
b. VII
c. DDD
d. VVI
e. VOO

A

e. VOO

> Asynchronous mode most often the result of magnet application. In a ventricular PPM, this means VOO

> However, various sources recommend against use of magnet for PPM management due to inconsistent effects on different devices

Equipment in Anaesthesia and Critical Care:

> The use of a magnet as a solution for pacemaker problems, either in theatre or otherwise is not recommended.
The application of a magnet to the pacemaker can have unpredictable results, from causing it to change to a back-up mode such as VOO, to reverting to factory settings, to performing various self-tests, to switching off entirely.

551
Q

Of the following, the side-effect LEAST likely to be caused by adenosine administration is

A. chest pain
B. bronchospasm
C. GI upset
D. Flushing
E. Dyspnoea

A

B. bronchospasm

Blue book 2017 article (see image)

CLASS
short acting anti-arrhythmic
naturally occurring purine nucleoside

MECHANISM OF ACTION
depression of SA & AV nodal activity
antagonises cAMP-mediated catecholamine stimulation of ventricular muscle
-> negative chronotropy & dromotropy

direct agonist at specific cell membrane receptors (A1 & A2) A1 = coupled to K+ channels by a guanine nucleotide-binding protein in supraventricular tissue.

PHARMACEUTICS
clear, colourless
3mg/mL
in saline

DOSE
rapid IV bolus followed by saline flush
3mg -> 6mg -> 12mg (adult)
0.04 to 0.25mg/kg (children)

INDICATIONS
diagnosis and treatment of paroxysmal SVT

ADVERSE EFFECTS
bronchospasm
flushing
SOB
Chest pain

CONTRAINDICATIONS
second and third degree AV block
allergy
care with asthma and COPD

PHARMACOKINETICS
Onset – 10 seconds
Duration – 10 seconds

Absorption – must be given IV
Distribution
Metabolism – absorbed by RBC’s and endothelium
Elimination – t ½ = 10 seconds

552
Q

23.1 Compared to a continuous epidural infusion, patient controlled epidural analgesia does NOT reduce

A. cesarean section rate.
B. Instrumental delivery.
C. Total dose of local anaesthetic.
D. height of block, motor block.
E. clinical workload

A

A. cesarean section rate.

Blue Book 2021

PCEA benefits include a decrease in motor blockade and decreased total local anaesthetic consumption. PCEA also reduces anaesthetic workload by significantly decreasing the requirement for clinician boluses while delivering similar analgesia, patient satisfaction and obstetric outcomes44. PCEA is most frequently used in combination with either PIB or CEI.

There is an associated decrease in instrumental deliveries in nulliparous women with intermittent bolus techniques with no difference in caesarean section rate. Caesarean section rates are unchanged for women with either type of epidural, compared to those without.

https://www.ncbi.nlm.nih.gov/books/NBK69143/

Unscheduled anaesthetic interventions (6 RCTs): significantly fewer patients required clinician top-ups with PCEA compared with CEI. The RD was 27% (95% CI: 18, 36, p<0.00001). Similar findings were obtained when only studies with a quality score of 3 or more were analysed (RD 35%, 95% CI: 19, 51, p<0.0001). No heterogeneity was detected (p=0.36).

Amount of local anaesthetic (8 RCTs): different regimens were used and significant heterogeneity was detected (p<0.0001). All studies found that higher local anaesthetic doses were used with CEI than with PCEA.

Motor block (4 RCTs): significantly fewer patients had ‘no motor weakness’ with PCEA than with CEI; the RD was 18% (95% CI: 6, 31, p=0.003).

Analgesia: there was no difference in the visual analogue scores between the treatments in the 7 RCTs that used them.

There was no significant difference between PCEA and CEI for: patient satisfaction (5 RCTs); total duration of first and second stage of labour (3 RCTs); incidence of Caesarean section or instrumental delivery (9 RCTs); incidence of low Apgar scores at 1 and 5 minutes after birth (6 RCTs); hypotension (6 RCTs); high block (3 RCTs); pruritus (2 RCTs); shivering (1 RCT) or nausea (1 RCT). The results for all of these outcomes were reported in the review.

Both PCEA and CEI appear to be safe for the mother and the neonate. Patients who receive PCEA are less likely to require anaesthetic intervention, require lower doses of local anaesthetic and have less motor block than those who receive CEI.

https://www.ncbi.nlm.nih.gov/books/NBK69143/

553
Q

23.1 Burns sustained from electrocardiography equipment during magnetic resonance imaging (MRI) scanning are minimised by

a. Low impedance ECG leads
b. Wet skin
c. Shaved skin
d. Looped leads
e. Ensure leads securely attached

A

e) ensure leads securely attached

https://journals.lww.com/nursing/Citation/2006/11000/Cables_and_electrodes_can_burn_patients_during_MRI.12.aspx#:~:text=The%20radiofrequency%20fields%20that%20occur,enough%20to%20require%20plastic%20surgery.

https://www.ahajournals.org/doi/10.1161/CIRCULATIONAHA.107.187256#d1e281

554
Q

21.1, 22.2 Intraoperative lung protective ventilation strategies include all of the following EXCEPT
A. Vt 6-8ml/kg
B. Patient titrated PEEP
C. Recruitment manoeuvre
D. I:E ratio 1:3

A

I:E ration 1:3

BJA Lung-protective ventilation for the surgical patient: international expert panel-based consensus recommendations:

An expert consensus was reached for 22 recommendations and four statements.

The following are the highlights:
(i) a dedicated score should be used for preoperative pulmonary risk evaluation; and
(ii) an individualised mechanical ventilation may improve the mechanics of breathing and respiratory function, and prevent PPCs.

The ventilator should initially be set to a tidal volume of 6–8 ml kg−1 predicted body weight and positive end-expiratory pressure (PEEP) 5 cm H2O.

PEEP should be individualised thereafter.

When recruitment manoeuvres are performed, the lowest effective pressure and shortest effective time or fewest number of breaths should be used.

Inspiratory/expiratory ratio:
Several studies have compared prolonged inspiratory-to-expiratory (I:E) ratios to the 1:2 ratio commonly used during mechanical ventilation.

An I:E ratio of 1:1, which has been characterised as providing a ‘balanced stress to time product’, was associated with attenuation of lung damage.
Prolonged I:E ratio increases mean airway pressure and concomitantly reduces peak airway pressure.

Studies using prolonged inspiratory times have described beneficial effects, including increased CRS and PaO2, lower alveolar–arterial gradient, and reduced inflammatory markers.

Given the lack of evidence for a clear benefit of a specific I:E ratio, no recommendation was offered by the panel.

However, the panel noted that optimisation of inspiratory time for individual patients can be achieved by monitoring parameters, such as oxygenation, CRS, and ΔP.

Intraoperative FIO2

Increased FIO2 during mechanical ventilation is administered to prevent or correct hypoxaemia, but may result in hyperoxia.

The negative effects of hyperoxia are not clear, but it has been suggested that it may increase oxidative stress, peripheral vascular and coronary artery vasoconstriction, decrease cardiac output, increase resorption atelectasis, and increase the rate of PPCs.

Recommendations for optimal use of oxygen and current evidence regarding the association between hyperoxaemia and clinically relevant outcomes during intraoperative mechanical ventilation are lacking.

Few studies have revealed a protective effect of hyperoxaemia, some report an association with mortality, whilst others show no association with clinically relevant outcomes.

Therefore, in the absence of evidence, the most prudent course of action during mechanical ventilation is to maintain normoxaemia.

SpO2 monitoring can assist in the detection of hypoxaemia, but during oxygen therapy SpO2 cannot detect hyperoxia.

Whilst SpO2 monitoring reduces the incidence of hypoxaemia, it does not improve the overall patient outcomes and does not reduce morbidity and mortality.

Therefore, once the airway is secured, FIO2 should be set to ≤0.4 with the goal of using the lowest possible FIO2 to achieve normoxia (or SpO2 ≥94%)

Unnecessarily high FIO2 should be avoided.

Administering lower FIO2 will not only decrease the risk of hyperoxia, but will also reduce the masking effect of oxygen therapy and allow for earlier diagnosis of gas-exchange impairment.

555
Q

20.2 A 100 kg twenty-five year old male presents 2 hours after suffering a 30% Total Body Surface Area electrical burn. He has had no resuscitation fluids. The infusion rate of isotonic crystalloid resuscitation fluid required for this man for the next 6 hours is

a) 500 ml/hr
b) 750 ml/hr
c) 1000 ml/hr
d) 1200 ml/hr
e) 1500ml/hr

A

c) 1000 ml/hr

4ml/kg/hr x 30%TBSA x 100kg = 12,000mL

1/2 within the first 8hrs, remainder over the next 16 hours (starting at 6 hour mark in this stem so beware)

6,000mL/6hrs = 1000 mL/hr which is the rate you would run it over the next 6 hours.

556
Q

20.2 Prothrombinex VF is a factor concentrate. It is indicated for the management of bleeding caused by

a Von Willebrand disease
b Haemophilia a
c Haemophilia b
d Haemophilia c
e Congenital fibrin deficiency

A

c Haemophilia b

557
Q

22.2 Large doses of sugammadex can potentially lead to
a) hypoglycaemia
b) hyperglycaemia
c) bradycardia
d) Prolonged QT

A

c) bradycardia

from PI

558
Q

22.1 In adults the spinal cord usually extends from the brainstem to the level of the inferior margin of the

a. T12
b. L1
c. L2
d. L3

A

b. L1

559
Q

23.1 During neonatal resuscitation, the pulse oximeter should be placed on the

A. Right hand
B. Left hand
C. Right foot
D. Left foot

A

R hand

probe should be attached to a preductal location (ie, the right upper extremity, usually the wrist or medial surface of the palm)
N.B
right foot - post ductal

https://www.ahajournals.org/doi/pdf/10.1161/CIRCULATIONAHA.110.971119

560
Q

20.2 When performing an infraclavicular block of the brachial plexus under ultrasound guidance, the structure indicated by the arrow is the (ultrasound image shown

a) Posterior Cord
b) Lateral cord
c) Medial cord
d) Superior trunk
e) Inferior trunk

A

b) Lateral cord

561
Q

21.2 The most reliable clinical indicator of opioid-induced ventilatory impairment (OIVI) is
decreased

a) level of consciousness
b) RR
c) SpO2
d) Vt

A

A) level of consciousness

In any patient who is given an opioid, oversedation should be considered to indicate OIVI until proven otherwise, regardless of a patient’s respiratory rate or oxygen saturation levels.

Source ANZCA PS 41

562
Q

23.1 Of the following, the drug which is most effective in the management of severe hyperthermia in serotonergic syndrome is

A. Paracetamol
B. Diazepam
C. Dantrolene
D. rocuronium

A

B. Diazepam

UTD

Discontinuation of all serotonergic agents

●Supportive care aimed at normalization of vital signs

●Sedation with benzodiazepines

●Administration of serotonin antagonists

●Assessment of the need to resume use of causative serotonergic agents after resolution of symptoms

563
Q

20.1 A patient with RA has been on 5mg of prednisone long term and is coming in for a joint replacement what is the appropriate management of their steroids?

a) 5mg oral pred
b) 10mg oral pred
c) No steroids
d) 50mg hydrocortisone IV
e) 100mg hydrocortisone IV

A

e) 100mg hydrocortisone IV
> note: alternatively, 6-8mg dexamethasone IV would suffice

564
Q

23.1 Suxamethonium may be safely given to patients with

a. Becker muscular dystrophy
b. Friedreich’s ataxia
c. Guillain-Barre
d. Cerebral palsy
e. Duchenne muscular dystrophy

A

d) myasthenia gravis
or
d) Cerebral palsy
->sux and volatiles are not contraindicated
-> presence of extrajunctional receptors may cause hyperkalaemia

if responses remembered incorrectly but of this list CP is probably the answer

a. Becker muscular dystrophy
-> essentially milder Duchenne’s (see duchenne response to Sux)

b. Cerebral palsy
-> Sux and volatiles not contraindicated
-> reduced MAC requirement
-> increased sensitivity to muscle relaxants

c. Guillain Barre
-> sux contraindicated due to risk of hyperkalaemia
-> increased sensitivity to Non depolarising NB

d. Frederich’s ataxia
-> sux should be avoided due to risk of hyperkalaemia

e. Duchenne muscular dystrophy
-> sux and volatiles contraindicated due to rick of hyperkalaemia and rhabdomyolysis

In contrast to other neuromuscular disorders, succinylcholine may be used in myasthenia gravis. The required dose may need to be increased by up to two-fold, as those with the disease show a relative resistance to the drug.

Sux is not recommended in patients with neuromuscular disease due to:
1. presence of extrajunctional receptors and risk of hyperkalaemia and rhabodmyolysis
2. fasiculations causing temperomandibular muscle spasm preventing intubation

REPEAT

565
Q

21.1 You are resuscitating a 60 kg man in cardiac arrest secondary to severe hyperkalaemia. You decide to give intravenous sodium bicarbonate. Australian and New Zealand resuscitation guidelines state the initial dose of 8.4% sodium bicarbonate should be

a. 30ml
b. 40 ml
c. 50 ml
d. 60 ml
e. 70ml

A

60 mmol
1mmol/kg

The 8.4% bottles of sodium bicarbonate provide 1 mmol/mL of sodium and bicarbonate

566
Q

22.2 You are giving IPPV via a mapleson D (bain) circuit. Minimum FGF to maintain normocapnia is
a) 50ml/kg/min
b) 70ml/kg/min
c) 100ml/kg/min
d) 150ml/kg/min
e) 200ml/kg/min

A

70-80ml/ kg/ min
Controlled ventilation

https://www.frca.co.uk/article.aspx?articleid=100141
A fresh gas flow of only 70 ml/kg is required to produce normocarbia.

Bain and Spoerel have recommended the following:

2 L/min fresh gas flow in patients <10 kg
3.5 L/min fresh gas flow in patients 10-50 kg
70 ml/kg fresh gas flow in patients >60 kg

The recommended tidal volume is 10 ml/kg and respiratory rate is 12-16 breaths/minute.

567
Q

22.1 The part of the lung that is typically divided into apical, anterior and posterior segments is the

a. RUL
b. RML
c. RLL
d. LUL
e. LLL

A

RUL

APALM
APIS APAL

568
Q

21.2 A 30-year-old man with morbid obesity (body mass index [BMI] 55 kg/m2) presents for middle ear surgery. The most appropriate bolus dose of propofol for induction should be based on

a) IBW
b) TBW
c) ABW
d) LBW
e) PBW

A

d) LBW

569
Q

21.1 A patient who usually takes oral morphine 50 mg bd develops a bowel obstruction and experiences withdrawal symptoms. They may be described as having

a) Tolerance
b) Physical dependence
c) Addiction
d) Abuse

A

Physical dependence

Physical dependence = presence of withdrawal symptoms when the drug is not taken.

570
Q

20.1 You are resuscitating a 60 kg man in cardiac arrest secondary to severe hyperkalaemia. You decide to give intravenous sodium bicarbonate. Australian and New Zealand resuscitation guidelines state the initial dose of 8.4% sodium bicarbonate should be

A

a) 60mls

571
Q

20.2 According to the National Audit Project (NAP) 6 report the drug with the highest rate of anaphylaxis (events per exposure) is

a. Teicoplanin
b. Amoxicillin
c. Cephazolin
d. Clindamycin
e. Gentamicin

A

a. Teicoplanin

572
Q

21.2 When performing a paediatric pain assessment, the five elements assessed to obtain the FLACC score are
a) face, legs, activity, cry, consolability
b) face, legs, arms, cry, consolability
c) function, legs, arms, cry, crossed legs
d) frown, legs, activity, cry, crossed arms

A

a) face, legs, activity, cry, consolability

573
Q

22.1 A patient with a body mass index 34 kg/m2 with no other disease has an ASA (American Society of Anesthesiologists) Physical Classification of at least

a. I
b. II
c. III
d. IV

A

b. II

574
Q

22.1 Regarding the Australian and New Zealand categorisation system for prescribing medicines in pregnancy, Category C medicines are ones which

A

c= Drugs which, owing to their pharmacological effects, have caused or may be suspected of causing, harmful effects on the human fetus or neonate without causing malformations. These effects may be reversible.

Category A
Drugs which have been taken by a large number of pregnant women and women of childbearing age without any proven increase in the frequency of malformations or other direct or indirect harmful effects on the fetus having been observed.

Category B1
Drugs which have been taken by only a limited number of pregnant women and women of childbearing age, without an increase in the frequency of malformation or other direct or indirect harmful effects on the human fetus having been observed.

Studies in animals have not shown evidence of an increased occurrence of fetal damage.

Category B2
Drugs which have been taken by only a limited number of pregnant women and women of childbearing age, without an increase in the frequency of malformation or other direct or indirect harmful effects on the human fetus having been observed.

Studies in animals are inadequate or may be lacking, but available data show no evidence of an increased occurrence of fetal damage.

Category B3
Drugs which have been taken by only a limited number of pregnant women and women of childbearing age, without an increase in the frequency of malformation or other direct or indirect harmful effects on the human fetus having been observed.

Studies in animals have shown evidence of an increased occurrence of fetal damage, the significance of which is considered uncertain in humans.

Category C
Drugs which, owing to their pharmacological effects, have caused or may be suspected of causing, harmful effects on the human fetus or neonate without causing malformations. These effects may be reversible. Accompanying texts should be consulted for further details.

Category D
Drugs which have caused, are suspected to have caused or may be expected to cause, an increased incidence of human fetal malformations or irreversible damage. These drugs may also have adverse pharmacological effects. Accompanying texts should be consulted for further details.

Category X
Drugs which have such a high risk of causing permanent damage to the fetus that they should not be used in pregnancy or when there is a possibility of pregnancy.

575
Q

21.1 A patient undergoing robotic prostatectomy with controlled mandatory volume ventilation has the following measurements:

plateau pressure 32 cmH2O, extrinsicPEEP 8 cmH2O, autoPEEP 4 cmH2O, peak pressure 38 cmH2O, tidal volume 600mL

The static compliance is

20 ml/cmH20
23 ml/cmH2O
25 ml/cmH20
30 ml/cm H20

A

30ml/cm H2O

600/32-(8+4) = 30
Static lung compliance (Cstat), mL/cm H2O = TV / (Plateau pressure (Pplat) – TotalPEEP)

remembered parameters included PEEP = 8 and autop PEEP = 4
if actual answer states TotalPEEP= 8 then no need to add 4 to the calculation

576
Q

22.1 A 30-year-old woman has had a free flap operation of eight hours duration. She received an intraoperative remifentanil infusion and was given 10 mg morphine 30 minutes before the end of the operation. In recovery her pain score has increased from 6/10 on arrival in recovery to 9/10 in spite of a further 10 mg of intravenous morphine. The most likely diagnosis is

a. Acute behavioural change
b. OIH
c. Inadequate analgesia
D. Physical dependence

A

b. OIH

577
Q

According to National Audit Project (NAP) 5, the incidence of awareness during general anaesthesia using a non relaxant technique with a volatile agent is
a. 1:700
b. 1:8000
c. 1:10000
d. 1:19000
e. 1:136,000

A

e. 1:136,000

https://www.bjanaesthesia.org/article/S0007-0912%2817%2930746-8/fulltext
1/670 E-LSCS
1/8000 with muscle relaxation
1/8600 CTS
1/8200 Volatile + neuromuscular blocking
Overall 1:19000

578
Q

22.1 The fourth position of the international pacemaker (NBG) code represents the

A. Pacing
B. Programability
C. Sensing
D. Anti-dyrhythmic functions
E: Inhibition

A

B. Programability ? Multi-site pacing?

579
Q

20.2 Apert syndrome is associated with

A) Atlanto-occipital instability
B) Hypotonia
C) Increased ICP
D) hypercalcemia
E) Mucopolysaccharoidosis

A

Raised ICP
*also associated with a difficult airway (Difficult BMV Ventilation)

Apert syndrome:
Autosomal dominant abnormality of first branchial arch causing premature closure of cranial sutures, midface hypoplasia, choanal atresia, cleft palate, fusion of cervical spine (mainly C5-C6) and syndactyly.

May have associated cardiac and renal abnormalities as well as intellectual impairment due to megalocephaly, hypoplasia of white matter and agenesis of the corpus callosum.

Obstructive sleep apnea is present in 50% and there may be an increased incidence of upper airway obstruction at induction, which is mostly overcome by routine maneuvers.

Classically, craniosynostosis release with fronto-orbital advancement is completed at 6 to 12 months of age if intracranial pressure (ICP) is normal [​24-26​]. However, elevated ICP may occur in up to 43 percent of cases. In this event, prompt surgical advancement and potentially ventriculoperitoneal shunt placement is required

580
Q

23.1 According to the Revised Cardiac Risk Index, a 72-year-old male scheduled for a laparoscopic cholecystectomy with a history of hypertension, 20 pack-year history of smoking, type 2 diabetes requiring insulin and a previous stroke has a score of

A. 1
B. 2
C. 3
D. 4
E. 5

A

B. 2
( CVA, Insulin use)

UTD
CHF
Renal impairment
Insulin use
Stroke
Ischaemic heart
Surgically high risk

581
Q

22.2 The drug of choice for the treatment of duct-dependent congenital heart disease is

a) Sildenafil
b) Prostacyclin
c) Carboprost
d) Alprostadil
e) NSAID

A

d) Alprostadil

https://www.rch.org.au/piper/neonatal_medication_guidelines/Alprostadil_(Prostin_VR)%E2%80%93(Prostaglandin_E1)/

Alprostadil (PROSTAGLANDIN E1) is a synthetic prostaglandin used to relax the ductus arteriosus in early post-natal life, where a patent ductus is critical for survival, including Tetralogy of Fallot, pulmonary atresia, pulmonary stenosis, tricuspid atresia and transposition of the great arteries.

Dose
To open a closed ductus arteriosus:
0.1 micrograms/kg/minute (100 nanograms/kg/min). An effect is usually seen within 30-60 minutes. Reduce the dose once an effect is seen or as directed by a Consultant.1

Doses > 0.1 micrograms/kg/minute are rarely more effective and may cause serious adverse effects.3

To maintain patency of ductus arteriosus:
0.01 to 0.02 micrograms/kg/minute (10-20 nanograms/kg/min).1, 2

For persistent pulmonary hypertension of the newborn (PPHN):
0.01 to 0.05 micrograms/kg/minute (10-50 nanograms/kg/min).2

582
Q

20.1 During trauma resuscitation in adults, contraindications to blind nasogastric tube insertion include all of the following EXCEPT

A) Basal skull fracture
B) Midface fracture
C) Recent nasal surgery
D) Oesophageal varices
E) High C-spine fracture

A

E) High C-spine fracture

Contraindications
1. Oesophageal stricture because of the risk for esophageal perforation
2. Basilar skull fracture or facial fracture due to the potential for intracranial misplacement
3. Avoided in patients with oesophageal varices because tube placement may trigger variceal bleeding
4. Patients with a bleeding diathesis, minimal trauma to the pharynx, esophagus, or stomach from nasogastric tubes can also lead to severe bleeding, and, thus, tubes are avoided whenever possible.

583
Q

21.1 The function of the bottle labelled ‘D’ in the diagram below is to protect against the consequences of
(diagram of chest drain bottles)

a. Suction failure
b. Excess positive pressure
c. Drain kinking
d. Excess negative pressure

A

bottle A = fluid trap or collection bottle, can be independently emptied and allows accurate record of drainage amount
- first tube connecting drain to drainage bottles must be wide to decreased resistance
- volume capacity of this tube should exceed ½ of patient’s maximum inspiratory volume (otherwise H2O may enter chest)

bottle B = underwater seal drain, maintained at a predetermined level whilst still allowing for drainage of pleural fluid (if bubbling continuously -> bronchopleural fistula)
- volume of H2O in bottle B should exceed ½ patient’s maximum inspiratory volume to prevent indrawing of air during inspiration

bottle C = manometer or pressure-regulating bottle allows suction to be attached and should bubble continuously
- The maximum negative pressure (in cm H2O) generated by suction equals to the distance (in cm) the vent tube is below the water line (this can be adjusted)
-The negative pressure generated by the vent tube is independent of the amount of pleural drainage that is collected in the trap bottle
- If suction is turned off then tubing must be unplugged -> so air can escape into atmosphere

584
Q

20.2 You are performing a regional block for analgesia following knee surgery. You have an ultrasound probe scanning the anterior mid-thigh. The muscle indicated by the arrow in the ultrasound image below is the

A

Adductor Magnus

585
Q

20.2 The ANZCA Choosing Wisely recommendations advise avoiding all of the following EXCEPT

A. Offering an epidural on a patient who is labouring spontaneously with a normal pregnancy and no comorbidities, upon request

B. Giving blood transfusion on a healthy 20yo male with Hb > 70g/L, except when severe and symptomatic

C. Giving an anaesthetic to a high-risk patient with severe comorbidities without risk stratifying them and taking an anaesthetic history and assessment

D. Ordering cardiac stress testing for a patient undergoing high risk non-cardiac surgery.

A

A. Offering an epidural on a patient who is labouring spontaneously with a normal pregnancy and no comorbidities, upon request

586
Q

23.1 You have diagnosed malignant hyperthermia in a person weighing 80 kg. Australian
and New Zealand guidelines recommend an initial dose of dantrolene (Dantrium) of

a. 10 vials
b. 20 vials
c. 30 vials
d. 40 vials

A

a) 10

Dose of Dantrolene = 2.5mg/kg
Repeat every 10 minutes to a Maximum dose of 10mg/kg (Total Vials = 35)
Each Vial Dantrolene = 20mg

80 x 2.5mg = 200mg
Therefore 10 Vials of 20mg Dantrolene

Or,
TBW(kg)/8 = number of vials required for initial dose

587
Q

21.1 The most reliable clinical indicator of opioid-induced ventilatory impairment (OIVI) is decreased

a) resp rate
b) conscious state
c) BP
d) heart rate

A

b) conscious state

No mention of BP or HR in ANZCA OIVI monitoring document

In many published reports of patient deaths resulting from OIVI, undue reliance has been placed on respiratory rate as a unidimensional measure of OIVI, either without formal assessment of patient sedation, or without recognising the significance of excessive sedation

Respiratory rate and oxygen saturation levels are not direct measures of adequacy of ventilation.

Sedation scores should be assessed repeatedly at intervals that are appropriate to the route of opioid administration

Continuous measurement of a patient’s carbon dioxide concentrations is more likely to identify OIVI than continuous pulse oximetry

588
Q

23.1 A 69-year-old man is dyspnoeic and complains of right shoulder tip pain whilst in the postanaesthesia care unit after a laparoscopic-assisted anterior resection. A focused thoracic ultrasound is performed and an image of the right lung is shown below. This represents

a) Pneumothorax
b) Pulmonary Oedema
c) Normal Lung
d) Consolidated Lung

A

Normal Lung

589
Q

23.1 A 75-year-old man has this right heart catheter trace as part of his investigation of dyspnoea. His pulmonary capillary wedge pressure is 24 mmHg. The most likely
diagnosis is:

A. Idiopathic Pulmonary Arterial Hypertension
B. Portopulmonary Syndrome
C. Left Heart Failure
D. Pulmonary Embolism
E. Pulmonary Fibrosis

A

C. Left heart failure causing PulmHTN

Normal PAPs/d is 25/7. This would be classed as severe (55) - (if image is correct)
PAWP >15 means ‘ post-capillary’ cause or combined pre- and post.
This is either group 2 or 5.
A PVR might help differentiate.

All other options (group 1,3,4 and 5) would likely have a isolated ‘pre-capillary’ PAWP of <15

LITFL and blue book 2015 article

590
Q

22.2 When using the ECG to time intra-aortic balloon counterpulsation, balloon inflation should occur at the

a. start of T wave
b. peak of T wave
c. end of T wave
d. end of R wave
e. start of R wave

A

B peak of T wave

Triggering of the IABP is usually set according to the patient’s ECG tracing.
When an R wave is detected the balloon is triggered to automatically start inflating in the middle of the T wave.
Triggering can be impaired if the patient develops an arrhythmia, is paced or has a poor ECG trace.

LITFL

591
Q

22.1 St. John’s wort (herbal medicine Hypericum perforatum) will reduce the effects of

a. Aspirin
b. Clopidogrel
c. Warfarin
d. Heparin
e. NOAC

A

c. Warfarin

It is also a potent inducer of hepatic cytochrome P450 CYP3A4 isoform. Hence, it may significantly increase the metabolism of many concomitantly administered drugs such as alfentanil, midazolam, and lidocaine. It also induces the P450 2C9 isoform that results in the reduction in effect of warfarin and NSAIDs

592
Q

23.1 Despite two separate 300 IU/kg doses of heparin, you have failed to attain yourtarget activated clotting time prior to instituting cardiopulmonary bypass. An appropriate option now would be to give

a. More heparin
b. FFP
c. Dalteparin
d. bivalirudin

A

b. FFP

593
Q
  1. A 45 Year old man has poor oxygenation in the post anaesthesia care unit after a low anterior resection. His chest xray is below. The most likely diagnosis is

a. LLL collapse
b. Pneumothorax
c. L pleural effusion
d. R bronchopneumonia

A

The lungs are hyperinflated with relatively flat diaphragms - a sign of pulmonary emphysema. There is a dense triangular opacity overlying the cardiac shadow with increased lucency of the left upper zone relative to the right upper zone. This is the “sail sign” of left lower lobe collapse with subsequent left upper lobe hyper-expansion.

594
Q

A patient has undergone a laparotomy with a central line inserted intra-operatively. In the PACU, the patient is dyspnoeic and a lung ultrasound is performed. The ultrasound, shown below, is consistent with

A. Pneumonia
B. Effusion
C. Normal lung
D. Pneumothorax
E. Pleural odema

A

C. Normal lung

  • shows sandy shore sign of normal lung sliding

Alternative: Absent sliding & PTx: Stratosphere sign

595
Q

22.1 A 60-year-old woman presents for thrombectomy with left lower leg ischaemia. She has not received any medications since presentation, and takes none at home. The sole abnormality on laboratory testing is an activated partial thromboplastin time (APTT) of 52 seconds. The most likely cause of the raised APTT is

a. Lupus anticoagulant
b. Erroneous reading
c. Cold agglutinins
d. Factor VII deficiency
e. Haemophilia A

A

a. Lupus anticoagulant

Factor VII
-> prolonged PT but not APTT

Cold Agglutinins
-> prolonged PT and APTT
-> “sole abnormality”

Haemophilia A
-> isolated prolonged APTT
-> associated with bleeding and not clotting

Lupus Anticoagulation
-> increased risk of clotting
-> prolonged APTT and normal PT

596
Q

To minimise the risk of developing propofol infusion syndrome, the maximum recommended propofol infusion rate averaged over a 48-hour period is

A. 2.5mg/kg/hr
B. 5mg/kg/hr
C. 7.5mg/kg/hr
D. 10mg/kg/hr
E. 12.5mg/kg/hr

A

A. 2.5mg/kg/hr

Associated with high doses >4mg/kg/hr and prolonged use (>48hrs)
Safe doses of propofol infusion for sedation in ICU are considered to be 1-4mg/kg/hr
-> fatal Cases pf PRIS have been reported after infusion doses as low as 1.9-2.6mg/kg/hr

Risk factors:
i. Young age
ii. Critical illness
iii. High fat and low Carbohydrate intake
iv. Inborn errors of mitochondrial fatty acid oxidation
v. Catecholamine infusion/ High catecholamine and glucocorticoid levels
vi. Steroid therapy
vii. Severe head injuries

Characteristics:
i. Bradycardia
ii. Severe metabolic acidosis
iii. Cardiovascular collapse
iv. Rhabdomyolysis
v. Hyperlipidaemia
vi. Renal failure
vii. Hepatomegaly

Management:
- Routine monitoring of CK and triglycerides should be performed for the at risk population
○ Daily CK and triglyceridees after 48hrs of propofol infusion
○ Increasing CK in the absence of other pathology triggers suspiscion of PRIS
- Propofol immediately stopped and alternative (midazolam and alfentanil) are used
- PRIS is difficult to treat once it occurs
- CVS support provided as needed
- Renal replacement therapy may be required to treat lactic acidosis, clear propofol and its metabolites from the patient rapidly
- Catecholamine resistant shock has been reported
- Pacing has been used with limited success
ECMO has been reported and successfully used in the CVS support of PRIS

597
Q

20.1 The structure labelled A shows

a. Empty stomach
b. Clear fluids
c. Solids, early stage
d. Solids, late stage

A

a. Empty stomach

598
Q

22.2 Recirculation is a cannula position complication specific to the use of

a) ECCO2R
b) VV ECMO
c) VA ECMO
d) dialysis
e) AV ECVO

A

b) VV ECMO
VV ECMO Disadvantages
- no cardiac support
- local recirculation though oxygenator at high flows
- reverse gas exchange in lung if FiO2 low
- limited power to create high systemic arterial oxygen tension

599
Q

21.2 A man with atrial fibrillation has no valvular heart disease. According to joint American Heart Association (AHA), American College of Cardiology (ACC) and Heart Rhythm Society (HRS) guidelines, oral anticoagulants are definitely recommended if his CHA2DS2-VASc score is greater than or equal to

a) 1
b) 2
c) 3
d) 4
e) 5

A

b) 2

  • if male CHA2DS2-VASc score ≥2 to be recommended chronic OAC (Grade 1A).
  • if female CHA2DS2-VASc score ≥3 to be recommended chronic OAC (Grade 1A).
  • non-sex risk factor also holds bearing:
  • For patients with CHA2DS2-VASc score of 1 in males and 2 in females based on age 65 to 74 years, we recommend chronic OAC (Grade 1A).

Up to date:

Our approach to deciding whether to prescribe anticoagulant therapy for patients with AF (excluding those with rheumatic mitral stenosis that is severe or clinically significant [mitral valve area ≤1.5 cm2], a bioprosthetic valve [surgical or bioprosthetic] within the first three to six months after implantation, or a mechanical heart valve) is as follows:

*For a CHA2DS2-VASc score ≥2 in males or ≥3 in females, we recommend chronic OAC (Grade 1A).

*For a CHA2DS2-VASc score of 1 in males and 2 in females:
-For patients with CHA2DS2-VASc score of 1 in males and 2 in females based on age 65 to 74 years, we recommend chronic OAC (Grade 1A). Age 65 to 74 years is a stronger risk factor than the other factors conferring one CHA2DS2-VASc score point.
-For patients with other risk factors, the decision to anticoagulate is based upon the specific nonsex risk factor and the burden of AF. For patients with very low burden of AF (eg, AF that is well documented as limited to an isolated episode that may have been due to a reversible cause such as recent surgery, heavy alcohol ingestion, or sleep deprivation), it may be reasonable to forgo chronic OAC and institute close surveillance for recurrent AF, although it may not be possible to reliably estimate AF burden from surveying symptoms or infrequent monitoring. The frequency and duration of AF episodes vary widely over time, and episodes are often asymptomatic.

*For patients with a CHA2DS2-VASc of 0 in males or 1 in females, we suggest against OAC (Grade 2C). Patient values and preferences may impact the decision. For example, a patient who is particularly stroke averse and is not at increased risk for bleeding may reasonably choose anticoagulation, particularly if the patient is a candidate for treatment with a direct oral anticoagulant (DOAC).

2019 AHA/ACC/HRS Focused Update of the 2014 AHA/ACC/HRS Guideline

600
Q

20.1 The threshold plasma fibrinogen level at which you should start replacement during postpartum haemorrhage is

a. 1.0
b. 1.5
c. 2.0
d. 2.5
e. 3.0

A

C

https://ranzcog.edu.au/wp-content/uploads/2022/05/Prevention-and-Management-of-Postpartum-Haemorrhage.pdf

601
Q

20.1 You are using ultrasound with colour flow Doppler to scan a patient’s neck prior to placing an internal jugular line. In the short axis view of the carotid artery, the colour Doppler image will be

A. Red because blood is going away from the probe
B. Blue because blood is going away from the probe
C. Blue when the blood is coming to the probe, red when the blood is going away from the probe
D. Red when the blood is coming toward the probe, blue when the blood is going away from the probe
E. The colour depends on the angle you hold the probe at

A

E. The colour depends on the angle you hold the probe at

Radiopaedia article: change of wording may change the answer to the question

602
Q

22.2 A patient under general anaesthesia monitored with transcranial cerebral oximetry has a decrease in their cerebral oxygen saturation. This is likely to be improved by an increase in all of the following EXCEPT

A. Increasing blood pressure
B. Deepening anaesthesia
C. Increased minute ventilation
D. Transfusion

A

C. Increased minute ventilation

Cerebral blood flow
Cardiac output
Acid–base status
Major haemorrhage
Arterial inflow/venous outflow obstruction

Oxygen content
Haemoglobin concentration
Haemoglobin saturation
Pulmonary function
Inspired oxygen concentration
Inspired oxygen concentration

603
Q

23.1 A 25-year-old woman has critical bleeding following major trauma. Her blood group is unknown. Fresh frozen plasma that she receives should ideally be from

A. Any
B. A
C. B
D. AB
E. O

A

D - AB
Group AB plasma or group A plasma that is high-titre negative can be given in an emergency when the blood group is unknown. Group AB plasma is universal but in short supply.

604
Q

21.2 The relatively slower onset of action of bupivacaine with adrenaline in brachial plexus anaesthesia compared to other local anaesthetics relates to

a) lipid solubility
b) pKa
c) protein binding
d) vasoconstriction

A

b) pKa
Onset = pKa
Duration = Lipophilicity
Offset = protein binding
BJA: Basic pharmacology of local anaesthetics

https://www.bjaed.org/article/S2058-5349(19)30152-0/fulltext

Local anaesthetic agents are amphipathic molecules.

They bind primarily to sodium channels but also to potassium and calcium channels, and G-protein-coupled receptors.

Structural modifications alter the physicochemical characteristics of a local anaesthetic.

Speed of onset, potency, and duration depend on the pKa, lipid solubility and protein binding, respectively.

All local anaesthetic agents carry a risk of toxicity.

605
Q

22.1 Moderate obstructive sleep apnoea in children is diagnosed by an apnoea-hypopnoea index of

a. 5-10
b. 10-15
c. 15-20
d. 20-25
e. 25-30

A

a. 5-10

606
Q

20.2 The initial dose of IV adrenaline recommended for Grade 2 (moderate) anaphylaxis in an adult is

a) 10mcg
b) 20mcg
c) 100mcg
d) 500mcg
e) 1000mcg

A

b) 20mcg

Grade (ANZAAG)
1 - mucocutaneous only (mild)
2 - mucocutaneous and hypotension and/or bronchospasm (moderate)
3 - life threatening hypotension and/or high airway pressure (severe)
4 - arrest

For adults, put 3mg into a 50ml syringe
(or 6mg into 100mls saline; and running in mls/hr = mcg/min)
Doses:
- 20mcg = Grade 2
- 100-200mcg = Grade 3
- 1mg = Grade 4

For Paediatrics:
- put 1mg into 50ml syringe, (20mcg/ml; run @ 0.3ml/kg/hr = 0.1mcg/kg/min)
- 2mcg/kg = Grade 2 (0.1ml/kg of this dilution)
- 4-10 mcg/kg = Grade 3
- 10 mcg/kg = Grade 4 (0.1ml/kg of 1:10 000 (i.e. 100mcg/ml concentration))

  • IM doses are:
    > 150mcg if <6 yrs
    > 300mcg if 6-12yrs;
607
Q

23.1 In a 20-year-old with cystic fibrosis, the most likely finding on pulmonary function
tests is

a. Mixed obstruction and restrictive pattern
b. Restrictive with normal DLCO
c. Restrictive with low DLCO
d. Obstruction with reduced RV
e. Obstructive with reduced FEV1

A

e. Obstructive w/ reduced FEV1

Mucous narrowing airways = obstructive
Parenchymal damage = restrictive

Obstructive PFP remains the most common pulmonary function pattern in adult CF and is associated with
-decrease FEV1 & FVC/FEV1

For patients with CF, an obstructive pattern is generally seen, with a decrease in forced expiratory volume in 1 s (FEV1), and forced vital capacity (FVC) to FEV1 ratio.

https://academic.oup.com/bjaed/article/11/6/204/263786

608
Q

21.1 The direct physiological effects of electroconvulsive therapy include

a) reduced contractility
b) initial htn
c) initial bradycardia
d) initial tachycardia
e) reduced ICP

A

c) initial bradycardia

ANS Stimulation (PNS first, then SNS)
- Bradycardic/Tachycardic
- Hypertensive
Neuro
- Increased CMR/CBF/ICP
- Increased IOP
Increased Gastric pressure

609
Q

23.1 A 72-year-old woman on aspirin therapy presents to her ophthalmologist for follow up three days after you performed a transconjunctival peribulbar block for cataract surgery on her left eye. She complains of painless periorbital swelling, erythema and mild chemosis which started the day after surgery but is improving. She also had a peribulbar block three weeks ago for surgery on the other eye. The most likely diagnosis is

a. Retrobulbar haemorrhage
b. Residual swelling from peribulbar block
c. Periorbital cellulitis
d. Hyalase/hyaluronidase reaction/allergy
E. Conjunctivitis

A

d. Hyalase/hyaluronidase reaction/allergy

https://www.ncbi.nlm.nih.gov/pmc/articles/PMC4850816/

610
Q

21.1 A third heart sound at the apex may be heard with

a) pulmonary stenosis
b) pulmonary hypertension
c) pericarditis
d) pregnancy

A

d) pregnancy

A third heart sound reflects rapid left ventricular distention along with an increased atrioventricular flow

Heard in Congestive heart failure

Associated with Dilated Cardiomyopathy with dilated ventricles

Less commonly valvular regurgitation and left to right shunts

May be normal physiological finding in patients less than 40yrs old

611
Q

21.1, 22.2 A patient requiring an elective joint replacement has had a recent stroke. The minimum time to wait after the stroke before proceeding with surgery is

a. 3 months
b. 6 months
c. 9 months
d. 12 months

A

c. 9
AHA guidelines

12 Months
But 12 weeks minimum

Although the evidence between surgical timing and stroke risk is limited to only these 2 studies, we suggest that elective noncardiac surgery be deferred at least 6 months after a prior stroke, and possibly as long as 9 months to reduce the risk of perioperative stroke in patients undergoing noncardiac surgery.

Alternatively, patients who stand to gain significant improvements in quality of life with elective surgery may consider waiting only 6 months after a prior stroke

612
Q

21.1 In cardiac surgery, volatile-based anaesthesia compared to total intravenous anaesthesia

a) Lower 30 day post-op mortality
b) Higher 30 day post-op mortality
c) Lower post-operative MI
d) No difference

A

d) No difference

no observed beneficial effect of sevoflurane on the composite endpoint of prolonged ICU stay, mortality, or both in patients undergoing high-risk cardiac surgery

613
Q

20.1 What is the arrow pointing to?

a. Ilioinguinal
b. Iliohypogastric
c. Genitofemoral
d. Psoas
e. Obturator

A

b. Iliohypogastric

614
Q

22.1 In the World Maternal Antifibrinolytic (WOMAN) trial, tranexamic acid administration within three hours of birth reduced the

a) Decreased all cause mortality
b) Decreased mortality due to bleeding
c) Decreased transfusion
d) Decreased use of Bakri balloons
e) Increased rate of VTE

A

b) Decreased mortality due to bleeding

TXA decreased death due to bleeding.

No difference in all cause mortality.
No difference in use of blood products. No difference in surgical interventions. No difference in thromboembolic events.

615
Q

21.2 The use of erythropoietin before major surgery results in
a) Less transfusion, same thrombosis
b) Less transfusion, more thrombosis
c) No change in transfusion or thrombosis
d) No change in transfusion, more thrombosis

A

a) Less transfusion, same thrombosis

●A 2019 meta-analysis of randomized trials comparing preoperative administration of EPO versus placebo (32 trials; 4750 patients, mostly orthopedic and cardiac surgery) found reduced blood transfusions in the EPO groups. Decreased blood transfusions were seen in the entire population (RR 0.59, 95% CI 0.47-0.73; 28 trials), as well as the subgroups undergoing cardiac surgery (RR 0.55, 95% CI 0.47-0.73; nine trials) and major orthopedic surgery (RR 0.36, 95% CI 0.28-0.46; five trials). In addition, the EPO group had increased hemoglobin levels. There was no increase in the incidence of thromboembolic events with EPO.

616
Q

21.2 Painless post-operative visual loss with preserved pupillary reflexes is most likely due to

a) Retinal detachment
b) Anterior ischaemic optic neuropathy
c) Corneal abrasion
d) Posterior ischaemic optic neuropathy
e) Posterior cerebral ischaemia

A

PCA

e) Posterior cerebral ischaemia

UTD: Postoperative visual loss after anaesthesia for nonocular surgery

Pupillary light reflexes*
Unilateral central retinal artery occlusion, ischemic optic neuropathy, and retrobulbar hematoma result in a poor or absent pupillary response to light (“direct” response) with a normal response when light is directed to the other pupil (“indirect” response); this “relative afferent pupillary defect” is revealed when tested with the swinging flashlight maneuver; if these processes are bilateral, there will be poor or absent direct pupillary responses and a relative afferent pupillary defect only if asymmetric.
Mid-dilated and nonreactive pupils are consistent with acute angle-closure glaucoma, while sluggish to fixed and dilated pupils are seen with glycine-induced visual loss.
Pupillary light reflexes are normal in cases of corneal abrasion, cerebral or cortical visual loss, and in cases of PRES. Examination of pupils is discussed more fully separately.

617
Q

23.1 The difference between a size 5.0 microlaryngeal tube (MLT) and a standard size 5.0 endotracheal tube is that the size 5 MLT

A. Smaller cuff
B. Longer length
C. Larger external diameter

A

Longer length

Different cuff size/ length: The MLT® has a cuff size/ length that would be typical for an adult-sized ‘standard’ ETT. A ‘standard’ pediatric 5.0 enndotracheal tube has a smaller cuff made for a pediatric-sized trachea (see picture below).

Distance of cuff from tube tip: In an MLT® the cuff is further away from the tube tip which is acceptable as the adult trachea is obviously longer than the pediatric one (see picture below).

https://aam.ucsf.edu/microlaryngoscopy-tube-mlt%C2%AE

618
Q

20.1 A patient with persistent pain on oral hydromorphone 12mg per day is admitted to hospital unable to tolerate oral intake. The equivalent parenteral morphine dose per day is:

a) 12mg
b) 20mg
c) 40mg
d) 60mg
e) 80mg

A

b) 20mg

hydromorphone PO: morphine PO = 1: 5
So 12mg x 5= 60mg Morphine PO
Which to convert to PO morph: IV morph is 3:1, so 60mg/3 = 20mg of parenteral morphine

IV hydromorphone:
IV hydromorphone 1mg = 15mg PO Morphine = 5mg IV morphine.
How to remember this:
- hydromorphone PCA is a 200mcg (1ml) bolus; 20mg into 100mls for PCA; therefore 20mg IV hydromorphone = 100mg IV morphine (i.e. 300mg PO morphine)
- vial in recovery for pain protocol also comes as 2mg (i.e. 10mg IV morphine equivalent)

619
Q

23.1 An adult patient undergoing surgical aortic valve replacement is in ventricular fibrillation after the removal of the aortic cross clamp and requires internal defibrillation. It has been shown it is safe to deliver a charge of up to:

a) 10J
b) 20J
c) 30J
d) 50J
e) 100J

A

B. 50J ?

In internal defibrillation, an initial dose of 20 joules is recommended to avoid burn-like injury to the myocardium. Care should be taken to avoid coronary vessels to prevent vessel damage. Subsequent doses can be increased to a maximum of 40 joules. Sterile internal pads must be used for internal defibrillation and should be readily available during any thoracotomy procedures

Manufacturing manual for internal definition paddles say 50J maximum as higher has been shown to be damaging

https://www.ncbi.nlm.nih.gov/books/NBK499899/

620
Q

21.1 The image below shows a normal central venous pressure (CVP) trace on the left. The CVP trace shown on the right is most consistent with

a. Tricuspid regurg
b. Mitral stenosis
c. Mitral regurg
d. Pericarditis
e. Tamponade

A

a. Tricuspid regurg

Regurgitant CV waves: tricuspid regurgitation

In tricuspid regurgitation, the backflow of blood out of the right ventricle obliterates the normal x descent. The c wave becomes accentuated and fuses with the v wave, as both are the results of right ventricular contraction (and the v wave peak pressure is often the same as the right ventricular peak systolic pressure).

the reality is that they usually fuse completely to produce huge mutant waves, as seen here:

621
Q

21.2 Allergic cross-reactivity between penicillins and cephalosporins is mediated by the

a) R1 side chain
b) R2 side chain
c) Beta lactam ring
d) Imidazole group

A

a) R1 side chain

UP TO DATE:
- sensitisation to R1 side chain in cephalosporins important in determining cross reactivity with penicillins.

622
Q

20.2, 23.2 Complications of hyperbaric oxygen therapy do NOT include

a) Myopia
b) Central retinal occlusion
c) Seizures
d) Hypoglycaemia
e) Bradycardia

A

b) Central retinal occlusion

SE’s from HBOT:
- progressive myopia (reversible)
- seizures
- hypoglycaemia
- sinus bradycardia from stimulation of vagal activity bassociated with hyperbaric pressures

623
Q

In elderly non-diabetic patients, the use of aspirin in primary prevention of disease

A) Increased risk of bleeding
B) Reduced overall mortality
C) Reduced CVS mortality
D) Reduced cancer mortality
E) ?

A

Unclear: age of patient not given in question, real answer for patient >70yrs old appears to be lack of net benefit but this is not a remembered option

Answer: A) increased incidence of major bleeding

Low-dose aspirin should not be administered on a routine basis for primary prevention of ASCVD among adults >70 years.

Low-dose aspirin should not be administered on a routine basis for primary prevention of ASCVD among adults >70 years.

2019 ACC/AHA Guideline on the Primary Prevention of Cardiovascular Disease

Aspirin
Candidates — For the secondary prevention of ASCVD in patients with diabetes, we recommend aspirin (75 to 162 mg daily).

For the primary prevention of ASCVD in patients with diabetes at increased cardiovascular risk (10-year risk >10 percent), we suggest aspirin (75 to 162 mg daily), although the evidence supporting this approach is weak and needs to be balanced with the increased risk of gastrointestinal bleeding.

We do not routinely use aspirin for the prevention of ASCVD in adults with diabetes at low risk (10-year ASCVD risk <10 percent). (See ‘Guidelines’ below.)
2019 ACC/AHA Guideline on the Primary Prevention of Cardiovascular Disease
Aspirin

Aspirin is well established for secondary prevention of ASCVD and is widely recommended for this indication, but recent studies have shown that in the modern era, aspirin should not be used in the routine primary prevention of ASCVD due to lack of net benefit. Most important is to avoid aspirin in persons with increased risk of bleeding including a history of GI bleeding or peptic ulcer disease, bleeding from other sites, age >70 years, thrombocytopenia, coagulopathy, chronic kidney disease, and concurrent use of nonsteroidal anti-inflammatory drugs, steroids, and anticoagulants.

The following are recommendations based on meta-analysis and three recent trials:

Low-dose aspirin might be considered for primary prevention of ASCVD in select higher ASCVD adults aged 40-70 years who are not at increased bleeding risk.

Low-dose aspirin should not be administered on a routine basis for primary prevention of ASCVD among adults >70 years.

Low-dose aspirin should not be administered for primary prevention among adults at any age who are at increased bleeding risk.

624
Q

21.2 The cardiac axis of this electrocardiogram is

a) -30
b) 0
c) 45
d) -90

A

a) -30

Normal Axis

https://litfl.com/super-axis-man-sam/

LITFL

625
Q

20.1 A patient has prolonged surgery with a laryngeal mask airway. The following day she reports a problem with her tongue. You examine her and see the following when she protrudes her tongue. The most likely cause of the abnormality is (facial picture shown)

a. Left hypoglossal nerve
b. Left glossopharyngeal
c. Right hypoglossal
d. Right glossopharyngeal
e. Right recurrent laryngeal

A

C

Ipsilateral deviation

https://www.ncbi.nlm.nih.gov/pmc/articles/PMC4308816/

626
Q

21.2 A trainee becomes aware that a patient they have just anaesthetised for emergency surgery is breastfeeding and seeks your advice regarding recommencement of breast feeding. You advise that breast feeding is contraindicated because during the admission today the patient
received

a) Tramadol
b) Codeine
c) Ketamine
d) Midazolam

A

Codeine

Source Appendix ANZCA PG 07

627
Q

22.1 When using ROTEM thromboelastometry, the APTEM test is used to assess

a. Fibrinolysis
b. Platelet function
c. Coagulation factors

A

Fibrinolysis

In APTEM, coagulation is also activated as in EXTEM. By the addition of aprotinin or tranexamic acid in the reagent, fibrinolytic processes are inhibited in vitro.

The comparison of EXTEM and APTEM allows for a rapid detection of fibrinolysis. Furthermore, APTEM enables the estimation if an antifibrinolytic therapy alone normalises the coagulation or if additional measures have to be taken (e.g. administration of fibrinogen).

628
Q

21.1 The intubating dose of atracurium in a patient with post-polio syndrome should be

a. 10 %
b. 20
c. 50
d. 100
e. 200

A

c. 50%
0.25mg/kg (Half)

Source: PolioSA

And ANZCA bulletin 2015 ‘Anaesthetists need to be wary of post polio syndrome’ -“twice as sensitive to non-depolarising muscle relaxants”

629
Q

20.2 A 50 year old man has the following pulmonary function test result.

FEV1 68% predicted,
FVC 68% predicted,
DLCO 46% predicted

The most consistent diagnosis is

a) Asthma
b) Myasthenia Gravis
c) Emphysema
d) Sarcoidosis
e) Pulmonary Hypertension

A

Repeat

d) Sarcoidosis

Normal FEV1/FVC ratio = no obstruction
Low FVC = restrictive pattern
Low DLCO = interstitial lung disease

Asthma and emphysema would have obstructive pattern.
Myasthenia gravis would have normal DLCO
Pulmonary HTN would have normal spirometry and low DLCO.

630
Q

23.1 In Australia and New Zealand, the proportion of blood donors who are cytomegalovirus (CMV) seropositive is

a. 65 to 85 per million
b. 650 to 850 per million
c. 6.5 to 8.5 per hundred
d. 65 to 85 per hundred

A

d. 65 to 85 per hundred
Risk of acquiring CMV through a leucodepleted blood product is estimated at around 1 in 13,575,000. This compares to a community acquired risk where 85% of Australian adults are infected by the age of 40.

85% of australians are CMV positive by the age of 40

https://www.blood.gov.au/system/files/documents/cmv-blood-components.pdf

631
Q

An ASA 1 28-year-old man attends for inguinal hernia repair under general anaesthesia. He is administered propofol 180mg morphine 8mg rocuronium 50mg cephazolin 2g Post induction he develops an erythematous rash on his chest and arms, swelling of his lips and face, and severe hypotension. Preliminary blood results show: (allergy related tests shown).

Tryptase at 1 hour 321 (11)
Tryptase at 3 hours 58 (11)
RAST Morphine 29 (15)
Serum IgE 88 (300)

The most likely diagnosis is

a. Morphine anaphylaxis
b. Rocuronium anaphylaxis
c. Cephazolin Anaphylaxis
d. Propofol Anaphylaxis
e. Opioid related histamine release

A

Answer: b. rocuronium anaphylaxis

NB
RadioAllergoabsorbentSpecificTesting is a serum test for specific IgE antibodies
RAST morphine is both more sensitive and more specific than the RAST for individual NMBDs (due to reaction with quaternary ammonium) and is being used increasingly to determine NMBDs as cause of anaphylaxis. IKR!

http://www.anzaag.com/anaphylaxis-management/testing-guidelines.pdf

632
Q

21.2 Risks associated with robot-assisted laparoscopic prostatectomy surgery in comparison with
open prostatectomy include all of the following EXCEPT

a) CO2 embolism
b) cerebral oedema
c) corneal burns
d) major haemorrhage

A

d) major haemorrhage
- blood loss is significantly less with RALP

Up to date: RALP

633
Q

22.2 Cyclooxygenase type 2 inhibitors (COX-2) in pregnancy are considered

a. Not safe
b. safe
c. safe only in 1st trimester
d. safe only in 1st and 3rd trimester
e. not safe for 3rd trimester and 48 hours post delivery

A

a. Not safe
or
c. safe only in 1st trimester

While relatively safe in early and mid pregnancy, NSAIDs can precipitate fetal cardiac and renal complications in late pregnancy, as well as interfere with fetal brain development and the production of amniotic fluid; they should be discontinued in gestational wk 32

APMSE

634
Q

20.2 Adverse effects of the use of SGLT 2 inhibitors in the perioperative period do NOT include

a) Hypocalcemia
b) Hypotension
c) Euglycemic dka
d) Hyperglycemic DKA
e) Urinary Tract Infections

A

Answer: Hypocalcaemia
● AMH
○ Increased risk of adverse effects related to volume depletion(e.g. hypotension, fainting)
○ UTI

635
Q

A patient with known suxamethonium allergy is most likely to demonstrate cross reactivity with

a. Mivacurium
b. Cisatracurium
c. Atracurium
d. Rocuronium
e. Cephazolin

A

Answer: d. Rocuronium

BJA Anaphylaxis to neuromuscular blocking drugs: incidence and cross-reactivity in Western Australia from 2002 to 2011
https://academic.oup.com/bja/article/110/6/981/245571

Rocuronium has a higher rate of IgE-mediated anaphylaxis compared with vecuronium, a result that is statistically significant and clinically important.

Cisatracurium had the lowest rate of cross-reactivity in patients who had previously suffered anaphylaxis to rocuronium or vecuronium.

Anaphylaxis rates (highest to lowest)
Primary anaphylaxis: rocuronium > atracurium > vecuronium > pancuronium = cisatracurium
Cross-reactivity: suxamethonium > rocuronium > vecuronium > pancuronium > atracurium > cisatracurium

636
Q

21.1 The recommended antibiotic prophylaxis for insertion of an intrauterine device is

a. cephalexin PO
b. cefazolin IV
c. doxycycline PO
d. none

A

d. none

Increase in presence of mycobacterium vaginosis, doxycylcine will kill commensal bacteria

Doxycycline is used for copper IUD in the setting of emergency insertion with PID

637
Q

Of the following, the condition that is an absolute contraindication to administration of electroconvulsive therapy is

A. Cochlear implant
B. PPM
C. Elevated ICP
D. Epilepsy
E. Pregnancy

A

C. Elevated ICP

  • No Absolute contraindications
  • Relative contraindications
    1. Raised ICP or space occupying lesion
    2. MI within the last 3 months
    3. Severe arterial hypertension
    4. Acute Glacoma
    5. Changes in the cerebral arteries e.g. aneurysm

Pregnancy and Pacemakers are not contraindications to ECT

Indications:
1. Depression (most common)
2. MDD with psychotic features
3. Schizoaffective disorder
4. Schizophrenia with catatonia
5. Highly suicidal or depressed pregnant patients (not first line)
6. Bipolar affective disorder
7. Neuroleptic malignant syndrome

638
Q

22.2 A 25-year-old male has continued postoperative bleeding after an extraction of an impacted third molar tooth under a general anaesthetic. The patient mentions that his father bruises quite easily. His coagulation screen reveals: (provided). The most likely diagnosis is
(APTT raised, PT normal?)

a. Factor V leiden
b. haemophilia A
C. Von willebrand’s disease
D. Haemophilia B

A

b. von willebrand’s disease

  • autosomal dominant inheritance
  • may have normal or prolonged APTT, PT is normal

*Haem A: X-linked recessive disorder; would expect prolonged aPTT, and normal PT
*Haem B: X-linked recessive disorder; would expect normal aPTT and normal PT

REPEAT

vWD can have prolonged APTT or normal APTT. Haemophilias are X-linked

639
Q

22.1 A patient has severe hypokalaemia and is in cardiac arrest. The Australian Resuscitation Council and the New Zealand Resuscitation Council recommend intravenous potassium should be given as

a) 5mmol bolus KCl
b) 10mmol bolus KCl
c) 5mmol KCl over 5min
d) 5mmol KCl over 10min
e) 20mmol KCl over 10min

A

5 mmol bolus KCl

3.6 Potassium
Potassium is an electrolyte essential for membrane stability. Low serum potassium, especially in conjunction with digoxin therapy and hypomagnesaemia, may lead to life threatening
ventricular arrhythmias.

Consider administration for:
* Persistent VF due to documented or suspected hypokalaemia.
[Class A; Expert consensus opinion]
ANZCOR Guideline 11.5 August 2016 Page 9 of 13
Adverse effects:
* Inappropriate or excessive use will produce hyperkalaemia with bradycardia,
hypotension and possible asystole
* Extravasation may lead to tissue necrosis.

Dosage:
A bolus of 5 mmol of potassium chloride is given intravenously

640
Q

22.1A 63-year-old woman is to undergo an elective total hip replacement. Her past medical history includes hypertension, stroke, type 2 diabetes mellitus, chronic atrial fibrillation and chronic renal impairment with an estimated creatinine clearance of 46 mL/min. Her medications include dabigatran 150 mg bd for stroke prevention. Perioperatively, her dabigatran therapy should

a. Be withheld 2 days
b. Withhold 3 days
c. Withhold 5 days
d. Withhold 6 days
e. Continue

A

5d

ANZCA - CrCl >80 (3D) 80-50 (4D) <50 (5D)

641
Q

21.2 Findings associated with massive pericardial tamponade include

a) Pulsus paradoxus
b) Electrical alternans
c) Kussmaul sign
d) Pericardial rub

A

a) Pulsus paradoxus electric alternans

Physical findings in Tamponade:
- A number of findings may be present on physical examination, depending upon the type and severity of cardiac tamponade
- None of the findings alone are highly sensitive or specific for the diagnosis.

Beck’s triad
1. Low arterial blood pressure
2. Dilated neck veins
3. Muffled heart sounds
- Are present in only a minority of cases of acute cardiac tamponade.

Physical findings of sinus tachycardia and the absence of frank hypotension
may indicate significant hemodynamic compromise from cardiac tamponade and serve as an indication for immediate pericardiocentesis.

In contrast, Kussmaul sign (the absence of an inspiratory decline in jugular venous pressure) is not usually seen in cardiac tamponade.

Tachycardia and hypotension
- Sinus tachycardia is seen in almost all patients, in an attempt to maintain cardiac output
- Hypotension is somewhat more variably present
- One exception is when the underlying disease is associated with bradycardia, as with a pericardial effusion and subacute cardiac tamponade associated with hypothyroidism.
- Tachycardia also may not be seen in patients with early cardiac tamponade even though they have signs of a hemodynamically significant effusion, such as an elevated jugular venous pressure.

Elevated jugular venous pressure
- The JVP is almost always elevated in cardiac tamponade and may be associated with venous distension in the forehead and scalp.
- CVP wave form:
->x descent is preserved
->y descent is attenuated or absent
(due to the limited or absent late diastolic filling of the ventricle)

Pulsus paradoxus
-defined as: abnormally large decrease in systolic blood pressure (>10 mmHg) on inspiration
- common finding in moderate to severe cardiac tamponade and is the direct consequence of ventricular interdependence.
- not all patients with cardiac tamponade have pulsus paradoxus (eg, those with chronic hypertension leading to elevated ventricular diastolic pressures or those with a co-existent atrial septal defect).

Pericardial rub — A pericardial rub may be heard in patients with cardiac tamponade due to inflammatory pericarditis.

Electrocardiography
- ECG in cardiac tamponade typically shows sinus tachycardia and may also show low voltage.
- If pericarditis is present, the ECG findings typical of that disorder are also seen.
- Electrical alternans is characterized by beat-to-beat alterations in the QRS complex and, in some cases, other electrocardiographic waves that reflect the swinging of the heart in the pericardial fluid.
- Electrical alternans is relatively specific but not very sensitive for cardiac tamponade; rarely, this phenomenon is seen with very large pericardial effusions alone.

Diagnosis:
Clinical diagnosis is usually suspected based on the history and physical examination findings, which may include:
●Chest pain
●Syncope or presyncope
●Dyspnea and tachypnea
●Hypotension
●Tachycardia
●Peripheral edema
●Elevated jugular venous pressure
●Pulsus paradoxus

Presence of a pericardial effusion on echocardiography with evidence of cardiac chamber collapse, flow variation, or dilation of the inferior vena cava is consistent with, and highly suggestive of, cardiac tamponade.

However, the diagnosis of cardiac tamponade can only be confirmed by the hemodynamic and clinical response to pericardial fluid drainage.

642
Q

21.2 A patient with an acute subarachnoid haemorrhage arrives in the emergency department. Her Glasgow Coma Scale is 10 and she has no motor deficit. A CT brain shows diffuse subarachnoid haemorrhage with no localised areas of blood > 1mm thick, and no intracerebral nor intraventricular blood. Her World Federation of Neurosurgical Societies
(WFNS) grade of subarachnoid haemorrhage is

a) 1
b) 2
c) 3
d) 4
e) 5

A

d) 4
- WFNS is 4
* alternatively her Fisher score is: grade 2 (​diffuse thin (<1 mm) SAH, no clots; which estimates an incidence of symptomatic vasospasm of 25%)

Note the new modified Fischer scale.
G0 No SAH or IVH (0%)
G1 Focal or diffuse thin SAH but no IVH (6-24%)
G2 Focal or diffuse thin SAH with IVH (15-33%)
G3 Thick SAH no IVH (33-35%)
G4 Thicc SAH with IVH (34-40%)

The main differences between the Fisher scale and modified Fisher scale are:
1) Fisher scale, no SAH is grade 1, but 0 in modified Fisher scale
2) Fisher scale, thin SAH & no IVH is grade 2, but 1 in modified Fisher scale
3) Fisher scale, thick SAH with no IVH is grade 3 and the same 3 in modified Fisher scale
4) Fisher scale, any IVH is grade 4, irrespective of the presence of SAH but in modified Fisher scale it is either 2 if thin or no SAH, or grade 4 if thick SAH

643
Q

20.1 Bleeding post AFE what’s contraindicated?

a) FFP
b) Cryoprecipitate
c) Platelets
d) Novoseven (Factor 7a)
e) Prothombinex

A

e) Prothrombinex

may potentiate DIC due to increasing thrombotic tendancy

Australian redcross

644
Q

23.1 The medication most strongly associated with an acute primary hypotensive reaction following transfusion of blood products is

A. Aspirin
B. Metoprolol
C. Hydralazine
D. perindopril

A

D. perindopril

Acute hypotensive transfusion reaction (AHTR) is characterized by the abrupt onset of hypotension immediately after the start of transfusion and usually resolves when transfusion ceases. Recent studies have shown an association with pre-operative treatment with an angiotensin-converting enzyme (ACE) inhibitor

https://www.lifeblood.com.au/health-professionals/clinical-practice/adverse-events/hypotension

645
Q

20.2 A patient is in sinus rhythm at a heart rate of 60 /minute and a QT interval of 420 msec. The corrected QT interval is

a) 360 ms
b) 380 ms
c) 420 ms
d) 460 ms
e) 480 ms

A

c) 420 ms
i.e. RR interval = 60 / HR; therefore RR interval is 60/60 = 1
420 / square root of 1 is 420ms

From LITFL:
The corrected QT interval (QTc) estimates the QT interval at a standard heart rate of 60 bpm
QT:
Time from the start of the Q wave to the end of the T wave
Represents time taken for ventricular depolarisation and repolarisation, effectively the period of ventricular systole from ventricular isovolumetric contraction to isovolumetric relaxation
The QT interval is inversely proportional to heart rate:
The QT interval shortens at faster heart rates
The QT interval lengthens at slower heart rates
An abnormally prolonged QT is associated with an increased risk of ventricular arrhythmias, especially Torsades de Pointes

Corrected QT interval (QTc)
The corrected QT interval (QTc) estimates the QT interval at a standard heart rate of 60 bpm
This allows comparison of QT values over time at different heart rates and improves detection of patients at increased risk of arrhythmias
Bazett formula: QTC = QT / √ RR

Note: The RR interval is given in seconds (RR interval = 60 / heart rate).

646
Q

22.1 To allow cardiopulmonary bypass in a patient with heparin resistance, fresh frozen plasma may be administered in order to increase the level of

A

ATIII

647
Q

23.1 A diagnosis of metabolic syndrome is NOT supported by

A. Impaired glucose tolerance
B. High HDL
C. Obesity
D. High triglycerides
E. Hypertension

A

b. high HDL-C

https://www.ahajournals.org/doi/10.1161/CIRCULATIONAHA.105.169405

648
Q

20.1 Patient on chronic daily oral hydromorphone 12mg, what is an appropriate daily parenteral morphine dose

a. 5
b. 10
c. 15
d. 20
e. 25mg

A

20mg

12mg PO hydromorphone = 60mg PO morphine
(Factor of 5)

PO - IV Morphine = factor of 3

FPM App

649
Q

22.2 A patient with acute right heart failure secondary to acute myocardial infarction is likely to have a/an

a. Increased PA pulsatility index
b. Increased tricuspid annular plane systolic excursion
c. Decreased PAP
d. Raised JVP
e. Decreased PA pulsatility index

A

PAPi goes down in acute RVF
- actually designed to prognosticate in acute MI

E is the answer

https://www.ahajournals.org/doi/10.1161/CIRCHEARTFAILURE.121.009085#:~:text=The%20pulmonary%20artery%20pulsatility%20index,left%20ventricular%20assist%20device%20implantation.

650
Q

22.2 You have been managing a case of malignant hyperthermia in an 80 kg man and have given a total of 400 mg of dantrolene (Dantrium). The amount of mannitol you have also administered is

a. None
b. 1.6g
c. 12g
d. 40g
e. 60g

A

e. 60g
Each 20mg dantrolene contains 3g mannitol

651
Q

20.1 A 56 year old man has suffered a TBI. What SBP (mmHg) would you aim for?

a) 90
b) 100
c) 110
d) 120
e) 140

A

b) 100

Brain trauma foundation
Level III recommendation.
To decrease mortality and improve outcomes:

Maintain SBP at >100mmHg for patients 50 - 69
Maintain SBP at >110 for patients 15 - 49
Maintain SBP at >110 for patients 70 or older

652
Q

20.2 In the fluid resuscitation of a patient with an isolated severe head injury, the LEAST appropriate fluid is

a) Hypertonic saline 7.5%
b) 4% albumin
c) NaCl 0.9%
d) Plasmalyte
e) Saline 3%

A

b) 4% albumin

  • SAFE study (2004) showed increased mortality at 24 months when albumin used as resuscitation fluid cf normal saline.
  • Also caused higher ICP at 1 week post injury.
  • 4% albumin (274 mOsm/L & 266 mOsm/kg) is hypotonic and hypoosmolar.
653
Q

21.1 The abnormality shown in this image is LEAST likely to be caused by an injury to the

a) Accessory nerve
b) Dorsal scapular nerve
c) Long thoracic nerve
d) Supraclavicular nerve

A

Supraclavicular nerve – it is a superficial sensory nerve arising from C3/C4.

Long thoracic nerve innervates serratus anterior muscle. Paralysis of this leads to medial winging of scapula (weakness of shoulder abduction).

Lateral winging of scapula (where lateral side protrudes) is due to weakness of trapezius or rhomboid muscles which are innervated by spinal accessory nerves and dorsal scapular nerve respectively.

Dorsal scapular nerve -> winging
Accessory nerve -> winging S
uprascapular nerve -> maybe
Subscapular nerve - > maybe

654
Q

22.1 Extended life plasma is thawed fresh frozen plasma which can be stored at 2 to 6 C for a
maximum period of

a. 2 days
b. 3 days
c. 5 days
d. 7 days

A

5 days

Previous MCQ2015A – cryoprecipitate once thawed must use within 4 hours.

Previous MCQ2015B – FFP must be transfused within 4 hours once thawed, or stored at 2-6 degrees for 5 days.

655
Q

22.2 The rate of drainage of cerebrospinal fluid via a lumbar drain is NOT influenced by the

a. Height of bed
b. Height of drainage chamber
c. Height of highest part of drainage system
d. Position of patient
e. Spinal level of drain

A

e. Spinal level of drain

According to AANN2 and SNACC4 Guidelines:
* Patient positioning and leveling is crucial to prevent complications from lumbar drainage
* The head of the bed, height of drainage chamber, and changes in patient positioning must be monitored closely to prevent sudden overdrainage
* While making changes to the patient’s positioning, the lumbar drainage device should be clamped so that overdrainage does not occur

https://www.integralife.com/file/general/1604065981.pdf
(manufacturer’s instructions)

656
Q

23.1 You are called to recovery to review an 80-year-old woman after neck of femur fracture fixation performed under general anaesthesia with a fascia iliaca block. She has a history of mild dementia. She has become confused and agitated after initially being cooperative and is pain-free. The most appropriate drug therapy to manage her is intravenous

a. Clonidine
b. dexmedetomidine
c. propofol
d. midazolam
e. haloperidol

A

e) haloperidol

Bluebook - suggest antipsychotics with caution

657
Q

22.2 A 6-year-old patient (140 cm, 24 kg, BSA 0.97m2) is on hydrocortisone 15 mg/day. Perioperative glucocorticoid supplementation is (considered if)

a.
b. Taking >1week
c. Taking >1 month
d. Taking >2 months
e. Taking >4 months

A

Taking > 1 month

https://associationofanaesthetists-publications.onlinelibrary.wiley.com/doi/full/10.1111/anae.14963

Daily doses of prednisolone of 5 mg or greater in adults and 10–15 mg.m−2 hydrocortisone equivalent or greater in children may result in hypothalamo–pituitary–adrenal axis suppression if administered for 1 month or more by oral, inhaled, intranasal, intra-articular or topical routes; this chronic administration of glucocorticoids is the most common cause of secondary adrenal suppression, sometimes referred to as tertiary adrenal insufficiency

All children who have known glucocorticoid deficiency (primary or secondary), or who are at risk of glucocorticoid deficiency (on significant exogenous dose of glucocorticoid >10–15 mg.m-2 per day) 38, should receive an i.v. dose of hydrocortisone at induction (2 mg.kg−1 for minor or major surgery under general anaesthesia).

658
Q

20.1 In patients with IgE-mediated allergy to penicillin, the rate of anaphylaxis to cefazolin is estimated to be

a) 1%
b) 2%
c) 5%
d) 10%
e) 20%

A

1%

the overall cross-reactivity rate is approximately 1% when using first-generation cephalosporins or cephalosporins with similar R1 side chains.”

J Emerg Med. 2012 May;42(5):612-20.
doi: 10.1016/j.jemermed.2011.05.035.Epub 2011 Jul 13.

659
Q

21.1 Globe perforation during eye block is more common in myopic eyes because

a) Higher rate of increased IOP
b) Globe is too short
c) Incidence of staphyloma
d) Corneal thickness is less

A

c) Incidence of staphyloma

But also reduced space between globe and orbit

Axial eye length: implications for globe perforation during regional block –

  • The axial length of the eye (distance from the cornea to the retina) is routinely measured by ultrasound before cataract surgery to determine the proper intraocular lens size to be implanted.
  • It has been noted that patients with long eyes (axial length >25 mm) have an increased risk of needle injury during a retrobulbar (intraconal) block, usually due to penetration of the posterior pole of the globe.
  • Indications that the eye may be longer than average include a history of myopia in childhood (confirmed by an affirmative answer to the question, “Did the patient need to wear glasses as a child to see distant objects?”) or the presence of globe-enveloping intraorbital hardware, such as a scleral buckle.
  • Also, patients with an abnormal outpouching of the eye called a staphyloma, which is usually associated with an axial length >25 mm and is usually located in the posterior portion of the globe, are at increased risk for globe perforation by a retrobulbar needle
  • In such patients, retrobulbar block is usually avoided in favor of a peribulbar (extraconal) or sub-Tenon block, topical anesthesia, or general anesthesia.
660
Q

20.1 Following uneventful sinus surgery, a 40-year-old, otherwise healthy male taking no medications, wakes up with confusion, agitation, headache and photophobia. The anaesthetist provided induced hypotension with a 40 % reduction in mean arterial pressure intraoperatively. It is suspected that there has been a period of cerebral ischaemia. Over 24 hours the patient makes a full recovery. The best description of this episode is:

a) Near miss
b) Adverse event
c) Sentinel event
d) Malfeasance
e) Misconduct

A

C) Adverse event—a clinical incident in which unintended or unneccessary harm resulted.

Sentinel event: Sentinel events are a subset of adverse patient safety events that are wholly preventable and result in serious harm to, or the death of, a patient

Adverse event—a clinical incident in which unintended or unneccessary harm resulted.

Harm—impairment of structure or function of the body and/or any deleterious effects arising there from. Harm includes disease, injury, suffering, psychological harm, disability and death.*

Near miss: an incident or potential incident that was averted and did not cause harm, but had the potential to do so.

Near miss = an act that could have caused harm but was avoided
Sentinel event = serious permanent harm (there are 12 listed)
Adverse event = preventable event that did result in harm
Malfeasance = less clear, more lawyer talk, but caused harm
Misconduct = deliberate wrongful act

661
Q

20.2 The nerve(s) that need to be blocked with local anaesthetic to achieve complete anaesthesia for amputation of the fifth toe is/are

a) Posterior tibial and sural
b) Posterior tibial and superficial peroneal
c) Sural and superficial peroneal
d) Deep and superficial peroneal
e) Sural, deep peroneal, and posterior tibial

A

b) Posterior tibial and superficial peroneal

If answer includes:
Posterior Tibial, Sural and superficial peroneal this would be the most appropriate

Or

Posterior Tibial + Sciatic nerve as Sural, deep and superficial peroneal are all branches of the sciatic nerve

662
Q

20.1 In a patient with known COPD, which of the following post bronchodilator spirometry results is consistent with a GOLD 3 classification? (Global initiative for chronic Obstructive Lung Disease)

a) FEV1 83%
b) FEV1 57%
c) FEV1 43%
d) FEV1 27%
e) FEV1 19%

A

c) FEV1 43%

In pulmonary function testing, a post-bronchodilator FEV1/FVC ratio of <0.70 is commonly considered diagnostic for COPD. The Global Initiative for Chronic Obstructive Lung Disease (GOLD) system categorises airflow limitation into stages. In patients with FEV1/FVC <0.70:

GOLD 1 - mild: FEV1 ≥80% predicted

GOLD 2 - moderate: 50% ≤ FEV1 <80% predicted

GOLD 3 - severe: 30% ≤ FEV1 <50% predicted

GOLD 4 - very severe: FEV1 <30% predicted.
663
Q

22.2 Drug classes demonstrated to reduce mortality in chronic heart failure with reduced ejection fraction include all of the following EXCEPT

A. ACE inhibitors
B. Beta blockers
C. Angiotensin receptor blockers
D. Spironolactone
E. Digoxin

A

Digoxin

664
Q

23.1 The BALANCED Anaesthesia Study compared older patients having deep
anaesthesia (bispectral index target of 35) to lighter anaesthesia (bispectral index
target of 50). It assessed postoperative mortality, and a substudy assessed
postoperative delirium. These showed that, compared to light anaesthesia, deep
anaesthesia causes

a) Decreased mortality, no change in post op delirium (POD)
b) No change mortality, reduced POD
c) Decreased mortality, reduced POD
d) No change in Mortality, no change in POD

A

No change in Mortality, no change in POD

No evidence was found that mortality or serious complication were modified by targeting either a BIS of 50 or 35

A broad range of anaesthetic depth can be delivered safely when using volatile anaesthetic agents and processed electroencephalographic monitoring

https://www.thebottomline.org.uk/summaries/pom/balance/

665
Q

The needle tip pictured is called a

a Sprotte
b Whittacre
c Quincke
d Trocar
e Tuohy

A

c Quincke

Needles for spinal anesthesia or lumbar puncture can be classified according to the needle tip.

Cutting-tip, or Quincke, needles have sharp, cutting tips, with the hole at the end of the needle.

Whitacre and Sprotte needles are two types of pencil point, or noncutting tip needles. They have a closed tip shaped like a pencil, with the hole on the side of the needle near the tip.

Pencil point needles are designed to minimize leak of cerebrospinal fluid after puncture and reduce the chance of postdural puncture headache.

666
Q

23.1 Sacubitril use reduces the plasma levels of

A. NT proBNP
B. Angiotensin II
C. BNP
D. Neprolysin
E. Bradykinin

A

a) NT ProBNP

Sacubitril inhibits the enzyme neprilysin, which is responsible for the degradation of atrial and brain natriuretic peptide, two blood pressure–lowering peptides that work mainly by reducing blood volume.

Neprolysin breaks down AT2 which leads to an accumulation, therefore an ARB is required in combination to block the effects of increased AT2.

BNP may increase with treatment - particularly in the early phase. NT-proBNP is not a substrate of neprolysin and so will not increase. Successful treatment will show a fall in NT-ProBnP.

“patients receiving LCZ696 had consistently lower levels of NTproBNP (reflecting reduced cardiac wall stress) and troponin (reflecting reduced cardiac injury) throughout the trial.”

https://www.ahajournals.org/doi/10.1161/CIRCULATIONAHA.114.013748?url_ver=Z39.88-2003&rfr_id=ori:rid:crossref.org&rfr_dat=cr_pub%20%200pubmed

667
Q

21.2 A 45-year-old man has the following results on his blood biochemistry testing: The most likely diagnosis is

  • Bili 30*
  • AST 1000*
  • ALT 500*
  • Albumin 30*
    *These blood results are not the original stem.

The most likely diagnosis is:

a) Hepatitis
b) Alcoholic liver disease
c) Paracetamol toxicity
d) Cholecystitis

A

b) Alcoholic liver disease
- AST>ALT

In hepatitis and paracetamol toxicity would expect ALT>AST.

In cholecystitis, would expect a cholestatic picture with raised conjugated bilirubin and raised GGT/ALP.

LITFL: Overall analysis of Liver Function Tests (LFT)

Transaminitis: Aminotransferases (AST, ALT)
- Generally associated with hepatocellular damage
- Generally not associated with cholestasis

Ratio of AST and ALT can be useful in differential
ALT is more specific for liver damage than AST

AST: ALT =1
-> Associated with ischaemia (CCF and ischaemic necrosis and hepatitis)

AST: ALT >2.5
-> Associated with Alcoholic hepatitis
-> Alcohol induced deficiency of pyridoxal phosphate
AST: ALT <1
-> High rise in ALT specific for Hepatocellular damage
-> Paracetamol OD with hepatocellular necrosis
-> Viral hepatitis, ischaemic necrosis, toxic hepatitis
-> Elevation with cholestasis (ALP, GGT)

ALP – primarily associated with cholestasis and malignant hepatic infiltration
Marker of rapid bone turnover and extensive bony metastasis

GGT – sensitive to alcohol ingestion
Marker of hepatocellular damage but non-specific
Sharpest rise associated with biliary and hepatic obstruction

668
Q

22.2 The influence of end-stage renal disease on the plasma clearance and dose of sugammadex is that the

a) Increased clearance – increased dose
b) Decreased clearance – reduced dose
c) Decreased clearance – same dose
d) No change in clearance or dose

A

c) Decreased clearance – same dose

669
Q

21.2 A patient who usually takes oral morphine 50 mg bd develops a bowel obstruction and
experiences withdrawal symptoms. They may be described as having

a) Tolerance
b) Physical dependence
c) Psychological dependence
d) Pseudo-addiction
e) Addiction

A

physical dependance

BARASH:
Physical dependence is a “physiologic state of adaptation to a specific psychoactive substance characterized by the emergence of a withdrawal syndrome during abstinence, which may be relieved in total or in part by re-administration of the substance.”

670
Q

20.1 Nitrous oxide chronic use complications:

a) Anaemia due to decreased EPO
b) Anaemia from glutathione deficiency
c) Neurological damage due to methionine deficit
d) Pulmonary hypertension
e) Hypoxia

A

Chronic neurological symptoms from methionine depletion

671
Q

20.2 A preeclamptic woman presents with a blood pressure of 150/100 mmHg. An appropriate first line treatment to reduce the blood pressure is

a. Labetalol
b. Nifedipine
c. Magnesium
d. Methyldopa
e. Perindopril

A

Labetalol

RANZCOG
first line drugs are
> methyldopa
> labetalol
> oxprenolol.

Second line agents are:
- hydralazine
- nifedipine
- prazosin.

Acute Rx:
Labetalol: 20-40mg IV q10min
Nifedipine 10-20mg PO Q45min
Hydralazine 5-10mg IV q20min

672
Q

21.1 Blocking the sciatic nerve results in loss of function of all of the following EXCEPT

a) Weak dorsiflexion
b) Dorsal foot sensation loss
c) Knee flexion weakness
d) Knee extension weakness

A

d) Knee extension weakness
-> this is femoral innervation

The sciatic nerve block results in anesthesia of the
- posterior aspect of the knee
- hamstring muscles
- entire lower limb below the knee, both motor and sensory block, with the exception of skin on the medial leg and foot (supplied by the saphenous nerve).

The skin of the posterior aspect of the thigh is supplied by the posterior femorocutaneous nerve, which deviates away from in the sciatic nerve proximal to the level of the anterior approach, and is therefore not blocked.

NYSORA

673
Q

21.2 High-risk transthoracic echocardiogram findings associated with aortic dissection include all of the following EXCEPT

a) RWMA
b) Pericardial effusion
c) Dilated aortic root
d) Aortic regurgitation
e) LV hypertrophy

A

E) LV hypertrophy

ECHO FINDINGS

intimal flap

TYPING (type A):
aortic regurgitation (acute dilatation of the aortic root, aortic leaflet prolapse, dissection flap prolapse, pre-existing disease, e.g. bicuspid valve)

pericardial effusion and/or tamponade

regional wall motion abnormality heralding coronary artery occlusion

DOPPLER
identifies true and false lumen

detect aortic branch occlusion/ dissection (absent flow)

Source LITFL

674
Q

20.1 To perform regional anaesthesia suitable for a fourth toe amputation, it is essential to block the:

a) posterior tibial
b) superficial peroneal
c) deep peroneal
d) sciatic

A

d) sciatic

Sciatic best answer given dermatomes and osteotomes

675
Q

21.2 The drug of choice for the treatment of duct dependent congenital heart disease is

a) Alprostadil
b) Prostacyclin
c) Carboprost
d) Sildenafil
e) NSAID

A

a) Alprostadil

Prostin (PGE1)

676
Q

20.2 Severe obstructive sleep apnoea in adults is confirmed if during polysomnography if the apnoea/hypopnea index (AHI) is greater than or equal to

A) 10
B) 20
C) 30
D) 40
E) 50

A

C) 30

677
Q

22.1 Ehlers-Danlos Syndrome is associated with each of the following EXCEPT

a. Blood vessel fragility
b. LA resistance
c. Intellectual impairment
d. Glaucoma

A

Intellectual impairment

No solid refs

678
Q

23.1 The technique of airway pressure release ventilation

a. Has a prolonged expiratory time
b. Augments cardiac output in hypovolaemic patients
c. Results in reduced mean airway pressures

A

remembered options

Airway pressure release ventilation (APRV) is an open-lung mode of invasive mechanical ventilation mode, in which spontaneous breathing is encouraged.
APRV uses longer inspiratory times; this results in increased mean airway pressures, which aim to improve oxygenation.
Brief releases at a lower pressure facilitate carbon dioxide clearance.
The terminology and methods of initiation, titration, and weaning are distinct from other modes of mechanical ventilation.
The use of APRV is increasing in the UK despite a current paucity of high-quality evidence

high intrathoracic pressure decreases the transmural left ventricular pressure, reducing the work of contraction and increasing cardiac output. In the context of hypoxaemia, a mode of mechanical ventilation that improves arterial oxygenation will improve myocardial oxygen delivery, myocardial function and cardiac output. As APRV is a spontaneous breathing mode, in addition to the benefits of spontaneous ventilation, reduced doses of sedative drugs can often be used, with subsequent reduction of requirement for vasoactive drugs and improvement in haemodynamic state.

Airway pressure release ventilation (APRV) is an open-lung mode of invasive mechanical ventilation mode, in which spontaneous breathing is encouraged. APRV uses longer inspiratory times; this results in increased mean airway pressures, which aim to improve oxygenation

https://www.bjaed.org/article/S2058-5349(19)30178-7/fulltext

https://derangedphysiology.com/main/required-reading/respiratory-medicine-and-ventilation/Chapter%20518/airway-pressure-release-ventilation-aprv-ards

679
Q

22.2 All patients over 70 years of age having received either spinal or general anaesthesia at Hospital X are reviewed 3 years later to assess cognitive function. The aim of the study is to determine whether exposure to general anaesthesia or spinal anaesthesia impacts cognitive function. This trial design is best described as a

a) RCT
b) cohort study
c) case-control study
d) case series
e) cross-sectional study

A

b) cohort study

What is a Cohort Study design?
- Cohort studies are longitudinal, observational studies, which investigate predictive risk factors and health outcomes.
- They differ from clinical trials, in that no intervention, treatment, or exposure is administered to the participants.
- The factors of interest to researchers already exist in the study group under investigation.
- Study participants are observed over a period of time. The incidence of disease in the exposed group is compared with the incidence of disease in the unexposed group.
- Because of the observational nature of cohort studies they can only find correlation between a risk factor and disease rather than the cause.

Cohort studies are useful if:
- There is a persuasive hypothesis linking an exposure to an outcome.
- The time between exposure and outcome is not too long (adding to the study costs and increasing the risk of participant attrition).
- The outcome is not too rare.

680
Q

22.1 You have anaesthetised a 25-year-old woman for a sleeve gastrectomy. She normally takes the oral contraceptive pill. You used rocuronium and at the end of the case reversed it with 4 mg/kg of sugammadex. Prior to discharge you should advise her to use non-hormonal contraception for the next

a. 1 day
b. 3 days
c. 5 days
d. 7 days

A

d. 7 days

A bolus dose of sugammadex is thought to have the following consequences:
(i) the equivalent of missing one daily dose of oral contraceptives, and
(ii) reduced efficacy of other hormonal contraceptives (e.g. implant, vaginal ring, or intrauterine system) requiring additional non-hormonal contraception be used for 7 days.

https://www.bjanaesthesia.org/article/S0007-0912(18)30198-3/fulltext

681
Q

20.1 The function of the bottle labelled D in the diagram is to protect against the consequences of

a. Loss of vacuum
b. Kinking ICC
c. Overflow of first bottle
d. High negative pressure

A

A

682
Q

21.2 A ten year old child (weight 30 kg) presents to the emergency department in status epilepticus. He has received one dose of 15 mg midazolam buccally prior to his arrival. According to Advanced Paediatric Life Supprt, Australia (APLS) guidelines the next drug treatment should be intravenous

a) Midazolam
b) Propofol
c) Levetiracetam
d) Phenytoin

A

a) Midazolam 0.15mg/kg

1st line: Midazolam IV/IO/IM –> 0.15mg/kg
2nd line: Midazolam IV/IO/IM –> 0.15mg/kg
3rd line: Keppra 40mg/kg (max 3g)
4th line: Phenytoin 20mg/kg or phenobarbitone
5th line: Intubation and deep sedation with midazolam, propofol +/- phenobarbitone

683
Q

22.2 You are asked to review a 65-year-old man in the emergency department who has presented with hypoxia and confusion. The chest x-ray shows a left-sided

a. Pneumothorax
b. pneumonia
c. one sided pulmonary oedema
d. pleural effusion
e. haemothorax

A

b. pneumonia

Air bronchogram

684
Q

22.1 The image shows results from noninferiority trials. The trial labelled N is best described as

(the confidence interval crosses midline of no effect and margin of non-inferiority)

a. Inferiority demonstrated
b. Non-inferiority demonstrated
c. Superiority demonstrated
d. Non-inferiority not demonstrated/Inconclusive

Green is N
A

Inconclusive/Noninf not demonstrated

685
Q

22.2 You are performing a focused cardiac ultrasound in the postanaesthesia care unit on a patient who is hypotensive for unclear reasons. His heart rate is 100 beats/min. The left ventricular velocity time integral is 10 cm. The left ventricular outflow area is 3 cm2. The left ventricular ejection fraction is 25%. The right ventricular systolic pressure is 40 mmHg. The inferior vena cava diameter is 20 mm. The estimated cardiac output is

a. 1L/min
b. 2L/min
c. 3L/min
d. 4L/min
e. 5L/min

A

C 3L/min

LVOT area x VTI = SV
3cm2 x 10cm = 30ml

SV x HR = CO
30 x 100 = 3000

686
Q

22.1 A patient requires a peripherally inserted central venous catheter. Electrocardiographic (ECG)-aided tip localisation is used to site the tip of the catheter. The initial ECG from the catheter is shown.

The ECG when the catheter is placed appropriately will be

A

Maximum P-wave corresponds to placement in the Cavo-atrial Junction (CAJ)

Benefits of CAJ placement
- furthest distance from “high risk” areas
- Largest vein diameter
- highest blood flow capacity
- minimal risk for catheter migration and looping

Too short placement:
Increased risk of:
- DVT
- Phlebitis

Too long placement:
Increased risk of:
- arrhythmias
- tricuspid valve dysfunction
atrial dysfunction

ECG and corresponding anatomy:

  • Normal P-wave = upper vasculature prior to CAJ
  • Max P-wav= CAJ
  • Initial negative P-wave deflection = Right Atrium
  • Biphasic P-wave = Right Atrium
  • Inverted P-wave = Right Ventricle
687
Q

22.2 A 72-year-old man with peripheral vascular disease presents for a femoral angioplasty and is currently taking aspirin. Regarding the perioperative management of his aspirin,

a) Cessation leads to increased risk of stroke
b) Cessation leads to increased risk of MI
c) Continuation leads to increased risk of major bleeding
d) Continuation leads to reduced rate of MI
e) Continuation leads to reduced rate of perioperative mortality

A

c) Continuation leads to increased risk of major bleeding

Aspirin in patients undergoing non cardiac surgery
https://www.nejm.org/doi/full/10.1056/nejmoa1401105

Conclusions

Administration of aspirin before surgery and throughout the early postsurgical period had no significant effect on the rate of a composite of death or nonfatal myocardial infarction but increased the risk of major bleeding. (Funded by the Canadian Institutes of Health Research and others; POISE-2 ClinicalTrials.gov number

688
Q

20.2 The maximum warm ischaemic time (in minutes) acceptable for procuring the lungs following donation after cardiac death is

a. 30
b. 60
c. 90
d. 120
e. 240

A

c. 90

Warm ischaemia time:
- Time from treatment withdrawal to the start of cold perfusion of the donated organs
- Significance is the impact on graft function
- Most important phase of WIT begins when the systolic BP is < 60mmHg
- This includes the waiting period from the absence of circulation to the declaration of death and the time before initiating the flow of cold perfusate through the cannula

Maximum WARM Ischaemia time
- Heart 30 mins
- Liver 30 mins
- Pancreas 30 mins
- Kidney 60 mins
- Lungs 90 mins

Maximum COLD Ischaemia time:
- Heart = 4 hrs
- Lungs = 6-8hrs
- Liver/Pancreas = 12hrs (DBD)/6 hrs (DCD)
- Kidneys = 18hrs (DBD)/ 12 hrs (DCD)

689
Q

22.1 A 45-year-old woman is being assessed for liver transplantation. In order to determine the severity of her liver disease the Model for End-stage Liver Disease score is derived using the international normalised ratio, serum bilirubin and

a. GGT
b. Albumin
c. Sodium
d. ALT
e. Creatinine

A

Creatinine

Model for End-stage Liver Disease
- Estimates disease severity and survival in patients with Liver Disease
- Objective assessment
- Score from 6-40
- Validated across a number of liver diseases and surgeries
- MELD score is used as a method of allocation of organs to estimate survival

MELD uses the following parameters:
- Bilirubin
- INR
- Creatinine
- [Hyponatraemia]
○ Part of the MELD-Na score update in 2016
○ Sodium (Na) Values < 125 are set to 125 and values >137 are set to 137
4 MELD levels are:
- >/=25 (gravely ill)
- 24-19
- 18-11
- </=10

In patients who have undergone abdominal surgery an elevated MELD score was a better predictor of poor perioperative outcome than Child-Pugh Classification
- MELD score >15 should avoid elective surgery

Calculation:
MELD =
3.8loge(serum bilirubin [mg/dL]) + 11.2loge(INR) + 9.6*loge(serum creatinine [mg/dL]) + 6.4

MELD-Na =
MELD + 1.32 * (137-Na) - [0.033*MELD * (137-Na)]

690
Q

23.1 To assist with guiding intravenous fluid resuscitation in adults with sepsis or septic shock, the 2021 Surviving Sepsis Guidelines suggest using any of the following EXCEPT

a. PPV
b. Response to straight leg raise
c. Response to fluid bolus
d. ECHO
e. Urine output

A

E. Urine output

For adults with sepsis or septic shock, we suggest using dynamic measures to guide fluid resuscitation over physical examination or static parameters alone.
Weak recommendation, very low-quality evidence.

Remarks: Dynamic parameters include response to a passive leg raise or a fluid bolus, using stroke volume (SV), stroke volume variation (SVV), pulse pressure variation (PPV), or echocardiography, where available.

https://journals.lww.com/ccmjournal/Fulltext/2021/11000/Surviving_Sepsis_Campaign__International.21.aspx

691
Q

21.2 An awake patient in the post-anaesthesia care unit complains of breathlessness. The FiO2 is 0.4 via a facemask. An arterial blood gas taken at the time shows PaO2 135 mmHg, PaCO2 48 mmHg, and SpO2 100% The alveolar-arterial gradient (in mmHg) is
approximately

a) 60
b) 90
c) 120
d) 150

A

b) 90

PAO2: 0.4 (760 - 47) - 48/0.8 = 285 - 60 = 225mmHg
225 - 135 = 90mmHg.

692
Q

22.1 Prolonged paralysis associated with mivacurium is most appropriately managed with

a. Give FFP
b. Give pradolixime
c. Ventilate and wait for recovery
d. Sugammadex

A

Ventilate and wait for recovery

693
Q

22.2 A patient is bleeding and her ROTEM displays a Fibtem A5 of 2 mm (normal > 4 mm). The most appropriate treatment is

a. FFP
b. fib conc
c. cryoprecipitate
d. TXA

A

b) fibrinogen concentrate

bleeding and low fib = concentrate
not bleding and low = cryo

694
Q

22.2 Dabigatran differs from rivaroxaban and apixaban because it inhibits
a. prothrombin
b. thrombin
c. factor X
d. fibrin
e. fibrinogen

A

Thrombin
rivaroxiban 10
dabigatran thrombin

695
Q

21.2 Complications of hyperbaric oxygen therapy include all of the following EXCEPT

a) Hypoglycaemia
b) Cataract
c) Worsening CCF
d) Seizures
e) Reversible hypermetropia

A

e) Reversible hypermetropia

696
Q

20.2 A 55-year-old patient who has undergone trans-sphenoidal hypophysectomy for a growth-hormone secreting adenoma has a urine output of one litre in the first postoperative hour. The following results are obtained. The most appropriate early management is

Na 145, Urinary osm ~200, Serum Osmolarity ~320

a) DDAVP
b) Hypertonic saline
c) Normal Saline 1 L bolus
d) 100 ml/hr of saline
e) Fluid restrict

A

a) DDAVP

Polyuria
Low urine osm
High serum osm
High Na
post transsphenoidal sx
= Central DI

697
Q

21.2 Of the following, the deficit that DOES NOT result from damage to the common peroneal nerve is

a) Weak dorsiflexion
b) Dorsal foot sensation loss
c) Knee flexion weakness
d) Knee extension weakness

A

d) Knee extension weakness
- most correct based on answers remembered
- this is femoral innervation

Superficial peroneal nerve injury (L4–5 S1–2)

Mechanism of injury
Lithotomy and the lateral position are the common risk factors as the nerve is potentially compressed at the fibular head. Length of time in lithotomy has not been associated with an increased risk of developing a PPNI.

Clinical presentation
There is loss of dorsiflexion and eversion of the foot (equinovarus deformity). Sensory manifestations are described along the anterolateral border of the leg and the dorsum of the digits except those supplied by saphenous and sural nerves.

Orthobullets:
Common peroneal nerve
- superficial & deep branches

Deep peroneal
- motor: extensor digitorum longus, extensor hallucis longus (dorsiflexion)
- sensory: 1st dorsal webspace

Superficial peroneal
- motor: peroneus longus and brevis (eversion)
- sensory: dorsum foot (except for 1st dorsal webspace & 5th toe)

Lower limb peripheral nerve injuries

Sciatic nerve injury (L4–S3)

Mechanism of injury
Stretch, compression, ischaemia, and direct damage are the primary mechanisms. The lithotomy, frog leg, and sitting positions have been implicated in perioperative injury to this nerve (hyperflexion of the hip, abduction, and extension of the leg causes stretching). Regional anaesthetic techniques and hip arthroplasty may also cause injury. A higher incidence is seen in men aged 45–55 yr and in patients suffering with diabetes mellitus. The common peroneal component is usually affected, as this is more superficial compared with the tibial component.

Clinical presentation
Injury manifests as paralysis of the hamstring muscles and all the muscles below the knee leading to weak knee flexion and foot drop. All sensation below the knee except the medial aspect of the leg and foot is impaired.

698
Q

23.1 The causes of macrocytic anaemia include

A. Liver failure
B. Renal failure
C. Thalassaemia
D. Thyrotoxicosis
E. Vitamin e deficiency

A

A

A - Alcohol is a common cause of macrocytosis and macrocytic anemia. (UpToDate)
B - No - normally normocytic chronic disease anaemia
C - No - microcytic
D - I can’t find anything on macrocytosis with thyrotoxicosis, but hypothyroidism definitely does
E - Possibly…. https://hemonc.mhmedical.com/content.aspx?bookid=1783&sectionid=121720217

699
Q

20.1 You are asked to review a patient two days after a difficult total knee replacement, which was undertaken under tourniquet with spinal anaesthesia in combination with an ultrasound- guided adductor canal block and high volume local anaesthetic infiltration by the surgeon. The patient complains of a new onset of leg weakness on the operative side. The nerve LEAST likely to be involved in this weakness is the

a) Common peroneal
b) Deep peroneal
c) Sciatic
d) Femoral
e) Saphenous

A

Saphenous nerve
It is a purely sensory nerve

  • rapid onset more suggestive or direct injury to nerve, later onset suggestive of ischaemia relating to oedema
  • mulscular injury related to tourniquet results in swelling/pain/weakness of affected muscle
  • post tourniquet syndrome - swollen, pale, stiff, weakness but not paralysis
  • L5 radiculopathy would affect knee flexion, but would have presented immediately post op if spinal related
700
Q

22.1 An adult male patient has a haemoglobin level of 80 g/L and his blood film shows a reticulocyte count of 10%. These findings are compatible with

a. ALL
b. Spherocytosis
c. Aplastic anaemia
d. Pernicious anaemia
e. Anaemia of chronic disease

A

Hereditary spherocytosis.

Auto-haemolytic, intraplenic haemolysis. High reticulocyte count (6-20%) (normal range 0.5-2%)

701
Q

21.1 Of the following, the LEAST likely cause of high anion gap metabolic acidosis is

a) lactic acidosis
b) renal failure
c) tuberculosis on isoniazid
d) renal tubular acidosis
e) salicylate overdose

A

renal tubular acidosis-> NAGMA

HAGMA:
Causes CATMUDPILES

- CO/CN
- Alcoholic ketoacidosis/ starvation ketoacidosis
- Toluene (paint thinners)
- Metformin/Methanol
- Uraemia
- DKA
- Pyroglutamic Acidosis/paracetamol/ phenformin/propylene glycol/paraldehyde
- Iron/ Isoniazid
- Ethylene glycol (anti-freeze)
- Salicylic Acid

NAGMA:
Causes CAGE:
- Chloride
- Acetazolamide and Addison’s
- GI causes (vomiting, diarrhoea, fistula)
Extras: RTA

702
Q

20.1 A 75-year-old man has this right heart catheter trace as part of his investigation of dyspnoea. His pulmonary capillary wedge pressure is 24mmHg. The most likely diagnosis is (pressure trace shown)
Right heart cath: PA pressure 75/26, CWP24

a) Pulmonary Arterial Hypertension
b) LV failure
c) Pulmonary embolus
d) Emphysema
E) aortic stenosis

A

Left heart failure (pulmonary htn with raised Left sided pressure (PCWP 24mmHg)

PCWP >15mmHg.
Therefore post-capillary or combined pre-and post-capillary PH = Left heart failure (clinical group 2)
Other Options are clinical groups 1,3,4 of PH and therefore not correct (as characteristic is PAWP<15mmHg).

WHO pulmonary Htn classification
1: Primary arterial Hypertension
2: 2ry to left heart disease
3: 2ry to lung disease/hyypoxia
4: Chronic Thromboembolic disease
5: Unclear/multifactorial

703
Q

23.1 To provide anaesthesia to the medial malleolus, the key nerve to block is the

a. Saphenous
b. Deep peroneal
c. Superficial peroneal
d. Tibial

A

a) saphenous

704
Q

22.1 A 45-year-old woman is reviewed in the preadmission clinic. She is scheduled to undergo a microwave endometrial ablation for menorrhagia in one week’s time. Her preoperative laboratory investigations include the following blood results (full blood examination and iron studies shown).
The most appropriate course of action would be to

a. Proceed
b. Iron IV then proceed
c. Transfuse 2 RBC intraop
d. Use cell saver intraop
e. Defer and refer to haematology for further Ix

A
705
Q

22.2 When using cardioversion to revert a patient in atrial fibrillation to sinus rhythm, the direct current shock is synchronised with the ECG to coincide with the

a) Start of R wave
b) Start of Q wave
c) Middle of T wave
d) Peak of R wave

A

d) Peak of R wave

The appropriate energy level is then selected, and the discharge/shock button is pressed and held. The defibrillator does not release the shock immediately. Instead, it waits for the next R-wave to appear and delivers the shock at the time of the R-wave. This allows the shock to be provided safely away from the T wave, avoiding the R-on-T phenomenon.

706
Q

23.1 You are asked to review a 5-year-old child weighing 24 kg in the recovery room for acute pain management after a tonsillectomy performed for obstructive sleep apnoea. The most appropriate analgesic regimen would be

Painstop formulation (codeine 1mg/ml, paracetamol 24mg/ml). (interestingly composition of painstop not included in the released stem but was on the day of the exam)

A Painstop q6h PRN, ibuprofen, tramadol
B Painstop q6h, oxycodone PRN
C Paracetamol 300mg q6h oxycodone
D Paracetamol 300mg QID, ibuprofen 200mg TDS, tramadol 20mg PRN

A

Poorly remembered options
Definitely do not give Painstop as contains codeine
Opioids should be PRN only

c or d
-go with D - tramadol versus oxycodone re OSA

paracetamol 15mg/kg (360mg) QID
ibuprofen 10mg/kg (240mg) TDS
tramadol 1mg/kg (24mg) QID
oxycodone 0.1-0.2mg/kg (2.4-4.8mg) 4hourly

Codeine should not be used. Deaths. Ultrafast metabolisers –> high levels of morphine.

Nonselective NSAIDs may increase the risk of any bleeding-related outcome after
tonsillectomy in adults (U) (Level I); however, not in paediatric patients

Prospect advice:

The basic analgesic regimen should include paracetamol and non-steroidal anti-inflammatory drugs (NSAIDs) administered pre-operatively or intra-operatively and continued postoperatively.

A single dose of intravenous (i.v.) dexamethasone is recommended for its analgesic and anti-emetic effects.
Pre-operative gabapentinoids, intra-operative ketamine (only in children) and dexmedetomidine are recommended in patients with contra-indications to the basic analgesic regimen.

Analgesic adjuncts such as intra-operative and postoperative acupuncture and postoperative honey are recommended.

Opioids should be reserved as rescue analgesics in the postoperative period

707
Q

20.1 Epidural filters are designed to retain particles down to a diameter of

A. 20 nanometers
B. 200 nanometers
C. 2 micrometers
D. 20 micrometers
E. 200micrometers

A

B. 200 nanometers (0.2 micrometres)

708
Q

22.2 AFE incidence highest in
a) HELP Syndrome
b) LSCS
c) Instrumental delivery
d) Preeclampsia
e) Spontaneously

A

LUCS

709
Q

22.2 According to National Audit Project (NAP) 5, the incidence of awareness during general anaesthesia is

a) 1:800
b) 1:1600
c) 1:8000
d) 1:19000
e) 1:30000

A

d) 1:19000

The estimated incidence of patient reports of AAGA was ~1:19,000 anaesthetics.

However, this incidence varied considerably in different settings.
The incidence was
~1:8,000 when neuromuscular blockade was used and
~1:136,000 without it.

Two high risk surgical specialties were
Cardiothoracic anaesthesia (1:8,600) and
Caesarean section (~1:670).

710
Q

21.1 The following ECG is consistent with

a) Atrial fibrillation
b) Lown-Ganong-Levine (LGL) Syndrome
c) Previous Myocardial Infarction
d) Wolf-Parkinson White Type B
d) Left Ventricular Hypertrophy

A

D) WPW B

Sinus rhythm with very short PR interval (< 120 ms)
Broad QRS complexes with a slurred upstroke to the QRS complexes — the delta wave
Dominant S wave in V1 indicates a right-sided accessory path — sometimes referred to as “Type B” WPW
Tall R waves and inverted T waves in the inferior leads and V4-6 mimic the appearance of left ventricular hypertrophy (LVH) — again, this is due to WPW and does not indicate underlying LVH

711
Q

21.1 The minimum microshock current required to elicit ventricular fibrillation is

A) 0.1 mA
B) 1 mA
C) 10 mA
D) 100 mA

Alt: What about Macroshock?

A

0.05-0.1mA

Source: LITFL

Macro is 100mA

712
Q

22.2 Of the following, the substance LEAST likely to cause lactic acidosis is

a. methanol
b. propofol
c. metformin
d. acetazolamide

A

d. acetazolamide

acetazolamdie has been known to cause lactic acidosis but is less common than the other drugs listed unless there is a 5th option not remembered

713
Q

21.2 A peripheral intravenous cannula is being inserted in the forearm of a man having a hemicolectomy. The skin asepsis preparation NOT suitable for this procedure is

a) Povidone iodine
b) Chlorhexidine 2%
c) Alcohol 70%
d) Chlorhexidine 0.5% with alcohol
e) Tincture of iodine

A

c) Alcohol 70%
- only suitable for short-term cannulation (<24 hours)

714
Q

22.1 Maintaining a CO2 pneumoperitoneum at a pressure of 15 mmHg is most likely to lead to

a) Lactic acidosis
b) Decreased arterial blood pressure
c) Decreased heart rate
d) Increased CVP
e) Increased renal blood flow
f) Increased SVR

A

f) Increased SVR

715
Q

22.1 A 72-year-old female smoker with hypertension presents to the emergency department with a wrist fracture after a fall. She has been increasingly tired and confused over the previous week. Her serum and urine electrolytes are (supplied). The most likely diagnosis is

(Low K, low Na, Normal Ur and Cr, Ur sodium <10mmol/L)

a. SIADH
b. Addison’s
c. Diuretic

A

Uncertain

SIADH:
1. hypotonic hyponatraemia
2. urine osmolality > plasma osmolality (<275mOsm/kg) (i.e. concentrated urine despite hypotonic blood)
3. urinary Na+ > 20mmol/L
4. normal renal, hepatic, cardiac, pituitary, adrenal and thyroid function
5. euvolaemia (absence of hypotension, hypovolaemia, and oedema)
6. correction by water restriction

Addison’s
Hypo natraemia
HYPER kalaemia
Hypo glycaemia
Acidosis

Diuretics
Hypo natraemia
Hypo kalaemia
High urinary Na and K

https://www.derangedphysiology.com/files/Electrolyte%20Disturbance.pdf

JAMA article on hyponatraemia
https://emergencymed.org.il/wp-content/uploads/2022/08/jama_adrogu_2022_rv_220011_1657919726.49616.pdf

716
Q

23.1 The glossopharyngeal nerve does NOT supply sensory innervation to the

a. Anterior third of tongue
b. Walls of pharynx
c. Motor to stylopharyngeal muscle
d. Pharyngeal plexus

A

a) anterior third of the tongue

717
Q

A 72-year-old man with a murmur has a left heart catheter. Shown are the simultaneous waveforms in the aorta and left ventricle. The most likely diagnosis is

A. Aortic regurgitation
B. Aortic Stenosis
C. HOCM
D. Mitral regurg
E. Normal

A

Answer: A Aortic regurgitation
- lack of dichrotic notch, high pulse pressure

https://www.ahajournals.org/doi/full/10.1161/CIRCULATIONAHA.111.060319#d1e195

718
Q

22.2 You are reviewing a primigravida at 32 weeks gestation with a Fontan circulation in the anaesthetic preassessment clinic. Peripartum care should avoid the use of

a. Terbutaline
b. Nitrous oxide
c. Ergometrine
d. Lignocaine 2% with adrenaline 1:200 000
e.

A

Ergometrin increases PVR and SVR

719
Q

22.2 The amount of fresh frozen plasma that needs to be administered (in mL/kg) to increase plasma fibrinogen levels by 1 g/L is

a) 10ml/kg
b) 20ml/kg
c) 30ml/kg
d) 40ml/kg
e) 50ml/kg

A

c) 30ml/kg

After a dose of 10 to 15 mL/kg of FFP, plasma clotting factors rise about 15%, and the fibrinogen level rises by 40 mg/dL (0.4g/l)

https://www.sciencedirect.com/topics/medicine-and-dentistry/fresh-frozen-plasma

1g/0.4g= 2.5
2.5 x 10ml/kg= 25ml/kg
2.5 x 15ml/kg= 37.5ml/kg
30ml/kg best answer

For cryoprecipitate:

One unit of Cryo is 15-20 mL in volume and contains 150-250 mg of fibrinogen. Cryo is generally transfused in pools of 10 units, which should increase an adult recipient’s fibrinogen level by 50-100 mg/dL. (0.5-1g/l)

10 units of cryo= 200-300ml
200ml/70kg= 2.8ml/kg
300ml/70kg= 4.2ml/kg

720
Q

22.1 You are anaesthetising a patient for implantation of an automated implantable cardioverter defibrillator. The patient is a 48-year-old with dilated cardiomyopathy and pulmonary hypertension.

The preoperative echocardiogram report states that the estimated pulmonary artery systolic pressure is 55 mmHg, and that there is mild right ventricular systolic dysfunction. To avoid
worsening right ventricular function during induction, it would be best to consider using

a. Milrinone
b. Dopamine
c. Dobutamine
d. Adrenaline

A

c. Dobutamine

In 2017 a similar questions was asked and an option for metaraminol was given, metaraminol could be a better answer as it will increase systemic pressure and reduce heart rate maintaining RCA perfusion at induction. Dobutamine and Milrinone can cause systemic vasodilation leading to reduction in systemic blood pressure and RCA perfusion pressure, Both adrenaline and Dopamine do not cause pulmonary vasodilation and can lead to tachyarhythmias

Pulmonary hypertension and its management in patients undergoing non-cardiac surgery
https://associationofanaesthetists-publications.onlinelibrary.wiley.com/doi/10.1111/anae.12831

Vasoconstrictors, inotropes and inodilators

Maintaining the gradient between aorta and right ventricle is achieved by using sympathomimetic and non-sympathomimetic vasopressors. Noradrenaline and vasopressin improve perfusion of the right coronary artery, reduce the pulmonary/systemic vascular resistance ratio, enhance right ventricular performance and marginally improve cardiac output

However, the evidence of their impact on mortality related to right heart failure is weak. Inotropes that enhance right ventricular performance, such as adrenaline, dobutamine and levosimendan are effective in treating right-sided heart failure.

The use of inotropes has a modest impact in reducing the overall mortality related to PH, and their wide availability and ease of administration make this group of drugs very attractive for use in the peri-operative setting.

Inodilators, such as the phosphodiesterase-3 inhibitors milrinone and enoximone, have been shown to be beneficial when compared with conventional inotropic support only. It appears that the influence of phosphodiesterase-3 inhibitors on reducing pulmonary vascular resistance is more pronounced than the reduction in systemic vascular resistance. However, reduction in systemic vascular resistance can compromise right coronary artery blood flow in patients with severe PH and therefore they should be administered cautiously.

Treatment of pulmonary hypertensive crisis:

General principles
- Avoid hypoxic pulmonary vasoconstriction
- Avoid hypercarbia, acidosis and hypothermia
- Avoid high airway pressures
- Optimise right ventricular preload
- Reduce right ventricular afterload
- Maintain coronary blood flow
- Maintain sinus rhythm
- Maintain arterial blood pressure and cardiac output

Vasopressors– noradrenaline; vasopressin

Inotropes– adrenaline; dobutamine

Inodilators– milrinone; enoximone

Intravenous vasodilators (caution if low systolic blood pressure)
- Milrinone (25–50 μg.kg−1 bolus, followed by 0.5–0.75 μg.kg−1.min−1 continuous infusion)
- Prostacyclin (4–10 ng.kg−1.min−1 continuous infusion)
- Iloprost (1–3 ng.kg−1.min−1 continuous infusion)
- Sildenafil (10 mg bolus three times a day)

Selective pulmonary vasodilation
- Iloprost (5–10 μg diluted in 10 ml saline, nebulised over 10 min, repeated every 2–4 h)
- Prostacyclin (25–50 μg diluted in 50 ml saline, nebulised over 15 min, repeated every hour)
- Nitric oxide (5–40 ppm continuously)

721
Q

21.1 Severe obstructive sleep apnoea in a 6-year-old child is confirmed if during polysomnography the
apnoea/hypopnea index (AHI) is greater than or equal to

A >5
B >10
C >15
D >20
E >30

A

> 10

722
Q

Prior to neuraxial block in a patient with normal renal function, apixaban should be ceased for

a. 1 day
b. 2 days
c. 3 days
d. 5 days
e. 7 days

A

c. 3 days

723
Q

23.1 A 65-year-old man with hypertension, type 2 diabetes and significant obstructive sleep apnoea on CPAP is scheduled for an abdominoperineal resection, with a high dependency unit admission planned postoperatively. He currently takes a calcium channel blocker, a sodium-glucose cotransporter 2 (SGLT2) inhibitor and metformin. ANZCA guidelines recommend withholding SGLT2 inhibitors

A. Day of and 2 days prior
B. Day of and 3 days prior
C. Continue on the day of surgery.
D. Stop day of surgery.

A

a) day of and 2 days prior

724
Q

22.2 Anterior spinal artery syndrome would NOT result in

a. Motor
b. Proprioception
c. Pain sensation
d. Temperature

A

Proprioception

Ventral (anterior) cord syndrome
- Involves cords in the anterior 2/3rds of the spinal cord
○ Corticospinal tract
○ Spinothalamic tract
○ Descending autonomic tracts to the sacral centers for bladder control
- Signs/symptoms
○ Weakness (CST)
○ Reflex changes (CST)
○ B/L temp and pain sensation (Spino)
○ Tactile and vibratory sense are normal
○ Urinary incontinence (Auto)
- Causes:
○ Spinal cord infarction
○ Intervertebral disc herniation
○ Radiation myelopathy

725
Q

22.1 You inadvertently place a 7.5Fr central venous catheter into the carotid artery of a patient undergoing an emergency laparotomy for peritonitis. The best course of management is to

a) Leave in, call vascular to repair at end of case
b) Heparin, remove, apply pressure

A

Leave in situ and contact vascular surgeons

726
Q

20.2 A 40 year old man suffers a hydrofluoric acid burn to 60% of his total body surface area in an industrial accident. An expected electrolyte disturbance is

a. Hypokalaemia
b. Hyponatremia
c. Hypophosphatemia
d. Hypomagnesemia
e. Hypocalcemia

A

e. Hypocalcemia

UTD:

> HF penetrates quickly through the epidermal layer into the dermis and deeper.
Fluoride ions complex with calcium and magnesium, which can lead to hypocalcemia and hypomagnesemia.
These electrolyte abnormalities and the direct cardiotoxic effects of fluoride ions contribute to the development of cardiac arrhythmias, which are the primary cause of death in HF burns.
Hypocalcemia may stimulate an efflux of potassium ions from cells resulting in hyperkalemia, and predisposing to cardiotoxicity.
QTc interval prolongation, due to hypokalemia, hypomagnesemia, and/or hypocalcemia may be seen.
Calcium salts are the mainstay of treatment of hydrofluoric acid burns; the dose and route depend upon the clinical situation

727
Q

22.2 A 54-year-old woman has a laryngeal mask airway (LMA) inserted for a surgical procedure. The following day she complains of tongue numbness and abnormal taste over the anterior two-thirds of the tongue. The most likely site of the nerve injury is the

a) Glossopharyngeal nerve
b) Lingual nerve
c) Facial nerve
d) Vagus nerve
e) Hypoglossal nerve

A

b) Lingual nerve

Has fibres from both mandibular branch of CN V3 and CN VII

728
Q

21.2 The diffusing capacity of the lungs for carbon monoxide (DLCO) is likely to be decreased with

a) Sarcoidosis
b) Asthma
c) Obesity
d) Pulmonary haemorrhage

A

sarcoid

729
Q

23.1 Of the following drugs, the LEAST likely to cause pulmonary vasodilation when used at low doses in patients with chronic pulmonary hypertension is

a) Vasopressin
b) Dobutamine
c) Dopamine
d) Milrinone

A

Dopamine

  • least likely to cause pulmonary vasodilation (all the others do to my knowledge)
  • From UP TO DATE:
    > At low doses of 1 to 3 mcg/kg per min, dopamine acts primarily on dopamine-1 receptors to dilate the renal and mesenteric artery beds
    > At 3 to 10 mcg/kg per min (and perhaps also at lower doses), dopamine also stimulates beta-1 adrenergic receptors and increases cardiac output, predominantly by increasing stroke volume with variable effects on heart rate.
    > At medium-to-high doses, dopamine also stimulates alpha-adrenergic receptors, although a small study suggested that renal arterial vasodilation and improvement in cardiac output may persist as the dopamine dose is titrated up to 10 mcg/kg per min
    *clinically, the haemodynamic effects of dopamine demonstrate individual variability

Dobutamine (inodilator):
- selective β1-agonist that increases cardiac contractility and reduces pulmonary vascular and systemic vascular resistances

Vasopressin:
- vasopressin may have pulmonary vasodilatory effects in addition to a systemic vasoconstrictive effect

Milrinone (inodilator):
- the phosphodiesterase-3 inhibitors, milrinone and enxoimone, have positive inotropic effects combined with the capacity to reduce RV afterload (‘inodilators’) without significant chronotropic effect, but they can be associated with significant systemic hypotension

https://pubs.asahq.org/anesthesiology/article/121/5/914/13855/VasopressinThe-Perioperative-Gift-that-Keeps-on

730
Q

22.2 A 55-year-old man with no past history of ischaemic heart disease is three days post-total hip replacement surgery. He has an episode of chest pain that sounds ischaemic, began at rest and lasts 30 minutes before resolving fully. There are no ECG changes nor troponin rise. The diagnosis is

a. No diagnosis made
b. Unstable angina
c. STEMI
d. NSTEMI
e. MINS

A

b. Unstable angina

UTD:

Unstable angina (UA) and acute non-ST elevation myocardial infarction (NSTEMI) differ primarily in whether the ischemia is severe enough to cause sufficient myocardial damage to release detectable quantities of a marker of myocardial injury (troponins):

●UA is considered to be present in patients with ischemic symptoms suggestive of an ACS and no elevation in troponins, with or without electrocardiogram changes indicative of ischemia (eg, ST segment depression or transient elevation or new T wave inversion).

●NSTEMI is considered to be present in patients having the same manifestations as those in UA, but in whom an elevation in troponins is present.

MINS: Myocardial injury after non-cardiac surgery (up to 30 days post-op):
1. Elevated postop troponin
2. Resulting from myocardial ischaemia (i.e. no evidence of a non-ischaemic aetiology), not requiring an ischaemic feature (i.e. no chest pain, no ECG change)

VISION studies (Vascular Events in Noncardiac Surgery Patients Cohort Evaluation) demonstrated that severity of MINS strongly associated with 30-day mortality after NCS.

hs-cTnT
<20ng/L ~ 0.5% 30 day mortality
20-64ng/L ~3% 30 day mortality
65-999 ng/L ~9% 30 day mortality
>1000ng/L ~30% 30 day mortality

Whilst VISION trial identified MINS in at risk patients, the question now becomes what interventions are available to prevent this complication?

731
Q

20.2 The flow volume loop is most consistent with (Flow-volume loop shown)

a) Variable intra-thoracic obstruction
b) Variable extra-thoracic obstruction
c) Lower airway obstruction
d) Restrictive lung pattern
e) Mixed pattern

A

c) Lower airway obstruction

Peripheral or lower airways obstruction: expiratory limb demonstrates concave upward, also called “scooped-out” or “coved” pattern.

732
Q

23.1 Cerebral salt wasting and syndrome of inappropriate antidiuretic hormone secretion
(SIADH) have the following common features EXCEPT for

a. High urinary concentration
b. High urinary osmolality
c. Increased extracellular fluid

A

c. inc extracellular fluid

https://derangedphysiology.com/main/required-reading/electrolytes-and-fluids/Chapter%20531/hyponatremia-lazy-mans-classification

733
Q

21.1 High-risk transthoracic echocardiogram findings associated with aortic dissection include all of the following EXCEPT

a) pericardial effusion
b) dilated Ao root
c) RV dilatation
d) RWMA
e) AR

A

EXCEPT C) RV dilation

Echo findings in Aortic Dissection:
1. Intimal flap

  1. Type A dissection:
    - Aortic regurgitation
    -Acute dilation of aortic root
    -Aortic leaflet prolapse
    -Dissection flap prolapse
    -Pre-existing disease
    -Pericardial Effusion/Tamponade
    -RWMA
  2. Colour flow doppler
    -identifies true and false lumen
    -aortic branch occlusion/dissection
734
Q

21.1 When performing a brachial plexus block at the level of the axilla, the structure indicated by the arrow is the (ultrasound image shown)

a. Musculocutaneous nerve
b. Median
c. Radial
d. Ulnar

A

Ultrasound view of right axillary brachial plexus AA = axillary artery AV = axillary vein McN = musculocutaneous nerve RN = radial nerve UN = ulnar nerve MN = median nerve CoBM = coracobrachialis muscle CT = conjoint tendon

735
Q

23.1 The antiemetic action of aprepitant is via receptors for

A. Serotonin
B. Neurokinin-A
C. Dopamine
D. Substance P
E. Glycine

A

D. Substance P

Development of aprepitant, the first neurokinin-1 receptor antagonist for the prevention of chemotherapy-induced nausea and vomiting (2011)
https://www.ncbi.nlm.nih.gov/pubmed/21434941

Aprepitant acts centrally at NK-1 receptors in vomiting centres within the central nervous system to block their activation by substance P released as an unwanted consequence of chemotherapy.

REPEAT

736
Q

The part of the lung that is typically divided into medial and lateral segments is the

a. Left upper lobe
b. Lingula
c. Right upper lobe
d. Right middle lobe
e. Right lower lobe

A

d. Right middle lobe

737
Q

21.1 An ECG abnormality which is NOT usually associated with severe anorexia nervosa is

a) QT prolongation
b) TWI
c) ST depression
d) prolonged PR interval
e) sinus tachycardia

A

e) sinus tachycardia

https://academic.oup.com/bjaed/article/9/2/61/299563

Typically anorexic patients are hypotensive and bradycardic.
Bradycardia reflects the decrease in basal metabolic rate that arises as an adaptive response to starvation
Electrocardiographic abnormalities are common and may be found in >80% of strict dieters. These include:
1. atrioventricular block
2. ST depression
3. T wave inversion
4. QT prolongation.

738
Q

20.2 International guidelines state that patients presenting for major surgery have inadequate or low iron stores if their serum ferritin level is less than

a. 20 mcg/L
b. 30 mcg/L
c. 40 mcg/L
d. 50 mcg/L
e. 100 mcg/L

A

E. 100 mcg/L

?? < 30mcg/L

< 100mcg/L IF CRP > 5 and/or Transferrin saturation < 20

https://associationofanaesthetists-publications.onlinelibrary.wiley.com/doi/10.1111/anae.13773#:~:text=Recommendations%20for%20best%20clinical%20practice,-Physicians%20should%20consider&text=The%20presence%20of%20anaemia%20should,identification%20of%20absolute%20iron%20deficiency.

739
Q

23.1 A patient you anaesthetised for a cervical fusion reports rapidly progressing unilateral visual loss commencing two days postoperatively. Fundoscopic examination reveals optic disc oedema. The most likely diagnosis is

A. AION
B. PION
C. CRAO
D. Vertebrobasilar stroke
E. Retinal detachment

A

A. AION

Answer is more likely ‘A - Anterior Ischaemic Optic Neuropathy, because:
1. Most common
2. One or two days post - up to 12
3. Optic disc oedema (CRAO - fundoscopic appearance is that retina appears pale with cherry red central spot). PION fundoscopy is normal at first but has late developing oedema. It is less common than AION.

https://eyewiki.aao.org/Non-Arteritic_Anterior_Ischemic_Optic_Neuropathy_(NAION)

As mentioned earlier, optic disc edema is always present in the acute phase of NAION (the reason will be discussed in the section under Pathophysiology) and comes in two varieties, diffuse or segmental.

Posterior ischemic optic neuropathy (PION) encompasses those conditions that result in ischemia to any portion of the optic nerve posterior to the optic disc. By definition, PION will not cause disc edema.

Symptoms:
The classic description of patients with NAION presenting with acute, painless unilateral vision loss that is often described as a blurring or cloudiness of vision, often inferiorly, has been expanded. Although the majority of patients do not have accompanying pain, headache or periocular pain is reported in 8-12% of patients, which can make it difficult to differentiate from optic neuritis

740
Q

The transducer that provides the best resolution for an ultrasound guided median nerve block is

a) 2 MHz
b) 2-5 MHz
c) 5-8 MHz
d) 5-10 MHz
e) 6-13 MHz

A

e) 6-13 MHz

Atlas of Ultrasound-Guided Procedures in Interventional Pain Management, 13 (2011)
The wavelength and frequency of US are inversely related, i.e., ultrasound of high frequency has a short wavelength and vice versa. US waves have frequencies that exceed the upper limit for audible human hearing, i.e., greater than 20 kHz.3

High-frequency ultrasound waves (short wavelength) generate images of high axial resolution. Increasing the number of waves of compression and rarefaction for a given distance can more accurately discriminate between two separate structures along the axial plane of wave propagation.
However, high-frequency waves are more attenuated than lower frequency waves for a given distance; thus, they are suitable for imaging mainly
superficial structures.5

Conversely, low-frequency waves (long wavelength) offer images of lower resolution but can penetrate to deeper structures due to a lower degree of attenuation. For this reason, it is best to use high-frequency transducers (up to 10–15 MHz range) to image superficial structures (such as for stellate ganglion blocks) and low-frequency transducers (typically 2–5 MHz) for imaging the lumbar neuraxial structures that are deep in most adults.

741
Q

23.1 In order to provide anaesthesia of the scalp for awake craniotomy, it is necessary to
block branches of the

a. Greater and lesser occipital and greater auricular nerves
b. Trigeminal, greater and lesser occipital nerves
c. Trigeminal, greater occipital and greater auricular nerves
d. Facial, trigeminal and greater occipital nerves
e. Facial, greater and lesser occipital nerves

A

b) Trigeminal, greater and lesser occipital nerves

2005 blue book article: six nerves need to be blocked bilaterally
- supratrochlear
- supraorbital
- zygomaticotemporal
- auriculotemporal
- lesser occipital nerve
- greater occipital nerve
Minor contributions from the greater auricular nerve and third occipital nerve rarely encroach into the surgical field

742
Q

21.2 Identified risk factors for opioid-induced ventilatory impairment DO NOT include

a) Opiate use preoperatively
b) Male gender
c) Sleep disordered breathing
d) Obesity
e) Renal impairment

A

b) Male gender

Patient-related risk factors for OIVI are

older age,
female gender,
sleep disordered breathing (SDB),
obesity,
renal impairment,
pulmonary disease (in particular chronic obstructive pulmonary disease),
cardiac disease,
diabetes,
hypertension,
neurologic disease,
two or more comorbidities,
genetic variations in opioid metabolism,
and opioid-tolerant patients.

Modifiable risk factors include:
* Coadministration of sedatives (e.g. benzodiazepines, gabapentinoids, antipsychotics and sedating antihistamines)

  • Simultaneous use of multiple opioid agents (this does not include verified doses of opioids taken for management of chronic pain, where the patient has developed a tolerance to and physical dependence on these medications)
  • Continuous infusions of opioids
  • Initiation of long-acting opioid preparations (including methadone)
  • Multiple prescribers
  • Inadequate nursing assessments or responses
  • Reliance on unidimensional pain scores alone to assess adequacy of analgesia, and chasing’ pain scores – that is, titrating opioids to pain scores alone to reduce them to a predetermined acceptable number
  • Using opioids for pain that is not opioid-responsive

Source ANSCA PS 41

743
Q

21.1 A structure that is NOT clamped during a Pringle manoeuvre is the

a. Hepatic artery
b. Hepatic vein
c. Portal vein
d. Bile duct
e. Hepato-duodenal ligament

A

b. hepatic vein

Pringle Manoeuvre = clamping hepatoduodenal ligament (clamps hepatic artery, portal vein, CBD)

744
Q

21.1 You are performing a regional block for analgesia following knee surgery. You have an ultrasound probe scanning the anterior mid-thigh. The muscle indicated by the arrow in the ultrasound image below is the

A: Sartorius
B: Vastus Medialis
C: Adductor Longus
D: Gracilis
E: Rectus femoris

A

C: Adductor Longus

745
Q

23.1 The tip of an ideally-placed intra-aortic balloon catheter should lie in the

A. Distal to aortic root
B. Distal to left subclavian artery
C. Distal to left carotid
D. Distal to renal veins.

A

B. distal to LSCA

The appropriate performance of the IABP is dependent on proper position. Ideally, the tip of the balloon should be positioned 2–3 cm distal to the origin of the left subclavian artery (LSCA)

https://academic.oup.com/bja/article/110/2/316/228037

746
Q

22.2 This ultrasound image is acquired in preparation for a thoracic erector spinae plane block. The structure indicated by the arrow is the

a) Errector Spinae
b) Latismus Dorsi
c) Trapezius
d) Rhomboid
e) Psoas

A

d) Rhomboid

747
Q

23.1 In preschool-aged children having tonsillectomy under general anaesthesia, delirium
is more likely with the use of

a. Inhalational anesthesia
b. Remifentanil at end of case
c. Dexamethasone
d. Intranasal ketamine

A

a) inhalational anaesthesia

https://resources.wfsahq.org/atotw/emergence-delirium-in-pediatric-patients/

748
Q

20.2 In order to provide adequate anaesthesia for operation on the earlobe, the following nerve/s need to be blocked

a) Greater auricular
b) Auriculotemporal
c) Vagal auricular branch
d) Lesser occiptal nerve
e) Zygomaticotemporal

A

a) Greater auricular

749
Q

21.2 A patient with known type 3 von Willebrand disease presents with persistent epistaxis. First-
line medical therapy should include

a) DDAVP
b) Prothrombin X
c) Factor VIIa
d) Factor VIII

A

TXA

DDAVP for T1
Factor 8 for T2/3 or unresponsive DDAVP
(RCH Guidelines)

750
Q

23.1 A central venous catheter is recognised as being inadvertently placed in the common carotid artery five hours after insertion. The most appropriate management is

A. Open repair
B. Percutaneous repair
C. Remove and put pressure on it.

A

a) Open repair

Flow chart from Blue book

https://jamanetwork.com/journals/jamasurgery/fullarticle/1741862

751
Q

20.2 An analgesic which is a category A drug using the Australian and New Zealand categories for prescribing medicines in pregnancy is

a) Codeine
b) Methadone
c) Tramadol
d) Oxycodone
e) Morphine

A

Answer: a) Codeine
TGA Pregnancy categories https://www.tga.gov.au/prescribing-medicines-pregnancy-database

Category A
■ Codeine
Category C
■ Methadone
■ Tramadol
■ Oxycodone
■ Morphine

752
Q

22.2 The curve labelled ‘b’ is most likely to represent the flow–volume loop of a patient with

a) Asthma
b) Post lung transplant
c) Pulmonary fibrosis
d) Tracheal stenosis
e) VC palsy

A

Tracheal stenosis

753
Q

21.2 A 50-year-old man is admitted with a stroke and undergoes cerebral angiography. The artery
marked with an ORANGE arrow on the angiogram below is the

a) Vertebral
b) Basilar
c) PICA
d) Superior cerebellar
e) Anterior cerebral

A

a) Vertebral
orange = vertebral
blue = basilar
purple = PCA
red arrows = AICA
yellow = pontine arteries

Circle of Willis:

754
Q

21.1 The following is an image from a focussed cardiac ultrasound in a patient with dyspnoea presenting for thoracic surgery. The diagnosis is

a) Pericardial effusion
b) Tamponade
c) Pleural effusion
d) Loculated lung abscess

A

c) Pleural effusion

Source: BJA Ultrasound in critical care

Thoracic US revealing a large pleural effusion (E) that has displaced the lung. The diaphragm (D) and liver (L) are visualized. The depth from the skin to the fluid has been measured. Insertion of the needle at this site is not advised; given the proximity to the liver, a more superior approach should be marked.

755
Q

22.2 Compared to a normothermic patient, a patient with mild intraoperative hypothermia (35.0 oC) will have
?no remembered options but if repeat of 20.1->

a. Decreased bleeding
b. increased bleeding and normal aptt and inr
c. Increased bleeding and decreased inr
d. Increased bleeding and decreased aptt

A

b. increased bleeding and normal aptt and inr

Bleeding because cold = we know this

Haemtology analyzer in labs warms blood to 37.2 degrees (fixes hypothermia on sample)

756
Q

23.1 The sensory supply of the external nose is provided by all of the following nerves EXCEPT the

A. Lacrimal
B. Supratrochlear
C. Infratrochlear
D. Infraorbital
E. Anterior ethmoidal

A

Lacrimal

757
Q

20.2 The following are all independent predictors for severe bone cement implantation syndrome (BCIS) in cemented hemiarthroplasty for hip fracture EXCEPT

a. Male
b. GA
c. Previous history of same
d. Diuretic use
e. Age

A

Repeat

b. GA

Grade 1: moderate hypoxia (Spo2<94%) or hypotension [fall in systolic blood pressure (SBP) >20%].

Grade 2: severe hypoxia (Spo2<88%) or hypotension (fall in SBP >40%) or unexpected loss of consciousness.

Grade 3: cardiovascular collapse requiring CPR.

Patient Risk factors:
1. old age
2. poor pre-existing physical reserve
3. impaired cardiopulmonary function
-> NYHA 3 or 4
4. pre-existing pulmonary htn
5. Male Sex
6. Diuretics
7. ASA grade 3 or 4
8. osteoporosis
9. bony metastases
10. concomitant hip fractures (particularly pathological and intertrochanteric)
(latter due to abnormal vascular channels through which marrow contents can enter the circulation)

Surgical Risk factors
1. patients with previously un-instrumented femoral canal > revision surgery
2. Use of long-stem femoral component

Anaesthetic Risk reduction:
- discussion between surgeons and anaesthetists over uncemented vs. cemented based on patient Hx particularly if lon-stem prosthesis, femoral fracture or patients with cardiorespiratory disease
- no clear evidence regarding the impact of anaesthetic technique
- increase inspired O2 considered in all patients at time of cementation
- avoid intravascular volume depletion
- Higher level of haemodynamic monitoring in high risk patients

Factors NOT predictive of severe BCIS include:

Arteriosclerosis
Angina pectoris
Congestive heart failure
Beta-blockers
Angiotensin-converting enzyme inhibitors.
758
Q

20.1 In patients with sepsis and acute kidney injury, early renal replacement therapy (<12 hours) compared to a delayed strategy (48 hours) results in

a) Decreased 90 day mortality
b) No difference
c) Decreased icu time
d) Decreased length of admission
e) Increased 90 day mortality

A

b) No difference

NEJM: Timing of Renal-Replacement Therapy in Patients with Acute Kidney Injury and Sepsis

Among patients with septic shock who had severe acute kidney injury, there was no significant difference in overall mortality at 90 days between patients who were assigned to an early strategy for the initiation of renal-replacement therapy and those who were assigned to a delayed strategy.

759
Q

22.1 According to the ‘Fourth Consensus Guidelines for the Management of Postoperative Nausea and Vomiting (PONV)’ published in 2020, multimodal PONV prophylaxis should be implemented in adult patients

a. For everyone
b. 1 or more RF
c. 2 or more RF
d. 3 or more RF
e. 4 or more RF

A

b. 1 or more RF

In this iteration of the PONV guideline, one of the major changes is that we now recommend the use of multimodal prophylaxis in patients with one or more risk factors. This decision was made due to the concern over inadequate prophylaxis as well as the availability of antiemetic safety data.

760
Q

22.1 A 36-year-old man complains of left calf pain for two weeks. His pain is worse on walking but not completely relieved by sitting or lying down. On examination, he has mild weakness of left big toe extension. The most likely finding on MRI would be

a. L4/5 central disc bulge with facet joint pathology
b. L4/5 disc prolapse with compression of interveterbral foramina pathology
c. L5/S1 central disc bulge with facet joint degeneration
d. L5/S1 disc prolapse with compression of interveterbral foramina pathology

A

d. L5/S1 disc prolapse with compression of interveterbral foramina pathology

BJA: Chronic BAck Pain
https://academic.oup.com/bjaed/article/6/4/152/387156?itm_medium=sidebar&itm_source=trendmd-widget&itm_campaign=BJA_Education&itm_content=BJA_Education_0

Neurological examination may reveal sensory, motor and reflex abnormalities. Nerve root pain can be caused by disc herniation, spinal stenosis and epidural adhesions. The nerve roots leave the spinal canal via the intervertebral foramina.

761
Q

21.1 A 26-year-old man is brought into the Emergency Department four hours after an accidental chemical exposure during crop spraying. His clinical signs include bradycardia, vomiting, diarrhoea, coughing, miosis and weakness. A drug which is NOT recommended during his resuscitation and treatment is

a. Pralidoxime
b. Glycopyrrolate
c. Benzodiazepine
d. Suxamethonium
e. Rocuronium

A

suxamethonium

Organophosphate nerve agent poisoning:
https://www.bjanaesthesia.org.uk/article/S0007-0912(19)30401-5/fulltext

The depolarising neuromuscular blocking agent suxamethonium:
- may have a longer onset (i.e. 2 min) and
- duration of action (up to 12 h) secondary to the OP inhibition of BuChE.

Caution should be exercised with non-depolarising neuromuscular blocking agents for up to 2 yr and lower doses used to avoid prolonged paralysis.

Caution should also be exercised when using other BuChE metabolised drugs such as ester local anaesthetics and mivacurium

Mainstay of treatment Pralidoxime and Atropine (5-10mg IV every 5-10mins until reversal of 3 B’s)
Benzodiazepines used for seizure termination
Glycopyrolate not mentioned in treatment but could be useful

762
Q

22.2 A 72-year-old woman on aspirin presents to her ophthalmologist for follow-up three days after you performed a transconjunctival peribulbar block for cataract surgery on her left eye. She complains of painless periorbital swelling, erythema, and mild chemosis which started the day after surgery but is improving. She had a peribulbar block three weeks ago for surgery on the other eye. The most likely diagnosis is

a. Retrobulbar bleeding?
b. Residual swelling from peribulbar block
c. Infection
d. hyalase reaction/allergy

A

d. hyalase reaction/allergy

763
Q

21.1 The substance that should be avoided in a patient with history of anaphylaxis to MMR vaccine is

a. Protamine
b. Gelofusine
c. Sulphonamides
d. Penicilins

A

b. Gelofusin
Gelatin

764
Q

21.2 A 74-year old man in the post-anaesthesia care unit complains of chest pain. An electrocardiogram (ECG) is performed. The occluded coronary artery is the

a) RCA
b) LCx
c) LAD
d) PDA

A

RCA (Inferior STEMI)
- 80% RCA
- 18% LCx
- 2% rare wrap around LAD

ST elevation in lead III > lead II
Presence of reciprocal ST depression in lead I
Signs of right ventricular infarction: STE in V1 and V4R

Source LITFL

765
Q

20.2 In the morbidly obese the induction dose of propofol should be calculated based on

a) Lean body weight
b) Total body weight
c) Ideal body weight
d) Ideal body weight + 70%
e) Adjusted body weight

A

a) Lean body weight

FROM SOBA:

LBW exceeds IBW in obese and plateaus at ~100kg in men and ~70kg in females
IBW used to calculate the adjusted body weight for maintenance infusion of propofol (IBW +40%).
IBW calculated using Broca formula (Ht in cm - 100; Ht in cm =105; as optimal weights for men and women respectively in kg)

766
Q

bonus neuro radiology questions

A 45 year-old man developed the lightning onset of an explosive headache followed by coma.
He undergoes cerebral angiography.
His cerebral aneurysm is in the

a. Anterior Cerebral Artery
b. Anterior communicating artery
c. MCA
d. PCA
e. Basilar artery

A

e. Basilar

https://case.edu/med/neurology/NR/SubarachnoidHemorrhag3%20Bas2.htm

767
Q

20.1 Of the following, the maternal cardiac condition that represents the highest risk of mortality associated with pregnancy is

a. Bicuspid aortic valve with significantly dilated aortic root
b. Fontan circulation
c. HOCM
d. PDA

A

A - Modified WHO class 4

B - Modified WHO class 3 (4 if ANY complication)
C - Modified WHO class 2-3 (if severe AS - 4)
D - Modified WHO class 1

Class 4 = 40-100% risk of event

Source
https://academic.oup.com/eurheartj/article/39/34/3165/5078465

Table:
https://academic.oup.com/view-large/186437995

768
Q

20.1, 21.2 A patient with a history of paroxysmal atrial fibrillation and chronic obstructive airways disease develops a wheeze intraoperatively which resolves with administration of salbutamol via the endotracheal tube.
Soon after, he develops rapid atrial fibrillation with a ventricular rate of 120 beats per minute, a BP of 90/60 and an ETCO2 of 40mmHg. His regular medications are
inhaled salbutamol, inhaled salmeterol and digoxin 125mcg daily. The next most suitable treatment is

a) Amiodarone 150mg over 30minutes, then 1mg/min for 6 hours
b) Esmolol 500mcg/kg and infusion
c) Direct cardioversion with 50J
d) Metoprolol 2.5mg IV up to 3 doses

A

a) Amiodarone 150mg over 30minutes, then 1mg/min for 6 hours

UP TO DATE: Arrhythmias in COPD
For patients with atrial fibrillation and COPD, we suggest using verapamil or diltiazem rather than metoprolol in patients who require ventricular rate control (Grade 2C).

Metoprolol is reserved for patients who do not respond to the calcium channel blockers and do not have uncontrolled bronchoconstriction. For those with an accessory pathway or heart failure, amiodarone or digoxin may be preferred as outlined in the table (table 3).

Addition of Digoxin in this answer stem could be prefered over Amiodarone

769
Q

20.2 During anaesthesia of a patient using sevoflurane as maintenance and who has been paralysed with a neuromuscular blocking agent, the following monitors must be in use EXCEPT

a) ETCO2
b) ET volatile
c) Pulse oximetry
d) ECG
e) O2 analysis

A

d) ECG

ECG – have AVAILABLE for every anaesthetised patient. Should be used for patients undergoing general and major regional anaesthesia AS CLINICALLY INDICATED.

Oxygen analyser – continuous operation for every patient when anaesthesia breathing system in use.

Pulse oximeter – use for every patient undergoing general anaesthesia or sedation.

Carbon dioxide monitor – use for every patient undergoing general anaesthesia, and have immediately available for sedation cases.

ET Volatile – should be in use for every patient undergoing general anaesthesia from an anaesthesia delivery system where inhalational anaesthetic agents are delivered.

770
Q

21.1 According to the ANZCA ‘Guideline on infection control in anaesthesia’, skin preparation prior to central neuraxial blockade should be performed using

a. 10% Povidine iodine
b. 0.5% Chlorhexidine/ETOH
c. 5% Chlorhexidine
d. 3% chlorhexidine

A

b. 0.5% Chlorhexidine/ETOH

For skin preparation, 0.5 per cent chlorhexidine in alcohol, where available, is recommended for neuraxial techniques although it should be noted that very small quantities of neuraxial chlorhexidine have been implicated in cases of severe neurotoxicity

771
Q

23.1 This 12 lead ECG shows

A. Complete heart block
B. Mobitz I
C. Mobitz II
D. LPFB + RBBB
E. LAFB + RBBB

A

E
LAFB:
> Left axis deviation (usually -45 to -90 degrees)
> qR complexes in leads I, aVL
> rS complexes in leads II, III, aVF
> Prolonged R wave peak time in aVL > 45ms

LPFB:
> Right axis deviation (RAD) (> +90 degrees)
> rS complexes in leads I and aVL
> qR complexes in leads II, III and aVF
> Prolonged R wave peak time in aVF

772
Q

21.2 The most likely complication from ultrasound guided left internal jugular central venous line insertion is

a) Arterial puncture
b) Thoracic duct injury
c) Pneumothorax
d) Haematoma

A

a) Arterial puncture
- thoracic duct injury is a risk with left sided IJ CVC insertion, but it is a rarer complication.

773
Q

22.1 You are about to anaesthetise a 25-year-old man for an open appendicectomy. He has a history of tricuspid atresia for which he has had a Fontan procedure. An important goal in managing his ventilation under anaesthesia is to ensure

a. Long I time, low pressures
b. Long I time, PEEP
c. Long E time
d. Spontaneous ventilation

A

c. Long E time

Spont vent not appropriate for this surgery as will require RSI so spont vent can’t be ensured

BJA: fontan circulation:
For relatively short procedures, Fontan patients are probably better off breathing spontaneously, as long as severe hypercarbia is avoided. For major surgery, or when prolonged anaesthesia is required, control of ventilation and active prevention of atelectasis is usually advisable. Potential disadvantages of mechanical ventilation in Fontan patients relate to the inevitable increase in mean intrathoracic pressure. This causes decreased venous return, decreased pulmonary blood flow, and hence, decreased cardiac output. Low respiratory rates, short inspiratory times, low PEEP, and tidal volumes of 5–6 ml kg−1 usually allow adequate pulmonary blood flow, normocarbia, and a low PVR. Hyperventilation tends to impair pulmonary blood flow, despite the induced respiratory alkalosis, because of the increased mean intrathoracic pressure.

https://academic.oup.com/bjaed/article/8/1/26/277637

774
Q

20.1 Sublingual (intralingual) sux 15kg kid what dose:
a) 20mg
b) 40mg
c) 50mg
d) 60mg
e) 15mg
? 30mg as other option

A

30 (2 mg/kg)

CEACCP Laryngospasm in anaesthesia (2014)
https://academic.oup.com/bjaed/article/14/2/47/271333

Intravenous (IV):
- 0.1-2 mg/kg
- lower doses used to break laryngospasm, but keep patient spont vent

Intramuscular (IM):
- 4 mg/kg (max 200 mg)
- break laryngospasm: 45-60 seconds
- full paralysis: 3-4 minutes

Intralingual (IL):
- 2 mg/kg
- an IM injection into body of tongue
- full relaxation after 75 seconds

Intraosseous (IO):
- 1 mg/kg
- onset 35 seconds

775
Q

21.2 A woman experiences a post-partum haemorrhage associated with uterine atony that is unresponsive to oxytocin and ergometrine. The recommended intramuscular dose of carboprost (15-methyl prostaglandin F2 alpha ) to be administered is

a) 250mcg IM once
b) 250mcg IM q15mins, up to 2mg
c) 500mcg IM
d) 250mcg IV
e) 500mcg IV

A

15-methyl-PGF2α (carboprost; Prostinfenem) which may be administered in one of two ways:
Intra-muscular injection of 0.25mg, in repeated doses as required at intervals of not less than 15
minutes to a maximum total cumulative dose of 2.0mg (ie up to 8 doses)

Source RANZCOG PPH Guideline 2021

776
Q

20.2 When performing an infraclavicular block of the brachial plexus under ultrasound guidance, the structure indicated by the arrow is the

a) Musculocutaneous nerve
b) Lateral Cord
c) Medial Cord
d) Superior trunk
e) Inferior trunk

A

c) Medial Cord

777
Q

20.1 What is the arrow pointing to?

a. Ilioinguinal
b. Psoas
c. Genitofemoral
d. Lateral cutaneous nerve of thigh
e. Obturator

A

c. Genitofemoral

778
Q

22.2 Adverse effects of the use of sodium-glucose co-transporter 2 inhibitors in the perioperative period do NOT include

a) UTI
b) Hyperglycaemic DKA
c) Hypovolaemia
d) Hypercalcaemia

A

d) Hypercalcaemia
SGLT2 inhibitors are relatively new and have several side effects that warrant caution, including the unique risks of diabetic ketoacidosis (DKA), mycotic genital infections and possibly lower limb amputations. Also polyuria, volume depletion, hypoT

Hypoglycaemia
As the glucose-lowering mechanism of SGLT2 inhibitors is glycaemia-dependent, hypoglycaemia risk is low. However, hypoglycaemia may occur when SGLT2 inhibitors are used in conjunction with sulphonylurea or insulin therapy.

https://www1.racgp.org.au/ajgp/2021/april/use-of-sodium-glucose-co-transporter-2-inhibitors#:~:text=Safety%20and%20tolerability,and%20possibly%20lower%20limb%20amputations.

779
Q

23.1 Ongoing cerebral seizure activity induced by electroconvulsive therapy should be medically terminated after

a) 30s
b) 60s
c) 90s
d) 120s
e) 150s

A

d) 120s

780
Q

23.1 The use of erythropoietin before major surgery results in

a) Less transfusion, same thrombosis
b) Less transfusion, more thrombosis
c) No change in transfusion or thrombosis
d) No change in transfusion, more thrombosis

A

repeat

a) Less transfusion, same thrombosis

●A 2019 meta-analysis of randomized trials comparing preoperative administration of EPO versus placebo (32 trials; 4750 patients, mostly orthopedic and cardiac surgery) found reduced blood transfusions in the EPO groups. Decreased blood transfusions were seen in the entire population (RR 0.59, 95% CI 0.47-0.73; 28 trials), as well as the subgroups undergoing cardiac surgery (RR 0.55, 95% CI 0.47-0.73; nine trials) and major orthopedic surgery (RR 0.36, 95% CI 0.28-0.46; five trials). In addition, the EPO group had increased hemoglobin levels. There was no increase in the incidence of thromboembolic events with EPO.

781
Q

22.2 A 34-year-old for a diagnostic laparoscopy has a height of 158 cm and a weight of 120 kg (BMI 48 kg/m2). For induction of anaesthesia, appropriate drug dosing includes

a) Fentanyl based on TBW
b) Rocuronium based on LBW
c) Propofol induction based on ABW
d) Propofol infusion based on LBW
e) Suxamethonium based on IBW

A

b) Rocuronium based on LBW

782
Q

21.1 The nerve labelled with the arrow in the diagram is the (diagram of a nerve plexus shown)

A
783
Q

22.2 A woman experiences a postpartum haemorrhage associated with uterine atony that is unresponsive to oxytocin and ergometrine. The recommended intramuscular dose of carboprost (15-methyl prostaglandin F2 alpha) to be administered is

a) 250mcg IM once
b) 250mcg IM q15mins, up to 2mg
c) 500mcg IM
d) 250mcg IV
e) 500mcg IV

A

b) 250mcg IM q15mins, up to 2mg

QLD maternity guidelines
Carpoprost 250mcg IM
Repeat every 15-90min as r

784
Q

23.1 The dose of hydrocortisone that has equivalent glucocorticoid effect to dexamethasone 8 mg is

a. 50mg hydrocortisone
b. 100mg hydrocortisone
c. 150mg hydrocortisone
d. 200mg hydrocortisone
e. 250mg hydrocortisone

A

c. 200mg hydrocortisone

200mg Hydrocortisone or 25mg Prednisolone

Conversion
Prednisone 1mg =
Hydrocortisone 4mg =
Dexamethasone 0.15mg =
Triamcinolone 0.8mg =
Methylprednisolone 0.8mg =
Betamethasone 0.15mg =

(https://litfl.com/corticosteroids-overview/)

785
Q

21.2, 22.2 The estimated proportion of human induced climate change attributable to nitrous oxide is

a) 0.01
b) 0.06
c) 1
d) >6

A

d) >6

Medical emissions of N2O account for <4% of all emissions of N2O, the majority originating from microbial action on nitrogenous fertilizers

786
Q

20.2 Features indicating an arterial retrobulbar haemorrhage sustained during a peribulbar eyeblock administered for cataract surgery include all of the following EXCEPT

a) Chemosis
b) Proptosis
c) Decreased visual acuity
d) Increased intraocular pressure

A

Chemosis is NOT a sign of arterial retrobulbar haemorrhage

Signs of arterial retrobulbar haemorrhage:
1. Sudden onset proptosis
2. Raised IOP
3. Reduced acuity.

787
Q

23.1 Double sequential external defibrillation is performed by applying two shocks from

a. Single set of pads, <1 second apart
b. Single set of pads, <5 seconds apart
c. Two sets of pads, <1 second apart
d. Two sets of pads, <5 seconds apart
e. Two sets of pads, simultaneously

A

c. Two sets of pads, <1 second apart

For DSED, to avoid possible defibrillator damage caused by shocks applied at the same instant, a short delay (<1 second) between shocks was created by having a single paramedic depress the “shock button” on each defibrillator in rapid sequence (anterior–lateral followed by anterior–posterior)

Among patients with refractory ventricular fibrillation, survival to hospital discharge occurred more frequently among those who received DSED or VC defibrillation than among those who received standard defibrillation.

https://www.nejm.org/doi/full/10.1056/NEJMoa2207304

788
Q

20.1 NAP 5 incidence of awareness with GA LSCS:
A) 1:700
B) 1:4,000
C) 1:8,000
D) 1:16,000
E) 1:32,000

A

Answer: a) 1:700

NAP5 Summary
The incidence of reports of AAGA after general anaesthetic Caesarean section was much higher, ~1:670.

789
Q

23.1 The following pressure-volume loop is displayed on your ventilator screen. The
shape of this loop indicates

a. Over-expansion
b. Under-expansion
c. Normal ventilation
d. PEEP too high
e. PEEP too low

A

a) over-expansion

The first graphic (loop a) shows the pattern of a typical pressure-volume loop, which rises in a counterclockwise direction until forming a complete loop. It also displays inflection points, which display rapid changes to the slope of the limb.

The lower inflection point (LIP) occurs due to the opening of collapsed alveoli, resulting in a sharp increase in volume. The upper inflection point (UIP) occurs near the end of inspiration when more pressure leads to only a minimal increase in volume.

The second graphic (loop b) displays how overdistension and hysteresis appear on a pressure-volume loop. Overdistention occurs when the lungs receive too much volume or pressure and can result in injury. Hysteresis refers to lung tissue that behaves differently on inspiration and expiration.

In other words, it takes more energy for the lungs to inflate than it does to deflate. Therefore, hysteresis on a pressure-volume loop refers to the space between the inspiratory and expiratory limbs. When the patient’s lung compliance or airway resistance changes, so will the hysteresis and, thus, the appearance of the loop.

Note: A pressure-volume loop under normal conditions should resemble the shape of a football (American). A curve with a flat appearance indicates decreased lung compliance. A steep curve, on the other hand, indicates increased lung compliance. A wide curve indicates increased airway resistance, whereas the opposite is true if the loop appears more narrow.

https://www.respiratorytherapyzone.com/ventilator-waveforms/#:~:text=Note%3A%20A%20pressure%2Dvolume%20loop,hand%2C%20indicates%20increased%20lung%20compliance.

790
Q

22.1 A 54-year-old woman has a laryngeal mask airway inserted for a surgical procedure. The following day she complains of tongue numbness and abnormal taste over the posterior third of the tongue.
The most likely site of the nerve injury is the

a) Glossopharyngeal nerve
b) Lingual nerve
c) Facial nerve
d) Vagus nerve

A

Glossopharyngeal

791
Q

20.1 A 55-year-old man is found to be in atrial fibrillation. He has no previous medical history. Physical examination, blood pressure and fasting blood glucose are normal. Appropriate long-term management is

A. Aspirin
B. Dabigatran
C. No anticoagulation
D. Warfarin
E. Rivaroxaban

A

C. No Anticoagulation

  • if male CHA2DS2-VASc score ≥2 to be recommended chronic OAC (Grade 1A).
  • if female CHA2DS2-VASc score ≥3 to be recommended chronic OAC (Grade 1A).
  • non-sex risk factor also holds bearing:
  • For patients with CHA2DS2-VASc score of 1 in males and 2 in females based on age 65 to 74 years, we recommend chronic OAC (Grade 1A).

Up to date:

Our approach to deciding whether to prescribe anticoagulant therapy for patients with AF (excluding those with rheumatic mitral stenosis that is severe or clinically significant [mitral valve area ≤1.5 cm2], a bioprosthetic valve [surgical or bioprosthetic] within the first three to six months after implantation, or a mechanical heart valve) is as follows:

*For a CHA2DS2-VASc score ≥2 in males or ≥3 in females, we recommend chronic OAC (Grade 1A).

*For a CHA2DS2-VASc score of 1 in males and 2 in females:
-For patients with CHA2DS2-VASc score of 1 in males and 2 in females based on age 65 to 74 years, we recommend chronic OAC (Grade 1A). Age 65 to 74 years is a stronger risk factor than the other factors conferring one CHA2DS2-VASc score point.
-For patients with other risk factors, the decision to anticoagulate is based upon the specific nonsex risk factor and the burden of AF. For patients with very low burden of AF (eg, AF that is well documented as limited to an isolated episode that may have been due to a reversible cause such as recent surgery, heavy alcohol ingestion, or sleep deprivation), it may be reasonable to forgo chronic OAC and institute close surveillance for recurrent AF, although it may not be possible to reliably estimate AF burden from surveying symptoms or infrequent monitoring. The frequency and duration of AF episodes vary widely over time, and episodes are often asymptomatic.

*For patients with a CHA2DS2-VASc of 0 in males or 1 in females, we suggest against OAC (Grade 2C). Patient values and preferences may impact the decision. For example, a patient who is particularly stroke averse and is not at increased risk for bleeding may reasonably choose anticoagulation, particularly if the patient is a candidate for treatment with a direct oral anticoagulant (DOAC).

2019 AHA/ACC/HRS Focused Update of the 2014 AHA/ACC/HRS Guideline

792
Q

21.1 A man is brought into hospital after a motor vehicle accident 45 minutes ago. His chest x-ray is shown.

This is most consistent with a left-sided

A

?? XR Reviews

793
Q

22.1 Of the following, the drug that is LEAST likely to provide effective analgesia following paediatric tonsillectomy is

a. Ketamine
b. Clonidine
c. NSAIDs
d. Paracetamol
e. Dexamethasone

A

b. Clonidine
PROSPECT 2021
https://associationofanaesthetists-publications.onlinelibrary.wiley.com/doi/full/10.1111/anae.15299

Pre-operative and intra-operative interventions that improved postoperative pain were:
- paracetamol;
- non-steroidal anti-inflammatory drugs;
- intravenous dexamethasone;
- ketamine (only assessed in children);
- gabapentinoids;
- dexmedetomidine;
- honey;
- acupuncture.

Inconsistent evidence was found for:
- local anaesthetic infiltration;
- antibiotics;
- magnesium sulphate.
Limited evidence was found for
- clonidine.

The analgesic regimen for tonsillectomy should include:
1. paracetamol;
2. non-steroidal anti-inflammatory drugs; and
3. intravenous dexamethasone,
4. with opioids as rescue analgesics.

Analgesic adjuncts such as:
1. intra-operative and postoperative acupuncture as well as
2. postoperative honey are also recommended.
3. Ketamine (only for children); dexmedetomidine; or gabapentinoids may be considered when some of the first-line analgesics are contra-indicated

794
Q

21.1 The most likely cause of hip adduction in a patient undergoing transurethral resection of a bladder tumour is

a) Neuraxial anaesthesia to T8
b) Inadequate depth of anaesthesia
c) Lateral bladder wall resection
d) Bladder perforation

A

c) Lateral bladder wall resection

obturator nerve stimulation

BARASH:
A serious intraoperative complication of TURBT is bladder perforation by the rigid cystoscope during tissue resection, which occasionally occurs owing to unexpected patient movement. For this reason, muscle relaxation is preferred during general anesthesia, particularly in lateral wall resections, where the obturator nerve may be stimulated by electrocautery, producing a violent contraction of the ipsilateral thigh muscles. Neuraxial anesthesia to the T9 to T10 dermatomal level also provides adequate anesthesia for the procedure and prevents the obturator reflex. Regional anesthesia may facilitate detection of bladder perforation. Postoperative pain is usually minimal and responds well to nonopiate and opiate medications.

795
Q

22.2 You are called to recovery to review an 80-year-old woman post neck of femur fracture fixation performed under general anaesthesia with a fascia iliaca block. She has a history of mild dementia. She has become confused and agitated after initially being cooperative and pain-free. The most appropriate drug therapy to manage her is intravenous

a. Clonidine
b. dexmedetomidine
c. propofol
d. midazolam
e. haloperidol

A

e. haloperidol

Clonidine-> no mention in the evidence
dexmedetomidine-> as an infusion seems to reduce risk of post-op delerium and could be used to treat but not necessarily practical in combative patient
Propofol-> not mentioned
Midazolam-> avoid benzos as can worsen delerium

If pharmacological approaches are required to reduce
risk of harm to the person with agitated delirium, then
haloperidol can be administered in incremental 0.5-mg
doses. Benzodiazepines should be used for people with
alcohol-related cognitive disorders or in people with
Parkinsonian dementia. There is no evidence to support the
use of prophylactic pharmacological measures
(cholinesterase inhibitors, antipsychotics, melatonin) in
routine peri-operative care for patients at risk of POD

https://anaesthetists.org/Portals/0/PDFs/Guidelines%20PDFs/Guideline_Perioperative_care_of_people_with_dementia_2019.pdf?ver=2019-02-11-121238-777&timestamp=1549888049165&ver=2019-02-11-121238-777&timestamp=1549888049165

Duan and colleagues conducted a meta-analysis of 18 clinical trials and found that intraoperative and postoperative dexmedetomidine administration significantly reduces the risk postoperative delirium (odds ratio 0.35).
->
https://www.bjanaesthesia.org/article/S0007-0912(20)30566-3/fulltext

796
Q

23.1 The main advantage of using noradrenaline (norepinephrine) over phenylephrine for
the prevention of hypotension as a result of spinal anaesthesia for elective
caesarean section is

a) Better APGAR
b) Better foetal acid-base balance
c) Less nausea & vomiting
d) Less maternal bradycardia

A

d) less maternal bradycardia (repeat)

797
Q

23.1 One metabolic equivalent (1 MET) is defined as the

a. O2 consumption walking 4km/h
b. O2 consumption when sitting
c. Energy expenditure walking 4km/h
d. Energy expenditure when sitting.

A

b) O2 consumption when sitting

One metabolic equivalent (MET) is defined as the amount of oxygen consumed while sitting at rest and is equal to 3.5 ml O2 per kg body weight x min.

798
Q

21.1 The best patient position to evaluate the gastric contents with ultrasound is

a. Right lateral
b. Trendelburg
c. Supine
d. Left lateral
e. Reverse trendelenberg

A

Right lateral Decubitus

BJA: ultrasound

799
Q

21.1 A 30-year-old previously healthy woman is four days post-caesarean section. You are asked to see her to manage her abdominal pain. Over the last two days she has had increasing abdominal pain, increasing abdominal distension, tachycardia and nausea. An abdominal x-ray shows a caecal diameter of 9 cm. After excluding mechanical obstruction, an appropriate management option is

a) neostigmine infusion
b) morphine PCA
c) Naloxone
d) Lactulose

A

a) neostigmine infusion

Consider this Ogilve’s Syndrome
Psuedo-obstruction.
If > 9cm dilation, would need surgical management.

https://www.ncbi.nlm.nih.gov/pmc/articles/PMC3168359/#!po=17.5000

800
Q

21.2 Intraoperative cell salvage is contraindicated in

a) LSCS
b) Revision of infected THR
c) Heparin allergy
d) Severe coagulopathy
e) Phaeochromocytoma

A

phaeochromatoma

801
Q

21.2 A drug which is likely to slow the heart rate in a patient with a heart transplant is

a. Phenylephrine
b. Digoxin
c. Metaraminol
d. Adenosine

A

Adenosine (effect is exagerated)

802
Q

20.2 A patient with a history of restless leg syndrome is experiencing significant agitation post op in recovery. After excluding other precipitating causes, the best treatment of the agitation in this patient is

a) Midazolam
b) Olanzepine
c) Haloperidol
d) Clozapine
e) Droperidol

A

Repeat

a) Midazolam

Blue Book 2019

RLS Definition
-Common neurological sensorimotor disorder characterised by the urge to move ones legs
-It is associated with unpleasnat paraesthesias deep within the legs during periods of rest or inactivity, whihc are relieved by movement

Pathophysiology
> RLS can be primary (idiopathic) or secondary
> patients with secondary RLS develop symptoms secondary to another disease process or drug
> causes of 2ry RLS include Iron deficiency, pregnancy, kidney disease, rheumatic disease and medications
> 1ry RLS Pathophysiology is partially known and includes genteic component along with theories of dopamine and brain iron dysregulation

Anaesthetic implications
- RLS may worsen recur or present perioperatively
- Common triggers include sleep deprivation and immobilisation
- Drug therapy for RLS shopuld be continued perioperatively where possible
- interuptions to treatment should be for the shortest time possible to prevent rebound effects
- if imobilised for a long period of time, dopamine agonists such as rotigotone may be required
- premedication with benzodiazepines or pregabalin may be useful
- prolonged medical imaging procedures or procedures under local anaesthetic alone may not be possible.
- post-op agitation 2ry to akathisia may be misinterpretted as delerium and treated with dopamine antagonists such as haloperidol which will worsen symptoms, Benzodiazepines should be used instead.

Drugs that may exacerbate RLS
1. Classic neuroleptics
- Haloperidol, prochlorperazine, promethazine
2. Atypical antipsychotics
- clozapine, olanzapine, quetiapine
3. Antidepressants
- amitriptyline, citalopram, lithium
4. Antihistamines
- promethazine
5. Dopamine antagonist anti-emetics
- metoclopramide
6. opioids
- Tramadol (serotonin), naloxone/naltrexone (antagonists)

> Opioids, oxycodone, fentanyl, morphine etc generally have a beneficial effect

> IV anaesthetics, Inhalational anaesthetics, muscle relaxants, local anaesthetics, NSAIDs, Antiemetics have no effect

Goals:
1. Prevent RLS exacerbation
- avoid drug triggers
- premedicate with benzos
- use benzos for sedation
- continue treatment for RLS
- consider topical dopamine agonists when oral route unavailable

  1. Alleviate post-op exacerbations
    - use parenteral opioids
    - Apomorphine
    - mobilise patient ASAP
  2. Alleviate long-term exacerbation of RLS after surgery
    - monitor ferritin levels
    - If ferritin level < 75mcg/ml treat with oral or IV iron replacement
    - transiently increase dopamine agonist dose to TDS or QID if unable to leave bed
803
Q

22.1 The washing process of modern cell savers for intraoperative blood salvage removes all the following EXCEPT

a) Microaggregates of leucocytes
b) Platelets
c) Clotting factors
d) Fetal cells
e) Free Hb

A

d) Fetal cells

Does not remove foetal red cells or vasoactive molecules (eg don’t use in pheochromocytoma surgery).

804
Q

22.2 A six-year-old child weighing 20 kg presents to hospital two hours after sustaining a burn to 25% of her body. Appropriate fluid management should include 1000 mL Hartmann’s solution in the next

a. 4 hours
b. 6 hours
c. 8 hours
d. 12 hours
e. 24 hours

A

B 6 hours

20 x 25 x 4 = 2000 L
(Parklands)

In first 8 hours 50%
1 L in 8 hours FROM TIME OF BURN

So in 6 hours.

805
Q

22.1 When compared to the interscalene block, the supraclavicular block has the advantage that

a. Less PTX
b. Less phrenic nerve block

A

Less phrenic nerve block

806
Q

21.2 A new volatile agent is developed. The property it shares with sevoflurane that will enable it to
be used in a sevoflurane vapouriser and deliver an accurate concentration is its

a) Blood:gas partition coefficient
b) Oil:gas partition coefficient
c) Saturated vapour pressure
d) Boiling point

A

same SVP

807
Q

20.2 This lung ultrasound shows

a) Normal lungs
b) Pulmonary odema
c) Pneumothorax
d) Pleural effusion
e) Pneumonia

A

a) Normal lungs

Probe selection

Linear probe (8–12 MHz)

These high-frequency probes give good resolution of superficial structures. As the anterior pleura is relatively superficial, excellent images of the pleura and lung sliding can be obtained. The poor penetration of high-frequency US and the narrow sector width mean deeper structures are poorly imaged.
Curvilinear probe (3–5 MHz)

This is the best all-round probe for LU. Lung sliding can be easily visualized as can IS. Effusions, consolidated lung, and the diaphragm are also well imaged because of the good penetration and large sector width. The large footprint of the probe means some angulation is needed to avoid the ribs when scanning postero-laterally.
Phased array (3–4.5 MHz)

These probes have a useful footprint for getting in between the ribs. They can be used to demonstrate all the signs of LU but the clarity of the images is not as good.
General points

The clearest images are obtained by having the image as shallow as possible with the focus point at the level of interest. The frequency can be adjusted to enhance the image, depending on the depth. Increasing the frequency on a curvilinear probe will improve the appearance of lung sliding whilst worsening the appearance of a consolidated lung base.

808
Q

21.1 A patient has bipolar disorder and is on long term lithium therapy. An analgesic which should be avoided is

a. Diclofenac
b. Tramadol
c. Oxycodone
d. Methadone

A

a. Diclofenac

LIthium perioperative concerns:
- Prolongation of NMB
- Reduction in anaesthetic agent requirement
- Avoid NSAIDs
- No withdrawl symptoms
- Discontinue 24hrs before surgery

NSAIDs differentially alter lithium concentrations by multiple mechanisms, and one of these is to reduce prostaglandin E2

BJA: perioperative advice for psychotropic drugs

809
Q

23.1 Anaesthesia-induced rhabdomyolysis differs from malignant hyperthermia in that it is NOT

a. Reduced Myoglobinaemia
b. Less increase in ETCO2
C. Less muscle rigidity

A

a. Reduced Myoglobinaemia

Repeat but its not myoglobinuria it was myoglobinaemia

  • There is NOT reduced myoglobinuria with AIR compared to MH (both have myoglobinuria)
  • There IS less increase in ETCO2
  • There IS less muscle rigidity
810
Q

23.1 A non-obese adult patient is administered a target-controlled propofol infusion for more than 15 minutes, with a constant target plasma concentration of 4 μg/mL propofol. Compared to the Marsh model, the propofol dose given by the Schnider model will be a:

a) Smaller bolus smaller total dose
b) Smaller bolus larger total dose
c) Larger bolus smaller total dose
d) Larger bolus larger total dose
e) Smaller bolus same total dose

A

a) Smaller bolus smaller total dose

811
Q

In the management of anaphylaxis in a 5-year-old with no intravenous or intra-osseous access, the correct dose of intramuscular adrenaline is

A. 20mcg
B. 50mcg
C. 100mcg
D. 150mcg
E. 300mcg

A

D. 150mcg

812
Q

22.1 In long-term use of nonsteroidal anti-inflammatory drugs, the risk of thromboembolic complications is lowest with

a. Ibuprofen
b. Celecoxib
c. Diclofenac
d. Naproxen

A

b. Celecoxib

The ANZCA pain booklet also references this study:

Risk of acute myocardial infarction with NSAIDs in real world use: bayesian meta-analysis of individual patient data

With use for one to seven days the probability of increased myocardial infarction risk (posterior probability of odds ratio >1.0) was 92% for celecoxib, 97% for ibuprofen, and 99% for diclofenac, naproxen, and rofecoxib. The corresponding odds ratios (95% credible intervals) were 1.24 (0.91 to 1.82) for celecoxib, 1.48 (1.00 to 2.26) for ibuprofen, 1.50 (1.06 to 2.04) for diclofenac, 1.53 (1.07 to 2.33) for naproxen, and 1.58 (1.07 to 2.17) for rofecoxib. Greater risk of myocardial infarction was documented for higher dose of NSAIDs. With use for longer than one month, risks did not appear to exceed those associated with shorter durations.

The ANZCA pain booklet also references this study

https://www.ncbi.nlm.nih.gov/pmc/articles/PMC6281031/
Naproxen OR 1, Celecoxib OR 1.3, Ibuprofen OR 1.49, Diclofenac OR 1.63 in UK study 2016 investigating NSAID use in knee OA.

813
Q

21.2 The intrinsic muscles of the larynx do NOT include

a) Cricothyroid
b) Suprahyoid
c) Thyroarytenoid
d) Transverse arytenoid

A

b) Suprahyoid

Extrinsic Muscles of the larynx:
1. Sternothyroid muscle
2. Thyrohyoid muscle
3. Inferior constrictor of the pahrynx

Indirect elevators of the larynx:
1. Mylohyoid
2. Stylohyoid
3. geniohyoid

Indirect depressors of the larynx:
1. Sternohyoid
2. Omohyoid

Intrinsic Muscles of the larynx:
1. Posterior Cricoarytenoid
2. Lateral Cricoarytenoid
3. Interarytenoid
4. Thyroarytenoid
5. Vocalis
6. Cricothyroid

actions of intrinsic laryngeal muscles
1. Abductor of the cords: posterior cricoarytenoids
2. Adductors of the cords: lateral cricoarytenoids, interarytenoids
3. Sphincter to the vestibule: aryepiglottics (interarytenoid), thyroepiglotics
4. Tension regulators of the cords: Cricothyroids (tensors), Thyroarytenoids (relaxors), Vocales (fine adjustment)

814
Q

20.1 A postpartum woman presents with numbness over the anterior thigh, and weakness on flexion of the hip and extension of the knee. An epidural was sited for labour and she underwent an instrumental delivery. The most likely site of the injury is the:

a) Femoral nerve
b) Lateral femoral cutaneous nerve
c) Lumbosacral plexus
d) Obturator nerve
e) Sciatic nerve

A

a) Femoral nerve

Nerve roots: L2-L4

Motor functions: Innervates the anterior thigh muscles that flex the hip joint (pectineus, iliacus, sartorius) and extend the knee (quadriceps femoris: rectus femoris, vastus lateralis, vastus medialis and vastus intermedius),

Sensory functions: Supplies cutaneous branches to the anteromedial thigh (anterior cutaneous branches of the femoral nerve) and the medial side of the leg and foot (saphenous nerve).

815
Q

20.2, 22.2 An open Ivor-Lewis oesophagectomy is performed via a

a Laparotomy then left thoracotomy
b Laparotomy, left neck incision
c Laparotomy, Right thoracotomy
d Left thoracotomy, left neck incision
d Right thoracotomy, Laparotomy

A

c Laparotomy, Right thoracotomy

Ivor-Lewis transthoracic esophagectomy — The Ivor-Lewis transthoracic esophagectomy can be used to resect cancers in the lower third of the esophagus but is not the optimal approach for cancers located in the middle third because of the limited proximal margin that can be achieved. This procedure combines a laparotomy with a right thoracotomy and an intrathoracic esophagogastric anastomosis. This approach permits direct visualization of the thoracic esophagus and allows the surgeon to perform a full thoracic lymphadenectomy. We prefer a minimally invasive Ivor-Lewis approach to a thoracotomy.

Transhiatal esophagectomy — A transhiatal esophagectomy (THE) can be performed to resect cervical, thoracic, and esophagogastric junction (EGJ) esophageal cancers; it is performed through an upper midline laparotomy incision and a left neck incision, typically without a thoracotomy.

Modified Ivor-Lewis transthoracic esophagectomy (left thoracoabdominal esophagogastrectomy) — A modification of the Ivor-Lewis transthoracic esophagectomy includes a left thoracoabdominal incision with a gastric pull-up and an esophagogastric anastomosis in the left chest. This approach is most useful for tumors involving the gastroesophageal junction. Only one incision is required, but disadvantages include a high incidence of complications such as postoperative reflux and limitation of the proximal esophageal margin by the aortic arch.

Tri-incisional esophagectomy — The tri-incisional esophagectomy combines the transhiatal and transthoracic approaches into a transthoracic total esophagectomy with a thoracic lymphadenectomy and cervical esophagogastric anastomosis. The three-incisional technique allows the surgeon to perform a complete two-field (mediastinal and upper abdominal) lymphadenectomy under direct vision and a cervical esophagogastric anastomosis. We prefer a thorascopic approach to the chest rather than a thoracotomy to minimize the risk of respiratory complications.

Esophagectomy is a technically difficult operation, and the complication rate is high due to the anatomic challenges of the procedure.

The choice of surgical approach depends upon many factors, including:
●Tumor location, length, submucosal extension, and adherence to surrounding structures
●The type or extent of lymphadenectomy desired
●The conduit to be used to restore gastrointestinal continuity
●Postoperative bile reflux
●The preference of the surgeon

816
Q

21.1 A condition or therapy that is NOT a contraindication to hyperbaric oxygen therapy is

A. Bleomycin
B. Cisplatin
C. Preterm neonate
D. Cerebral Abscess

A

D. Cerebral Abscess

HBOT Indications:
- Indications are related to need for enhanced Pressure or oOxygenation and treatment of infection:

Pressure
- air or gas embolism
- Decompression sickness

Oxygenation:
- arterial insufficiencies
(central retinal artery occlusion, enhancement of healing in wound problems)
- Carbon monoxide poisoning
- Compromised grafts and flaps
- Acute traumatic ischaemia
- Delayed radiation injuries
- Sudden sensorineural hearing loss
- Severe Anaemia
- Thermal burns

Infection:
- Clostridium myonecrosis (gas gangrene)
- Intracranial abscess
- Necrotising soft tissue infections
- Refractory osteomyelitis

Absolute Contraindications to HBOT:
- untreated PTx
- Premature Infants
- Bleomycin
- Disulfiram (antabuse)
- Cisplatin

Relative contraindications:
- Pregnancy
- Asthma
- Thoracic Surgery
- Emphysema with CO2 retention
- upper respiratory tract infections
- History of middle ear surgery or disorder
- History of seizures
- Fevers
- Congenital spherocytosis
- Optic neuritis

817
Q

22.1 According to the 6th National Audit Project, the likelihood that a patient who reports an allergy to penicillin has a true allergy is approximately

a. 10%
b. 30%
c. 50%
d. 70%
e. 90%

A

10%

Nap6

818
Q

21.1 The most common cause of mortality in children with diabetic ketoacidosis is

a. Cerebral oedema
b. Septic shock
c. Central pontine myelinolysis

A

a. Cerebral oedema

Cerebral Oedema

Source: UpToDate

819
Q

20.1

a) Gas trapping
b) Patient triggering
c) COPD
d) Circuit leak

A

c) gas trapping

Specific features of increased airway resistance seen here are:
High peak airway pressure, but a normal plateau pressure
Slow return of the flow-time curve to baseline
The flow-time curve does not reach baseline (indicating that emptying is incomplete)

820
Q

20.1 What is the arrow pointing to?

b. Psoas
c. Iliacus
c. Genitofemoral
d. Lateral cutaneous nerve of thigh
e. Lumbosacral trunk

A

c. Iliacus

821
Q

22.1 According to the Third International Consensus Definitions for Sepsis and Septic Shock (Sepsis-3), sepsis is defined as

a. SIRS criteria
b. Life threaning organ dysfunction with vasopressor requirement to maintain MAP >65 and lactate >2
c. Life threatening organ dysfunction caused by a dysregulated host response to infection
d. sBP <100, RR>22, altered mentation

A

Life threatening organ dysfunction caused by a dysregulated host response to infection

822
Q

22.2 A woman is diagnosed with preeclampsia and fetal growth restriction at 30 weeks gestation. Her haemodynamics are most likely to show
a) Increased CO, Decreased SVR
b) Decreased CO, Decreased SVR
c) Decreased CO, Increased SVR
d) No change CO, Increased SVR

A

c) Decreased CO, Increased SVR

It is plausible that a case of pre-eclampsia that occurs earlier in gestation and is associated with fetal growth restriction is related to low cardiac output and high peripheral vascular resistance with a much similar profile as observed in women with fetal growth restriction without HDP. In cases of later and term gestation pre-eclampsia, babies tend to be larger and there is a predominantly high cardiac output, low peripheral vascular resistance and raised intravascular volume state. Certainly, the clinical phenotype of a very ‘dry’, intravascularly depleted woman at 26 weeks with a growth restricted baby and conversely of a well-perfused oedematous woman with a bounding pulse and large baby at 38 weeks rings true: both have hypertension, but the mechanisms may be diametrically opposite.

https://www.ncbi.nlm.nih.gov/pmc/articles/PMC5569150/

823
Q

21.2 The CRASH-2 trial showed tranexamic acid administration to trauma victims results in a
reduction in

a. Decreased mortality
b. Increased mortality
c. Decreased blood product use
d. No change mortality
e. Increased bleeding

A

Death in bleeding trauma patients

Early administration of TXA safely reduced the risk of death in bleeding trauma patients and is highly cost-effective. Treatment beyond 3 hours of injury is unlikely to be effective.

  • Reduced death due to bleeding x 0.85
  • Equivocal blood transfusion
  • Equivocal thromboembolism
824
Q

22.1 You place a paravertebral catheter for postoperative analgesia at the level of T5 in an adult patient prior to a thoracotomy. Two minutes following the injection of 0.75% ropivacaine 10 mL, the patient becomes bradycardic, hypotensive and apnoeic. The most likely cause of the complication is

a) Subarachnoid injection
b) IV injection
c) LA toxicity

A

B. Intrathecal spread

c) = d) ?! possible, but respiratory function not effected until very late

ATOTW: COMPLETE SPINAL BLOCK FOLLOWING SPINAL ANAESTHESIA (2010)

CARDIO- RESPIRATORY
Hypotension*
Bradycardia*
Respiratory compromise*
Apnoea*
Reduced oxygen saturation
Difficulty speaking/coughing
Cardiac arrest (asystole)

NEUROLOGICAL
Nausea and anxiety*
Arm/hand dysaesthesia or paralysis*
High sensory level BLOCK
Cranial nerve involvement
Loss of consciousness*

CEACCP Paraveterbral Block (2009)
The overall incidence of reported complications with PVBs is between 2.6% and 5%; however, the risk of long-term morbidity is exceedingly low. No fatality directly attributable to PVBs has been reported. The failure rate in experienced hands varies between 6.8% and 10%, which is broadly comparable with epidural analgesia. Other specifically reported complications include: hypotension 4.6%, vascular puncture 3.8%, pleural puncture 1.1%, and pneumothorax 0.5%. Inadvertent pleural puncture may not be recognized, as a short but effective interpleural block will result. The actual frequency of this complication may therefore exceed 1.1%, particularly with the cranial approach. If pleural puncture is appreciated, an interpleural block can be performed intentionally and a catheter inserted to prolong analgesia. Pneumothorax only rarely follows pleural puncture but when it occurs, it is usually small and can therefore be managed conservatively. Tension pneumothorax is a potential complication in ventilated patients, but no cases have as yet been reported. Bilateral block has been reported in up to 10% of cases, which is usually due to epidural spread and less commonly to mass movement of the drug across the midline in the prevertebral plane. Epidural spread is more common with a more medial injection site and with catheter techniques, although block distribution tends to be less on the contralateral side. Ipsilateral Horner’s syndrome is a common side-effect with blocks extending to T1 and T2. Total spinal anaesthesia is very rare and has only been reported twice in the world literature. However, if the plane of approach of the needle is close to the midline, the dural cuff surrounding the intercostal nerve can be penetrated.

825
Q

22.1 The most common cause of bilateral blindness following spinal surgery and anaesthesia is

a. Ischaemic optic neuropathy
b. Retinal artery occlusion
c. Retinal detachment
d. Cortical stroke

A

ION
Post - spinal
Ant - cardiac

repeat

826
Q

22.1 The radial artery pressure trace shown below is from a patient who has an intra-aortic balloon pump in situ. The device has been switched to 1:2 augmentation to assess the timing. The trace shows an augmented beat followed by a nonaugmented beat. With respect to the augmentation, the trace shows

a. Early deflation
b. Late deflation
c. Late inflation
d. Early inflation
e. No change

A

c. Late inflation

827
Q

20.2 You are part of an international humanitarian aid mission. You have packed sevoflurane but the only local vaporiser is isoflurane specific with a maximum output of 5%. If you added sevoflurane to the isoflurane vaporiser the maximum sevoflurane output percentage would be approximately (Sevoflurane saturated vapour pressure 160mmHg, isoflurane 240mmHg)

a. 2
b. 3
c. 5
d. 7
e. 9

A

Answer: 3%.

(5%/240) x 160

Principle:
If Vaporizer specific for agent with low SVP (Enflurane or Sevoflurane) is misplaced with an agent that has high SVP (halothane or isoflurane) then actual output concentration will be greater than the concentration indicated by dial. (inverse is also true)

Administration of sevoflurane using other agent-specific vaporizers:

The current study investigated the concentration of sevoflurane that could be achieved when sevoflurane was administered using standard agent-specific halothane, isoflurane, and enflurane vaporizers. An artificial lung analog model was made by attaching the 3-L reservoir bag to the 15-mm end of the anesthesia circle system. The lung analog was attached and ventilated with oxygen and air at flow rates of 2 L/min each (total gas flow = 4 L/min), a tidal volume of 800 mL, a rate of 10 breaths/min, and an inspiratory-to-expiratory ratio of 1:2. The vaporizer was filled with sevoflurane and the dial turned to 1%. After a 10-minute equilibration period, the concentration of sevoflurane was measured. The vaporizer concentration was increased in 1% increments, and after a 10-minute equilibration, the sevoflurane concentration was recorded. The dial was increased from 1% to 5% for the halothane and isoflurane vaporizer and from 1% to 7% for the enflurane vaporizer. Each study was repeated five times at each incremental increase of 1% for each of the three vaporizers. The series of studies were repeated using a total gas flow of 8 L/min (oxygen 4 and air 4) instead of 4 L/min (oxygen 2 and air 2). Using the halothane or isoflurane vaporizers at the 5% setting, the maximum sevoflurane concentrations achieved were 3.0% and 3.1%, respectively. The sevoflurane concentration was a maximum of 6% using the enflurane vaporizer set at 7%. The sevoflurane concentration decreased significantly when using any of the three vaporizers at all concentrations when the gas flow was increased from 4 to 8 L/min. The current study demonstrates that clinically useful concentrations of sevoflurane can be achieved with the administration of sevoflurane through an enflurane vaporizer. Although this is not routinely recommended, in specific circumstances it may allow the use of sevoflurane in third-world countries if sevoflurane vaporizers are not available and the use of sevoflurane is clinically necessary.

828
Q

bonus neuro radiology questions

A 33 year-old man developed the explosive onset of a headache followed by nausea and vomiting.
He undergoes cerebral angiography.
His cerebral aneurysm is in the

a. Anterior Cerebral Artery
b. Anterior communicating artery
c. MCA
d. PCA
e. Basilar artery

A

a. Anterior Cerebral Artery

https://case.edu/med/neurology/NR/ACAaneur.html

829
Q

23.1 A patient’s glomerular filtration rate is estimated at 35 mL/min/1.73m2. The patient’s chronic kidney disease can be classified as Stage

a. 5
b. 4
c. 3a
d. 3b
e. 2

A

Category GFR
ml/min/1.73 m2 Terms
G1 ≥90 Normal or high
G2 60-89 Mildly decreased*
G3a 45-59 Mildly to moderately decreased
G3b 30-44 Moderately to severely decreased
G4 15-29 Severely decreased
G5 <15 Kidney failure

Assign Albuminuria category as follows:
Albuminuria categories in CKD
Category ACR (mg/g) Terms
A1 <30 Normal to mildly increased
A2 30-300 Moderately increased*
A3 >300 Severely increased**
Abbreviations: ACR, albumin-to-creatinine ratio; CKD, chronic kidney disease.
*Relative to young adult level.
**Including nephrotic syndrome (albumin excretion ACR >2220 mg/g)

**Collectively referred to as “CGA Staging”

REPEAT

830
Q

21.1 The apical four–chamber view of a transthoracic echocardiogram below shows all EXCEPT
a) Left pulmonary vein
b) Pulmonary Trunk
c) Right Atrium
d) Aortic valve

A

b) pulmonary trunk

831
Q

22.1 Of the following, the drug with the LEAST effect on serum potassium is

a. Calcium gluconate
b. NaHCO3
c. Resonium
d. Salbutamol
e. Frusemide

A

a. Calcium gluconate

832
Q

22.2 The image below shows a normal central venous pressure (CVP) trace on the left. The CVP trace shown on the right is most consistent with

(actual image on exam! found on deranged physiology)
a. Tricuspid regurg
b. Constrictive pericarditis
c. Restrictive cardiomyopathy
d. Cardiac tamponade
e. Complete heart block

A

d. Cardiac tamponade

Deranged physiology

In summary
The CVP is raised
All CVP waveform components are elevated
a and v waves are tall
x descent is steep
y descent is (usually) absent

833
Q

23.1 A 58-year-old man with alcohol-related cirrhosis is booked to undergo a transjugular intrahepatic portosystemic shunt (TIPS) procedure. The calculation of his MELD-Na score to estimate his mortality risk requires all of the following EXCEPT

A. Sodium
B. INR
C. Cr
D. Albumin
E. Bilirubin

A

D. Albumin

https://www.tamingthesru.com/blog/r1-diagnostics/labs-in-hepatic-failure

834
Q

21.1 A man who had successful treatment of a germ cell tumour 10 years ago presents for laparoscopic appendectomy. Your intraoperative management should consider

a) Lung protective ventilation
b) Oncoanaesthesia
c) Lowest FiO2 possible
d) MAP 60

A

c) Lowest FiO2 possible

oxygen administration/ low fio2
assumed bleomycin

835
Q

The most common cause of airway compromise after anterior cervical spine surgery is

A. Aspiration
B. RLN injury
C. Oedema
D. Phrenic nerve injury
E. Haematoma

A

C. Oedema

Blue book 2017

The aetiology of UAO differs from that of airway compromise seen after thyroid or carotid surgery.

Haematoma formation and cerebrospinal fluid leak are potential complications of CSS that usually present early in the postoperative period, whereas upper airway obstruction most commonly develops in the late postoperative period (days rather than hours).

UAO occurs because of prevertebral tissue swelling that evolves late in the postoperative course.

The danger is that the onset can be insidious in a ward environment, leading to late recognition and limited availability of practitioners with airway expertise12.

Development of prevertebral oedema has been implicated in several near misses and deaths, which became the sentinel events that stimulated creation of departmental protocols to safely manage these patients postoperatively

836
Q

A 43-year-old man is undergoing an elective endovascular coiling procedure for an 8 mm middle cerebral artery aneurysm. Midway through the procedure the interventionalist tells you they have ruptured the aneurysm. All of the following are appropriate initial interventions EXCEPT

A. Decrease BP
B. Give protamine
C. Urgent transfer to theatre
D. Continue coiling
E. Mild hyperventilation

A

Answer: c. Urgent transfer to theatre

BJA Anaesthesia for interventional neuroradiology
https://academic.oup.com/bjaed/article/8/3/86/293346

Clinical signs of a rise in ICP or a sudden rise in blood pressure with or without a fall in heart rate should alert the anaesthetist to this possibility. Extravasation of contrast may also be seen. The goals are to increase coagulability by reversing heparin, decrease bleeding by lowering blood pressure (to the level before the bleed), control ICP with hyperventilation, head elevation, steroids and osmotic agents, control seizures, and initiate cerebral protection. Once the bleeding is controlled, the pressure may be raised to check for leaks. Usually, the coiling continues; rarely, a ventriculostomy may be required. If the coiling is unsuccessful, a rescue craniotomy and clipping will be required. Management may also involve performance of CT scans and subsequent transfer to ICU.

837
Q

22.1 Bowel preparation prior to elective colorectal surgery is associated with

a. No change
b. Decreased risk of surgical site infection
c. Decreased risk of anastomotic breakdown
d. Something about mortality/morbidity

A

No change in outcomes

repeat

838
Q

23.1 Under the NEXUS criteria, requirements to clear the cervical spine of trauma patients without radiographic imaging include all of the following EXCEPT

A. No distracting injury
B. No limitation in neck movement
C. No midline tenderness
D. No focal neurological deficit
E. No altered level of consiousness

A

b) no limitation in neck movement

839
Q

20.2 This lung ultrasound shows

a) Normal lungs
b) Pulmonary odema
c) Pneumothorax
d) Pleural effusion
e) Pneumonia

A

e) Pneumonia

Signs of Pneumonia on lung ultrasound:
> Early Pneumonia
- B-lines and areas of sub pleural consolidation
- Fluid filled alveoli surrounded by air-filled lungs cause a short path reverberation and B-lines can be seen
- localised patches of numerous B-lines indicate sub-pleural consolidation

> Hepatization: solid appearing consolidated lung
- inflammatoy and purulent fluid filled alveoli makes the lung appear solid, with homogenous relatively fine echotexture similar to liver
- Atelectasis can also cause solid non-aerated lung and it can be difficult to distinguish the two conditions

> Shred sign: irregular consolidation/ air interface
- consolidated areas adjacent to aerated areas where the consolidated areas will be linear and well defined

> Air bronchograms and dynamic air bronchograms
- Air within consolidated area may remain in small aerated patches of lung or more commonly air remains within small bronchi, small air bubbles all lined up within a bronchus are known as sonographic air bronchograms
- when air bubbles are seen to bubble in and out with each breath the term “dynamic air bronchogram” is used

> Colour doppler interrogation: flow remains
- pulmonary arterial and venous vasculature are well demonstrated in areas of consolidation

Associated pleural effusion or empyema
- small hyperechoic parapneumonic effusion are frequently demonstrated
- echogenic debris within the effusion suggest empyema

840
Q

20.1
a. Left anterior hemiblock
b. Left posterior hemiblock
c. RBBB
d. LBBB

A

First degree heart block + Left Anterior hemiblock

ECG criteria for LAFB/LAHB

Left axis deviation (usually -45 to -90 degrees)
qR complexes in leads I, aVL
rS complexes in leads II, III, aVF
Prolonged R wave peak time in aVL > 45ms
841
Q

23.1 ANZCA recommends that after confirmed COVID-19 infection, non-urgent elective major surgery should be delayed for a minimum of

A. 4 weeks
B. 5 weeks
C. 6 weeks
D. 7 weeks
E. 8 weeks

A

d. 7 weeks UPDATE: 2-3 weeks

PG68(A)

https://www.anzca.edu.au/getattachment/af1fb728-5e87-413a-b006-c54cecf282b1/PG68(A)-Guideline-surgical-patient-safety-SARS-CoV-2

For most patients, it is safe to proceed with surgery TWO TO THREE WEEKS post SARS-CoV-2 infection provided no ongoing symptoms are present. For high-risk patients, it is recommended to perform an individualised risk assessment and utilise Shared Decision Making to determine optimal timing of surgery post SARS-CoV-2 infection.

Patients who are asymptomatic, have returned back to baseline, are vaccinated, aged <70 years and without comorbidity can proceed with non-urgent elective minor surgery (day case) and endoscopy procedures without delay beyond the infectious period (timeframe as per local guideline and expertise

ALL patient with ongoing symptoms, especially those who have not returned to baseline function and those patients with a history of moderate or more severe25 SARS-CoV-2 infection: recommended delay for non-urgent elective surgery is still 7 weeks

842
Q

21.2 International guidelines state that patients presenting for major surgery have inadequate or low iron stores if their serum ferritin level is less than

a) 20
b) 30
c) 50
d) 100

A

ANSWER: d. Ferritin <100mcg/L

Serum ferritin level < 30 μg.l−1 is the most sensitive and specific test used for the identification of absolute iron deficiency. However, in the presence of inflammation (C-reactive protein > 5 mg.l−1) and/or transferrin saturation < 20%, a serum ferritin level < 100 μg.l−1 is indicative of iron deficiency.

International consensus statement on peri-operative management of anaemia and iron deficiency

https://associationofanaesthetists-publications.onlinelibrary.wiley.com/doi/10.1111/anae.13773#:~:text=Recommendations%20for%20best%20clinical%20practice,-Physicians%20should%20consider&text=Serum%20ferritin%20level%20%3C%2030%20%CE%BCg,serum%20ferritin%20level%20%3C%20100%20%CE%BCg.

843
Q

22.1 A 35-year-old patient is undergoing a diagnostic laparoscopy. Three minutes after insufflation of CO2 her oxygen saturation falls to 85%. You note decreased air entry on the left side of her chest.
Lung ultrasound on the left reveals lung pulse and no lung sliding. The best first action is to

a. Tell surgeon to deflate
b. Needle decompression
c. Chest drain
d. Pigtail drain

A

Tell surgeon to deflate / remove pneumoperitoneum

?endobronchial intubation caused by pneumoperitoneum-> loss of lung sliding but continual lung pulse?

Lung pulse:
Cardiac contraction is sufficiently forceful as to move the lung in synchrony with the cardiac cycle. This movement is readily detected as cardiophasic movement of the pleural line. It has the same implication as detection of respirophasic movement of the pleural line which termed lung sliding (ie, there is no pneumothorax at the site of probe application on the chest wall).

844
Q

20.1 70 year old patient for revision THR, in clinic 10 days prior

Hb 110
Ferritin 51
CRP 10
What should you do?

a Transfuse 2u pRBC
b Give oral iron therapy and continue with surgery
c Give oral iron therapy and defer surgery for 6 weeks
d Give IV iron
e Do nothing

A

c Oral iron and defer

or

d give IV iron

  • most assume its IV iron and proceed but
  • Assuming IDA and raised CRP then iron therapy but
    ‘deferable’ surgery? then oral and come again in 6 weeks
    if not deferrable then IV iron - surely a revision THR is deferable??
  • If give IV iron and defer was an option I would choose that one, it would allow assessment of inflammatory process and to confirm Hb and ferritin are at an acceptable level

International consensus statement on the peri-operative management of anaemia and iron deficiency

https://associationofanaesthetists-publications.onlinelibrary.wiley.com/doi/10.1111/anae.13773

However, many patients will not respond to oral iron, especially those with functional iron deficiency and chronic illness or infection and those with ongoing blood loss 15, 25. Others will not tolerate oral iron due to gastro-intestinal side-effects. Once oral iron has been commenced, the Hb should be measured again, at least 4 weeks before surgery. In the absence of an increased Hb or if the patient is intolerant, i.v. iron is the preferred replacement route. If surgery is planned in less than 6 weeks time, i.v. iron may also be the most effective option.

845
Q

20.2 The approximate maximum flow rate expected with fluid administered (under a pressure bag inflated at 300 mmHg) via an intraosseous needle inserted into the humerus is

a 60 ml/min
b 90mL/min
c 120 ml/min
d 600 ml/min
e 1200 ml/min

A

C: 120ml/min

An observational, prospective study comparing tibial and humeral intraosseous access using the EZ-IO.

  • humeral flow rates were significantly faster using a pressure bag (153 mL/min) compared with humeral those achieved without pressure bag (84 mL/min)
  • tibial flow rates to be significantly faster using a pressure bag (165 mL/min) compared with those achieved without a pressure bag (73 mL/min)
846
Q

22.2 During an infraclavicular approach to the brachial plexus, the tip of the needle is positioned closest to the
a. roots
b. trunks
c. divisions
d. cords
e. branches

A

d. cords

847
Q

23.1 A 35-year-old woman is brought to the emergency department following a suspected amitriptyline overdose. She has a Glasgow Coma Scale score of 6 and her blood pressure is 90/46 mmHg. Her electrocardiogram is most likely to show

A. AF
B. CHB
C. Sinus tachy with prolonged QRS
D. Sinus brady with prolonged QRS
E. VT

A

c. sinus tachy with prolonged QRS

848
Q

The water capacity of an oxygen transport cylinder is 2 litres. The gauge is reading 150 bar. At an oxygen flow rate of 10 litres per minute, the number of minutes the cylinder will last is

A. 15 min
B. 30 min
C. 45 min
D. 60 min
E. 2 hours

A

B. 30 min

P1x V1= P2xV2

150bar x 2l = 1bar x Unknown Volume
150 x 2/1= Unknown Volume
300L = unknown volume
300/10l/min = 30mins

849
Q

22.2 A raised (> 140% predicted) single-breath diffusing capacity of the lung for carbon monoxide (DLCO) can be caused by

a. Emphysema
b. COPD
c. interstitial lung disease
d. Asthma
e. Sarcoidosis

A

d. Asthma

What are the causes of an elevated DL CO ?

The causes of an elevated DLCO are numerous, but is most commonly caused by asthma and obesity (increased pulmonary blood flow). Pulmonary hemorrhage is an additional important cause.

https://www.atsjournals.org/doi/pdf/10.1513/AnnalsATS.201605-355CC

850
Q

21.1 In the treatment of persistent mucosal bleeding in patients with von Willebrand disease Type 3, Desmopressin (DDAVP) is

a) contraindicated due to risk of thrombocytopenia
b) indicated if previous response documented
c) indicated to improve plt function
d) contraindicated as it won’t work

A

d) contraindicated as it won’t work

Type 1:
-Quantitative defect of VWF

Type 2:
-Qualitative Defect of VWF
-Type 2 subclassification depending on plt binding function, F8 binding capcacity, number of high molecular weight VWF multimers

Type 3:
- complete absence of VWF

Treatment:
- do not need blood components to control haemorrhage
-F8 plasma concentration >100 for major surgery and >50 for minor surgery
-DDAVP approved for use in Type 1, no use in type 3, discuss its use with haematology in type 2 due to its variable effect
-DDAVP given atleast 90mins before operation
-TXA may be useful
-VWF/F8 concentrates indicated in severe cases, type 3 and qualitiative defects in VWF
-Plt infusions should be considered in persistent bleeding
-Cryo has an unpredictable effect, only used if other treatments have failed

851
Q

20.1 Which tooth is most commonly damaged in anaesthesia practise

A. Right middle maxillary incisor
B. Left central maxillary incisor
C. Left middle mandibular incisor
D. Right middle mandibular incisor
E. Right 2nd mandibular molar

A

B. Left central maxillary incisor

BJA Education Dental Knowledge for Anaesthetists 2016 Abeysundera

“Direct laryngoscopy is implicated in 50-75% of all cases of dental injury. Maxillary incisors are the most commonly injured under GA. Representing 50% of cases, they are particularly prone to fracture, being small-rooted, of narrow cross-sectional area with a slight anterior axis. The left central maxillary incisor is most vulnerable to damage from the flange of the laryngoscope blade if used as a fulcrum, usually when attempting to improve the view during a difficult intubation.”

852
Q

21.2 A 50 year old man has the following pulmonary function test result:

FEV1 98% predicted
FVC 95% predicted
DLCO 43% predicted

The diagnosis is most consistent with:

a) Pulmonary fibrosis
b) Pulmonary hypertension
c) COPD
d) Obesity

A

b) Pulmonary hypertension

Up to date: Overview of pulmonary function testing in adults

Diffusing capacity — Measurement of the single-breath diffusing capacity for carbon monoxide (DLCO; also known as transfer factor or TLCO) is quick, safe, and useful in the evaluation of restrictive and obstructive lung disease, as well as pulmonary vascular disease. The technique and interpretation are discussed separately.

In the setting of restrictive disease, the diffusing capacity helps distinguish between intrinsic lung disease, in which DLCO is usually reduced, and other causes of restriction, in which DLCO is usually normal.

In the setting of obstructive disease, the DLCO helps distinguish between emphysema, in which it is usually reduced, and other causes of chronic airway obstruction, like asthma or chronic bronchitis, where it is usually normal.

The DLCO is also used in the assessment of pulmonary vascular disease (eg, thromboembolic disease, pulmonary hypertension), which typically causes a reduction in DLCO in the absence of significant restriction or obstruction

853
Q

21.1, 21.2, 20.1 Hepcidin production is inhibited in response to

a. Anaemia
b. Inflammation
c. Acute leukemia
d. Infection
e. Excess iron stores

A

anaemia

Iron deficiency can be caused by depletion of total Iron stores or a chronic loss of blood.

Metabolism of Iron is also influenced by disease states including inflammation and malignancy.

Raised Iron stores and inflammation upregulate the production of HEPCIDIN, a hormone responsible for the inhibition of enteral Iron absorption

HEPCIDIN degrades iton trans-membrane transporter ferroportin on duodenal enterocyte membranes. it also inhibits the transport of stored iron from hepatocytes and macrophages into plasma in a similar manner.

Upregulation of HEPCIDIN can produce functional iron deficiency, lading to what has been tradionally known as the anaemia of chronic disease

HEPCIDIN deficiency is the cause of iron overload in hereditary hemochromatosis, iron-loading anemias, and hepatitis C

HEPCIDIN is suppressed in iron deficiency, allowing increased absorption of dietary iron and replenishment of iron stores. Increased erythropoietic activity also suppresses HEPCIDIN production.

HEPCIDIN is decreased in iron deficiency anemia, hemolytic anemia, and anemias with ineffective erythropoiesis

854
Q

21.1 The optimal position, under ultrasound guidance, to place a catheter tip to provide continuous erector spinae plane block for post-thoracotomy analgesia is

a. Between the fascial plane of erector spinae and rhomboids
b. Posterior to both erector spinae and spinous process
c. Anterior to erector spinae and posterior to transverse process 5th rib
d. Superficial to the infraspinatus fossa
e .Superficial to the lamina

A

c. Anterior to erector spinae and posterior to transverse process 5th rib

Midpoint between T5-6
(Usual Incision T4-5, ICC T6)

Source - Blue book 2019

855
Q

bonus neuro radiology questions

A 45 year-old man developed the abrupt onset of an explosive headache followed by nausea and vomiting. On exam, he was drowsy with subtle left sided weakness.
He undergoes cerebral angiography.
His cerebral aneurysm is in the

a. Anterior Cerebral Artery
b. Anterior communicating artery
c. MCA
d. PCA
e. Basilar artery

A

c. MCA

https://case.edu/med/neurology/NR/mcaanery.htm

856
Q

20.1 You are urgently called to assist a colleague in a neighbouring theatre who has been having difficulty with intubation of a large adult male. They have managed to pass a double lumen tube airway exchange catheter. If the tip of the catheter is at the level of the carina, the approximate length outside of the mouth will be

a.31 cm
b.40 cm
c.45 cm
d.58 cm
e.75 cm

A

Answer : e. 75cm

DLT exchange catheter is 100cm long (AEC, extra firm with soft tip)
Mouth to carina ~28cm
Outside of mouth ~72cm

Aintree Catheter 56cm
outside of mouth 31cm

857
Q

22.2 A 21-year-old patient with a history of schizophrenia on quetiapine develops tremor, restlessness, hyperreflexia, nausea and vomiting in the post-anaesthesia care unit following an emergency laparoscopic cholecystectomy. Her heart rate is 80 / minute, blood pressure 130/90 mmHg, and her temperature is 37.0°C. The most likely diagnosis is

a. MH
b. NMS
c. serotonin syndrome
d. rhabdomyolysis
e. anticholinergic crisis

A

Serotonin Syndrome
Hyper reflexia
Usually has hypertension and hyperthermia

https://static1.squarespace.com/static/5e6d5df1ff954d5b7b139463/t/617242e2ab18df2dee31f417/1634878179720/ICU_one_pager_hyperthermic_toxidromes.png

858
Q

21.2 A 76 year old woman who is spontaneously breathing through a tracheostomy tube with an
inner cannula becomes acutely breathless. Despite application of high flow oxygen, her
respiratory rate is 40 breaths per minute and her SpO2 is 82%. The next most appropriate step in her airway management is to

a) Hand ventilate
b) Suction down the tracheostomy
c) Take down the cuff
d) Remove the inner cannula
e) Remove the tracheostomy

A

remove the inner cannula

The key principles of the algorithm are:
1.Waveform capnography has a prominent role at an early stage in emergency management.
2.Oxygenation of the patient is prioritised.
3.Trials of ventilation via a potentially displaced tracheostomy tube to assess patency are avoided.
4.Suction is only attempted after removing a potentially blocked inner tube.
5.Oxygen is applied to both potential airways.
6.Simple methods to oxygenate and ventilate via the stoma are described.
7.A blocked or displaced tracheostomy tube is removed as soon as this is established and not as a ‘last resort’

BJA: Update on management of tracheostomy
https://www.bjaed.org/article/S2058-5349(19)30125-8/fulltext

859
Q

20.1 A condition that is NOT associated with a raised baseline serum mast cell tryptase level is

a) Chronic renal failure
b) Alcoholic liver disease
c) Chronic eosinophilic leukaemia
d) Mastocystosis
e) Acute myeloid leukaemia

A

b) Alcoholic liver disease

A raised serum tryptase (hypertryptasemia) has numerous diagnostic uses. The main use is in the diagnosis of anaphylactic reactions; however, it does not distinguish between IgE-mediated versus non-IgE-mediated reactions.

Tryptase is a serine protease primarily produced by MCs, and to a lesser extent by mature basophils and myeloid progenitors. (Eosinophils have myeloid progenitor)

Elevated serum tryptase is an independent prognostic factor for progression to ESRF in patients treated with ACEi or ARB

https://associationofanaesthetists-publications.onlinelibrary.wiley.com/doi/10.1111/j.1365-2044.2004.03757.x

Systemic Anaphylaxis:
levels peak at 15-120mins and with half life of 1.5-2.5hrs

histamine levels peak at 5 mins and decrease to baseline within 15-30mins

B-Tryptase testing can be performed on blood samples obtained 1-6hrs after onset

Hihg tryptase concentration can be found 3 days after death from suspected anaphylaxis

Other allergic conditions:
Asthma
In asthma, β-tryptase is usually overexpressed or released from mast cells prematurely. It causes a cascade of events such as airway inflammation and bronchoconstriction. A mutation of the β-tryptase gene causes the overexpression

Sudden Infant Death Syndrome:
Anaphylaxis has been suggested to be a cause of the sudden infant death syndrome (SIDS), although allergen sensitivity and mast cell activation have not been demonstrated. Elevated postmortem levels of β-tryptase in victims of SIDS have been reported.

Amniotic Fluid Embolism:
A case study of an autopsy-proven fatal case of amniotic fluid embolism reported a significantly elevated serum β-tryptase level, suggesting possible mast cell activation. However, there are other reported cases of amniotic fluid embolism diagnosed clinically whose tryptase levels were not elevated.

Systemic Mastocytosis:
Characterised by mast cell hyperplasia in bone marrow, skin, liver, spleen and gastrointestinal mucosa.
A study of tryptase levels in patients with biopsy-proven mastocytosis reported concentrations of total tryptase > 20 ng.ml−1 and ratios of total tryptase to β-tryptase > 20, whereas normal patients had total tryptase levels < 14 ng.ml−1

MC activation Syndrome
These heterogeneous conditions are often associated with biochemical evidence of MC activation, but total tryptase may be normal.

Urticaria/Agioedema:
Basal total serum tryptase tends to be higher in patients with chronic urticaria over atopic and non-atopic healthy controls, likely due to increased MC burden and/or greater release of tryptase per MC. Other studies have found tryptase ≥15 μg/L in 12% of patients with urticaria/angioedema without anaphylaxis

Familial Hypertryptasaemia
These patients have a baseline tryp­tase of >8 μg/L and commonly have complaints of arthralgias, gastroesophageal reflux disease, flushing, urticaria, hypermobility, postural orthostatic tachycardia syndrome, increased risk of anaphylaxis to stinging insects, and irritable bowel syndrome

CKD and ESRF
Tryptase may be raised in patients with chronic kidney disease (CKD), accounting for around 7% of all elevated tryptase samples in one laboratory’s retrospective review of elevated tryptases. It tends to be elevated with more severe CKD, correlating with other markers of CKD including creatinine and proteinuria

Haematological conditions:
Hypereosinophilic syndrome (HES) is a heterogeneous group of systemic diseases of unknown cause characterised by excessive eosinophils invading the heart, lungs, brain and nerves, causing organ damage
Patients with HES and elevated serum tryptase levels were more likely to develop fibroproliferative end organ damage and had a shorter life expectancy.

Myelodysplastic syndrome (MDS) is a large group of acquired neoplastic disorders of the bone marrow most common in the elderly and is caused by an abnormal differentiation and maturation of haemopoietic cells.
Elevated levels of total tryptase (α-tryptase and β-tryptase) are found in a group of patients with MDS

Parasitic infection
MCs and tryptase play a significant role in parasitic infections [72], so it is of no surprise that serum tryptase can be elevated in these infections.

Gausher’s disease
Rare genetic disorder caused by the deficiency of β-glucocerebrosidase, an enzyme for the catabolism of glucocerebroside, a component of cell membranes.
A single case report exists of an 18-month-old boy who had an immediate hypersensitivity to imiglucerase enzyme replacement and an elevated baseline tryptase of 63.2 μg/L.

860
Q

21.1 A woman with preeclampsia presents with a blood pressure of 150/100 mmHg. An appropriate first line treatment to reduce the blood pressure is

a. Labetalol
b. Nifedipine
c. Magnesium
d. Levodopa

A

labetalol

https://www.bjaed.org/article/S2058-5349(20)30114-1/fulltext
The threshold for initiating antihypertensive treatment for all hypertensive disorders in pregnancy has been lowered. A sustained BP ≥140/90 mmHg warrants treatment, targeting a BP ≤135/85 mmHg.

The main aim of controlling the maternal BP is the prevention of intracerebral haemorrhage and stroke. The rate of stroke during the peripartum period in women with pre-eclampsia is 133 per 100,000, with haemorrhagic stroke being more common than ischaemic stroke.
NICE recommends offering oral labetalol as initial therapy, followed by nifedipine and then methyldopa as alternatives.

Second- and third-line agents include hydralazine and prazosin.

Women with severe hypertension (SBP ≥160 mmHg, DBP ≥110 mmHg, or both) should be admitted to hospital for assessment and treatment in a monitored setting.

Hypertensive emergencies can be treated with intravenous labetalol, hydralazine and immediate-release oral nifedipine without the need for invasive cardiac monitoring.

Labetalol 20 mg i.v. can be given over 2 min, and increased incrementally up to 80 mg i.v. If the BP remains high, another antihypertensive agent such as hydralazine can be added.

An initial dose of hydralazine 5–10 mg i.v. over 2 min can be followed by a further 10 mg i.v. after 20 min if the BP remains high.

A suggested initial dose of immediate-release oral nifedipine is 10 mg, followed by a further 20 mg if the BP remains high after 20 min.

861
Q

20.1 What is the arrow pointing to?

a. Ilioinguinal
b. Iliohypogastric
c. Genitofemoral
d. Accesory Obturator
e. Obturator

A

d. Accessory Obturator

862
Q

23.1 Reviewing the below image (ultrasound image shown), in order to safely perform an erector spinae block the probe needs to be moved

(exact exam image)

A. Move inferiorly
B. Move laterally
C. Move superiorly
D. Move medially

A

A. Move medial

863
Q

22.1 Anaesthesia-induced rhabdomyolysis differs from malignant hyperthermia in that it is NOT

a. Reduced myoglobinuria
b. Less increase in ETCO2
c. Less muscle rigidity

A

a. Reduced myoglobinuria

weird double negative question, answers don’t quite fit grammatically, however
- There is NOT reduced myoglobinuria with AIR compared to MH (both have myoglobinuria)
- There IS less increase in ETCO2
- There IS less muscle rigidity

Anesthesia-induced rhabdomyolysis or malignant hyperthermia: is defining the crisis important?
College library drop “is defining the crisis important?” from search
https://onlinelibrary-wiley-com.ezproxy.anzca.edu.au/doi/full/10.1111/pan.13130?sid=worldcat.org

864
Q

22.2 After ceasing smoking, a patient’s immune function has effectively recovered to normal after

a) 1 day
b) 3 weeks
c) 6-8 weeks
d) 6 months
e) 6 years

A

d) 6 months

ANZCA PS 12 perioperative smoking
https://www.anzca.edu.au/getattachment/5deb6800-e8f9-453f-b9a6-a151a9323249/PG12(POM)-Guideline-on-smoking-as-related-to-the-perioperative-period-(PS12)

Effects of quitting
1 day
- Reduced HbCO3-> increased O2 content
- Reduced nicotine/ SNS stimulation

3 weeks
- Increased wound healing

6-8 weeks
- Reduced sputum volume
- Increased lung function

6 months
- Increased immune function

865
Q

21.1 Risk factors for chronic post surgical pain do NOT include

a. Smoking
b. Pre-existing pain
c. High level of anxiety
d. Young age
e. High level of education

A

high level of education

BJA: CPSP

Previous chronic pain is the most important clinical risk factor.
Other risk factors for CPSP include:
1. Young age
2. alcohol use
3. smoking
4. unemployed
5. disability
6. obesity
7. type of surgery.

Risk factors for CPSP

Pre-op
1. existence and intensity of pre-op pain is a risk factor for developing CPSP after:
- Hernia repair
- Thoracotomy
- amputation
- mastectomy
- for mastectomy and amputation continuous pre-operative pain for more than 1 month predicts CPSP

  1. Genetic susceptibility
  2. Psychosocial factors

Intraoperative factors:
1. longer operations
2. Laparoscopic surgical approaches result in less chronic pain
3. Repeat surgery (for hernias) has higher incidene of moderate to severe pain

Post-operative factors:
1. Radiotherapy increases risk factors
2. Severity of postoperative pain predicts development of CPSP
- repetitive nociceptive stimulation during perioperative period resul;ts in nervous system changes e.g central sensitization

866
Q

21.2 A pregnant woman requires a caesarean section delivery within 30 minutes for fetal distress.
Her body mass index (BMI) is 26 kg/m2. She has multiple sclerosis with lesions in her brain
and spinal cord and receives monthly injections of the disease-modifying drug ofatumumab.
The most appropriate plan for her delivery is

a) Spinal
b) CSE
c) Epidural
d) GA

A

a) Spinal
Makarla

Epidural and vaginal delivery
? GA

all are safe in MS
The MAN I think is to signify advanced MS

(Really there isn’t heaps of evidence)

Source World Fed Anaesthetists

https://resources.wfsahq.org/wp-content/uploads/359_english.pdf
(What a terrible question)

867
Q

20.1 When topicalising the airway prior to a nasal awake fibreoptic intubation, it is necessary to anaesthetise all of the following nerves EXCEPT the

a) Anterior Ethmoidal
b) Tonsillar
c) Palatine
d) Glossopharyngeal
e) Lingual

A

e) Lingual

Nose
The nose is entirely innervated by branches of the trigeminal nerve.
Septum and anterior parts of the nasal cavity are affected by the anterior ethmoidal nerve (a branch of the ophthalmic nerve).
The rest of the nasal cavity is innervated by the greater and lesser palatine nerves (branches of the maxillary nerve).

Pharynx
The pharynx is largely innervated by the glossopharyngeal nerve.
Innervation of the whole pharynx, posterior third of tongue, the fauces, tonsils, and epiglottis is from the glossopharyngeal nerve.

Oropharynx
The oropharynx is innervated by branches of the vagus, trigeminal, and glossopharyngeal nerves.
The posterior third of the tongue, vallecula, and anterior surface of the epiglottis are innervated by the tonsillar nerve (a branch of the glossopharyngeal nerve).
The posterior and lateral wall of the pharynx are innervated by the pharyngeal nerve (a branch of the vagus nerve).
The tonsillar nerve affects the tonsils.
The anterior twothirds of the tongue are innervated by the lingual nerve (branch of the mandibular division of the trigeminal nerve).

Larynx
The larynx is innervated by the vagus nerve
Above the vocal cords (base of tongue, posterior epiglottis, aryepiglottic folds, and arytenoids), the internal branch of the superior laryngeal nerve (a branch of the vagus nerve) supplies innervation.
For the vocal cords and below the vocal cords, the recurrent laryngeal nerve (a branch of the vagus nerve) is the supplier.

868
Q

20.1 What is the arrow pointing to?

a. Ilioinguinal
b. Iliohypogastric
c. Genitofemoral
d. Lateral cutaneous nerve of thigh
e. Obturator

A

a. Ilioinguinal

869
Q

22.2 This lung ultrasound image is consistent with

a. pulmonary oedema
b. pneumonia
c. pneumothorax
d. pleural effusion
e. Normal lung

A

c. pneumothorax

870
Q

23.1 Consideration for same-day discharge in an ex-premature infant after orchidopexy for undescended testis would be suitable at a minimum postmenstrual age of

A. 44 weeks
B. 46 weeks
C. 50 weeks
D. 54 weeks

A

d. 54

Ex-preterm infants at risk of post-operative apnoea should not be considered for same day discharge unless they are medically fit and have reached a postmenstrual age of 54 weeks.

Term infants should not be considered for same day discharge unless they are
medically fit and have reached a postmenstrual age of 46 weeks.d) 54 weeks

https://www.anzca.edu.au/getattachment/568bad2d-7517-4eea-9c5d-cb7aa1c60

871
Q

22.2 The knee is NOT innervated by the

a) Common peroneal
b) Saphenous
c) Obturator
d) Posterior cutaneous nerve of the thigh
e) Posterior tibial

A

POSTERIOR CUTANOUS NERVE OF THE THIGH

lat / int and medial cutaneous of the thigh
femoral nerve (posterior division)
saphenous
obturator (post branch)
tibial nerve - articulates to the knee
sciatic (common perineal nerve)
L3/4 = extensors of knee
L5/S1 = flexors of the knee

Anatomy for Anaesthetists

872
Q

20.1 Patients with obstructive sleep apnoea undergoing surgery, have been shown to have an increased incidence of

A) AF
B) Acute renal failure
C) AMI
D) Perioperative mortality
E) Unplanned admission after ambulatory surgery

A

C) AMI

The association between OSA and perioperative mortality is unclear [19], with various studies showing increased [12,23] or comparable [14] mortality. While decreased mortality [7,18,43] has been reported in retrospective studies, this may be due to unrecognized or undiagnosed OSA in the control or non-OSA groups, and/or the possibility that patients with diagnosed OSA received monitoring and treatment of their OSA.

SOBA 2015:
Severe OSA occurs in 10–20% of patients with BMI > 35 kg.m2 and is often undiagnosed. Overall, a diagnosis of OSA is associated with a greater than doubling of the incidence of postoperative desaturation, respiratory failure, postoperative cardiac events and ICU admission [16]. The presence of multiple and prolonged oxygen desaturations increases the sensitivity to opioid-induced respiratory depression [17]. However, if identified pre-operatively and treated appropriately with continuous positive airway pressure (CPAP), the risk of complications is much reduced [18].

Obesity has no influence on the rate of unanticipated admission, postoperative complications, readmission or other unplanned contact with health professionals after home discharge.

873
Q

20.1 In planning the induction of anaesthesia in a morbidly obese patient, the total body weight should be used to calculate the dose of

A Suxamethonium
B Propofol
C Thiopentone
D Rocuronium

A

a) Suxamethonium

874
Q

21.1 A patient with a history of hereditary angioedema requires an appendectomy for acute appendicitis.
The most effective therapy for the prevention of an acute attack in the perioperative period is

a) FFP
b) Icatibant
c) Hydrocortisone
d) Danazole
e) cetirizine

A

d) Danazol
https://www.allergy.org.au/hp/papers/hereditary-angioedema

Treatment options:
Plasma derived C1-esterase inhibitor = Berinert/Cinryze,
Androgens = Danazol
B2 Bradykinin REceptor antagonist = Icatibant
FFP.

Danazol (an androgen) is recommended as first line PROPHYLAXIS for planned procedures (need to give for 5-10 days prior and 2-5 days post)

For emergency or high risk procedures C1 esterase inhibitor concentrate (Berinert or Cinryze) is recommended
- give 1 hour before procedure
- more effective than danazol but more expensive

Berinert:
- 20units/kg IV over 10 min
- Symptoms usually stabilise in 30 mins
- 2nd dose uncommon, but may be given 30mins to 2hrs after 1st dose

Icatibant:
- 30mg slow subcut infusion in abdominal area

Due to the risk of precipitating laryngeal oedema, oropharyngeal procedures should usually involve general anaesthesia with endotracheal intubation

Short answer:
- if you have days before surgery increase danazole, if complex surgery increase danazole and give C1Inh
- If you have acute emergency surgery give C1Inh Concentrate (Berinert/Cinryze) before and after
- if you have an acute attack use C1Inh or Bradykinin antagonist (Icatibant)
- If C1 Inh and Bradykinin antagonoist are not available then use FFP but this may worsen the attack due to the presence of C4 in the FFP
- Has Cetirizine been misremembered instead of Cinryze as an option in this question? No it wasn’t
-> adrenaline, steroids, antihistamines have no role in treatment of HAE acute attack

875
Q

21.2 The advantage of the Mapleson E circuit in paediatric anaesthesia is due to its

a) Use with low FGF
b) Ability to assess compliance
c) Ability to assess tidal volume
d) Ability to rapidly change levels of CPAP
e) Low resistance

A

e) Low resistance

876
Q

Independent risk factors for bone cement implantation syndrome include all
of these EXCEPT:

A) diuretics
B) general anaesthesia
C) increasing age
D) male gender
E) severe cardiopulmonary disease

A

B) general anaesthesia

Grade 1: moderate hypoxia (Spo2<94%) or hypotension [fall in systolic blood pressure (SBP) >20%].

Grade 2: severe hypoxia (Spo2<88%) or hypotension (fall in SBP >40%) or unexpected loss of consciousness.

Grade 3: cardiovascular collapse requiring CPR.

Patient Risk factors:
1. old age
2. poor pre-existing physical reserve
3. impaired cardiopulmonary function
-> NYHA 3 or 4
4. pre-existing pulmonary htn
5. Male Sex
6. Diuretics
7. ASA grade 3 or 4
8. osteoporosis
9. bony metastases
10. concomitant hip fractures (particularly pathological and intertrochanteric)
(latter due to abnormal vascular channels through which marrow contents can enter the circulation)

Surgical Risk factors
1. patients with previously un-instrumented fenoral canal > revision surgery
2. Use of long-stem femoral component

Anaesthetic Risk reduction:
- discussion between surgeons and anaesthetists over uncemented vs. cemented based on patient Hx particularly if lon-stem prosthesis, femoral fracture or patients with cardiorespiratory disease
- no clear evidence regarding the impact of anaesthetic technique
- increase inspired O2 considered in all patients at time of cementation
- avoid intravascular volume depletion
- Higher level of haemodynamic monitoring in high risk patients

Factors NOT predictive of severe BCIS include:

Arteriosclerosis
Angina pectoris
Congestive heart failure
Beta-blockers
Angiotensin-converting enzyme inhibitors.
877
Q

22.2 The electrolyte abnormality most associated with an increased risk of laryngospasm is
a. Hypokalaemia
b. Hyponatraemia
c. Hypocalcaemia
d. Hypercalcaemia
e. Hypernatraemia

A

c. Hypocalcaemia

Laryngospasm is a rare, but serious and potentially lethal, complication of hypocalcemia in adults. In every adult presenting with acute dyspnea and stridor, the possibility of hypocalcemia should be considered. Hypocalcemia should be treated promptly.

878
Q

21.1 All of the following conditions are associated with acromegaly EXCEPT

a. Myocardial fibrosis
b. biventricular enlargement
c. Arrhythmia
d. Left ventricular enlargement
e. AAA

A

e. AAA

Osteoarthritis
nerve compression syndrome due to bony overgrowth, and carpal tunnel syndrome
Hypertension
Diabetes mellitus
Cardiomyopathy/HF
Colorectal cancer
Sleep Apnea
Thyroid nodules and thyroid cancer
Hypogonadism
Compression of the optic chiasm

Source: BJA

879
Q

21.2 Local anaesthetic blockade of the sciatic nerve results in loss of function of all of the following EXCEPT

a) Weak dorsiflexion
b) Dorsal foot sensation loss
c) Knee flexion weakness
d) Knee extension weakness

A

d) Knee extension weakness
- this is femoral innervation

BJA: Perioperative peripheral nerve injuries
https://academic.oup.com/bjaed/article/12/1/38/260058

Lower limb peripheral nerve injuries

Sciatic nerve injury (L4–S3)

Mechanism of injury
Stretch, compression, ischaemia, and direct damage are the primary mechanisms. The lithotomy, frog leg, and sitting positions have been implicated in perioperative injury to this nerve (hyperflexion of the hip, abduction, and extension of the leg causes stretching). Regional anaesthetic techniques and hip arthroplasty may also cause injury. A higher incidence is seen in men aged 45–55 yr and in patients suffering with diabetes mellitus. The common peroneal component is usually affected, as this is more superficial compared with the tibial component.

Clinical presentation
Injury manifests as paralysis of the hamstring muscles and all the muscles below the knee leading to weak knee flexion and foot drop. All sensation below the knee except the medial aspect of the leg and foot is impaired.

880
Q

21.2 A derived value from an arterial blood gas sample is

a) PaO2
b) PaCO2
c) pH
d) BE

A

HCO3- is derived from pCO2 and pH
Base excess is derived from pH
SaO2 is derived from oxyHb and Hb

Source LITFL

881
Q

22.2 A 60-year-old man remains unconscious after an isolated head injury. The systolic blood pressure (in mmHg) should be kept above

a) 90
b) 100
c) 110
d) 120
e) 140

A

b) 100

Brain trauma foundation
Level III recommendation.
To decrease mortality and improve outcomes:

Maintain SBP at >100mmHg for patients 50 - 69
Maintain SBP at >110 for patients 15 - 49
Maintain SBP at >110 for patients 70 or older

882
Q

20.2 An ASA 1 28 year old male attends for inguinal hernia repair under general anaesthesia. He is administered propofol 180mg morphine 8mg rocuronium 50mg cephazolin 2g Post induction he develops an erythematous rash on his chest and arms, swelling of his lips and face, and severe hypotension. Preliminary blood results show …

Elevated tryptases (100 -> 40)
normal Ig E level
elevated morphine RAST.

The most likely diagnosis is

a) Ig E mediated morphine allergy
b) IgE mediated rocuronium allergy
c) Morphine induced histamine release
d) IgE mediated cephazolin allergy
e) Mastocytosis

A

b) IgE mediated (i.e. anaphylaxis) rocuronium allergy

Morphine RAST is most sensitive (88%) and specific (100%) test for NMBD as cause of anaphylaxis (quaternary ammonium epitope)

883
Q

22.1 Of the following clinical conditions, difficult intubation is LEAST likely to be associated with

a. Apert syndrome
b. Hurler
c. Pierre Robin
d. Downs Syndrome
e. Treacher collins

A

d. Downs Syndrome

884
Q

A patient who underwent a thoracotomy 6 months ago reports ongoing pain caused by light brushing of clothes against the skin on the chest wall. This is known as

a) Hyperalgesia
b) Allodynia
c) Hyperaesthesia
d) dysasthesia

A

Mechanical allodynia

Allodynia IASP definition: pain due to a stimulus that does not normally provoke pain

“The term allodynia was originally introduced to separate from hyperalgesia and hyperesthesia, the conditions seen in patients with lesions of the nervous system where touch, light pressure, or moderate cold or warmth evoke pain when applied to apparently normal skin.”

References IASP https://www.iasp-pain.org/resources/terminology/?ItemNumber=1698
And APMSE 5th Ed pg64.

Dysaesthesia: spontaneous and unpleasant sensation

885
Q

According to Australian and New Zealand Committee on Resuscitation (ANZCOR) guidelines, during advanced life support for ventricular fibrillation, adrenaline 1mg should be administered

a) As soon as possible
b) Before shock
c) After 2nd shock
d) After 3rd shock

A

C.

Shockable:
Adrenaline 1mg after 2nd shock
Then every second cycle
Amioderone 300mg after 3 shocks

Non-shockable
Adrenaline 1mg immediately
(then every second cycle)

886
Q

The Sequential Organ Failure Assessment (SOFA) score is used in intensive care for the
assesment of sepsis. This score does NOT include the:

a) MAP
b) FiO2/PaO2
c) INR
d) GCS
e) Plts

A

Previous Q (23.1) with different options.
ANSWER C (INR)

887
Q

In an adult patient with reduced mouth opening, insertion of a classic design LMA may be easier than with other supraglottic airways because of its

a) Bite block
b) Gastric port
c) Low profile
d) Preformed curve

A

a) low profile

Resource:
ANZCA PG56(A)BP Difficult airway equipment BP 2021

First generation SADs (page 19)
“classic design LMAs (cLMAs) with their low profile and lack of preformed curve have several advantages.”

888
Q

You are asked to assess a patient in the intensive care unit who has a tracheostomy that may have become dislodged. To assess if the tracheostomy is patent you should NOT

a) Put in a bougie
b) Suction cath
c) Deflate cuff
d) Remove speaking valve
e) Remove inner cannula

A

A

High risk of creating a false passage

Blue book 2017 page 21

No reference

889
Q

Albumin is contraindicated in

a) Traumatic brain injury

A

No remembered options.
Answer could be:
Traumatic Brain injury
Direct allergy
Cardiac Failure

SAFE trial

890
Q

A bleeding patient has ROTEM results including (ROTEM results shown). The most
appropriate treatment is

a) Plts
b) FFP
c) Cryo
d) TXA

A

c) Cryo

Cryo or TXA,

TXA first line treatment however patient has low fibrinogen and requires fibrinogen replacement.

891
Q

The shoulder joint receives sensory innervation from all of the following nerves EXCEPT the

a) Axillary
b) Long thoracic
c) Lateral pectoral nerve
d) Supra scapular
e) Sub scapularis

A

b) Long thoracic

892
Q

The most likely diagnosis for the following electrocardiograph is

a) VF
b) AF w bundle branch block
c) SVT w BBB
d) VT
e) Sinus w BBB

A

d) VT

Negative concordance

https://litfl.com/ventricular-tachycardia-monomorphic-ecg-library/

893
Q

A pregnant woman requires a caesarean section delivery within 30 minutes for fetal distress.

Her body mass index (BMI) is 26 kg/m2. She has multiple sclerosis with lesions in her brain and spinal cord and receives monthly injections of the disease-modifying drug ofatumumab.

The most appropriate plan for her delivery is

a) Spinal
b) GA
c) CSE
d) Epi

A

a) Spinal
Makarla

Epidural and vaginal delivery
? GA

all are safe in MS
The MAN I think is to signify advanced MS

(Really there isn’t heaps of evidence)

Source World Fed Anaesthetists

https://resources.wfsahq.org/wp-content/uploads/359_english.pdf
(What a terrible question)

894
Q

The needle whose tip is pictured is a

a) Sprotte
b) Quinke
c) Touhy
d) Whitacre

A

c) Touhy

895
Q

Pulmonary hypertension is defined as a mean pulmonary arterial pressure greater than
a) 15mmHg
b) 20mmHg
c) 25mmHg
d) 30mmHg

A

b) 20mmHg

896
Q

An inappropriate irrigation solution when using monopolar diathermy during transurethral resection of prostate would be

a) 1.5% Glycine
b) 5% dextrose
c) 3% Mannitol
d) 0.9% Saline
e) Sorbitol

A

d) 0.9% Saline

Other fluids are all electrolyte free except 0.9% Saline

897
Q

The technique of airway pressure release ventilation

a. Has a prolonged expiratory time
b. Augments cardiac output in hypovolaemic patients
c. Results in reduced mean airway pressures

A

none of the remembered options

Airway pressure release ventilation (APRV) is an open-lung mode of invasive mechanical ventilation mode, in which spontaneous breathing is encouraged.
APRV uses longer inspiratory times; this results in increased mean airway pressures, which aim to improve oxygenation.
Brief releases at a lower pressure facilitate carbon dioxide clearance.
The terminology and methods of initiation, titration, and weaning are distinct from other modes of mechanical ventilation.
The use of APRV is increasing in the UK despite a current paucity of high-quality evidence

high intrathoracic pressure decreases the transmural left ventricular pressure, reducing the work of contraction and increasing cardiac output. In the context of hypoxaemia, a mode of mechanical ventilation that improves arterial oxygenation will improve myocardial oxygen delivery, myocardial function and cardiac output. As APRV is a spontaneous breathing mode, in addition to the benefits of spontaneous ventilation, reduced doses of sedative drugs can often be used, with subsequent reduction of requirement for vasoactive drugs and improvement in haemodynamic state.

Airway pressure release ventilation (APRV) is an open-lung mode of invasive mechanical ventilation mode, in which spontaneous breathing is encouraged. APRV uses longer inspiratory times; this results in increased mean airway pressures, which aim to improve oxygenation

https://www.bjaed.org/article/S2058-5349(19)30178-7/fulltext

https://derangedphysiology.com/main/required-reading/respiratory-medicine-and-ventilation/Chapter%20518/airway-pressure-release-ventilation-aprv-ards

898
Q

The modified Aldrete scoring system uses all of the following EXCEPT the

a) BP
b) Pain score
c) Resp rate
d) sedation level

A

b) Pain score

Aldrete score, which includes five elements (activity, respiration, circulation, consciousness, oxygen saturation) [16].

The original scoring system was developed before the invention of pulse oximetry and used the patient’s colouration as a surrogate marker of their oxygenation status. A modified Aldrete scoring system was described in 1995 which replaces the assessment of skin colouration with the use of pulse oximetry to measure SpO2.

The Modified Aldrete system includes five additional elements that are particularly useful during the Phase II recovery period prior to discharge to home (dressing, pain, ambulation, feeding, urine output)

899
Q

The ventilator waveforms shown represent (actual image from exam)

a) Triggered breaths
b) Bronchospasm
c) Obstructive pattern
d) Gas trapping

A

C) Obstructive Pattern

https://thoracickey.com/ventilator-graphics/
Image 9.6

900
Q

According to the ATACAS trial, the continuation of low-dose aspirin prior to cardiac surgery is associated, in the postoperative period, with

a) No increased risk of bleeding
b) Decreased risk of MI
c) Increased risk of Thrombotic events
d) Increased risk of seizures

A

a) No increased risk of bleeding

There is no evidence that pre-operative aspirin administration resulted in a lower risk of death or thrombotic complications, or a higher risk of haemorrhage.

The study aim (and title) was to compare stopping vs continuing aspirin, however the design insisted on all patients stopping aspirin and then being given a single dose of aspirin or placebo prior to surgery (and presumably all patients were given aspirin after surgery) – this method hasn’t really investigated the theory

TheBottomLine.org.uk

901
Q

A patient will open her eyes in response to voice, speak with inappropriate words and
withdraw to a painful stimulus. Her Glascow Coma Scale score is

A

E3 V3 M4 = GCS 10

902
Q

Intravenous dexmedetomidine use does NOT result in

a) hypotension
b) Unchanged PACU length of Stay
c) residual sedation
4) Reduced in pain

A

c) residual sedation

https://pubmed.ncbi.nlm.nih.gov/35085107/#:~:text=Conclusions%3A%20The%20use%20of%20dexmedetomidine,sedation%20or%20bradycardia%20in%20PACU

903
Q

The risk of developing postherpetic neuralgia may be reduced by treating acute herpes zoster (shingles) with

A. Ibuprofen
B. Gabapentin
C. Aciclovir
D. Amitriptyline
E. Oxycodone

A

D. Amitriptyline

Amitriptyline (used in low doses for 90 days from onset of the herpes zoster rash) reduces the incidence of postherpetic neuralgia

N.B
Antiviral agents started within 72 hours of onset of the herpes zoster rash accelerate the resolution of acute pain (U) (Level I) but do not reduce the incidence, severity and duration of postherpetic neuralgia

UTD
Both Gabapentinoids and TCAs are effective at TREATING postherpetic neuralgia. The former have lower risk of discontinuation due to adverse side effects.
For moderate or severe pain, use gabapentinoids.

904
Q

Rapid reversal of the anticoagulant effect of dabigatran can be achieved with

a) Andexenet Alfa
b) rotuzimab
c) Idarucizumab (Praxbind)
d) Infliximab

A

Idarucizumab (Praxbind) is a monoclonal antibody to dabigatran

Dabigatran bleeding may be treated with:
- idarucizumab
- haemodialysis
-PCC 25-50IU/kg
- TXA will decrease fibrinolysis and has some effect
- FFP also has some effect

Humanized monoclonal antibody fragment (Fab) indicated in patients treated with dabigatran (Pradaxa) when reversal of the anticoagulant effects are needed for emergency surgery or urgent procedures, or in the event of life-threatening or uncontrolled bleeding
- very high affinity for dabigatran (300x vs affinity for thrombin)
- 5 g IV, provided as 2 separate vials each containing 2.5 g/50 mL (see Administration)
- RE-VERSE-AD trial: undetectable levels <20ng/ml within minutes and for 24 hours
- Limited data support administration of an additional 5 g depending on clinical situation

Dosage Modifications

Renal impairment: Renal impairment did not impact the reversal effect of idarucizumab; no dosage adjustment required
Hepatic impairment:
Dosing Considerations

This indication is approved under accelerated approval based on a reduction in unbound dabigatran and normalization of coagulation parameters in healthy volunteers; continued approval for this indication may be contingent upon the results of an ongoing cohort case series study

905
Q

A patient under general anaesthesia monitored with transcranial cerebral oximetry has a decrease in their cerebral oxygen saturation. This is likely to be improved by an increase in all of the following EXCEPT

A. Increasing blood pressure
B. Deepening anaesthesia
C. Increased minute ventilation
D. Transfusion

A

C. Increased minute ventilation

Cerebral blood flow
Cardiac output
Acid–base status
Major haemorrhage
Arterial inflow/venous outflow obstruction

Oxygen content
Haemoglobin concentration
Haemoglobin saturation
Pulmonary function
Inspired oxygen concentration
Inspired oxygen concentration

906
Q

A patient who has had a previous axillary nodal dissection and who does not have
lymphoedema of the affected arm presents for surgery. On the affected arm

A

https://www.anzca.edu.au/resources/professional-documents/professional-document-appendix-topics/appendix-1-pg18(a).pdf

Appendix 1 - Intravenous access and blood pressure monitoring in patients with previous axillary nodal dissection

Recommendations
1. Shared decision-making:
* Anaesthetists should communicate the importance of safety and optimisation of anaesthesia administration in this setting to allow patients to make informed decisions.

  1. Institutional considerations.
    * To facilitate shared decision-making, as well as staff and patient education, institutions should remove universal recommendations, such as wrist-band or standardised alerts, that relate to patients who have a history of axillary dissection.
  2. For patients who have had axillary nodal dissection and do not have lymphoedema of the affected arm, there is no contraindication to:
    * Non-invasive blood pressure monitoring using the affected arm.
    * Peripheral intravenous cannulation in the affected arm.
    * Arterial line insertion and monitoring in the affected arm if clinically indicated.
  3. For patients WITH lymphoedema of the affected arm following axillary nodal dissection (Stages 0-III):
    * An alternative site should be contemplated where practicable, considering the relative risk associated with more invasive procedures. There is no absolute contraindication to using the affected limb for monitoring and intravenous access.
    * Ongoing monitoring of peripheral intravenous cannulae should be done in accordance with the
    ACSQHC Clinical Care Standard.
907
Q

Synchronised direct current cardioversion is NOT indicated when the arrhythmia is

a) AF
b) Flutter with rate <100
c) Multifocal atrial tachy
d) SVT with
e) Conscious torsades

A

C- Multifocal Atrial Tachycardia

Cardioversion is contraindicated in MAT. Due to the multiple atrial foci, direct current (DC) cardioversion is not effective in restoring normal sinus rhythm and can precipitate more dangerous arrhythmias.
- https://emedicine.medscape.com/article/155825-overview#a10

DCCV is indicated for
1. Any haemodynamically unstable narrow or wide QRS complex tachycardia
2. AF <48hrs
3. AF >48hrs with adequate anticoag/TOE to exclude thrombus
4. SVTs and monomorphic TVs not responding to trial of IV medical therapy

DCCV is CONTRAindicated in:
a. Digitalis toxicity and associated tachycardia
b. AF >48hrs without adequate anticoagulation/TOE
-BJAEducation 2017
https://academic.oup.com/bjaed/article/17/5/166/2669966

908
Q

Dulaglutide reduces blood glucose by

A - Binding Glucagon-like peptide 1 receptors and causing activation
B - Binding Glucagon-like peptide 1 receptors and competitively inhibiting GLP1 binding
C - Binding Glucagon-like peptide 1 receptors and causing conformational change leading to cell death
D - Binding L cells of the gastrointestinal mucosa leading to GLP-1 secretion
E - Binding L cells of the gastrointestinal mucosa leading to GLP-1 sequestration

A

A - GLP1 receptor agonist
(rest of options made up)

“Dulaglutide binds to glucagon-like peptide 1 receptors, slowing gastric emptying and increases insulin secretion by pancreatic Beta cells. Simultaneously the compound reduces the elevated glucagon secretion by inhibiting alpha cells of the pancreas, as glucagon is known to be inappropriately elevated in diabetic patients. GLP-1 is normally secreted by L cells of the gastrointestinal mucosa in response to a meal”
- Wikipedia, Dulaglutide
- Once weekly injection, “trulicity”

https://www.asahq.org/about-asa/newsroom/news-releases/2023/06/american-society-of-anesthesiologists-consensus-based-guidance-on-preoperative

909
Q

Ongoing cerebral seizure activity induced by electroconvulsive therapy should be medically terminated after

a) 30 seconds
b) 1 minute
c) 2 minutes
d) 3 minutes
e) 5 minutes

A

c) 2 minutes

910
Q

Characteristics of post-operative visual loss due to vertebrobasilar ischaemia include

a) inattention
b) Vision returns in 24hrs
c) relevant afferent pupillary defect
d) diplopia

A

d) diplopia

Bilateral visual loss associated with insufficiency to posterior circulation so: parieto-occipital ischaemia, signs of stroke, visual agnosia, ophthalmoplegia or diplopia.

911
Q

The odds ratio is the measure of choice for a

a. Case control
b. Cohort
c. RCT
d. Epidemiological study

A

a) case control

https://www.cdc.gov/csels/dsepd/ss1978/lesson3/section5.html

912
Q

In a cardiac transplant recipient, hypotension due to general anaesthesia is least likely to respond to

a) noradrenaline
b) Ephedrine
c) adrenaline
d) Atropine

A

d) Atropine

Blue book 2019

913
Q

When the infraclavicular approach is used, the brachial plexus is blocked at the level of the

a. roots
b. trunks
c. divisions
d. cords
e. branches

A

d. cords

914
Q

A patient’s glomerular filtration rate is estimated at 35 mL/min/1.73m2. The patient’s chronic kidney disease can be classified as Stage
a) 2
b) 3a
c) 3b
d) 4
e) 5

A

c) 3b

Stage 1 >90
Stage 2 60-89
Stage 3a 45-59
Stage 3b 30-44
Stage 4 15-29
Stage 5 <15

915
Q

The Myocardial Injury after Non Cardiac Surgery study showed elevated troponin in the first three post-operative days was strongly associated with

a) 30 day mortality
b) 30 day MI

A

New question.

a) 30 day mortality

Postoperative myocardial injury was associated with an increased risk of death. Twenty-seven of the 315 patients (8.6%; 95% CI, 6.0–12.2%) with myocardial injury died within 30 days compared with 29 of the 1312 patients (2.2%; 95% CI, 1.5–3.2%) with normal troponin I levels (P<0.01)

Reference: Myocardial Injury After Noncardiac Surgery and its Association With Short-Term Mortality (Circulation 2013)

916
Q

The National Audit Project 6 found that the most common early clinical feature of perioperative anaphylaxis was

a) Arrest
b) Urticaria
c) Bronchospasm
d) Hypotension
e) CO2 down

A

d) Hypotension

The commonest presenting feature of perioperative anaphylaxis by far was hypotension (accounting for 46%), followed by bronchospasm/high airway pressure (18%), tachycardia (9.8%), flushing/non-urticarial rash 6.6% and cyanosis/oxygen desaturation (4.7%).

917
Q

You are inducing anaesthesia in a 20-year-old female through a cannula which was inserted in the right antecubital fossa while she was in the emergency department. After 10 ml of propofol has been injected, she complains of severe pain and it becomes clear that the cannula is intra-arterial. The most appropriate management is

a) aspirate
b) flush with N.Saline
c) flush with lignocaine
d) observe
e) flush with Heparin

A

You are inducing anaesthesia in a 20-year-old female through a cannula which was inserted in the right antecubital fossa while she was in the emergency department. After 10 ml of propofol has been injected, she complains of severe pain and it becomes clear that the cannula is intra-arterial. The most appropriate management is

https://www.anztadc.net/Publications/Images/ANZCA/Unintended%20Intraarterial%20injection%20WebAIRS%20news%20ANZCA%20Bulletin%20September%202019.pdf

918
Q

A patient with a history of restless leg syndrome is agitated in the post-anaesthesia care unit.
After excluding other causes, the best treatment of the agitation in this patient is

a) Pethidine
b) Clonidine
c) Droperidol
d) Haloperidol
e) Midazolam

A

midazolam

  • Opioids, benzodiazepines and pregabalin may also be used to alleviate symptoms.

Perioperative treatment of symptoms
If RLS symptoms occur perioperatively, patients should be allowed to walk or move their legs in bed as soon as possible.
If prolonged bed rest is required, the frequency of RLS medications may be increased to three times a day.
If oral intake is feasible, a patient’s usual oral medication may be given.
Levodopa (a dopamine agonist) may be administered by nasogastric tube.
Alternatively, parenteral apomorphine or a rotigotine patch may be used.
Apomorphine (1 milligram) may be injected subcutaneously on an hourly basis.
Nausea is a common side effect so it may need to be given with an antiemetic.
Rotigotine patches may be used every 24 hours.
Opioids, benzodiazepines and pregabalin may also be used to alleviate symptoms.
Patients should be proactively investigated and treated for iron deficiency, targeting ferritin level greater than 300 micrograms/ litre in adults, and 50 micrograms/litre in children.

919
Q

A man with a history of obesity and obstructive sleep apnoea has just had a transsphenoidal pituitary resection. Soon after extubation he is semi-conscious and is making a respiratory effort but has near complete upper airway obstruction with stridor. His arterial oxygen saturation is 93% and starting to fall. Your first actions should be to

a) Deepen with propofol and insert LMA
b) Insert Oropharyngeal airway and provided positive pressure ventilation
c) Insert Nasopharyngeal airway and provided positive pressure ventilation
d) Insert Nasopharyngeal airway and provide CPAP

A

a) Deepen with propofol and insert LMA

Nasal continuous positive airway pressure (CPAP) is contraindicated after transsphenoidal surgery due to the risk of tension pneumocephalous. The level of consciousness, eye movements, visual fields, and acuity should be tested frequently and any deterioration discussed with the surgeon, and radiological investigation and/or re-exploration considered.

https://academic.oup.com/bjaed/article/11/4/133/266875#3195876

920
Q

Measures to avoid venous air embolism when inserting an internal jugular central venous catheter in an awake patient include all of the following EXCEPT

a) Trendelenburg position
b) Occlude needle hub with thumb
c) Insert during inspiration
d) Pre-insertion IV fluid bolus

A

New question

Measures to avoid venous air embolism when inserting an internal jugular central venous catheter in an awake patient include all of the following EXCEPT

c) Insert during inspiration

Negative pressure generated by inspiration in an AWAKE patient

https://www.ncbi.nlm.nih.gov/pmc/articles/PMC5126790/

921
Q

Diffusing capacity of the lungs for carbon monoxide (DLCO) is decreased in all of the following EXCEPT

made up potential answers:

a) Pulmonary Fibrosis
b) Interstitial Lung disease
c) Obesity
d) Pulmonary haemorrhage

A

d) Pulmonary haemorrhage

Rewording of 21.2 Question

Won’t increase in Myasthenia Gravis

Causes of HIGH value include:
Asthma
Left-right intracardiac shunt
polycythaemia
Pulmonary haemorrhage
Obesity - Dlco will increase but kco will not

922
Q

A patient who is day 3 post laparotomy has used 30 mg oxycodone intravenously via patient controlled analgesia in the last 24 hours. The approximate oral morphine equivalent daily
dose is

a) 30mg
b) 45mg
c) 60mg
d) 90mg

A

90mg PO morphine

Oral Tapentadol 25mg = 8mg Oral Morphine

Oral Oxycodone 5mg = 8mg Oral Morphine

Oral Tramadol 25mg = Oral Morphine 5mg

Oral Hydromorphone 4mg = Oral Morphine 20mg

S/L Buprenorphine 200mcg = 8mg Oral Morphine

IV Oxycodone 5mg = Oral Morphine 15mg

IV Morphine 5mg = Oral Morphine 15mg

IV Hydromorphone 1mg = Oral Morphine 15mg

923
Q

The recommended dose of IV adrenaline in a 15 kg, 5 year old child with grade 2 (moderate) perioperative anaphylaxis is

a) 15mcg
b) 30mcg
c) 50mcg
d) 100mcg
e) 150mcg

A

b) 30mcg

Moderate = 2mcg/kg
Life threatening = 4-10mcg/kg

file:///Users/jbjon/Downloads/Australian_and_New_Zealand_Anaesthetic_Allergy_Gro.pdf

924
Q

The condition for which you would have a lower arterial oxygen saturation target is

Made up responses

a) Carbon monoxide poisoning
b) Bronchopulmonary Pneumonia
c) Bleomycin toxicity
d) Pulmonary Fibrosis

A

Answers could also possibly be COPD, Acute stroke or Neonates

c) Bleomycin toxicity

Bleomycin is a particularly important chemotherapy drug for the anaesthetist to be aware of. Bleomycin is often used to treat germ cell tumours and Hodgkin’s disease in a curative setting. The major limitation of bleomycin therapy is the potential for subacute pulmonary damage that can progress to life-threatening pulmonary fibrosis. Pulmonary toxicity occurs in 6–10% patients and can be fatal.2 Exposure to high-inspired concentration oxygen therapy, even for short periods, as experienced during anaesthesia, is often implicated in causing rapidly progressive pulmonary toxicity in patients previously treated with bleomycin.3 These claims have been considered controversial by some, but it is the authors’ recommendation that any patient previously exposed to bleomycin therapy should be treated as high risk, and summary guidance regarding oxygen therapy is shown in Table 4.

Summary guidance—oxygen therapy for patients who have received bleomycin > Patients have a life-long risk of bleomycin-induced lung injury
> Oxygen therapy should be avoided if at all possible
> Clinical procedures (and leisure activities) involving a high should be avoided If a patient is hypoxic
> O2 therapy should be minimized to maintain O2 saturation of 88–92%
> High oxygen concentrations should be used with extreme caution for immediate life-saving indications only (to maintain O2 saturation of 88–92%)

925
Q

In the thigh, the adductor canal is bordered by all of the following EXCEPT

a) Adductor Longus
b) Adductor Magnus
c) Sartorius
d) Vastus Lateralis
e) Vastus Medialis

A

d) Vastus Lateralis

Anteromedial: sartorius
Lateral: vastus medialis
Posterior: adductor longest and magnus

926
Q

When performing cannulation of the median cubital vein the structure that is LEAST likely to be inadvertently punctured or damaged is the

A) Radial artery
B) Median nerve
C) Brachial artery
D) Ulnar artery
E) Ulnar nerve

A

Repeat

e) Ulnar nerve

The cubital fossa is triangular in shape and consists of three borders, a roof, and a floor:
Lateral border – medial border of the brachioradialis muscle.
Medial border – lateral border of the pronator teres muscle.
Superior border – horizontal line drawn between the epicondyles of the humerus.
Roof – bicipital aponeurosis, fascia, subcutaneous fat and skin.
Floor – brachialis (proximally) and supinator (distally).
Contents:
- radial nerve
- biceps tendon
- brachial artery
- median nerve

Mnemonic for contents of the cubital fossa:
Really Need (radial nerve) Beer To (biceps tendon) Be At (brachial artery) My Nicest (median nerve).

927
Q

Following denervation injury to muscles, critical hyperkalaemia associated with suxamethonium administration can occur as early as

a) 12hrs
b) 18hrs
c) 24hrs
d) 48hrs

A

d) 24hrs

Extrajunctional receptors are not found in normal active
muscle but appear very rapidly whenever muscle activity has
ended or after injury has been sustained. They can appear
within 18 h of injury and an altered response to neuromuscu-
lar blocking drugs can be detected within 24 h of the insult.
They disappear when muscle activity returns to normal.

928
Q

For an adult patient with septic shock, the 2021 Surviving Sepsis Guidelines suggest using
procalcitonin to guide

a) Start/stop steroids
b) Stop antibiotics
c) Start CRRT
d) Source control

A

b) Stop/stop antibiotics

For adults with suspected sepsis or septic shock, we suggest AGAINST using procalcitonin plus clinical evaluation to decide when to start antimicrobials, as compared to clinical evaluation alone.
Weak, very low quality of evidence

For adults with an initial diagnosis of sepsis or septic shock and adequate source control where optimal duration of therapy is unclear, we suggest using procalcitonin AND clinical evaluation to decide when to discontinue antimicrobials over clinical evaluation alone.
Weak, low quality of evidence

929
Q

ANZCA guidelines recommend that under general anaesthesia, blood pressure should be
measured no less frequently than every

a) 2 mins
b) 3 mins
c) 5 mins
d) 10 mins

A

10mins
PG18

930
Q

Appropriate surgical anaesthesia with sevoflurane is characterized by a frontal EEG showing

a) Decreased alpha and delta waves
b) Increased alpha waves
c) anteriorisation alpha waves
d) Increased gamma and epsilon
e) increased spectral edge frequency

A

Increased alpha and slow delta power

During general anaesthesia with sevoflurane, the EEG shows increased α (8–12 Hz) and slow-δ oscillation power.9 This dynamic also closely approximates the EEG of general anaesthesia with propofol.9 Alpha oscillations are likely to originate from a mechanism similar to that proposed for the β oscillations. An increase in GABAA decay time and conductance results in cortical α oscillations and enhanced rebound spiking of thalamic relay cells, strengthening the intrinsic α oscillatory dynamic of the thalamus. The net result is reciprocal thalamic–cortical α oscillation coupling.13 Mechanisms to explain the slow-δ oscillations are being investigated. However, slow-δ oscillations may be associated with an alternation between ‘on’ states, in which neurones are able to fire, and ‘off’ states, in which neurones are silent.9 Different from propofol, sevoflurane general anaesthesia is also associated with increased frontal θ (4–8 Hz) oscillation power.1,9 The increase in θ oscillation power creates a distinctive pattern of distributed EEG power from the slow-δ oscillation through to the α oscillation range.

At an end-tidal sevoflurane concentration of 1.1%, the EEG shows increased slow-δ (0.1–4 Hz) and β (13–33 Hz) oscillations

BJA Ed

931
Q

The smallest endotracheal tube that can be railroaded over an Aintree Intubation Catheter has an internal diameter of

A. 4.0
B. 5.0
C. 6.0
D. 7.0
E. 8.0

A

Size 7.0

The Tube

The endotracheal tube has a length and diameter. The endotracheal tubes size (“give me a 6.0 tube”) refers to its internal diameter in millimeters (mm). The ETT will typically list both the inner diameter and outer diameter on the tube (for example, a 6.0 endotracheal tube will list both the internal diameter, ID 6.0, and outer diameter, OD 8.8).

932
Q

The drug that is LEAST likely to decrease blood flow to the splanchnic circulation is:

a) Noradrenaline
b) Adrenaline
c) Vasopressin
d) Dopamine
e) Phenylephrine

A

d) Dopamine

Dobutamine (β1 and β2), dopexamine (DA1, some β2) and low-dose dopamine (DA1 and DA2, β1 and β2, α1 in high dose) all have vasodilatory effects on the splanchnic circulation, and have been shown to improve markers of perfusion. For many years, low-dose infusions of dopamine were used as a prophylactic and therapy for acute renal failure, using the logic that DA1- and DA2-mediated vasodilation in renal and splanchnic beds would be protective.

https://pubmed.ncbi.nlm.nih.gov/12794401/

933
Q

A randomised control trial is performed on a new antiemetic medication. The rate of nausea in the placebo group is 20% and in the treatment group the rate is 5%. The number needed to treat to prevent nausea with this new drug is

A

NOT for this question

(base rate is 10%, experimental group is 8% (20% below 10%) therefore 100/ 2 = 50
or 1 divided by risk reduction

population risk = 10/100 patients get PONV
population risk + new antiemetic = 8/100 patients get PONV (8/100 as reduction by 20% with new drug)

RR= 0.10-0.08=0.02
NNT= 1/RR
=1/0.02
=50

934
Q

The cardiac arrhythmia most commonly associated with the chronic use of methadone is:

a) Torsades
b) VF
c) Tachycardia

A

a) Torsades

2ry to prolonged QT leading to R on T
PETKOV

935
Q

The maintenance anaesthetic technique that has the lowest environmental impact from
greenhouse gas is

a) sevoflurane
b) desflurane
c) Halothane
d) Ketamine
e) Propofol

A

e) Propofol

https://www.bjanaesthesia.org/article/S0007-0912(20)30547-X/pdf

936
Q

Cryoprecipitate is a concentrated source of all the following EXCEPT

a) Factor I
b) Factor VII
c) Factor VIII
d) VWF
e) Fibronectin

A

b) Factor VII

Redcross:
Cryoprecipitate contains most of the following found in fresh frozen plasma:
1. factor VIII
2. fibrinogen
3. factor XIII
4. von Willebrand factor
5. fibronectin

Prothrombinex-VF® is a lyophilised concentrate of human coagulation factors it contains:

Factors:
II
IX
X
small amount of factor VII.

Also contains:
plasma proteins (human)
Antithrombin III (human)
Heparin sodium (porcine)
Sodium
Phosphate
Citrate
Chloride

https://litfl.com/cryoprecipitate/

Fractionated plasma product consisting of Fibrinogen (Factor I), von Willebrand Factor, Factor VIII, and small amounts of Factor XIII and Fibronectin

https://www.anzca.edu.au/getattachment/9ec71c61-8a66-4f81-b0f8-c87d65e36298/Australasian-Anaesthesia-2023

937
Q

With regard to Donation after Circulatory Determination of Death (DCDD), the maximum
acceptable time from withdrawal of cardio-respiratory support to cold perfusion for liver
donation is

a) 30mins
b) 45 mins
c) 60 mins
d) 90 mins

A

Warm ischaemia time:
- Time from treatment withdrawal to the start of cold perfusion of the donated organs
- Significance is the impact on graft function
- Most important phase of WIT begins when the systolic BP is < 60mmHg
- This includes the waiting period from the absence of circulation to the declaration of death and the time before initiating the flow of cold perfusate through the cannula

Maximum WARM Ischaemia time
- Heart 30 mins
- Liver 30 mins
- Pancreas 30 mins
- Kidney 60 mins
- Lungs 90 mins

Maximum COLD Ischaemia time:
- Heart = 4 hrs
- Lungs = 6-8hrs
- Liver/Pancreas = 12hrs (DBD)/6 hrs (DCD)
- Kidneys = 18hrs (DBD)/ 12 hrs (DCD)

938
Q

When commencing treatment of proximal deep vein thrombosis or pulmonary embolus, factor Xa inhibitors (apixaban, rivaroxaban) are preferred to dabigatran or warfarin because they do not require

a. A need to dose reduce in pregnancy
b. No need to dose reduce in renal failure
c. No need to bridge
d. Need for monitoring
e. Once daily dosing

A

c. No need to bridge

?D
Dabigatran needs testing of renal function.
Warfarin needs testing of INR

Higher risk of bleeding with Dabigatran c/f other DOACs

See ETG recommendations

https://www.ahajournals.org/doi/full/10.1161/JAHA.120.017559

939
Q

A patient with a history of hereditary angioedema presents to the emergency department with difficulty with breathing, abdominal pain and swelling of the face, hands and feet. The most effective therapy for managing this is

a) C1-esterase inhibitor
b) Danazole
c) Cetirizine
d) FFP

A

a) C1-esterase inhibitor

Treatment options:
Plasma derived C1-esterase inhibitor = Berinert/Cinryze,
Androgens = Danazol
B2 Bradykinin Receptor antagonist = Icatibant
FFP.

Danazol (an androgen) is recommended as first line PROPHYLAXIS for planned procedures (need to give for 5-10 days prior and 2-5 days post)

For emergency or high risk procedures C1 esterase inhibitor concentrate (Berinert or Cinryze) is recommended
- give 1 hour before procedure
- more effective than danazol but more expensive

Berinert:
- 20units/kg IV over 10 min
- Symptoms usually stabilise in 30 mins
- 2nd dose uncommon, but may be given 30mins to 2hrs after 1st dose

Icatibant:
- 30mg slow subcut infusion in abdominal area

Due to the risk of precipitating laryngeal oedema, oropharyngeal procedures should usually involve general anaesthesia with endotracheal intubation

Short answer:
- if you have days before surgery increase danazole, if complex surgery increase danazole and give C1Inh
- If you have acute emergency surgery give C1Inh Concentrate (Berinert/Cinryze) before and after
- if you have an acute attack use C1Inh or Bradykinin antagonist (Icatibant)
- If C1 Inh and Bradykinin antagonoist are not available then use FFP but this may worsen the attack due to the presence of C4 in the FFP
- Has Cetirizine been misremembered instead of Cinryze as an option in this question? No it wasn’t
-> adrenaline, steroids, antihistamines have no role in treatment of HAE acute attack

940
Q

In the three-bottle chest drainage system set up shown, the maximum suction pressure (cmH2O) generated inside the underwater seal bottle would be minus

A

Depth of tube in water in bottle 3

941
Q

Somatic pain in the second stage of labour is NOT transmitted via the

a) Pudendal nerve
b) Illioinguinal
c) pelvic splanchnic
d) genitofemoral

A

c) pelvic splanchnic
-> visceral not somativ nerve

942
Q

During a thyroidectomy, the surgeon is concerned the parathyroid glands have been
devascularised. From the time of potential damage, a serum calcium level should be checked in

a) 6hrs
b) 12hrs
c) 24 hrs
d) 36hrs

A

24hrs

Oxford handbook

943
Q

A patient with a perioperative troponin rise above the upper limit of normal, chest pain, left ventricular anterior regional wall motion abnormality, and atheroma with a partially occluding thrombus of the left anterior descending coronary artery has had a/an

A) Type 1 MI
B) Type 2 MI
C) NSTEMI
D) MINS

A

A) Type 1 MI

Clinical classification based on the assumed proximate cause of the MI:
- Type 1
○ MI caused by atherothrombotic coronary artery disease
○ And usually precipitated by atherosclerotic plaque disruption ( rupture or erosion)
- Type 2
○ MI consequent to a mismatch between oxygen supply and demand
○ Multiple potentional mechanisms:
§ Coronary dissection
§ Vasospasm
§ Emboli
§ Microvascular dysfunction
§ Increases in demand without underlying coronary artery disease
- Type 3
○ Patient with typical presentation of MI (ECG changes or VF) with unexpected death before blood samples for biomarkers could be drawn
- Type 4a
○ MI associated with Percutaneous Coronary intervion (PCI)
- Type 4b
○ Subcategory of PCI related MI related to stent/scaffold thrombosis
- Type 5
CABG related MI

944
Q

A patient requires elective surgery under general anaesthesia with neuromuscular relaxation.
The recommended preoperative management of donepezil is to

a) cease day before
b) cease 2 weeks before
c) Cease day of surgery
d) continue

A

d) continue

to avoid cognitive decline post-op

Donepezil is in a class of medications called cholinesterase inhibitors. It improves mental function

https://www.ukcpa-periophandbook.co.uk/medicine-monographs/donepezil

945
Q

A 75 year-old patient is given a Fleet® sodium phosphate enema prior to a colonoscopy. The hyperphosphataemia from the laxative can directly cause

a) renal failure
b) cardiac failure
c) Arrhythmia
d) severe sleep apnoea

A

a) renal failure

‘…phosphate containing laxatives can lead to acute phosphate nephropathy’
https://academic.oup.com/bjaed/article/16/9/305/1743822#35669023 - BJA Ed article

Phosphate binds to calcium leading to crystal calcium phosphate deposition in tubules.

Old repeat 2020

https://academic.oup.com/bjaed/article/16/9/305/1743822#35669023

946
Q

An anaesthetic drug that is safe to use for a patient with porphyria is

a) propofol
b) ketamine
c) thiopentone
d) etomidate

A

a) propofol

947
Q

A patient has received high dose hydroxycobalamin for refractory vasoplegia post cardiac
surgery. Observed effects include all of the following EXCEPT

a) leukopenia
b) red urine
c) falsly low SpO2
d) thrombocytosis

A

c) falsly low SpO2

Effects of hihg dose hydroxycobalamin:
- red urine
- thrombocytosis
- leukopenia

948
Q

The bioavailability of an oral dose of ketamine is approximately

A. 10%
B. 20%
C. 40%
D. 70%
E. 80%

A

B. 20%

25% (a few studies have higher ranges but typically around 20-25%)

https://doi.org/10.1192/bjp.bp.115.165498

949
Q

A venturi mask delivers a fraction of inspired oxygen of 0.28 at the recommended fresh gas flow rate of 6 litres per minute. Increasing the flow rate to 12 litres per minute will deliver a fraction of inspired oxygen of

a) 0.24
b) 0.28
c) 0.36
d) 0.40

A

b) 0.28

950
Q

An inverted u wave is an electrocardiographic sign of

a) Hypokalaemia
b) Raised ICP
c) Digoxin treatment
d) Myocardial ischaemia

A

D> Myocardial ischaemia

An inverted U wave may represent myocardial ischemia (and especially appears to have a high positive predictive accuracy for left anterior descending coronary artery disease[7] ) or left ventricular volume overload.
^Wikipedia
——–
U-wave inversion is abnormal (in leads with upright T waves)
A negative U wave is highly specific for the presence of heart disease
Common causes of inverted U waves

Coronary artery disease
Hypertension
Valvular heart disease
Congenital heart disease
Cardiomyopathy
Hyperthyroidism
In patients presenting with chest pain, inverted U waves:

Are a very specific sign of myocardial ischaemia
May be the earliest marker of unstable angina and evolving myocardial infarction
Have been shown to predict a ≥ 75% stenosis of the LAD / LMCA and the presence of left ventricular dysfunction
^LITFL: https://litfl.com/u-wave-ecg-library/

951
Q

The use of direct oral anticoagulants [DOAC] in atrial fibrillation is contraindicated in the
presence of

a) Bioprosthetic Heart Valve
b) Mitral Regurgitation
c) mild hepatorenal impairment
d) Mitral Stenosis, moderate to severe

A

D) Mitral Stenosis (Rheumatic, moderate to severe)

DOAC use is contraindicated in certain clinical conditions, notably, in patients who have a mechanical heart valve and those with rheumatic mitral stenosis. Moderate to severe renal impairment or significant hepatic disease is also a contraindication to DOAC treatment

Bioprosthetic valves are less thrombogenic thus DOAC use is acceptable. https://www.ahajournals.org/doi/epdf/10.1161/JAHA.120.017559

952
Q

A 30 year old parturient presents in labour. She has a history of Addison’s disease from
autoimmune adrenalitis and has been taking prednisolone 6 mg daily for ten years. On
presentation the patient is given hydrocortisone 100 mg intravenously. The most appropriate steroid replacement regime the patient should receive during labour is

a. 25mg TDS hydrocortisone
b. 8mg/hr hydrocortisone
c. 6mg PO prednisone

A

8mg/hr

Guidelines for mx of glucocorticoids during the perioperative period for patients with adrenal insufficiency

https://associationofanaesthetists-publications.onlinelibrary.wiley.com/doi/10.1111/anae.14963

953
Q

One metabolic equivalent (MET) is equal to

a. O2 consumption walking 4km/h
b. O2 consumption when sitting
c. Energy expenditure walking 4km/h
d. Energy expenditure when sitting.

A

b) O2 consumption when sitting

One metabolic equivalent (MET) is defined as the amount of oxygen consumed while sitting at rest and is equal to 3.5 ml O2 per kg body weight x min.

954
Q

A 42-year-old female is admitted with subarachnoid haemorrhage. She has a severe
headache, has eyes open spontaneously, and is confused but is obeying commands. She is unable to move her left side. The World Federation of Neurological Surgeons grade is

a) 1
b) 2
c) 3
d) 4
e) 5

A

C:3 (Pt is GCS 14 E4V4M6, with motor deficit)

The WFNS scale:

Grade 1: GCS 15, no motor deficit.
Grade 2: GCS 13-14 without deficit
Grade 3: GCS 13-14 with focal neurological deficit
Grade 4: GCS 7-12, with or without deficit.
Grade 5: GCS <7 , with or without deficit.

(BJA Education, Deranged Physiology)

955
Q

Hepatopulmonary syndrome can be treated with

a) Methylene blue
b) Inhaled nitric oxide
c) Nitric oxide inhibitors
d) Oxygen therapy
e) Liver transplantation

A

e) Liver transplantation

  • Oxygen therapy for symptom relief
  • Liver transplant provides long term survival benefit
  • All other therapies tried but no conclusive evidence of benefit/nil are FDA approved

Hepatopulmonary Syndrome Article https://www.ncbi.nlm.nih.gov/books/NBK562169/

Hepatopulmonary syndrome (BJA)
- Prevalence up to 20% (end stage liver disease)
- Characterised by: disordered pulmonary capillary vasodilation and VQ mismatch
- Present with hypoxia, ortheodeoxia (decrease in PaO2 when standing)
- Diagnosis w/bubble echocardiography
- Risk factor for early post-transplant mortality
- If transplant successful, will resolve over time

956
Q

The most effective treatment for pain following wisdom teeth extraction as a single oral dose is

a) Paracetamol 1000mg
b) Tramadol 100mg
c) Parecoxib 40mg
d) Ibuprofen 400mg
e) Codeine 30mg

A

d) Ibuprofen 400mg
- Ibuprofen (I think, because of the single oral dose statement)

APMSE 5th edition

Acute pain after third molar extraction is the most extensively studied model for testing postoperative analgesics in single-dose investigations. Nonselective NSAIDs or coxibs are recommended as “first-line” analgesics following third molar extraction (Derry 2011 Level I, 155 RCTs, n=16,104), however paracetamol is also safe and effective with a dose of 1,000 mg providing better pain relief than lower doses (Weil 2007 Level I [Cochrane], 21 RCTs, n=1,968). The best available evidence suggests the use of NSAIDs either with or without paracetamol is effective and well-tolerated (Moore 2018 Level I, 5 SRs, n unspecified).

Nonselective NSAIDs are more effective than paracetamol or codeine (either alone or in combination) (Ahmad 1997 Level I, 33 RCTs, n=5,171). Ibuprofen (200–512 mg) specifically is superior to paracetamol (600–1,000 mg) in this setting and combining these two drugs improves analgesia further (Bailey 2014 Level I [Cochrane], 7 RCTs, n=2,241)

957
Q

You have induced a 20-year-old male for appendicectomy with propofol, fentanyl and suxamethonium. You are maintaining anaesthesia with oxygen, air and sevoflurane. His heart rate has climbed to 150 /minute, the ETCO2 is 50 mmHg and his temperature is 40°C.

After turning off the sevoflurane, you should

a) Commence TIVA
b) Give dantrolene 2.5mg/kg
c) Allocate task cards
d) Start active cooling
e) Remove vaporiser

A

e) Remove vaporiser

https://anaesthetists.org/Portals/0/PDFs/Guidelines%20PDFs/Guideline%20Malignant%20hyperthermia%202020.pdf?ver=2021-01-13-144236-793

as per guidelines, see link and attached image

As per anaesthetic crisis manual
1. Call for help, communicate and delegate
2. Stop any volatile and remove vaporiser
3. Allocated task cards
4. Give dantrolene
5. Hyperventilate with 100% high flow oxygen
6. Use activated charcoal filters on both limbs
7. Maintain anaesthesia with TIVA
8. Insert IAL +/- CVC
9. Actively cool if temperature > 38.5
10. Treat associated hyperkalaemia, acidosis, arrhythmias

958
Q

When using the ECG to time intra-aortic balloon counterpulsation, balloon deflation should occur at the

a. start of T wave
b. peak of T wave
c. end of T wave
d. end of R wave
e. peak of R wave

A

e. peak of R wave

https://derangedphysiology.com/main/required-reading/cardiothoracic-intensive-care/Chapter%20634/normal-iabp-waveform

https://litfl.com/intra-aortic-balloon-pump-trouble-shooting/

959
Q

Individuals with Prader-Willi syndrome having an anaesthetic are at most risk of

a) Hypocalcaemia
b) Hypoglycaemia
c) Neuroleptic malignant syndrome
d) Malignant hyperthermia
e) Hypothermia

A

b) Hypoglycaemia

Stoelting:
Prader-Willi syndrome is a rare genetic disorder characterized by
hypothalamic-pituitary abnormalities with severe hypotonia during the neonatal period and
during the first two years of life, hyperphagia with a risk of morbid obesity during infancy and
adulthood, learning difficulties and behavioural problems or severe psychiatric problems. The
disease affects 1/25,000 births.

https://www.orphananesthesia.eu/en/rare-diseases/published-guidelines/prader-willi-syndrome/1339-prader-willi-syndrome-2/file.html

Prader-Willi syndrome is a rare genetic disorder characterized by
hypothalamic-pituitary abnormalities with severe hypotonia during the neonatal period and
during the first two years of life, hyperphagia with a risk of morbid obesity during infancy and
adulthood, learning difficulties and behavioural problems or severe psychiatric problems. The
disease affects 1/25,000 births.

960
Q

Cross clamping of the descending aorta is NOT expected to cause
(MADE UP ANSWERS)

a) Bacterial translocation
b) Decreased Renal perfusion
c) Abdominal compartment syndrome
d) Organ ischaemia
e) Decreased afterload

A

e) decreased afterload

https://academic.oup.com/bjaed/article/13/6/208/246828#2904603

Aortic cross-clamping and physiological considerations

The physiological effect of aortic cross-clamping during surgery varies with the level of the clamp in relation to the main aortic branches. Perfusion to the lower half of the body is therefore dependent on collateral circulation while the clamp is applied.

Clamp application increases the afterload of the heart and a sudden increase in arterial pressure proximal to the clamp; this can be attenuated with vasodilators [e.g. glyceryl trinitrate (GTN), sodium nitroprusside], opioids, or deepening of anaesthesia. These measures may also allow fluid loading in preparation for clamp release; however, the effect of vasoactive drugs is unpredictable; they may change haemodynamics without improving cardiac output and tissue perfusion due to blood redistribution.10

Increased afterload and left ventricular end-diastolic volume both increase myocardial contractility and oxygen demand. This increase in myocardial oxygen demand is usually met by an increase in coronary blood flow and oxygen supply, but can cause myocardial ischaemia.

After aortic cross-clamp release, peripheral vascular resistance decreases by 70–80%, causing a decrease in arterial pressure. Hypotension can also be caused by blood sequestration in the lower half of the body, ischaemia–reperfusion injury, and the washout of anaerobic metabolites causing metabolic (lactic) acidosis. This can cause direct myocardial suppression and profound peripheral vasodilatation. Coronary blood flow and left ventricular end-diastolic volume also decrease (almost 50% from pre-clamp levels) after clamp release.

Strategies to manage hypotension after aortic cross-clamp release include gradual release of the clamp, volume loading, vasoconstrictors, or positive inotropic drugs (e.g. ephedrine, meteraminol, phenylephrine, epinephrine, and norepinephrine). It is important to be aware that vasoactive drugs should only be used after adequate volume repletion.10 Management of aortic cross-clamp application and release requires excellent communication with the surgeon in order to anticipate and manage the physiological effects.

961
Q

Of the following, the LEAST likely to occur during one-lung ventilation in the lateral decubitus position is

a. Intrapulmonary shunt
b. V/Q mismatch
c. Hypercarbia
d. Hypoxia
e. Hypoxic pulmonary vasoconstriction

A

c. Hypercarbia

Single-lung ventilation leads to a right-to-left intrapulmonary shunt as the nondependent lung continues to undergo perfusion with no ventilation, leading to a widened alveolar-to-arterial (A-a) oxygen gradient, which may contribute further to hypoxemia.

Factors leading to decreased blood flow to the ventilated lung also lead to hypoxemia.
Such factors include:
Low Fio2 leads to hypoxic pulmonary vasoconstriction in the dependent ventilated lung
High mean airway pressures in the dependent ventilated lung Vasoconstrictor agents
Intrinsic PEEP

The lateral decubitus position under anesthesia: Under anesthesia, there is a decrease in functional residual capacity. The upper lobe moves under anesthesia to a more favorable portion of the compliance curve versus the lower lung, which lies now on a less favorable portion of the compliance curve. Neuromuscular blockade contributes to abdominal contents pressing against the dependent hemidiaphragm, thereby restricting ventilation. Open non-dependent lung leads to variation in compliance and thus worsens ventilation-perfusion (V/Q) mismatch - thereby leading to hypoxemia. **Carbon dioxide elimination is usually unaffected **in using single-lung ventilation with adequate maintenance of minute ventilation. Both lungs may be affected independently by single-lung ventilation. The ventilated-dependent lung is prone to ventilator-induced lung injury due to higher tidal volumes used. The nondependent nonventilated lung is prone to injury by surgical trauma and ischemia-reperfusion injuries. Considering these physiological changes in single-lung ventilation is vital to safely performing the anesthetic technique and airway management.

Reference: StatPearls Single-Lung Ventilation https://www.ncbi.nlm.nih.gov/books/NBK538314/”

962
Q

Refeeding syndrome following the commencement of total parenteral nutrition is associated with the development of

A

Most likely answer will be related to hypophosphataemia

Refeeding syndrome is a constellation of biochemical abnormalities which occurs when normal intake is resumed after a period of starvation. Its characteristic features are low levels of phosphate, potassium, magnesium and sodium. Its major complications include cardiac arrhythmias, heart failure (due to hypophosphataemia), muscle weakness, rhabdomyolysis, seizures and an altered sensorium.

The major risk factors are calorie malnutrition of any cause, alcohol or drug use, low BMI (18-16) and starvation for 5-10 days.

Pathophysiology
With the restoration of glucose as a substrate, insulin levels rise and cause cellular uptake of these ions. Depletion of adenosine triphosphate (ATP) and 2,3-diphosphoglyceric acid (2,3-DPG) results in tissue hypoxia and failure of cellular energy metabolism. This may manifest as cardiac and respiratory failure, with paraesthesiae and seizures also reported. Thiamine deficiency may also play a part.

  • Exogenous sources of phosphate are inadequate to supplement the daily phosphate requirements
  • Intracellular phosphate stores are used to synthesise ATP (using protein and fat as fuel)
  • Homeostatic mechanisms maintain serum concentrations of these ions at the expense of intracellular stores

Reference: https://derangedphysiology.com/main/required-reading/endocrinology-metabolism-and-nutrition/Chapter%20315/refeeding-syndrome “

963
Q

The peak effect of intravenous insulin on serum potassium when treating hyperkalaemia
occurs at approximately

A. 2 mins
B. 4 mins
C. 10 mins
D. 20 mins
E. 30 mins

A

**D. 20 mins

**The time taken to reduce K+ with insulin/dextrose ranges from ~15-30 mins depending on source
**
https://www.uptodate.com/contents/treatment-and-prevention-of-hyperkalemia-in-adults Treatment approach to hyperkalemic emergencies — Patients with a hyperkalemic emergency should receive (table 1): Intravenous calcium and insulin are rapidly acting treatments that provide time for the initiation of therapies that remove the excess potassium from the body. ●Intravenous calcium to antagonize the membrane actions of hyperkalemia
●Intravenous insulin (typically given with intravenous glucose) to drive extracellular potassium into cells ●Therapy to rapidly remove excess potassium from the body (ie, loop or thiazide diuretics if renal function is not severely impaired, a gastrointestinal cation exchanger, and/or dialysis [preferably hemodialysis] if renal function is severely impaired)
●Treatment of reversible causes of hyperkalemia, such as correcting hypovolemia and discontinuing drugs that increase the serum potassium (eg, nonsteroidal anti-inflammatory drugs, inhibitors of the renin-angiotensin-aldosterone system)

RCH: http://www.rch.org.au/clinicalguide/guideline_index/Hyperkalaemia/ Insulin/glucose to be given at the same time

If severe hyperkalaemia:
- Dextrose 10% : 5ml/kg IV bolus (if no hyponatremia)
- Insulin short action: 0.1 U/kg IV bolus (Max 10 units) Then followed by infusion insulin/glucose (see below)

-If moderate hyperkalaemia:
- Dextrose 10% IV at maintenance with 0.9% sodium chloride (normal saline)
- Insulin short action infusion : 0.1 U/kg/h IV
Note: Close monitoring of glucose every 30-60 minutes
Onset of Action: 15 minutes, should reduce intravascular K+, reduction of 0.5-1.5mmol/L
Duration: peak 60 minutes, 2-3hours

American College of Emergency Physicians:
Nebulized albuterol by face mask begins to take measurable effect after 15 to 20 minutes and lowers the serum potassium level by up to 1 mEq/L, depending on the dose. β-Agonists are safe despite the side effect of tachycardia.

**Insulin, given intravenously in combination with glucose, also results in a similar fall in the potassium level after 20 to 30 minutes **and also lowers levels by up to 1 mEq/L. The combination of nebulized albuterol and intravenous insulin with glucose appears to be additive, lowering serum potassium by a mean of 1.21 mEq/L or more.11

Adult hyperkalemic patients who have ECG changes should receive continuous nebulized albuterol and 50 grams of intravenous dextrose plus 10 units of intravenous regular insulin.

Emergency Medicine Journal J Accid Emerg Med. 2000 May; 17(3): 188–191.
The management of hyperkalaemia in the emergency department

INSULIN WITH GLUCOSE
Insulin binds to specific membrane receptors and via an unknown second messenger, stimulates the sodium-potassium (Na-K) adenosine triphosphatase (ATP) pump resulting in intracellular uptake of K.5 This effect is independent of its hypoglycaemic action. Uraemia attenuates the hypoglycaemic response to insulin but does not affect its hypokalaemic action. Insulin has been the traditional temporising treatment against which newer treatments are compared. It is indicated in every case of hyperkalaemia that needs emergency treatment. Ten units (in adults) soluble insulin is given with 40–60 g glucose intravenously as a bolus. In children, a glucose load of 0.5 g/kg/h (2.5 ml/kg/h) should be given. This is because many of these patients increase their endogenous insulin production with the administration of a glucose load. If the blood glucose rises above 10 mml/l, insulin should be added at 0.05 u/kg/h.24 These studies show that the onset of hypokalaemic action is within 15 minutes and lasts for at least 60 minutes. The reduction in K observed is 0.65–1.0 mmol/l.5, 10 Delayed (30–60 minutes post insulin) hypoglycaemia is common (up to 75% of patients10) if less than 30 g glucose is given.

LITFL: Treatment of hyperkalaemia involves stabilizing the myocardium to prevent arrhythmias, shifting potassium back into the intracellular space and removing excess potassium from the body.

Drive Potassium into the Cell:
Insulin & Glucose
- Dose: IV fast acting insulin (actrapid) 10-20 units and glucose/dextrose 50g 25-50ml
- Insulin drives potassium into cells and administering glucose prevents hypoglycaemia.
- Begins to work in 20-30mins reduces potassium by 1mmol/L and ECG changes within the first hour Ca gluconate
- should be part of initial treatment but it does not lower either total body or serum potassium, it acts as a membrane stabiliser

LITFL: Correct Serious Conduction Abnormalities (Calcium)
- Calcium is a very useful agent. It does not lower the serum potassium level, but instead is used to stabilise the myocardium, as a temporising measure. Calcium is indicated if there is widening of QRS, sine wave pattern (when S and T waves merge together), or in hyperkalaemic cardiac arrest.
- The ‘cardiac membrane stabilising effects’ take about 15-30mins.

Calcium Chloride
- Dose: Calcium Chloride 10% 5-10mL = 6.8 mmol - 3 x more potent than Calcium Gluconate
- Complication: severe thrombophlebitis

  • Calcium Gluconate:
  • Dose: Calcium Gluconate 10% 5-10mL = 2.2 mmol
  • Less potent, less irritating to veins
  • Potential Complications of Calcium administration - Bradycardia, hypotension and peripheral vasodilation
  • Generally these occur if administered too quickly
  • Avoid in digoxin toxicity (use magnesium as alternative)
  • salbutamol
    Drive Potassium into the Cell: Salbutamol
  • Dose: 10-20mg via nebulizer - Beta 2 agonist therapy lower K via either IV or nebulizer route.
  • Salbutamol can lower potassium level 1mmol/L in about 30 minutes, and maintain it for up to 2 hours.
  • Very effective in renal patients that are fluid overloaded
  • Drive Potassium into the Cell: Sodium Bicarbonate
  • Dose: 50- 200mmol of 8.4% Sodium Bicarbonate
  • Bicarbonate is only effective at driving Potassium intracellullarly if the patient is acidotic
  • Begins working in 30-60 minutes and continues to work for several hours.

Eliminate Potassium From the Body: Calcium Resonium
- Dose: 15-45g orally or rectally, mixed with sorbitol or lactulose
- Calcium polystyrene sulfonate is a large insoluble molecule that binds potassium in the large intestine, where it is excreted in faeces
- Effects take 2-3 hours

M&M 2016: An intravenous infusion of glucose and insulin (30–50 g of glucose with 10 units of insulin) is also effective in promoting cellular uptake of potassium and lowering plasma [K+], but may take up to 1 h for peak effect

964
Q

A relative contraindication to a peribulbar needle technique for cataract surgery is:

a) Axial length of 24mm
b) INR 2.5 for mechanical aortic valve
c) Staphyloma
d) Scleral buckle
e) Pterygium

A

c) Staphyloma

https://eyewiki.aao.org/Ocular_Anesthesia#cite_note-:2-3

Contraindications

Absolute
Confirmed allergy to a necessary anesthetic, and nystagmus. Other contraindications are just those of the particular surgery that is to be performed.

Relative
Long eye (in the anterior to posterior axis, evidenced by high myopia), staphyloma (abnormal protrusion at a weak spot in the wall of the eye), enophthalmos (posteriorly or deep set eyes), and extended surgery duration are relative contraindications to retrobulbar and peribulbar anesthesia. In an uncooperative patient, patients deemed to be unable to follow commands during surgery, children and those with uncontrollable neurological movements, general anesthesia may be considered.

965
Q

Assuming a blood volume of 70 ml/kg, a massive transfusion in a 20 kg, 5-year-old child is
defined as a three-hour packed red blood cell (PRBC) transfusion volume of

a) 250ml
b) 500ml
c) 700ml
d) 1000ml

A

700ml
50% of blood volume in 3 hours

S Blaine. BLAE Paediatric massive transfusion

966
Q

According to the Association of Anaesthetists of Great Britain and Ireland (AAGBI)
guidelines, an acceptable reason to delay surgery in a patient with a fractured neck of femur is

A

https://associationofanaesthetists-publications.onlinelibrary.wiley.com/doi/10.1111/anae.15291

The 2011 guidelines list seven ‘acceptable’ reasons for delaying surgery:
1 Haemoglobin < 80 g.l−1
2 Plasma sodium concentration < 120 or > 150 mmol.l−1 and potassium concentration < 2.8 or > 6.0 mmol−1
3 Uncontrolled diabetes
4 Uncontrolled or acute onset left ventricular failure.
5 Correctable cardiac arrhythmia with a ventricular rate > 120.min−1
6 Chest infection with sepsis
7 Reversible coagulopathy

967
Q

Local anaesthetic blockade of the musculocutaneous nerve in the upper limb will result in
weakness of

A

All muscles in the anterior compartment of the arm are innervated by the musculocutaneous nerve.
- biceps brachii: forearm flexion and supination. Accessory shoulder flexor
- coracobrachialis: shoulder flexion, arm adduction.
- Brachialis: forearm flexion

The musculocutaneous nerve innervates skin on the anterolateral side of the forearm.

968
Q

In patients with symptomatic carotid stenosis, carotid endarterectomy can be performed
within two weeks of initial symptoms if there is/are

a) large stroke area
b) crescendo TIA symptoms
c) haemodynamic instability
d) Tandem Stenosis
e) contralateral occlusion

A

b) crescendo TIA symptoms

https://academic.oup.com/bja/article/99/1/119/269458

Ideally, patients presenting with a suspected TIA should have undergone investigation and, if appropriate, surgery within 2 weeks of presentation.

969
Q

Of the following, the drug most likely to cause pulmonary arterial vasodilation with systemic arterial vasoconstriction when used in low doses is

a) Adrenaline
b) Noradrenaline
c) Vasopressin
d) Dopamine
e) Dobutamine

A

c) Vasopressin

https://emcrit.org/ibcc/pressors/

  • From UP TO DATE:
    > At low doses of 1 to 3 mcg/kg per min, dopamine acts primarily on dopamine-1 receptors to dilate the renal and mesenteric artery beds
    > At 3 to 10 mcg/kg per min (and perhaps also at lower doses), dopamine also stimulates beta-1 adrenergic receptors and increases cardiac output, predominantly by increasing stroke volume with variable effects on heart rate.
    > At medium-to-high doses, dopamine also stimulates alpha-adrenergic receptors, although a small study suggested that renal arterial vasodilation and improvement in cardiac output may persist as the dopamine dose is titrated up to 10 mcg/kg per min
    *clinically, the haemodynamic effects of dopamine demonstrate individual variability

Dobutamine (inodilator):
- selective β1-agonist that increases cardiac contractility and reduces pulmonary vascular and systemic vascular resistances

Vasopressin:
- vasopressin may have pulmonary vasodilatory effects in addition to a systemic vasoconstrictive effect

Milrinone (inodilator):
- the phosphodiesterase-3 inhibitors, milrinone and enxoimone, have positive inotropic effects combined with the capacity to reduce RV afterload (‘inodilators’) without significant chronotropic effect, but they can be associated with significant systemic hypotension

970
Q

The option below which ranks these pressures from highest to lowest is (atm = atmosphere, cmH2O = centimetres of water, kPa = kilopascals, mmHg = millimetres of mercury, psi = pounds per square inch)

10 atm, 10 cmH2O, 10kPa, 10mmHg, 10PSI

A

All People Kick My Cat
Atm> PSI > KPA > mmHg > cmH2O

1ATM = 14.69 PSI = 101.325 kPa = 760mmHg = 1033 cmH20

971
Q

A patient taking tranylcypromine, a monoamine oxidase inhibitor, requires elective surgery.
The best management is to

(made up answers)

a) Cease 1 month before surgery
b) Do not Cease
c) Cease day of surgery
d) Cease 2 weeks before surgery
e) stop 2 weeks before, start moclobemide and omit Moclobemide day of surgery

A

e) stop 2 weeks before, start moclobemide and omit Moclobemide day of surgery
-> probably in discussion with the patients psychiatrist

Tranylcypromine, sold under the brand name Parnate among others, is a monoamine oxidase inhibitor. More specifically, tranylcypromine acts as nonselective and irreversible inhibitor of the enzyme monoamine oxidase.

In the elective setting, there is some debate regarding the management of patients on MAOI. Although the risks associated with anaesthesia in those taking this group of drugs are significant, abrupt withdrawal may precipitate serious psychiatric relapse. Traditionally, irreversible MAOIs have been stopped 2 weeks before operation; however, omitting the dose of moclobemide on the day of surgery is acceptable. It has been suggested that in the elective situation, patients could be switched from an irreversible MAOI to moclobemide to avoid a prolonged period of discontinuation.

972
Q

Jet ventilation for shared airway surgery is traditionally delivered at pressures in atmospheres (atm) of

a) 1 ATM
b) 2 ATM
c) 3 ATM
d) 4 ATM

A

b) 2 ATM

https://academic.oup.com/bjaed/article/7/1/2/509371

**A typical parameter-set for HFJV via a subglottic catheter is DP, 2 atm; f, 150 min−1; Fio2, 1.0; I-time, 50%.
**
Driving pressure 1-2 atm
(250-500ml/s)
RR 8-10

Automated jet ventilator – typical starting jet pressure for an adult is 1.5 bar (~1.5 atm).
Manual jet ventilators deliver up to 3.5-4 bar.

973
Q

A 46-year-old woman with menorrhagia is booked for abdominal hysterectomy. Her
preoperative bloods show

creatinine 55
Ca2+ 2.2
PO43- 0.34.

The most likely reason for these findings is

a) Diuretic use
b) Fanconi syndrome
c) Hyperparathyrodisim
d) Vit D deficiency
a) Iron transfusion

A

a) Iron transfusion

Iron infusion (ferric carboxymaltose) – can cause renal wasting of phosphate resulting in severe hypophosphataemia

Vitamin D deficiency and hyperparathyroidism can also cause hypophosphataemia. Vitamin D deficiency would result in low calcium whereas hyperparathyroidism would result in hypercalcaemia.
Fanconi syndrome: rare defect of proximal tubule leading to decreased reabsorption -> results in hypokalaemia, hypophosphataemia, hyperchloraemic metabolic acidosis. https://www.ncbi.nlm.nih.gov/pmc/articles/PMC6689119/

974
Q

The BALANCED Anaesthesia Study compared older patients having deep anaesthesia
(bispectral index target of 35) to lighter anaesthesia (bispectral index target of 50). It
assessed postoperative mortality and a substudy assessed postoperative delirium. These
showed that, compared to light anaesthesia, deep anaesthesia causes

a) Decreased mortality, no change in post op delirium (POD)
b) No change mortality, reduced POD
c) Decreased mortality, reduced POD
d) No change in Mortality, no change in POD

A

No change in Mortality, no change in POD

No evidence was found that mortality or serious complication were modified by targeting either a BIS of 50 or 35

A broad range of anaesthetic depth can be delivered safely when using volatile anaesthetic agents and processed electroencephalographic monitoring

https://www.thebottomline.org.uk/summaries/pom/balance/

975
Q

A 58-year-old man with ischaemic cardiomyopathy is undergoing a ventricular tachycardia ablation procedure in the catheter laboratory. Partway through the procedure his systolic blood pressure abruptly falls from 110 mmHg to 50 mmHg. The most likely cause for his hypotension is

a) Tamponade
b) RV failure
c) Arrhythmia
d) Anaphylaxis
e) Oesophageal aortic fistula

A

a) Tamponade

Cardiac tamponade occurs ~1%. Can usually be managed with reversal of anticoagulation and percutaneous drainage.

Vascular complications most common followed by tamponade.

https://www.ahajournals.org/doi/10.1161/circep.113.000768

https://academic.oup.com/bjaed/article/12/5/230/289246#3659733

976
Q

The coronary artery most likely occluded in this ECG of an acute ST-elevation myocardial
infarction is the

a) RCA
b) L Cx
c) LAD
d) Left Main CA
e) Posterior Descending CA

A

c) LAD

977
Q

The most appropriate initial diagnostic test for a suspected phaeochromocytoma is a/an

A

serum free metanephrines and nor-metanephrines

https://www.ncbi.nlm.nih.gov/pmc/articles/PMC3230088/

https://www1.racgp.org.au/ajgp/2021/january-february/adrenal-disease-an-update

978
Q

An adult patient is administered a target controlled propofol infusion for more than 30
minutes with a constant effect-site target of 4 mcg/ml propofol plasma concentration.
Compared to the Schnider model, the propofol dose given by the Eleveld model will be a

a) Smaller bolus lower infusion rate
b) Smaller bolus hihger infusion rate
c) Larger bolus lower infusion rate
d) Larger bolus highier infusion rate
e) Smaller bolus same infusion rate

A

c) Larger bolus lower infusion rate

https://associationofanaesthetists-publications.onlinelibrary.wiley.com/doi/10.1111/anae.13345

https://journals.lww.com/anesthesia-analgesia/fulltext/2014/06000/a_general_purpose_pharmacokinetic_model_for.12.aspx

979
Q

A 4 week old full term neonate with an inguinal hernia, who is otherwise healthy, has an ASA (American Society of Anesthesiologists) classification of at least

a) 1
b) 2
c) 3
d) 4

A

ASA 3

980
Q

You are inserting a pulmonary artery catheter in an intubated patient prior to cardiac surgery, and a significant amount of blood appears in the endotracheal tube. The most appropriate specific initial management is to

A. Remove PAC and insert DLT
B. Wedge PAC and insert DLT
C. Wedge PAC and insert bronchial blocker
D. Withdraw PAC 2 cm and insert DLT
E. Withdraw PAC and insert bronchial blocker
F. Inflate balloon

A

D. Withdraw PAC 2 cm and insert DLT

LITFL: Pulmonary haemorrhage after PAOP measurement

a life threatening time-critical emergency
pulmonary artery rupture caused by inflation of the pulmonary artery catheter (PAC) balloon during ‘wedging’ (measurement of the pulmonary artery occlusion pressure)
some experts advise against measuring PAWP because of the risk of pulmonary artery rupture
0.2% risk,  30% mortality

RISK FACTORS

pulmonary hypertension
mitral valve disease
anticoagulants
age >60 years

MANAGEMENT

Goals

prevent further pulmonary haemorrhage
stop bleeding
resuscitate

Call for help

ICU consultant
anaesthetist/ OT
cardiothoracic surgeon
interventional radiology

Resuscitation

A
    may have to be emergently intubated if not already
B
    FiO2 1.0
    controlled ventilation
    if able to recognize which lung is haemorrhaging may be able to perform lung isolation (insert single lumen tube into unaffected side, exchange for a double lumen tube or use bronchial blocker with bronchoscopic assistance)
    apply PEEP to tamponade wound
C
    large bore IV cannulae, fluids, blood products, inotropes

Specific therapy

Lay the patient ruptured side down
withdraw pulmonary catheter 2-3 cm with balloon down then refloat PAC with balloon inflated to occlude pulmonary artery (to try to tamponade bleeding)
stop antiplatelet agents and anticoagulants
give reversal agents:
— protamine for heparin
— platelets for anti-platelet agents
give blood products as indicated by FBC, coags and clinical state
interventions
— angiogram or bronchoscopy to isolate pulmonary vessel involved
— if bleeding doesn’t settle will require lobectomy
981
Q

This patient has been requested to look straight ahead. He is suffering from a right

a) Horner’s Syndrome
b) 3rd nerve palsy
c) 4th nerve palsy
d) 6th nerve palsy

A

b) 3rd nerve palsy

https://derangedphysiology.com/main/required-reading/neurology-and-neurosurgery/Chapter%204631/lesions-oculomotor-nerve-cn-iii
This is the “down and out” eye syndrome. It is characterised by ptosis, a down-and-out pupil, mydriasis, absent light reflex with intact consensual constriction of the opposite eye, and failure of accommodation. Classically, this is the lesion which develops during uncal herneation, due to an ipsilateral cerebral injury.

Causes of unilateral CN III lesions:
- Uncal herneation: Pressure from herniating uncus on nerve
- Fracture involving ipsilateral cavernous sinus
- Cavernous sinus thrombosis (ipsilateral)
- Aneurysm (ipsilateral)
- Midbrain lesion (see Question 26.2 from the second paper of 2011)

Causes of bilateral CN III lesions:
- Cavernous sinus thrombosis
- Aneurysm
- Contralateral brainstem lesion (midbrain)

Exclusion of a 4th nerve lesion
- Tilt the head to the same side as the lesion
- The affected eye will intort if the fourth nerve is intact.

982
Q

The antiemetic least likely to precipitate an arrhythmia in a patient with this ECG is

a) Droperidol
b) Metoclopramide
c) Promethazine
d) Dexamethasone
e) Ondansetron

A

d) Dexamethasone
The ECG shows LONG QT

https://litfl.com/qt-interval-ecg-library/

983
Q

Of the following, the lowest level at which neurogenic shock is likely if an acute spinal cord
injury were to occur at that level is

a) C2
b) C6
b) T4
c) T6
d) T10

A

c) T6

LITFL: https://litfl.com/trauma-spinal-injury/

Neurogenic shock is classically characterised by hypotension, bradycardia and peripheral vasodilatation. Neurogenic shock is due to loss of sympathetic vascular tone and happens only after a significant proportion of the sympathetic nervous system has been damaged – as may occur with lesions at the T6 level or higher.

Spinal shock is not a true form of shock. It refers to the flaccid areflexia that may occur after spinal cord injury, and may last hours to weeks. It may be thought of as ‘concussion’ of the spinal cord and resolves as soft tissue swelling improves. Priapism may be present.